Download as docx, pdf, or txt
Download as docx, pdf, or txt
You are on page 1of 286

1.

THE APPLICABLE LAWS

2.BASIC PRINCIPLES

1. Jao vs. BCC Products Sales Inc. G.R. No. 163700, April 18, 2012

PRINCIPLE: The existence of an employer-employee relationship is a question of fact. Generally, a re-examination


of factual findings cannot be done by the Court acting on a petition for review on  certiorari  because the Court is not a
trier of facts but reviews only questions of law. Nor may the Court be bound to analyze and weigh again the evidence
adduced and considered in the proceedings below.  This rule is not absolute, however, and admits of exceptions. For
one, the Court may look into factual issues in labor cases when the factual findings of the Labor Arbiter, the NLRC,
and the CA are conflicting.

FACTS:

Petitioner maintained that respondent BCC Product Sales Inc. (BCC) and its President, respondent Terrance Ty (Ty),
employed him as comptroller starting from September 1995 with a monthly salary of P20,000.00 to handle the
financial aspect of BCC’s business; that on October 19,1995, the security guards of BCC, acting upon the instruction
of Ty, barred him from entering the premises of BCC where he then worked. He filed a complaint dated December
28, 1995 for illegal dismissal, reinstatement with full backwages, non-payment of wages, damages and attorney’s
fees.

Respondents countered that petitioner was not their employee but the employee of Sobien Food Corporation (SFC),
the major creditor and supplier of BCC

ISSUE:

The sole issue is whether or not an employer-employee relationship existed between petitioner and BCC. A finding
on the existence of an employer-employee relationship will automatically warrant a finding of illegal dismissal,
considering that respondents did not state any valid grounds to dismiss petitioner.

RULING:

The existence of an employer-employee relationship is a question of fact. Generally, a re-examination of factual


findings cannot be done by the Court acting on a petition for review on certiorari because the Court is not a trier of
facts but reviews only questions of law. Nor may the Court be bound to analyze and weigh again the evidence
adduced and considered in the proceedings below. This rule is not absolute, however, and admits of exceptions. For
one, the Court may look into factual issues in labor cases when the factual findings of the Labor Arbiter, the NLRC,
and the CA are conflicting.

Here, the findings of the NLRC differed from those of the Labor Arbiter and the CA. This conflict among such
adjudicating offices compels the Court’s exercise of its authority to review and pass upon the evidence presented and
to draw its own conclusions therefrom.

Our perusal of the affidavit of petitioner compels a conclusion similar to that reached by the CA and the Labor Arbiter
to the effect that the affidavit actually supported the contention that petitioner had really worked in BCC as SFC’s
representative. It does seem more natural and more believable that petitioner’s affidavit was referring to his
employment by SFC even while he was reporting to BCC as a comptroller in behalf of SFC. As respondents pointed
out, it was implausible for SFC to still post him to oversee and supervise the collections of accounts receivables due
from BCC beyond December 1995 if, as he insisted, BCC had already illegally dismissed him and had even
prevented him from entering the premises of BCC. Given the patent animosity and strained relations between him
and respondents in such circumstances, indeed, how could he still efficiently perform in behalf of SFC the essential
responsibility to “oversee and supervise collections” at BCC? Surely, respondents would have vigorously objected to
any arrangement with SFC involving him.

Moreover, in determining the presence or absence of an employer-employee relationship, the Court has consistently
looked for the following incidents, to wit:  (a) the selection and engagement of the employee; (b) the payment of
wages; (c) the power of dismissal; and (d) the employer’s power to control the employee on the means and methods
by which the work is accomplished. The last element, the so-called control test, is the most important element.

Hereunder are some of the circumstances and incidents occurring while petitioner was supposedly employed by BCC
that debunked his claim against respondents. It can be deduced from the March 1996 affidavit of petitioner that
respondents challenged his authority to deliver some 158 checks to SFC. Considering that he contested respondents’
challenge by pointing to the existing arrangements between BCC and SFC, it should be clear that respondents did
not exercise the power of control over him, because he thereby acted for the benefit and in the interest of SFC more
than of BCC.  
In addition, petitioner presented no document setting forth the terms of his employment by BCC. The failure to
present such agreement on terms of employment may be understandable and expected if he was a common or
ordinary laborer who would not jeopardize his employment by demanding such document from the employer, but may
not square well with his actual status as a highly educated professional.
2. Legend Hotel (Manila) vs. Realuyo, G.R. No. 153511, July 18, 2012

PRINCIPLE: Generally, the Court does not review factual questions, primarily because the Court is not a trier of
facts. However, where, like here, there is a conflict between the factual findings of the Labor Arbiter and the NLRC,
on the one hand, and those of the CA, on the other hand, it becomes proper for the Court, in the exercise of its equity
jurisdiction, to review and re-evaluate the factual issues and to look into the records of the case and re-examine the
questioned findings.

FACTS:

This labor case for illegal dismissal involves a pianist employed to perform in the restaurant of a hotel. On August 9,
1999, respondent, whose stage name was Joey R. Roa, filed a complaint for alleged unfair labor practice,
constructive illegal dismissal, and the underpayment/nonpayment of his premium pay for holidays, separation pay,
service incentive leave pay, and 13th month pay. He prayed for attorney's fees, moral damages off P100,000.00 and
exemplary damages for P100,000.00.

Respondent averred that he had worked as a pianist at the Legend Hotel’s Tanglaw Restaurant from September
1992 with an initial rate of P400.00/night that was given to him after each night’s performance; that his rate had
increased to P750.00/night; and that during his employment, he could not choose the time of performance, which had
been fixed from 7:00 pm to 10:00 pm for three to six times/week. He added that the Legend Hotel’s restaurant
manager had required him to conform with the venue’s motif; that he had been subjected to the rules on employees’
representation checks and chits, a privilege granted to other employees; that on July 9, 1999, the management had
notified him that as a cost-cutting measure his services as a pianist would no longer be required effective July 30,
1999; that he disputed the excuse, insisting that Legend Hotel had been lucratively operating as of the filing of his
complaint; and that the loss of his employment made him bring his complaint.

ISSUES:

(a) whether or not respondent was an employee of petitioner; and


(b) if respondent was petitioner’s employee, whether he was validly terminated.

RULING:

Substantive Issue No. 1: Employer-employee relationship existed between the parties


We next ascertain if the CA correctly found that an employer-employee relationship existed between the parties.

The issue of whether or not an employer-employee relationship existed between petitioner and respondent is
essentially a question of fact. The factors that determine the issue include who has the power to select the employee,
who pays the employee’s wages, who has the power to dismiss the employee, and who exercises control of the
methods and results by which the work of the employee is accomplished. Although no particular form of evidence is
required to prove the existence of the relationship, and any competent and relevant evidence to prove the relationship
may be admitted, a finding that the relationship exists must nonetheless rest on substantial evidence, which is that
amount of relevant evidence that a reasonable mind might accept as adequate to justify a conclusion.

Generally, the Court does not review factual questions, primarily because the Court is not a trier of facts. However,
where, like here, there is a conflict between the factual findings of the Labor Arbiter and the NLRC, on the one hand,
and those of the CA, on the other hand, it becomes proper for the Court, in the exercise of its equity jurisdiction, to
review and re-evaluate the factual issues and to look into the records of the case and re-examine the questioned
findings.

A review of the circumstances reveals that respondent was, indeed, petitioner’s employee. He was undeniably
employed as a pianist in petitioner’s Madison Coffee Shop/Tanglaw Restaurant from September 1992 until his
services were terminated on July 9, 1999.

A review of the records shows, however, that respondent performed his work as a pianist under petitioner’s
supervision and control. Specifically, petitioner’s control of both the end achieved and the manner and means used to
achieve that end was demonstrated by the following, to wit:

a. He could not choose the time of his performance, which petitioners had fixed from 7:00 pm to 10:00 pm,
three to six times a week;
b. He could not choose the place of his performance;
c. The restaurant’s manager required him at certain times to perform only Tagalog songs or music, or to wear
barong Tagalog to conform to the Filipiniana motif; and
d. He was subjected to the rules on employees’ representation check and chits, a privilege granted to other
employees.
Substantive Issue No. 2: Validity of the Termination
Having established that respondent was an employee whom petitioner terminated to prevent losses, the conclusion
that his termination was by reason of retrenchment due to an authorized cause under the Labor Code is inevitable.

The Court has laid down the following standards that an employer should meet to justify retrenchment and to foil
abuse, namely:

a) The expected losses should be substantial and not merely de minimis in extent;
b) The substantial losses apprehended must be reasonably imminent;
c) The retrenchment must be reasonably necessary and likely to effectively prevent the expected losses; and
d) The alleged losses, if already incurred, and the expected imminent losses sought to be forestalled must be
proved by sufficient and convincing evidence.

Was the retrenchment of respondent valid?

In termination cases, the burden of proving that the dismissal was for a valid or authorized cause rests upon the
employer. Here, petitioner did not submit evidence of the losses to its business operations and the economic havoc it
would thereby imminently sustain. It only claimed that respondent’s termination was due to its "present
business/financial condition." This bare statement fell short of the norm to show a valid retrenchment. Hence, we hold
that there was no valid cause for the retrenchment of respondent.

Indeed, not every loss incurred or expected to be incurred by an employer can justify retrenchment.1âwphi1 The
employer must prove, among others, that the losses are substantial and that the retrenchment is reasonably
necessary to avert such losses. Thus, by its failure to present sufficient and convincing evidence to prove that
retrenchment was necessary, respondent’s termination due to retrenchment is not allowed.
3. The New Philippine Skylanders, Inc., vs. Dakila, G.R. No. 199547, Sept. 24, 2012

PRINCIPLE: The issue of illegal dismissal is premised on the existence of an employer-employee relationship
between the parties herein. It is essentially a question of fact, beyond the ambit of a petition for review
on  certiorari  under Rule 45 of the Rules of Court unless there is a clear showing of palpable error or arbitrary
disregard of evidence which does not obtain in this case.

FACTS:

Respondent Dakila was employed by petitioner corporation as early as 1987 and terminated for cause in April 1997
when the corporation was sold. In May 1997, he was rehired as consultant by the petitioners under a Contract for
Consultancy Services6 dated April 30, 1997.

Thereafter, in a letter7 dated April 19, 2007, respondent Dakila informed petitioners of his compulsory retirement
effective May 2, 2007 and sought for the payment of his retirement benefits pursuant to the Collective Bargaining
Agreement. His request, however, was not acted upon. Instead, he was terminated from service effective May 1,
2007.

Consequently, respondent Dakila filed a complaint for constructive illegal dismissal, non-payment of retirement
benefits, under/non-payment of wages and other benefits of a regular employee, and damages against petitioners.
On the other hand, petitioners, in their position paper,8 asserted that respondent Dakilawas a consultant and not their
regular employee. The latter was not included in petitioners' payroll and paid a fixed amount under the consultancy
contract. He was not required to observe regular working hours and was free to adopt means and methods to
accomplish his task except as to the results of the work required of him. Hence, no employer-employee relationship
existed between them. Moreover, respondent Dakila terminated his contract in a letter dated April 19, 2007, thus,
negating his dismissal.

ISSUE:

Whether petitioner was illegally dismissed.

RULING:

The issue of illegal dismissal is premised on the existence of an employer-employee relationship between the parties
herein. It is essentially a question of fact, beyond the ambit of a petition for review on certiorari under Rule 45 of the
Rules of Court unless there is a clear showing of palpable error or arbitrary disregard of evidence which does not
obtain in this case. Records reveal that both the LA and the NLRC, as affirmed by the CA, have found substantial
evidence to show that respondent Dakila was a regular employee who was dismissed without cause.

Following Article 279 of the Labor Code, an employee who is unjustly dismissed from work is entitled to reinstatement
without loss of seniority rights and other privileges and to his full backwages computed from the time he was illegally
dismissed. However, considering that respondent Dakila was terminated on May 1, 2007, or one (1) day prior to his
compulsory retirement on May 2, 2007, his reinstatement is no longer feasible. Accordingly, the NLRC correctly held
him entitled to the payment of his retirement benefits pursuant to the CBA. On the other hand, his backwages should
be computed only for days prior to his compulsory retirement which in this case is only a day. Consequently, the
award of reinstatement wages pending appeal must be deleted for lack of basis.
4. Tesoro et al., vs. Metro Manila Retreaders Inc., et al., GR No. 171482, March 12, 2014

This case concerns the effect on the status of employment of employees who entered into a Service Franchise
Agreement with their employer.

PRINCIPLE: Franchising involves the use of an established business expertise, trademark, knowledge, and training.
As such, the franchisee is required to follow a certain established system. Accordingly, the franchisors may impose
guidelines that somehow restrict the petitioners’ conduct which do not necessarily indicate “control.” The important
factor to consider is still the element of control over how the work itself is done, not just its end result.

FACTS:

On various dates between 1991 and 1998, petitioners Ashmor M. Tesoro, Pedro Ang, and Gregorio Sharp used to
work as salesmen for respondents Metro Manila Retreaders, Inc., Northern Luzon Retreaders, Inc., or Power Tire
and Rubber Corporation, apparently sister companies, collectively called “Bandag.” Bandag offered repair and
retread services for used tires. In 1998, however, Bandag developed a franchising scheme that would enable others
to operate tire and retreading businesses using its trade name and service system.
Petitioners quit their jobs as salesmen and entered into separate Service Franchise Agreements (SFAs) with Bandag
for the operation of their respective franchises. Under the SFAs, Bandag would provide funding support to the
petitioners subject to a regular or periodic liquidation of their revolving funds. The expenses out of these funds would
be deducted from petitioners’ sales to determine their incomes.

ISSUE:

The only issue presented in this case is whether or not petitioners remained to be Bandag’s salesmen under the
franchise scheme it entered into with them.

RULING:

Franchising is a business method of expansion that allows an individual or group of individuals to market a product or
a service and to use of the patent, trademark, trade name and the systems prescribed by the owner.

The question is: did petitioners remain to be Bandag’s employees after they began operating those branches? The
tests for determining employer–employee relationship are: (a) the selection and engagement of the employee; (b) the
payment of wages; (c) the power of dismissal; and (d) the employer’s power to control the employee with respect to
the means and methods by which the work is to be accomplished. The last is called the “control test,” the most
important element.

When petitioners agreed to operate Bandag’s franchise branches in different parts of the country, they knew that this
substantially changed their former relationships. They were to cease working as Bandag’s salesmen, the positions
they occupied before they ventured into running separate Bandag branches. They were to cease receiving salaries or
commissions. Their incomes were to depend on the profits they made. Yet, petitioners did not then complain of
constructive dismissal. They took their chances, ran their branches, Gregorio Sharp in La Union for several months
and Ashmor Tesoro in Baguio and Pedro Ang in Pangasinan for over a year. Clearly, their belated claim of
constructive dismissal is quite hollow.

It is pointed out that Bandag continued, like an employer, to exercise control over petitioners’ work. It points out that
Bandag: (a) retained the right to adjust the price rates of products and services; (b) imposed minimum processed tire
requirement (MPR); (c) reviewed and regulated credit applications; and (d) retained the power to suspend petitioners’
services for failure to meet service standards.

But uniformity in prices, quality of services, and good business practices are the essence of all franchises. A
franchisee will damage the franchisor’s business if he sells at different prices, renders different or inferior services, or
engages in bad business practices. These business constraints are needed to maintain collective responsibility for
faultless and reliable service to the same class of customers for the same prices.
This is not the “control” contemplated in employer–employee relationships. Control in such relationships addresses
the details of day to day work like assigning the particular task that has to be done, monitoring the way tasks are
done and their results, and determining the time during which the employee must report for work or accomplish his
assigned task.

Franchising involves the use of an established business expertise, trademark, knowledge, and training. As such, the
franchisee is required to follow a certain established system. Accordingly, the franchisors may impose guidelines that
somehow restrict the petitioners’ conduct which do not necessarily indicate “control.” The important factor to consider
is still the element of control over how the work itself is done, not just its end result.
The Labor Arbiter, the NLRC, and the CA, are unanimous that petitioners were no longer “route salesmen, bringing
previously ordered supplies and goods to dealers, taking back returned items, collecting payments, remitting them,
etc. They were themselves then the dealers, getting their own supply and bringing these to their own customers and
sub–dealers, if any.”
5. Royale Homes Marketing Corp., vs. Alcantara, GR No. 195190, July 28, 2014

PRINCIPLE: Not every form of control that a hiring party imposes on the hired party is indicative of employee-
employer relationship. Rules and regulations that merely serve as guidelines towards the achievement of a mutually
desired result without dictating the means and methods of accomplishing it do not establish employer-employee
relationship.

FACTS:

Royale Homes, a corporation engaged in marketing real estates, appointed Alcantara as its Marketing Director for a
fixed period of one year.  His work consisted mainly of marketing Royale Homes’ real estate inventories on an
exclusive basis.  Royale Homes reappointed him for several consecutive years, the last of which covered the period
January 1 to December 31, 2003 where he held the position of Division 5 Vice-President-Sales.

Alcantara filed a Complaint for Illegal Dismissal9 against Royale Homes and its President Matilde Robles, Executive
Vice-President for Administration and Finance Ma. Melinda Bernardino, and Executive Vice- President for Sales
Carmina Sotto. 

Royale Homes, on the other hand, vehemently denied that Alcantara is its employee. It argued that the appointment
paper of Alcantara is clear that it engaged his services as an independent sales contractor for a fixed term of one
year only.  He never received any salary, 13th month pay, overtime pay or holiday pay from Royale Homes as he was
paid purely on commission basis. In addition, Royale Homes had no control on how Alcantara would accomplish his
tasks and responsibilities as he was free to solicit sales at any time and by any manner which he may deem
appropriate and necessary.  He is even free to recruit his own sales personnel to assist him in pursuance of his sales
target.

ISSUE:

The pivotal issue to be resolved in this case is whether Alcantara was an independent contractor or an employee of
Royale Homes.

RULING:

The juridical relationship of the parties based on their written contract

The primary evidence of the nature of the parties’ relationship in this case is the written contract that they signed and
executed in pursuance of their mutual agreement.  While the existence of employer-employee relationship is a matter
of law, the characterization made by the parties in their contract as to the nature of their juridical relationship cannot
be simply ignored, particularly in this case where the parties’ written contract unequivocally states their intention at
the time they entered into it.

In this case, the contract,27 duly signed and not disputed by the parties, conspicuously provides that “no employer-
employee relationship exists between” Royale Homes and Alcantara, as well as his sales agents.  It is clear that they
did not want to be bound by employer-employee relationship at the time of the signing of the contract. Since “the
terms of the contract are clear and leave no doubt upon the intention of the contracting parties, the literal meaning of
its stipulations should control.”

The juridical relationship of the parties based on Control Test 

In determining the existence of an employer-employee relationship, this Court has generally relied on the four-fold
test, to wit: (1) the selection and engagement of the employee; (2) the payment of wages; (3) the power of dismissal;
and (4) the employer’s power to control the employee with respect to the means and methods by which the work is to
be accomplished.29  Among the four, the most determinative factor in ascertaining the existence of employer-
employee relationship is the “right of control test”.

Not every form of control is indicative of employer-employee relationship.  A person who performs work for another
and is subjected to its rules, regulations, and code of ethics does not necessarily become an employee.34   As long
as the level of control does not interfere with the means and methods of accomplishing the assigned tasks, the rules
imposed by the hiring party on the hired party do not amount to the labor law concept of control that is indicative of
employer-employee relationship. 

Logically, the line should be drawn between rules that merely serve as guidelines towards the achievement of the
mutually desired result without dictating the means or methods to be employed in attaining it, and those that control
or fix the methodology and bind or restrict the party hired to the use of such means. The first, which aim only to
promote the result, create no employer-employee relationship unlike the second, which address both the result and
the means used to achieve it.
In this case, the Court agrees with Royale Homes that the rules, regulations, code of ethics, and periodic evaluation
alluded to by Alcantara do not involve control over the means and methods by which he was to perform his job. 
Understandably, Royale Homes has to fix the price, impose requirements on prospective buyers, and lay down the
terms and conditions of the sale, including the mode of payment, which the independent contractors must follow.  It is
also necessary for Royale Homes to allocate its inventories among its independent contractors, determine who has
priority in selling the same, grant commission or allowance based on predetermined criteria, and regularly monitor the
result of their marketing and sales efforts.  But to the mind of this Court, these do not pertain to the means and
methods of how Alcantara was to perform and accomplish his task of soliciting sales.  They do not dictate upon him
the details of how he would solicit sales or the manner as to how he would transact business with prospective clients. 
In Tongko, this Court held that guidelines or rules and regulations that do not pertain to the means or methods to be
employed in attaining the result are not indicative of control as understood in labor law. 

Guidelines indicative of labor law “control,” as the first Insular Life case tells us, should not merely relate to the
mutually desirable result intended by the contractual relationship; they must have the nature of dictating the means or
methods to be employed in attaining the result, or of fixing the methodology and of binding or restricting the party
hired to the use of these means.  In fact, results-wise, the principal can impose production quotas and can determine
how many agents, with specific territories, ought to be employed to achieve the company’s objectives. These are
management policy decisions that the labor law element of control cannot reach.
6. Sameer Overseas Placement Agency vs. Cabiles, GR No. 170139, August 3, 2014, En Banc

Facts:

Petitioner, Sameer Overseas Placement Agency, Inc., is a recruitment and placement agency. Respondent submitted
her application for a quality control job in Taiwan. Joy’s application was accepted.7 Joy was later asked to sign a one
year employment contract for a monthly salary. She alleged that Sameer Overseas Agency required her to pay a
placement fee of 70,000.00 when she signed the employment contract. Joy was deployed to work for Taiwan Wacoal,
Co. Ltd. (Wacoal) on June 26, 1997.10 She alleged that in her employment contract, she agreed to work as quality
control for one year.11 In Taiwan, she was asked to work as a cutter.

Sameer Overseas Placement Agency claims that on July 14, 1997, a certain Mr. Huwang from Wacoal informed Joy,
without prior notice, that she was terminated and that “she should immediately report to their office to get her salary
and passport. Thereafter, Joy filed a complaint with the National Labor Relations Commission against petitioner and
Wacoal. She claimed that she was illegally dismissed.

Petitioner reiterates that there was just cause for termination because there was a finding of Wacoal that respondent
was inefficient in her work. Therefore, it claims that respondent’s dismissal was valid.

Issue:

Whether respondent was illegally dismissed by her employer.

Ruling:

Protection to labor both local and overseas

Sameer Overseas Placement Agency failed to show that there was just cause for causing Joy’s dismissal. The
employer, Wacoal, also failed to accord her due process of law. Indeed, employers have the prerogative to impose
productivity and quality standards at work. They may also impose reasonable rules to ensure that the employees
comply with these standards. Failure to comply may be a just cause for their dismissal. This prerogative, however,
should not be abused. It is “tempered with the employee’s right to security of tenure. Employees are not stripped of
their security of tenure when they move to work in a different jurisdiction. With respect to the rights of overseas
Filipino workers, we follow the principle of lex loci contractus.

Thus, in Triple Eight Integrated Services, Inc. v. NLRC,65 this court noted:
Here in the Philippines, employment agreements are more than contractual in nature. The Constitution itself, in Article
XIII, Section 3, guarantees the special protection of workers, to wit:

The State shall afford full protection to labor, local and overseas, organized and unorganized, and promote full
employment and equality of employment opportunities for all. It shall guarantee the rights of all workers to self-
organization, collective bargaining and negotiations, and peaceful concerted activities, including the right to strike in
accordance with law. They shall be entitled to security of tenure, humane conditions of work, and a living wage. They
shall also participate in policy and decision-making processes affecting their rights and benefits as may be provided
by law.

However, petitioners contend that the twin requirements of notice and hearing applies strictly only when the
employment is within the Philippines and that these need not be strictly observed in cases of international maritime or
overseas employment. The Court does not agree. The provisions of the Constitution as well as the Labor Code which
afford protection to labor apply to Filipino employees whether working within the Philippines or abroad. Moreover, the
principle of lex loci contractus (the law of the place where the contract is made) governs in this jurisdiction.

By our laws, overseas Filipino workers (OFWs) may only be terminated for a just or authorized cause and after
compliance with procedural due process requirements.

Illegal dismissal (inefficiency in work)

Petitioner’s allegation that respondent was inefficient in her work and negligent in her duties69 may, therefore,
constitute a just cause for termination under Article 282(b)[gross and habitual negligence], but only if petitioner was
able to prove it.

The burden of proving that there is just cause for termination is on the employer. “The employer must affirmatively
show rationally adequate evidence that the dismissal was for a justifiable cause.”

To show that dismissal resulting from inefficiency in work is valid, it must be shown that: 1) the employer has set
standards of conduct and workmanship against which the employee will be judged; 2) the standards of conduct and
workmanship must have been communicated to the employee; and 3) the communication was made at a reasonable
time prior to the employee’s performance assessment.

The employee cannot be expected to meet any standard of character or workmanship if such standards were not
communicated to him or her. Courts should remain vigilant on allegations of the employer’s failure to communicate
work standards that would govern one’s employment “if [these are] to discharge in good faith [their] duty to
adjudicate.” In this case, petitioner merely alleged that respondent failed to comply with her foreign employer’s work
requirements and was inefficient in her work.74 No evidence was shown to support such allegations.

Petitioner did not even bother to specify what requirements were not met, what efficiency standards were violated, or
what particular acts of respondent constituted. There was also no showing that respondent was sufficiently informed
of the standards against which her work efficiency and performance were judged. Thus, respondent was illegally
dismissed.
3.RIGHT TO HIRE

4.WAGES & WAGE RATIONALIZATION ACT

7. Royal Plant Workers Union vs. Coca-Cola Bottlers Phils Inc. -Cebu Plant, G.R. No. 198783, April 15, 2013

Facts:

Petitioner Coca-Cola Bottlers Philippines, Inc. (CCBPI) is a domestic corporation engaged in the manufacture, sale
and distribution of softdrink products. Under the employ of each bottling plant are bottling operators. In the case of the
plant in Cebu City, there are 20 bottling operators who work for its Bottling Line 1 while there are 12-14 bottling
operators who man its Bottling Line 2. All of them are male and they are members of herein respondent Royal Plant
Workers Union (ROPWU). The bottling operators work in two shifts. The first shift is from 8 a.m. to 5 p.m. and the
second shift is from 5 p.m. up to the time production operations is finished. Thus, the second shift varies and may end
beyond eight (8) hours. However, the bottling operators are compensated with overtime pay if the shift extends
beyond eight (8) hours.

Each shift has rotations of work time and break time. Prior to September 2008, the rotation is this: after two and a half
(2 ½) hours of work, the bottling operators are given a 30-minute break and this goes on until the shift ends. In
September 2008 and up to the present, the rotation has changed and bottling operators are now given a 30-minute
break after one and one half (1 ½) hours of work.

In 1974, the bottling operators of then Bottling Line 2 were provided with chairs upon their request. Their request was
likewise granted. Sometime in September 2008, the chairs provided for the operators were removed pursuant to a
national directive of petitioner. This directive is in line with the "I Operate, I Maintain, I Clean" program of petitioner for
bottling operators.The program reinforces the task of bottling operators to constantly move about in the performance
of their duties and responsibilities.

Furthermore, CCBPI rationalized that the removal of the chairs is implemented so that the bottling operators will avoid
sleeping, thus, prevent injuries to their persons. In addition, sleeping will hamper the efficient flow of operations as the
bottling operators would be unable to perform their duties competently. . Petitioner argued that the removal of the
chairs is valid as it is a legitimate exercise of management prerogative.

The Union further claims that management prerogatives are not absolute but subject to certain limitations found in
law, a collective bargaining agreement, or general principles of fair play and justice. The operators have been
performing their assigned duties and responsibilities satisfactorily for thirty (30) years using chairs. Thus, a violation
of the principle of non-diminution of benefits provided in Article 100 of the Labor Code.

Issue:

Whether there’s a valid exercise of management prerogative. Whether the principle of non-diminution of benefits is
applicable in this case.

Ruling:

Management Prerogative

The Court has held that management is free to regulate, according to its own discretion and judgment, all aspects of
employment, including hiring, work assignments, working methods, time, place, and manner of work, processes to be
followed, supervision of workers, working regulations, transfer of employees, work supervision, lay-off of workers, and
discipline, dismissal and recall of workers. The exercise of management prerogative, however, is not absolute as it
must be exercised in good faith and with due regard to the rights of labor.

In the present controversy, it cannot be denied that CCBPI removed the operators’ chairs pursuant to a national
directive and in line with its "I Operate, I Maintain, I Clean" program, launched to enable the Union to perform their
duties and responsibilities more efficiently. The chairs were not removed indiscriminately. They were carefully studied
with due regard to the welfare of the members of the Union. The removal of the chairs was compensated by: a) a
reduction of the operating hours of the bottling operators from a two-and-one-half (2 ½)-hour rotation period to a one-
and-a-half (1 ½) hour rotation period; and b) an increase of the break period from 15 to 30 minutes between rotations.

Apparently, the decision to remove the chairs was done with good intentions as CCBPI wanted to avoid instances of
operators sleeping on the job while in the performance of their duties and responsibilities and because of the fact that
the chairs were not necessary considering that the operators constantly move about while working. In short, the
removal of the chairs was designed to increase work efficiency. Hence, CCBPI’s exercise of its management
prerogative was made in good faith without doing any harm to the workers’ rights.
Principle of non-diminution of benefits

The operators’ chairs cannot be considered as one of the employee benefits covered in Article 10016 of the Labor
Code. In the Court’s view, the term "benefits" mentioned in the non-diminution rule refers to monetary benefits or
privileges given to the employee with monetary equivalents.

Such benefits or privileges form part of the employees’ wage, salary or compensation making them enforceable
obligations.Examples(13th month pay, legal/special holiday pay, night premium pay and vacation and sick leaves;
service awards with cash incentives, premium pay, Christmas party with incidental benefits and promotional increase)
8. Vergara, Jr. vs. Coca-Cola Bottlers Phils Inc. G.R. No. 176985, April 1, 2013

Facts:

Vergara, Jr. was an employee of respondent Coca-Cola Bottlers Philippines, Inc. from May 1968 until he retired on
January 31, 2002 as a District Sales Supervisor (DSS) for Las Piñas City, Metro Manila. As stipulated in respondent’s
existing Retirement Plan Rules and Regulations at the time, the Annual Performance Incentive Pay of RSMs, DSSs,
and SSSs shall be considered in the computation of retirement benefits.

Petitioner filed a complaint before the NLRC on June 11, 2002 for the payment of his "Full Retirement Benefits,
claiming that his entitlement to Service Management Incentive (SMI) was not considered in the computation of his
retirement package. Petitioner alleged that he is entitled to the SMI because the provision of such benefit has ripened
into a company practice. Thus, the unilateral elimination of the SMI violates the principle of non-diminution of benefits.

Issue:
Whether the application of the principle of non-diminution of benefits is warranted by the facts of the case.

Ruling:

No. Generally, employees have a vested right over existing benefits voluntarily granted to them by their
employer.Thus, any benefit and supplement being enjoyed by the employees cannot be reduced, diminished,
discontinued or eliminated by the employer. The principle of non-diminution of benefits is actually founded on the
Constitutional mandate to protect the rights of workers, to promote their welfare, and to afford them full protection. In
turn, said mandate is the basis of Article 4 of the Labor Code which states that "all doubts in the implementation and
interpretation of this Code, including its implementing rules and regulations, shall be rendered in favor of labor."

There is diminution of benefits when the following requisites are present: (1) the grant or benefit is founded on a
policy or has ripened into a practice over a long period of time; (2) the practice is consistent and deliberate; (3) the
practice is not due to error in the construction or application of a doubtful or difficult question of law; and (4) the
diminution or discontinuance is done unilaterally by the employer.

To be considered as a regular company practice, the employee must prove by substantial evidence that the giving of
the benefit is done over a long period of time, and that it has been made consistently and deliberately. The common
denominator in previously decided cases appears to be the regularity and deliberateness of the grant of benefits over
a significant period of time. It requires an indubitable showing that the employer agreed to continue giving the benefit
knowing fully well that the employees are not covered by any provision of the law or agreement requiring payment
thereof. In sum, the benefit must be characterized by regularity, voluntary and deliberate intent of the employer to
grant the benefit over a considerable period of time.

Upon review of the entire case records, We find no substantial evidence to prove that the grant of SMI to all retired
DSSs regardless of whether or not they qualify to the same had ripened into company practice. Despite more than
sufficient opportunity given him while his case was pending before the NLRC, the CA, and even to this Court,
petitioner utterly failed to adduce proof to establish his allegation that SMI has been consistently, deliberately and
voluntarily granted to all retired DSSs without any qualification or conditions whatsoever. The only two pieces of
evidence that he stubbornly presented throughout the entirety of this case are the sworn statements of Renato C.
Hidalgo (Hidalgo) and Ramon V. Velazquez (Velasquez), former DSSs of respondent who retired in 2000 and 1998,
respectively. They claimed that the SMI was included in their retirement package even if they did not meet the sales
and collection qualifiers. The declarations of Hidalgo and Velazquez were sufficiently countered by respondent
through the affidavits executed by Norman R. Biola (Biola), Moises D. Escasura (Escasura), and Ma. Vanessa R.
Balles (Balles).They attested that contrary to petitioner’s claim, Hidalgo was in fact qualified for the SMI. As for
Velazquez, Escasura asserted that even if he (Velazquez) did not qualify for the SMI, respondent’s General Manager
in its Calamba plant still granted his (Velazquez) request, along with other numerous concessions, to achieve
industrial peace in the plant which was then experiencing labor relations problems. Lastly, Balles confirmed that
petitioner failed to meet the trade receivable qualifiers of the SMI. She also cited the cases of Ed Valencia (Valencia)
and Emmanuel Gutierrez (Gutierrez), both DSSs of respondent who retired on January 31, 2002 and December 30,
2002, respectively. She noted that, unlike Valencia, Gutierrez also did not receive the SMI as part of his retirement
pay, since he failed to qualify under the policy guidelines.

Therefore, respondent's isolated act of including the SMI in the retirement package of Velazquez could hardly be
classified as a company practice that may be considered an enforceable obligation.It presupposes that a company
practice, policy and tradition favorable to the employees has been clearly established; and that the payments made
by the company pursuant to it have ripened into benefits enjoyed by them.
9. Royal Plant Workers Union vs. Coca-Cola Bottlers Phils Inc. -Cebu Plant, G.R. No. 198783, April 15, 2013

Facts:

Petitioner Coca-Cola Bottlers Philippines, Inc. (CCBPI) is a domestic corporation engaged in the manufacture, sale
and distribution of softdrink products. Under the employ of each bottling plant are bottling operators. In the case of the
plant in Cebu City, there are 20 bottling operators who work for its Bottling Line 1 while there are 12-14 bottling
operators who man its Bottling Line 2. All of them are male and they are members of herein respondent Royal Plant
Workers Union (ROPWU). The bottling operators work in two shifts. The first shift is from 8 a.m. to 5 p.m. and the
second shift is from 5 p.m. up to the time production operations is finished. Thus, the second shift varies and may end
beyond eight (8) hours. However, the bottling operators are compensated with overtime pay if the shift extends
beyond eight (8) hours.

Each shift has rotations of work time and break time. Prior to September 2008, the rotation is this: after two and a half
(2 ½) hours of work, the bottling operators are given a 30-minute break and this goes on until the shift ends. In
September 2008 and up to the present, the rotation has changed and bottling operators are now given a 30-minute
break after one and one half (1 ½) hours of work.

In 1974, the bottling operators of then Bottling Line 2 were provided with chairs upon their request. Their request was
likewise granted. Sometime in September 2008, the chairs provided for the operators were removed pursuant to a
national directive of petitioner. This directive is in line with the "I Operate, I Maintain, I Clean" program of petitioner for
bottling operators.The program reinforces the task of bottling operators to constantly move about in the performance
of their duties and responsibilities.

Furthermore, CCBPI rationalized that the removal of the chairs is implemented so that the bottling operators will avoid
sleeping, thus, prevent injuries to their persons. In addition, sleeping will hamper the efficient flow of operations as the
bottling operators would be unable to perform their duties competently. . Petitioner argued that the removal of the
chairs is valid as it is a legitimate exercise of management prerogative.

The Union further claims that management prerogatives are not absolute but subject to certain limitations found in
law, a collective bargaining agreement, or general principles of fair play and justice. The operators have been
performing their assigned duties and responsibilities satisfactorily for thirty (30) years using chairs. Thus, a violation
of the principle of non-diminution of benefits provided in Article 100 of the Labor Code.

Issues:

Whether there’s a valid exercise of management prerogative.


Whether the principle of non-diminution of benefits is applicable in this case.

Ruling:

Management Prerogative
The Court has held that management is free to regulate, according to its own discretion and judgment, all aspects of
employment, including hiring, work assignments, working methods, time, place, and manner of work, processes to be
followed, supervision of workers, working regulations, transfer of employees, work supervision, lay-off of workers, and
discipline, dismissal and recall of workers. The exercise of management prerogative, however, is not absolute as it
must be exercised in good faith and with due regard to the rights of labor.

In the present controversy, it cannot be denied that CCBPI removed the operators’ chairs pursuant to a national
directive and in line with its "I Operate, I Maintain, I Clean" program, launched to enable the Union to perform their
duties and responsibilities more efficiently. The chairs were not removed indiscriminately. They were carefully studied
with due regard to the welfare of the members of the Union. The removal of the chairs was compensated by: a) a
reduction of the operating hours of the bottling operators from a two-and-one-half (2 ½)-hour rotation period to a one-
and-a-half (1 ½) hour rotation period; and b) an increase of the break period from 15 to 30 minutes between rotations.

Apparently, the decision to remove the chairs was done with good intentions as CCBPI wanted to avoid instances of
operators sleeping on the job while in the performance of their duties and responsibilities and because of the fact that
the chairs were not necessary considering that the operators constantly move about while working. In short, the
removal of the chairs was designed to increase work efficiency. Hence, CCBPI’s exercise of its management
prerogative was made in good faith without doing any harm to the workers’ rights.

Principle of non-diminution of benefits

The operators’ chairs cannot be considered as one of the employee benefits covered in Article 10016 of the Labor
Code. In the Court’s view, the term "benefits" mentioned in the non-diminution rule refers to monetary benefits or
privileges given to the employee with monetary equivalents.
Such benefits or privileges form part of the employees’ wage, salary or compensation making them enforceable
obligations.Examples(13th month pay, legal/special holiday pay, night premium pay and vacation and sick leaves;
service awards with cash incentives, premium pay, Christmas party with incidental benefits and promotional increase)
10. The National Wages & Productivity Commission et al., vs. The Alliance of Progressive Labor et al., GR No.
150326, March 12, 2014

Facts: The RTWPB-NCR issued Wage Order No. NCR-07 on October 14, 1999 imposing an increase of P25.50/day
on the wages of all private sector workers and employees in the NCR and pegging the minimum wage rate in the
NCR at P223.50/day.6 However, Section 2 and Section 9 of Wage Order No. NCR-07 exempted certain sectors and
industries from its coverage, to wit:

Section 9. Upon application with and as determined by the Board, based on documentation and other requirements in
accordance with applicable rules and regulations issued by the Commission, the following may be exempt from the
applicability of this Order:

1. Distressed establishments as defined in the NPWC Guidelines No. 01, series of 1996;
2. Exporters including indirect exporters with at least 50% export sales and with forward contracts with their
foreign buyers/principals entered into on or twelve (12) months before the date of publication of this
Order may be exempt during the lifetime of said contract but not to exceed twelve (12) months from the
effectivity of this Order.

Feeling aggrieved by their non-coverage by the wage adjustment, the Alliance of Progressive Labor (APL) and the
Tunay na Nagkakaisang Manggagawa sa Royal (TNMR) filed an appeal with the NWPC assailing Section 2(A) and
Section 9(2) of Wage Order No. NCR-07. They contended

that neither the NWPC nor the RTWPB-NCR had the authority to expand the non-coverage and exemptible
categories under the wage order. Exporters are not among those listed in the law. Hence, the assailed sections of the
wage order should be voided. NWPC upheld the validity of the Wage Order. However, the C.A revered the decision
of the NWPC.

Issue: Whether or not the RTWPB-NCR had the authority to provide additional exemptions from the minimum wage
adjustments embodied in Wage Order No. NCR-07.

Ruling: Yes. Indisputably, the NWPC had the authority to prescribe the rules and guidelines for the determination of
the minimum wage and productivity measures, and the RTWPB-NCR had the power to issue wage orders.

Pursuant to its statutorily defined functions, the NWPC promulgated NWPC Guidelines No. 001-95 (Revised Rules of
Procedure on Minimum Wage Fixing) to govern the proceedings in the NWPC and the RTWPBs in the fixing of
minimum wage rates by region, province and industry. Section 1 of Rule VIII of NWPC Guidelines No. 001-95
recognized the power of the RTWPBs to issue exemptions from the application of the wage orders subject to the
guidelines issued by the NWPC, viz: (refer to regulation)

The NWPC also issued NWPC Guidelines No. 01, Series of 1996, to fix the rules on the exemption from compliance
with the wage increases prescribed by the RTWPBs. Section 2 of the Guidelines No. 01 reads:

SECTION 2. CATEGORIES OF EXEMPTIBLE ESTABLISHMENTS:

1. Distressed establishments
2. New business enterprises (NBEs)
3. Retail/Service establishments employing not more than ten (10)
4. workers
5. Establishments adversely affected by natural calamities

Exemptible categories outside of the abovementioned list may be allowed only if they are in accord with the rationale
for exemption\reflected in the first paragraph of this section. The concerned Regional Board shall submit strong and
justifiable reason/s for the inclusion of such categories which shall be subject to review/approval by the Commission.

Under the guidelines, the RTWPBs could issue exemptions from the application of the wage orders as long as the
exemptions complied with the rules of the NWPC. In its rules, the NWPC enumerated four exemptible
establishments, but the list was not exclusive. The RTWPBs had the authority to include in the wage orders
establishments that belonged to, or to exclude from the four enumerated exemptible categories. If the exemption was
outside of the four exemptible categories, like here, the exemptible category should be: (1) in accord with the
rationale for exemption; (2) reviewed/approved by the NWPC; and (3) upon review, the RTWPB issuing the wage
order must submit a strong and justifiable reason or reasons for the inclusion of such category. It is the compliance
with the second requisite that is at issue here. The very fact that the validity of the assailed sections of Wage Order
No. NCR-07 had been already passed upon and upheld by the NWPC meant that the NWPC had already given the
wage order its necessary legal imprimatur. Accordingly, the requisite approval or review was complied with.

In creating the RTWPBs, Congress intended to rationalize wages, firstly, by establishing full time boards to police
wages round-the-clock, and secondly, by giving the boards enough powers to achieve this objective. In Employers
Confederation of the Phils. v. National Wages and Productivity Commission, 16 this Court all too clearly pronounced
that Congress meant the RTWPBs to be creative in resolving the annual question of wages without Labor and
Management knocking on the doors of Congress at every turn. The R TWPBs are the thinking group of men and
women guided by statutory standards and bound by the rules and guidelines prescribed by the NWPC.
In the nature of their functions, the R TWPBs investigate and study all the pertinent facts to ascertain the conditions in
their respective regions. Hence, they are logically vested with the competence to determine the applicable minimum
wages to be imposed as well as the industries and sectors to exempt from the coverage of their wage orders.
11. Our Haus Realty Development Corp., vs. Parian et al., GR No. 204651, August 6, 2014

Principles:

i. If the provision of articles/services to workers is mandated by law and required by law to be integrated into
the overall business cost, the employer cannot pass the burden of the costs to its employees by deducting it
as facilities, because this is the employer’s obligation under the law.
ii. The lodging and meals furnished by the employer to its workers in the construction business are
supplements, not facilities, because they redound to the benefit and convenience of the employer. The
success of its projects is largely a function of the physical strength, vitality and efficiency of its laborers. Its
business will be jeopardized if its workers are weak, sickly, and lack the required energy to perform
strenuous physical activities. Moreover, it will be more convenient to the employer if its workers are housed
near the construction site to ensure their ready availability during urgent or emergency circumstances.

Facts:

Respondents Alexander Parian, Jay Erinco, Alexander Canlas, Jerry Sabulao and Bernardo Tenedero were all
laborers working for petitioner Our Haus Realty Development Corporation (Our Haus), a company engaged in the
construction business.

They filed with the LA a complaint for underpayment of their daily wages. They claimed that except for respondent
Bernardo N. Tenedero, their wages were below the minimum rates prescribed in the following wage orders from 2007
to 2010:

Wage Order No. NCR-13, which provides for a daily minimum wage rate ofP362.00 for the non-agriculture sector
(effective from August 28, 2007 until June 13, 2008);

Wage Order No. NCR-14, which provides for a daily minimum wage rate ofP382.00 for the non-agriculture sector
(effective from June 14, 2008 until June 30, 2010).

The respondents also alleged that Our Haus failed to pay them their holiday, service incentive leave (SIL),
13th month and overtime pays.

Before the LA, Our Haus primarily argued that the respondents’ wages complied with the law’s minimum requirement.
Aside from paying the monetary amount of the respondents’ wages, Our Haus also subsidized their meals (3 times a
day), and gave them free lodging near the construction project they were assigned to.10 In determining the total
amount of the respondents’ daily wages, the value of these benefits should be considered, in line with Article
97(f)11 of the Labor Code.

The respondents pointed out that Our Haus never presented any proof that they agreed in writing to the inclusion of
their meals’ value in their wages.16 Also, Our Haus failed to prove that the value of the facilities it furnished was fair
and reasonable.17 Finally, instead of deducting the maximum amount of 70% of the value of the meals, Our Haus
actually withheld its full value (which was Php290.00 per week for each employee).

In its petition, Our Haus propounded a new theory. It made a distinction between deduction and charging. A written
authorization is only necessary if the facility’s value will be deducted and will not be  needed  if it will merely be
charged or included in the computation of wages.25 Our Haus claimed that it did not actually deduct the values of the
meals and housing benefits. It only considered these in computing the total amount of wages paid to the respondents
for purposes of compliance with the minimum wage law. Hence, the written authorization requirement should not
apply.

Issues:

1. WON there is substantial distinction between deducting and charging a facility’s value to the wages,
justifying non-compliance with the written authorization requirement in the case of merely “charging” the
value of the facility
2. WON the lodging and subsidized meals furnished by Our Haus to respondents are considered facilities and
thus may be deducted from the latter’s wages.

Ruling:

1. No. In reality, deduction and charging both operate to lessen the actual take-home pay of an employee; they are
two sides of the same coin. In both, the employee receives a lessened amount because supposedly, the facility’s
value, which is part of his wage, had already been paid to him in kind. As there is no substantial distinction between
the two, the requirements set by law must apply to both.

2. No. The requirements in order to deduct facilities from the workers’ wages are the following:

a. Proof must be shown that such facilities are customarily furnished by the trade.

In a string of cases, we have concluded that one of the badges to show that a facility is customarily furnished by the
trade is the existence of a company policy or guideline showing that provisions for a facility were designated as part
of the employees’ salaries. Apart from company policy, the employer may also prove compliance with the first
requirement by showing the existence of an industry-wide practice of furnishing the benefits in question among
enterprises engaged in the same line of business. If it were customary among construction companies to provide
board and lodging to their workers and treat their values as part of their wages, we would have more reason to
conclude that these benefits were really facilities. However, Our Haus could not really be expected to prove
compliance with the first requirement since the living accommodation of workers in the construction industry is not
simply a matter of business practice. Peculiar to the construction business are the occupational safety and health
(OSH) services which the law itself mandates employers to provide to their workers. This is to ensure the humane
working conditions of construction employees despite their constant exposure to hazardous working environments.
Under Section 16 of DOLE Department Order (DO) No. 13, series of 1998,43 employers engaged in the construction
business are required to provide suitable living accommodation for workers, and as may be applicable, for their
families.

Moreover, DOLE DO No. 56, series of 2005, which sets out the guidelines for the implementation of DOLE DO No.
13, mandates that the cost of the implementation of the requirements for the construction safety and health of
workers, shall be integrated to the overall project cost.44 The rationale behind this is to ensure that the living
accommodation of the workers is not substandard and is strictly compliant with the DOLE’s OSH criteria.

As part of the project cost that construction companies already charge to their clients, the value of the housing of their
workers cannot be charged again to their employees’ salaries. Our Haus cannot pass the burden of the OSH costs of
its construction projects to its employees by deducting it as facilities. This is Our Haus’ obligation under the law.

Lastly, even if a benefit is customarily provided by the trade, it must still pass the purpose test set by jurisprudence.
Under this test, if a benefit or privilege granted to the employee is clearly for the employer’s convenience, it will not be
considered as a facility but a supplement. Here, careful consideration is given to the nature of the employer’s
business in relation to the work performed by the employee.  This test is used to address inequitable situations
wherein employers consider a benefit deductible from the wages even if the factual circumstances show that it clearly
redounds to the employers’ greater advantage.

Our Haus is engaged in the construction business, a labor-intensive enterprise. The success of its projects is largely
a function of the physical strength, vitality and efficiency of its laborers. Its business will be jeopardized if its workers
are weak, sickly, and lack the required energy to perform strenuous physical activities. Thus, by ensuring that the
workers are adequately and well fed, the employer is actually investing on its business. Moreover, in the construction
business, contractors are usually faced with the problem of meeting target deadlines.  More often than not, work is
performed continuously, day and night, in order to finish the project on the designated turn-over date. Thus, it will be
more convenient to the employer if its workers are housed near the construction site to ensure their ready availability
during urgent or emergency circumstances. Based on these considerations, we conclude that even under the
purpose test, the subsidized meals and free lodging provided by Our Haus are actually supplements.

b. The provision of deductible facilities must be voluntarily accepted in writing by the employee;

Again, in the motion for reconsideration with the NLRC, Our Haus belatedly submitted five kasunduans, supposedly
executed by the respondents, containing their conformity to the inclusion of the values of the meals and housing to
their total wages. Oddly, Our Haus only offered these documents when the NLRC had already ruled that respondents
did not accomplish any written authorization, to allow deduction from their wages. These five kasunduans were also
undated, making us wonder if they had really been executed when respondents first assumed their jobs.

c. The facilities must be charged at fair and reasonable value.

Our Haus’ valuation cannot be plucked out of thin air. The valuation of a facility must be supported by relevant
documents such as receipts and company records for it to be considered as fair and reasonable.

In the present case, Our Haus never explained how it came up with the values it assigned for the benefits it provided;
it merely listed its supposed expenses without any supporting document. 
5.VIOLATION OF WAGE ORDERS

6.WAGE ENFORCEMENT AND RECOVERY

12. People’s Broadcasting (Bombo Radyo Phils) vs. Sec. of DOLE et al., GR No. 179652, March 6, 2012
Resolution on the main Decision of May 8, 2009

Principle: It is the DOLE who determines WON an ER-EE relationship exists pursuant to the clause “in cases where
the relationship of employer-employee still exists” in Art. 128(b). If a complaint is brought before the DOLE to give
effect to the labor standards provisions of the Labor Code or other labor legislation, and there is a finding by the
DOLE that there is an existing employer-employee relationship, the DOLE exercises jurisdiction to the exclusion of
the NLRC.   If the DOLE finds that there is no employer-employee relationship, the jurisdiction is properly with the
NLRC.

Facts:

Private respondent Jandeleon Juezan filed a complaint against petitioner with the Department of Labor and
Employment (DOLE) Regional Office No. VII, Cebu City, for illegal deduction, nonpayment of service incentive leave,
13th month pay, premium pay for holiday and rest day and illegal diminution of benefits, delayed payment of wages
and noncoverage of SSS, PAG-IBIG and Philhealth.[1]  After the conduct of summary investigations, and after the
parties submitted their position papers, the DOLE Regional Director found that private respondent was an employee
of petitioner, and was entitled to his money claims.

The petitioner brought the matter before the CA. In the Decision of this Court, the CA Decision was reversed and set
aside, and the complaint against petitioner was dismissed. The Court found that there was no employer-employee
relationship between petitioner and private respondent.  It was held that while the DOLE may make a determination
of the existence of an employer-employee relationship, this function could not be co-extensive with the visitorial and
enforcement power provided in Art. 128(b) of the Labor Code, as amended by RA 7730.  The National Labor
Relations Commission (NLRC) was held to be the primary agency in determining the existence of an employer-
employee relationship.  This was the interpretation of the Court of the clause “in cases where the relationship of
employer-employee still exists” in Art. 128(b).

Issue:

WON the DOLE make a determination of whether or not an employer-employee relationship exists, and if so, to what
extent

Ruling:

Yes, and when the DOLE determines the existence of an employer-employee relationship, it must be respected.

No limitation in the law was placed upon the power of the DOLE to determine the existence of an employer-employee
relationship.  No procedure was laid down where the DOLE would only make a preliminary finding, that the power
was primarily held by the NLRC.  The law did not say that the DOLE would first seek the NLRC’s determination of the
existence of an employer-employee relationship, or that should the existence of the employer-employee relationship
be disputed, the DOLE would refer the matter to the NLRC.  The DOLE must have the power to determine whether or
not an employer-employee relationship exists, and from there to decide whether or not to issue compliance orders in
accordance with Art. 128(b) of the Labor Code, as amended by RA 7730.

 The DOLE, in determining the existence of an employer-employee relationship, has a ready set of guidelines to
follow, the same guide the courts themselves use.  The elements to determine the existence of an employment
relationship are: (1) the selection and engagement of the employee; (2) the payment of wages; (3) the power of
dismissal; (4) the employer’s power to control the employee’s conduct.[9]  The use of this test is not solely limited to
the NLRC. The DOLE Secretary, or his or her representatives, can utilize the same test, even in the course of
inspection, making use of the same evidence that would have been presented before the NLRC.

 The determination of the existence of an employer-employee relationship by the DOLE must be respected.  The
expanded visitorial and enforcement power of the DOLE granted by RA 7730 would be rendered nugatory if the
alleged employer could, by the simple expedient of disputing the employer-employee relationship, force the referral of
the matter to the NLRC.  The Court issued the declaration that at least a prima facie showing of the absence of an
employer-employee relationship be made to oust the DOLE of jurisdiction.  But it is precisely the DOLE that will be
faced with that evidence, and it is the DOLE that will weigh it, to see if the same does successfully refute the
existence of an employer-employee relationship.

 If the DOLE makes a finding that there is an existing employer-employee relationship, it takes cognizance of the
matter, to the exclusion of the NLRC.  The DOLE would have no jurisdiction only if the employer-employee
relationship has already been terminated, or it appears, upon review, that no employer-employee relationship existed
in the first place.
This is not to say that the determination by the DOLE is beyond question or review.  Suffice it to say, there are judicial
remedies such as a petition for certiorari under Rule 65 that may be availed of, should a party wish to dispute the
findings of the DOLE.
13. Superior Packaging Corp., vs. Balagsay et al., G.R. No. 178909, October 10, 2012

Principle: It can be assumed that the DOLE in the exercise of its visitorial and enforcement power somehow has to
make a determination of the existence of an employer-employee relationship. Such determination, however, is
merely preliminary, incidental and collateral to the DOLE’s primary function of enforcing labor standards provisions.

Facts:

The petitioner Superior Packaging engaged the services of Lancer to provide reliever services to its business, which
involves the manufacture and sale of commercial and industrial corrugated boxes.

Pursuant to a complaint filed by the respondents against the petitioner and its President, Cesar Luz (Luz), for
underpayment of wages, nonpayment of premium pay for worked rest, overtime pay and non-payment of salary, the
Department of Labor and Employment (DOLE) conducted an inspection of the petitioner’s premises and found
several violations, to wit: (1) non-presentation of payrolls and daily time records; (2) non-submission of annual report
of safety organization; (3) medical and accident/illness reports; (4) non-registration of establishment under Rule 1020
of Occupational and Health Standards; and (5) no trained first aide.1 Due to the petitioner’s failure to appear in the
summary investigations conducted by the DOLE, an Order2 was issued on June 18, 2003 finding in favor of the
respondents and adopting the computation of the claims submitted.

Petitioner and Luz were ordered, among others, to pay respondents their total claims in the amount of Eight Hundred
Forty Thousand Four Hundred Sixty-Three Pesos and 38/100 (P840,463.38).

They filed a motion for reconsideration on the ground that respondents are not its employees but of Lancer and that
they pay Lancer in lump sum for the services rendered.

The LA, NLRC, and CA ruled in favor of the respondents.

The petitioner objects to the finding that it is engaged in labor-only contracting and is consequently an indirect
employer, considering that it is beyond the visitorial and enforcement power of the DOLE to make such conclusion.
According to the petitioner, such conclusion may be made only upon consideration of evidentiary matters and cannot
be determined solely through a labor inspection.

Issue:

WON the DOLE has the authority to make a finding of an employer-employee relationship concomitant to its visitorial
and enforcement power.

Ruling: Yes.

The DOLE clearly acted within its authority when it determined the existence of an employer-employee relationship
between the petitioner and respondents as it falls within the purview of its visitorial and enforcement power under
Article 128(b) of the Labor Code.

In People’s Broadcasting (Bombo Radyo Phils., Inc.) v. Secretary of the Department of Labor and Employment, the
Court stated that it can be assumed that the DOLE in the exercise of its visitorial and enforcement power somehow
has to make a determination of the existence of an employer-employee relationship. Such determination, however, is
merely preliminary, incidental and collateral to the DOLE’s primary function of enforcing labor standards provisions.

Also, the existence of an employer-employee relationship is ultimately a question of fact. The determination made in
this case by the DOLE, albeit provisional, and as affirmed by the Secretary of DOLE and the CA is beyond the ambit
of a petition for review on certiorari.
7.WAGE PROTECTION PROVISIONS & PROHIBITIONS REGARDING WAGES

14. Netlink Computer Inc. vs. Delmo, GR No. 160827, June 18, 2014

Principle: In the absence of a written agreement between the employer and the employee that sales commissions
shall be paid in a foreign currency, the latter has the right to be paid in such foreign currency once the same has
become an established practice of the former. The rate of exchange at the time of payment, not the rate of exchange
at the time of the sales, controls.

Facts:

On November 3, 1991, Netlink Computer, Inc. Products and Services (Netlink) hired Eric S. Delmo (Delmo) as
account manager tasked to canvass and source clients and convince them to purchase the products and services of
Netlink. Delmo worked in the field most of the time. He and his fellow account managers were not required to
accomplish time cards to record their personal presence in the office of Netlink.1 He was able to generate sales worth
P35,000,000.00, more or less, from which he earned commissions amounting to P993,558.89 and US$7,588.30. He
then requested payment of his commissions, but Netlink refused and only gave him partial cash advances chargeable
to his commissions. Later on, Netlink began to nitpick and fault find, like stressing his supposed absences and
tardiness. In order to force him to resign, Netlink issued several memoranda detailing his supposed infractions of the
company’s attendance policy. Despite the memoranda, Delmo continued to generate huge sales for Netlink.

On November 28, 1996, Delmo was shocked when he was refused entry into the company premises by the security
guard pursuant to a memorandum to that effect. His personal belongings were still inside the company premises and
he sought their return to him. This incident prompted Delmo to file a complaint for illegal dismissal.

The NLRC and CA ruled that the respondent was terminated for just causes, but ordered petitioner to pay the latter
his unpaid commissions, including his dollar commissions for US dollar-denominated sales.

Issues:

1. WON the payment of the commissions should be in US dollars

2. WON the award of attorney’s fees was warranted.

Ruling:

1. Yes. As a general rule, all obligations shall be paid in Philippine currency. However, the contracting parties may
stipulate that foreign currencies may be used for settling obligations. This is pursuant to Republic Act No. 8183,which
provides as follows:

Section 1. All monetary obligations shall be settled in the Philippine currency which is legal tender in the Philippines.
However, the parties may agree that the obligation or transaction shall be settled in any other currency at the time of
payment.

The rate of exchange at the time of payment, not the rate of exchange at the time of the sales, controls.

There was no written contract between Netlink and Delmo stipulating that the latter’s commissions would be paid in
US dollars. The absence of the contractual stipulation notwithstanding, Netlink was still liable to pay Delmo in US
dollars because the practice of paying its sales agents in US dollars for their US dollar-denominated sales had
become a company policy. This was impliedly admitted by Netlink when it did not refute the allegation that the
commissions earned by Delmo and its other sales agents had been paid in US dollars. Instead of denying the
allegation, Netlink only sought a declaration that the US dollar commissions be paid using the exchange rate at the
time of sale. The principle of non-diminution of benefits, which has been incorporated in Article 10013 of the Labor
Code, forbade Netlink from unilaterally reducing, diminishing, discontinuing or eliminating the practice. Verily, the
phrase “supplements, or other employee benefits” in Article 100 is construed to mean the compensation and
privileges received by an employee aside from regular salaries or wages.

2. Yes. The award of attorney's fees must, likewise, be upheld in line with the decision of the Supreme Court in the
case of Consolidated Rural Bank (Cagayan Valley), Inc. vs. National Labor Relations Commission, 301 SCRA 223,
235, where it was held that "in actions for recovery of wages or where an employee was forced to litigate and thus
incur expenses to protect her rights and interests, even if not so claimed, an award of attorney's fees equivalent to
ten percent (10%) of the total award is legally and morally justifiable. There is no doubt that in the present case, the
private respondent has incurred expenses for the protection and enforcement of his right to his commissions.
15. Locsin II vs. Mekeni Food Corp., GR No. 192105, December 9, 2013

Principle: In the absence of specific terms and conditions governing a car plan agreement between the employer
and employee, the former may not retain the installment payments made by the latter on the car plan and treat them
as rents for the use of the service vehicle in the event that the employee ceases his employment and is unable to
complete the installment payments on the vehicle. The underlying reason is that the service vehicle was precisely
used in the former's business; any personal benefit obtained by the employee from its use is merely incidental.

Facts:

In February 2004, respondent Mekeni Food Corporation (Mekeni)–a Philippine company engaged in food
manufacturing and meat processing – offered petitioner Antonio Locsin II the position of Regional Sales Manager to
oversee Mekeni’s National Capital Region Supermarket/Food Service and South Luzon operations.

In addition to a compensation and benefit package, Mekeni offered petitioner a car plan, under which one-half of the
cost of the vehicle is to be paid by the company and the other half to be deducted from petitioner’s salary.

Mekeni’s offer was contained in an Offer Sheet which was presented to petitioner. Petitioner began his stint as
Mekeni Regional Sales Manager on March 17, 2004. To be able to effectively cover his appointed sales territory,
Mekeni furnished petitioner with a used Honda Civic car valued at P280,000.00, which used to be the service vehicle
of petitioner’s immediate supervisor. Petitioner paid for his 50% share through salary deductions of P5,000.00 each
month. Subsequently, Locsin resigned effective February 25, 2006. By then, a total of P112,500.00 had been
deducted from his monthly salary and applied as part of the employee’s share in the car plan.

Mekeni supposedly put in an equivalent amount as its share under the car plan. In his resignation letter, petitioner
made an offer to purchase his service vehicle by paying the outstanding balance thereon. The parties negotiated, but
could not agree on the terms of the proposed purchase. Petitioner thus returned the vehicle to Mekeni on May 2,
2006.

In May 3, 2007, petitioner filed against Mekeni and/or its President, Prudencio S. Garcia, a Complaint for the recovery
of monetary claims consisting of unpaid salaries, commissions, sick/vacation leave benefits, and recovery of monthly
salary deductions which were earmarked for his cost-sharing in the car plan.

The NLRC held that petitioner’s amortization payments on his service vehicle amounting to P112,500.00 should be
reimbursed; if not, unjust enrichment would result, as the vehicle remained in the possession and ownership of
Mekeni. The CA reversed the decision of the NLRC and held that this award should be deleted.

Issue:

WON petitioner is entitled to a refund of all the amounts applied to the cost of the service vehicle under the car plan.

Ruling:

Partly, yes. The petitioner is only entitled to a refund of his contributions to the car plan, but is not entitled to the
amount contributed by employer Mekeni.

From the evidence on record, it is seen that the Mekeni car plan offered to petitioner was subject to no other term or
condition than that Mekeni shall cover one-half of its value, and petitioner shall in turn pay the other half through
deductions from his monthly salary. Mekeni has not shown, by documentary evidence or otherwise, that there are
other terms and conditions governing its car plan agreement with petitioner. There is no evidence to suggest that if
petitioner failed to completely cover one-half of the cost of the vehicle, then all the deductions from his salary going to
the cost of the vehicle will be treated as rentals for his use thereof while working with Mekeni, and shall not be
refunded. Installments made on the car plan may be treated as rentals only when there is an express stipulation in
the car plan agreement to such effect.

Indeed, the Court cannot allow that payments made on the car plan should be forfeited by Mekeni and treated simply
as rentals for petitioner’s use of the company service vehicle. Nor may they be retained by it as purported loan
payments, as it would have this Court believe. In the first place, there is precisely no stipulation to such effect in their
agreement. Secondly, it may not be said that the car plan arrangement between the parties was a benefit that the
petitioner enjoyed; on the contrary, it was an absolute necessity in Mekeni’s business operations, which benefited it to
the fullest extent: without the service vehicle, petitioner would have been unable to rapidly cover the vast sales
territory assigned to him, and sales or marketing of Mekeni’s products could not have been booked or made fast
enough to move Mekeni’s inventory. Poor sales, inability to market Mekeni’s products, a high rate of product spoilage
resulting from stagnant inventory, and poor monitoring of the sales territory are the necessary consequences of lack
of mobility.

Any benefit or privilege enjoyed by petitioner from using the service vehicle was merely incidental and insignificant,
because for the most part the vehicle was under Mekeni’s control and supervision. Thus, it is clear that while
petitioner was paying for half of the vehicle’s value, Mekeni was reaping the full benefits from the use thereof.
In light of the foregoing, it is unfair to deny petitioner a refund of all his contributions to the car plan. In the absence of
specific terms and conditions governing the car plan arrangement between the petitioner and Mekeni, a quasi-
contractual relation was created between them. Consequently, Mekeni may not enrich itself by charging petitioner for
the use of its vehicle which is otherwise absolutely necessary to the full and effective promotion of its business.
Conversely, petitioner cannot recover the monetary value of Mekeni’s counterpart contribution to the cost of the
vehicle; that is not property or money that belongs to him, nor was it intended to be given to him in lieu of the car
plan. In other words, Mekeni’s share of the vehicle’s cost was not part of petitioner’s compensation package. To start
with, the vehicle is an asset that belonged to Mekeni. Just as Mekeni is unjustly enriched by failing to refund
petitioner’s payments, so should petitioner notbe awarded the value of Mekeni’s counterpart contribution to the car
plan, as this would unjustly enrich him at Mekeni’s expense.
16. TH Shopfitters Corp., et al., vs. T&H Shopfitters Corp., Union, GR No. 191714, Feb 26, 2014

Principle: On the award of attorney’s fees, the applicable law concerning the grant thereof in labor cases is Article
111 of the Labor Code. Pursuant thereto, the award of 10% attorney’s fees is limited to cases of unlawful withholding
of wages. In this case, however, the Court cannot find any claim or proof that petitioners unlawfully withheld the
wages of respondents. Consequently, the grant of 10% attorney’s fees in favor of respondents is not justified under
the circumstances. Accordingly, the Court deems it proper to delete the same.

Facts:

T&H Shopfitters Corporation/Gin Queen Corporation workers union (THS–GQ Union) filed their Complaint for Unfair
Labor Practice (ULP) by way of union busting, and Illegal Lockout, with moral and exemplary damages and attorney’s
fees, against T&H Shopfitters and Gin Queen Corporation before the Labor Arbiter.

The LA dismissed respondents’ complaint and all their money claims for lack of merit.

NLRC reversed the LA decision and ruled in favor of respondents. CA sustained the ruling of the NLRC.

Issues:

Whether ULP acts were committed by petitioners against respondents in the case at bench.

Ruling:

ULP was committed.

In the case at bench, petitioners are being accused of violations of paragraphs (a), (c), and (e) of Article 257 (formerly
Article 248) of the Labor Code, to wit:

Article 257. Unfair labor practices of employers.––It shall be unlawful for an employer to commit any of the following
unfair labor practices:

(a) To interfere with, restrain or coerce employees in the exercise of their right to self–organization;

xxxx

(c) To contract out services or functions being performed by union members when such will interfere with, restrain, or
coerce employees in the exercise of their right to self–organization;

xxxx

(e) To discriminate in regard to wages, hours of work, and other terms and conditions of employment in order to
encourage or discourage membership in any labor organization. x x x

The concept of ULP is embodied in Article 256 (formerly Article 247) of the Labor Code, which provides:

Article 256. Concept of unfair labor practice and procedure for prosecution thereof.––Unfair labor practices violate the
constitutional right of workers and employees to self–organization, are inimical to the legitimate interests of both labor
and management, including their right to bargain collectively and otherwise deal with each other in an atmosphere of
freedom and mutual respect, disrupt industrial peace and hinder the promotion of healthy and stable labor–
management relations.

In essence, ULP relates to the commission of acts that transgress the workers’ right to organize. As specified in
Articles 248 [now Article 257] and 249 [now Article 258] of the Labor Code, the prohibited acts must necessarily relate
to the workers’ right to self–organization x x x.

In the case of Insular Life Assurance Co., Ltd. Employees Association – NATU v. Insular Life Assurance Co.
Ltd.,16 this Court had occasion to lay down the test of whether an employer has interfered with and coerced
employees in the exercise of their right to self–organization, that is, whether the employer has engaged in conduct
which, it may reasonably be said, tends to interfere with the free exercise of employees’ rights; and that it is not
necessary that there be direct evidence that any employee was in fact intimidated or coerced by statements of threats
of the employer if there is a reasonable inference that anti–union conduct of the employer does have an adverse
effect on self–organization and collective bargaining.

The questioned acts of petitioners, namely: 1) sponsoring a field trip to Zambales for its employees, to the exclusion
of union members, before the scheduled certification election; 2) the active campaign by the sales officer of
petitioners against the union prevailing as a bargaining agent during the field trip; 3) escorting its employees after the
field trip to the polling center; 4) the continuous hiring of subcontractors performing respondents’ functions; 5)
assigning union members to the Cabangan site to work as grass cutters; and 6) the enforcement of work on a
rotational basis for union members, all reek of interference on the part of petitioners.
Indubitably, the various acts of petitioners, taken together, reasonably support an inference that, indeed, such were
all orchestrated to restrict respondents’ free exercise of their right to self–organization. The Court is of the considered
view that petitioners’ undisputed actions prior and immediately before the scheduled certification election, while
seemingly innocuous, unduly meddled in the affairs of its employees in selecting their exclusive bargaining
representative.

In fine, mindful of the nature of the charge of ULP, including its civil and/or criminal consequences, the Court finds
that the NLRC, as correctly sustained by the CA, had sufficient factual and legal bases to support its finding of ULP.

Anent the issue on the award of attorney’s fess, the applicable law concerning the grant thereof in labor cases is
Article 11120 of the Labor Code. Pursuant thereto, the award of 10% attorney’s fees is limited to cases of unlawful
withholding of wages. In this case, however, the Court cannot find any claim or proof that petitioners unlawfully
withheld the wages of respondents. Consequently, the grant of 10% attorney’s fees in favor of respondents is not
justified under the circumstances. Accordingly, the Court deems it proper to delete the same.
17. Wesleyan University-Phils., vs. Wesleyan University-Phils., Faculty & Staff Asso., GR No. 181806, March
12, 2014

Principle: The Non-Diminution Rule found in Article 100 of the Labor Code explicitly prohibits employers from
eliminating or reducing the benefits received by their employees.  This rule, however, applies only if the benefit is
based on an express policy, a written contract, or has ripened into a practice. To be considered a practice, it must be
consistently and deliberately made by the employer over a long period of time.

An exception to the rule is when “the practice is due to error in the construction or application of a doubtful or difficult
question of law.”  The error, however, must be corrected immediately after its discovery;  otherwise, the rule on Non-
Diminution of Benefits would still apply.

Facts:

Petitioner and respondent signed a 5-year CBA. Petitioner, through its President, Atty. Maglaya issued a
Memorandum providing guidelines on the implementation of vacation and sick leave credits as well as vacation leave
commutation.  The pertinent portions of the Memorandum to wit:

VACATION AND SICK LEAVE CREDITS

Vacation and sick leave credits are not automatic.  They have to be earned.  Monthly, a qualified employee earns an
equivalent of 1.25 days credit each for VL and SL.  Vacation Leave and Sick Leave credits of 15 days become
complete at the cut off date of May 31 of each year.  (Example,  only a total of 5 days credit will be given to an
employee for each of sick leave [or] vacation leave, as of month end September,  that is, 4 months from June to
September multiplied by 1.25 days).  An employee, therefore, who takes VL or SL beyond his leave credits as of date
will have to file leave without pay for leaves beyond his credit.

VACATION LEAVE COMMUTATION

Only vacation leave is commuted or monetized to cash.  Vacation leave commutation is effected after the second
year of continuous service of an employee.  Hence, an employee who started working June 1, 2005 will get his
commutation on May 31, 2007 or thereabout.

Respondent’s President, Cynthia L. De Lara (De Lara) wrote a letter to Atty. Maglaya informing him that respondent
is not amenable to the unilateral changes made by petitioner.  De Lara questioned the guidelines for being violative of
existing practices and the CBA, specifically Sections 1 and 2, Article XII of the CBA, to wit:

ARTICLE XII 

VACATION LEAVE AND SICK LEAVE

SECTION 1. VACATION LEAVE - All regular and non-tenured rank-and-file faculty and staff who are entitled to
receive shall enjoy fifteen (15) days vacation leave with pay annually.

1.1   All unused vacation leave after the second year of service shall be converted into cash and be paid to the
entitled employee at the end of each school year to be given not later than August 30 of each year.

SECTION 2. SICK LEAVE - All regular and non-tenured rank-and-file faculty and staff shall enjoy fifteen (15) days
sick leave with pay annually.

Subsequently, a Labor Management Committee (LMC) Meeting was held during which petitioner advised respondent
to file a grievance complaint on the implementation of the vacation and sick leave policy. In the same meeting,
petitioner announced its plan of implementing a one-retirement policy, which was unacceptable to respondent.

Unable to settle their differences at the grievance level, the parties referred the matter to a Voluntary Arbitrator. 
During the hearing, respondent submitted affidavits to prove that there is an established practice of giving two
retirement benefits, one from the Private Education Retirement Annuity Association (PERAA) Plan and another from
the CBA Retirement Plan. The Voluntary Abitrator rendered a Decision declaring the one-retirement policy and the
Memorandum contrary to law. 

The CA affirmed the findings and the ruling of the VA.

Issues:

Whether the CA committed grave and palpable error in sustaining the Voluntary Arbitrator’s ruling that a university
practice of granting its employees two (2) sets of Retirement Benefits had already been established as defined by the
law and jurisprudence especially in light of the illegality and lack of authority of such alleged grant.

Ruling:

The practice of giving two retirement benefits is supported by substantial evidence.


The Non-Diminution Rule found in Article 100 of the Labor Code explicitly prohibits employers from eliminating or
reducing the benefits received by their employees.  This rule, however, applies only if the benefit is based on an
express policy, a written contract, or has ripened into a practice. To be considered a practice, it must be consistently
and deliberately made by the employer over a long period of time.

An exception to the rule is when “the practice is due to error in the construction or application of a doubtful or difficult
question of law.”  The error, however, must be corrected immediately after its discovery; otherwise, the rule on Non-
Diminution of Benefits would still apply.
The practice of giving two retirement benefits to petitioner’s employees is supported by substantial
evidence.

In this case, respondent was able to present substantial evidence in the form of affidavits to support its claim that
there are two retirement plans.  Based on the affidavits, petitioner has been giving two retirement benefits as early as
1997.  Petitioner, on the other hand, failed to present any evidence to refute the veracity of these affidavits.  
Petitioner’s contention that these affidavits are self-serving holds no water.  The retired employees of petitioner have
nothing to lose or gain in this case as they have already received their retirement benefits.  Thus, they have no
reason to perjure themselves.  Obviously, the only reason they executed those affidavits is to bring out the truth.   As
we see it then, their affidavits, corroborated by the affidavits of incumbent employees, are more than sufficient to
show that the granting of two retirement benefits to retiring employees had already ripened into a consistent and
deliberate practice.

Moreover, petitioner’s assertion that there is only one retirement plan as the CBA Retirement Plan and the PERAA
Plan are one and the same is not supported by any evidence.  There is nothing in Article XVI of the CBA to indicate
or even suggest that the “Plan” referred to in the CBA is the PERAA Plan.  Besides, any doubt in the interpretation of
the provisions of the CBA should be resolved in favor of respondent.
18. Bluer Than Blue Joint Ventures Co., vs. Esteban, GR No. 192582, April 7, 2014, citing 2011 Nina Jewelry
Manufacturing of Metal Arts Inc. vs. Montecillo

Principle: Article 113 of the Labor Code provides that no employer, in his own behalf or in behalf of any person, shall
make any deduction from the wages of his employees, except in cases where the employer is authorized by law or
regulations issued by the Secretary of Labor and Employment, among others.

Facts:

Respondent Glyza Esteban (Esteban) was employed as Sales Clerk, and assigned at Bluer Than Blue Joint Ventures
Company's (petitioner) EGG boutique in SM City Marilao, Bulacan, beginning the year 2006. Part of her primary tasks
were attending to all customer needs, ensuring efficient inventory, coordinating orders from clients, cashiering and
reporting to the accounting department.

In November 2006, the petitioner received a report that several employees have access to its point-of-sale (POS)
system through a universal password given by Elmer Flores (Flores). Upon investigation, it was discovered that it was
Esteban who gave Flores the password. The petitioner sent a letter memorandum to Esteban asking her to explain in
writing why she should not be disciplinary dealt with for tampering with the company’s POS system through the use of
an unauthorized password. Esteban was also placed under preventive suspension for ten days.

In her explanation, Esteban admitted that she used the universal password three times on the same day in December
2005, after she learned of it from two other employees who she saw browsing through the petitioner’s sales inquiry.
She inquired how the employees were able to open the system and she was told that they used the "123456"
password.

On November 13, 2006, Esteban’s preventive suspension was lifted, but at the same time, a notice of termination
was sent to her, finding her explanation unsatisfactory and terminating her employment immediately on the ground of
loss of trust and confidence.

Esteban was given her final pay, including benefits and bonuses, less inventory variances incurred by the store
amounting to P8,304.93..

Esteban filed a complaint for illegal dismissal, illegal suspension, holiday pay, rest day and separation pay.

The Labor Arbiter (LA) ruled in favor of Esteban and found that she was illegally dismissed. NLRC reversed the
decision of the LA. However, the CA reinstated the decision of the LA.

Issues:

WON the deduction of P8,304.93, representing the store’s negative variance, from Esteban’s salary violates Article
113 of the Labor Code, which prohibits wage deduction.

Ruling:

Yes, petitioner violated the Labor Code provision on wage deduction.

The petitioner deducted the amount of P8,304.93 from Esteban’s last salary. According to the petitioner, this
represents the store’s negative variance for the year 2005 to 2006. The petitioner justifies the deduction on the basis
of alleged trade practice and that it is allowed by the Labor Code.

Article 113 of the Labor Code provides that no employer, in his own behalf or in behalf of any person, shall make any
deduction from the wages of his employees, except in cases where the employer is authorized by law or regulations
issued by the Secretary of Labor and Employment, among others. The Omnibus Rules Implementing the Labor Code,
meanwhile, provides:

SECTION 14. Deduction for loss or damage. – Where the employer is engaged in a trade, occupation or business
where the practice of making deductions or requiring deposits is recognized to answer for the reimbursement of loss
or damage to tools, materials, or equipment supplied by the employer to the employee, the employer may make wage
deductions or require the employees to make deposits from which deductions shall be made, subject to the following
conditions:

a) That the employee concerned is clearly shown to be responsible for the loss or damage;
b) That the employee is given reasonable opportunity to show cause why deduction should not be made;
c) That the amount of such deduction is fair and reasonable and shall not exceed the actual loss or damage;
and
d) That the deduction from the wages of the employee does not exceed 20 percent of the employee’s wages in
a week.

In this case, the petitioner failed to sufficiently establish that Esteban was responsible for the negative variance it had
in its sales for the year 2005 to 2006 and that Esteban was given the opportunity to show cause the deduction from
her last salary should not be made. The Court cannot accept the petitioner’s statement that it is the practice in the
retail industry to deduct variances from an employee’s salary, without more. In Niña Jewelry Manufacturing of Metal
Arts, Inc. v. Montecillo,the Court ruled that:

The petitioners should first establish that the making of deductions from the salaries is authorized by law, or
regulations issued by the Secretary of Labor. Further, the posting of cash bonds should be proven as a recognized
practice in the jewelry manufacturing business, or alternatively, the petitioners should seek for the determination by
the Secretary of Labor through the issuance of appropriate rules and regulations that the policy the former seeks to
implement is necessary or desirable in the conduct of business. The petitioners failed in this respect. It bears
stressing that without proofs that requiring deposits and effecting deductions are recognized practices, or without
securing the Secretary of Labor's determination of the necessity or desirability of the same, the imposition of new
policies relative to deductions and deposits can be made subject to abuse by the employers. This is not what the law
intends.
8.PAYMENT OF WAGES
9.CONDITIONS OF EMPLOYMENT
10.MINIMUM LABOR STANDARD BENEFITS

19. Radio Mindanao Network Inc. et al., vs. Ybarola, Jr. G.R. No. 198662, Sept. 12, 2012

Facts:

Respondents Domingo Z. Ybarola, Jr. and Alfonso E. Rivera, Jr. were hired on June 15, 1977 and June 1, 1983,
respectively, by RMN. They eventually became account managers, soliciting advertisements and servicing various
clients of RMN.

On September 15, 2002, the respondents' services were terminated as a result of RMN's reorganization/restructuring;
they were given their separation pay — P631,250.00 for Ybarola, and P481,250.00 for Rivera. Sometime in
December 2002, they executed release/quitclaim affidavits.

Dissatisfied with their separation pay, the respondents filed separate complaints (which were later consolidated)
against RMN and its President, Eric S. Canoy, for illegal dismissal with several money claims, including attorney's
fees. They indicated that their monthly salary rates were P60,000.00 for Ybarola and P40,000.00 for Rivera.

Issues:

Whether or not the commissions received by respondents be included in the computation of their separation pay.

Ruling:

Yes. If these commissions had been really profit-sharing bonuses to the respondents, they should have received the
same amounts, yet, as the NLRC itself noted, Ybarola and Rivera received P372,173.11 and P586,998.50
commissions, respectively, in 2002.  The variance in amounts the respondents received as commissions supports the
CA's finding that the salary structure of the respondents was such that they only received a minimal amount as
guaranteed wage; a greater part of their income was derived from the commissions they get from soliciting
advertisements; these advertisements are the "products" they sell. As the CA aptly noted, this kind of salary structure
does not detract from the character of the commissions being part of the salary or wage paid to the employees for
services rendered to the company, as the Court held in  Philippine Duplicators, Inc. v. NLRC.
20. David/Yiels Hog Dealer vs. Macasio, GR No. 195466, July 2, 2014

Facts:

Macasio filed before the LA a complaint against petitioner Ariel L. David, doing business under the name and style
"Yiels Hog Dealer," for non-payment of overtime pay, holiday pay and 13th month pay. He also claimed payment
for moral and exemplary damages and attorney's fees. Macasio also claimed payment for service incentive
leave (SIL).

Macasio alleged before the LA that he had been working as a butcher for David since January 6, 1995. Macasio
claimed that David exercised effective control and supervision over his work, pointing out that David: (1) set the work
day, reporting time and hogs to be chopped, as well as the manner by which he was to perform his work; (2) daily
paid his salary of P700.00, which was increased from P600.00 in 2007, P500.00 in 2006 and P400.00 in 2005; and
(3) approved and disapproved his leaves. Macasio added that David owned the hogs delivered for chopping, as well
as the work tools and implements; the latter also rented the workplace. Macasio further claimed that David employs
about twenty-five (25) butchers and delivery drivers.

In his defense, David claimed that he started his hog dealer business in 2005 and that he only has ten employees. He
alleged that he hired Macasio as a butcher or chopper on "pakyaw" or task basis who is, therefore, not entitled to
overtime pay, holiday pay and 13th month pay pursuant to the provisions of the Implementing Rules and
Regulations (IRR) of the Labor Code. David pointed out that Macasio: (1) usually starts his work at 10:00 p.m. and
ends at 2:00 a.m. of the following day or earlier, depending on the volume of the delivered hogs; (2) received the
fixed amount of P700.00 per engagement, regardless of the actual number of hours that he spent chopping the
delivered hogs; and (3) was not engaged to report for work and, accordingly, did not receive any fee when no hogs
were delivered.

David maintains that Macasio's engagement was on a "pakyaw" or task basis. Hence, the latter is excluded from the
coverage of holiday, SIL and 13th month pay.||| 

Issue:

Whether or not a worker engaged on “pakyaw” or task basis is entitled to Holiday, SIL, and 13th month pay.

Ruling:

Petition was partially granted.

SIL

The general rule is that holiday and SIL pay provisions cover all employees. To be excluded from their coverage, an
employee must be one of those that these provisions expressly exempt, strictly in accordance with the exemption.

Under the IRR, exemption from the coverage of holiday and SIL pay refer to "field personnel and other employees
whose time and performance is unsupervised by the employer including those who are engaged on task or contract
basis[.]" Note that unlike Article 82 of the Labor Code, the IRR on holiday and SIL pay do not exclude employees
"engaged on task basis" as a separate and distinct category from employees classified as "field personnel." Rather,
these employees are altogether merged into one classification of exempted employees.

The payment of an employee on task or pakyaw basis alone is insufficient to exclude one from the coverage of SIL
and holiday pay. They are exempted from the coverage of Title I (including the holiday and SIL pay) only if they
qualify as "field personnel." The IRR therefore validly qualifies and limits the general exclusion of "workers paid by
results" found in Article 82 from the coverage of holiday and SIL pay. This is the only reasonable interpretation since
the determination of excluded workers who are paid by results from the coverage of Title I is "determined by the
Secretary of Labor in appropriate regulations."|||

In short, in determining whether workers engaged on "pakyaw" or task basis" is entitled to holiday and SIL pay, the
presence (or absence) of employer supervision as regards the worker's time and performance is the key: if the
worker is simply engaged on pakyaw or task basis, then the general rule is that he is entitled to a holiday pay and
SIL pay unless exempted from the exceptions specifically provided under Article 94 (holiday pay) and Article 95 (SIL
pay) of the Labor Code. However, if the worker engaged on pakyaw or task basis also falls within the meaning of
"field personnel" under the law, then he is not entitled to these monetary benefits.|||

Based on the definition of field personnel under Article 82, that Macasio does not fall under the definition of "field
personnel." The finding is supported by the established facts of this case: first, Macasio regularly performed his duties
at David's principal place of business; second, his actual hours of work could be determined with reasonable
certainty; and, third, David supervised his time and performance of duties. Since Macasio cannot be considered a
"field personnel," then he is not exempted from the grant of holiday, SIL pay even as he was engaged on "pakyaw" or
task basis.|||

13th month pay

The governing law on 13th month pay is PD No. 851. As with holiday and SIL pay, 13th month pay benefits generally
cover all employees; an employee must be one of those expressly enumerated to be exempted. Section 3 of the
Rules and Regulations Implementing P.D. No. 851 enumerates the exemptions from the coverage of 13th month pay
benefits. Under Section 3 (e), "employers of those who are paid on . . . task basis, and those who are paid a fixed
amount for performing a specific work, irrespective of the time consumed in the performancethereof" are
exempted. DSAICa

Note that unlike the IRR of the Labor Code on holiday and SIL pay, Section 3 (e) of the Rules and Regulations
Implementing PD No. 851 exempts employees "paid on task basis" without any reference to "field personnel." This
could only mean that insofar as payment of the 13th month pay is concerned, the law did not intend to qualify the
exemption from its coverage with the requirement that the task worker be a "field personnel" at the same time.
11.OTHER SPECIAL BENEFITS

21. Radio Mindanao Network Inc, et al., vs. Ybarola, Jr. et al., G.R. No. 198662, September 12, 2012

Facts:

Respondents Domingo Z. Ybarola, Jr. and Alfonso E. Rivera, Jr. were hired on June 15, 1977 and June 1, 1983,
respectively, by RMN. They eventually became account managers, soliciting advertisements and servicing various
clients of RMN.

On September 15, 2002, the respondents' services were terminated as a result of RMN's reorganization/restructuring;
they were given their separation pay — P631,250.00 for Ybarola, and P481,250.00 for Rivera. Sometime in
December 2002, they executed release/quitclaim affidavits.

Dissatisfied with their separation pay, the respondents filed separate complaints (which were later consolidated)
against RMN and its President, Eric S. Canoy, for illegal dismissal with several money claims, including attorney's
fees. They indicated that their monthly salary rates were P60,000.00 for Ybarola and P40,000.00 for Rivera.

Issues:

Whether or not the commissions received by respondents be included in the computation of their separation pay.

Ruling:

Yes. If these commissions had been really profit-sharing bonuses to the respondents, they should have received the
same amounts, yet, as the NLRC itself noted, Ybarola and Rivera received P372,173.11 and P586,998.50
commissions, respectively, in 2002.  The variance in amounts the respondents received as commissions supports the
CA's finding that the salary structure of the respondents was such that they only received a minimal amount as
guaranteed wage; a greater part of their income was derived from the commissions they get from soliciting
advertisements; these advertisements are the "products" they sell. As the CA aptly noted, this kind of salary structure
does not detract from the character of the commissions being part of the salary or wage paid to the employees for
services rendered to the company, as the Court held in  Philippine Duplicators, Inc. v. NLRC.
22. Padillo vs. Rural Bank of Nabunturan Inc. G.r. No. 199338, Jan. 21, 2013

Facts:

Padillo, was employed by respondent Bank as its SA Bookkeeper. Due to liquidity problems which arose sometime in
2003, the Bank took out retirement/insurance plans with Philippine American Life and General Insurance Company
(Philam Life) for all its employees in anticipation of its possible closure and the concomitant severance of its
personnel. Respondent Mark S. Oropeza (Oropeza), the President of the Bank, bought majority shares of stock in
the Bank and took over its management which brought about its gradual rehabilitation. The Bank's finances improved
and eventually, its liquidity was regained.

Padillo suffered a mild stroke due to hypertension which consequently impaired his ability to effectively pursue his
work. In particular, he was diagnosed with Hypertension S/P CVA (Cerebrovascular Accident) with short term
memory loss, the nature of which had been classified as a total disability.  On September 10, 2007, he wrote a letter
addressed to respondent Oropeza expressing his intention to avail of an early retirement package. Despite several
follow-ups, his request remained unheeded. Padillo was separated from employment due to his poor and failing
health

Issue:
Whether or not petitioner is entitled to retirement pay.

Ruling:
In the absence of any applicable agreement, an employee must (1) retire when he is at least sixty (60) years of age
and (2) serve at least (5) years in the company to entitle him/her to a retirement benefit of at least one-half (1/2)
month salary for every year of service, with a fraction of at least six (6) months being considered as one whole year.
Notably, these age and tenure requirements are cumulative and non-compliance with one negates the employee's
entitlement to the retirement benefits under Article 300 of the Labor Code altogether.

In this case, it is undisputed that there exists no retirement plan, collective bargaining agreement or any other
equivalent contract between the parties which set out the terms and condition for the retirement of employees, with
the sole exception of the Philam Life Plan which premiums had already been paid by the Bank. Neither was it proven
that there exists an established company policy of giving early retirement packages to the Bank's aging employees.

All told, in the absence of any applicable contract or any evolved company policy, Padillo should have met the age
and tenure requirements set forth under Article 300 of the Labor Code to be entitled to the retirement benefits
provided therein. Unfortunately, while Padillo was able to comply with the five (5) year tenure requirement — as he
served for twenty-nine (29) years — he, however, fell short with respect to the sixty (60) year age requirement given
that he was only fifty-five (55) years old when he retired. Therefore, without prejudice to the proceeds due under the
Philam Life Plan, petitioners' claim for retirement benefits must be denied.

Nevertheless, financial assistance should be awarded but at an increased amount. With a veritable understanding
that the award of financial assistance is usually the final refuge of the laborer, considering as well the supervening
length of time which had sadly overtaken the point of Padillo's death — an employee who had devoted twenty-nine
(29) years of dedicated service to the Bank — the Court, in light of the dictates of social justice, holds that the CA's
financial assistance award should be increased from P50,000.00 to P75,000.00, still exclusive of the P100,000.00
benefit receivable by the petitioners under the Philam Life Plan which remains undisputed. 
12.RIGHT TO SECURITY OF TENURE

23. Gapayao vs. Fulo et al., G.R. No. 193493, June 13, 2013

Facts:

Fulo (deceased) died of "acute renal failure secondary to 1st degree burn 70% secondary electrocution" while doing
repairs at the residence and business establishment of petitioner located at San Julian, Irosin, Sorsogon. The ER and
the wife executed a compromise agreement whereby the ER agreed to pay 40K to the suriving spouse of Fulo.
Thereafter, private respondent filed a claim for social security benefits with the Social Security System (SSS) —
Sorsogon Branch. However, upon verification and evaluation, it was discovered that the deceased was not a
registered member of the SSS.

Private respondent claimed that the deceased was not the former's employee, but was rather an independent
contractor whose tasks were not subject to petitioner's control and supervision. Assuming arguendo that the
deceased was petitioner's employee, he was still not entitled to be paid his SSS premiums for the intervening period
when he was not at work, as he was an "intermittent worker who [was] only summoned every now and then as the
need [arose]."  Hence, petitioner insisted that he was under no obligation to report the former's demise to the SSS for
social security coverage.

Issue:

Whether or not there exists between the deceased Jaime Fulo and petitioner an employer-employee relationship that
would merit an award of benefits in favor of private respondent under social security laws.

Ruling:

Jurisprudence has identified the three types of employees mentioned in the provision: (1) regular employees or those
who have been engaged to perform activities that are usually necessary or desirable in the usual business or trade of
the employer; (2) project employees or those whose employment has been fixed for a specific project or undertaking,
the completion or termination of which has been determined at the time of their engagement, or those whose work or
service is seasonal in nature and is performed for the duration of the season; and (3) casual employees or those who
are neither regular nor project employees.

Farm workers generally fall under the definition of seasonal employees. We have consistently held that seasonal
employees may be considered as regular employees. Regular seasonal employees are those called to work from
time to time. The nature of their relationship with the employer is such that during the off season, they are temporarily
laid off; but reemployed during the summer season or when their services may be needed. They are in regular
employment because of the nature of their job, and not because of the length of time they have worked. 

The rule, however, is not absolute. Seasonal workers who have worked for one season only may not be considered
regular employees. When such seasonal employees are free to contract their services with other farm owners, then
the former are not regular employees.

For regular employees to be considered as such, the primary standard used is the reasonable connection between
the particular activity they perform and the usual trade or business of the employer. The deceased was indeed a farm
worker who was in the regular employ of petitioner. From year to year, starting January 1983 up until his death, the
deceased had been working on petitioner's land by harvesting abaca and coconut, processing copra, and clearing
weeds. His employment was continuous in the sense that it was done for more than one harvesting season.
Moreover, no amount of reasoning could detract from the fact that these tasks were necessary or desirable in the
usual business of petitioner. 

The other tasks allegedly done by the deceased outside his usual farm work only bolster the existence of an
employer-employee relationship. As found by the SSC, the deceased was a construction worker in the building and a
helper in the bakery, grocery, hardware, and piggery — all owned by petitioner. 63 This fact only proves that even
during the off season, the deceased was still in the employ of petitioner.

Pakyaw  workers are considered employees for as long as their employers exercise control over them. Being the
owner of the farm on which the latter worked, petitioner — on his own or through his overseer — necessarily had the
right to review the quality of work produced by his laborers

The right of an employee to be covered by the Social Security Act is premised on the existence of an employer-
employee relationship. The Court ruled in favor of private respondent.
24. Concrete Solutions Inc. et al., vs. Cabusas, G.R. No. 177812, June 19, 2013

Facts:
Respondent Arthur Cabusas (respondent) was hired by petitioner Primary Structures Corporation (PSC) as transit
mixer driver for petitioner Concrete Solutions, Inc. (CSI) — Batching Plant Project. The appointment letter  dated June
27, 2000, which was signed by petitioner PSC's Human Resource Division Assistant with respondent's conformity,
provided, among others: that respondent was hired for the period from June 28, 2000 until June 23, 2001; the status
of his employment was that of a project employee and, as such, his employment was co-terminus with the completion
of the project or any phase thereof; that upon completion of the particular project or phase, he was free to seek other
employment of his choice; and, that within the duration of the work, petitioners shall have the right to terminate his
employment without any liability on their part if his performance did not meet the company standards, or if he violated
petitioners' rules and regulations.

On February 16, 2001, a report reached petitioners that at around 5 o'clock in the afternoon of that day, respondent,
as the driver of Transit Mixer 13, unloaded less than a cubic meter of concrete mix at Cabancalan, Mandaue City,
more than two kilometers away from its project site located at Wireless, Mandaue City, instead of returning the
excess concrete mix to the plant; and that respondent sold the excess concrete mix to the residents of the place
where he unloaded the same.

On March 7, 2001, petitioners' Administrative Assistant, Carlo E. Gimena, submitted an Incident Report where he
stated that it is a company policy that washing/cleaning of drums must be done inside petitioners' plant to maximize
the utilization of concrete residues for precast use; and nearly a cubic meter of concrete mix as excess would have
been a substantial quantity for such purpose.

On March 8, 2001, petitioners' Manager, Anastacio G. Ardiente, Jr., required respondent to explain in writing why he
should not be meted with a disciplinary action for the alleged act of theft or dishonesty under the company's Code of
Conduct and Discipline. In his explanation, 7 respondent stated that he threw away the concrete mix at Cabancalan,
Mandaue City, instead of turning them over to the plant as he will wash the transit mixer at A.S. Fortuna, Mandaue
City. Respondent was meted a three (3)-day suspension effective March 20, 2001 to March 22, 2001.

On April 19, 2001, petitioners received an information that respondent allegedly took the company's plastic drum for
personal gain. In his Incident Report dated April 20, 2001, petitioners' Administrative Assistant Gimena reported that
at 10:00 a.m. of April 19, 2001, respondent took an empty plastic drum and hid it in the Transit Mixer 13 he was
driving on his way to deliver concrete mix to Ayala Heights; and that respondent even admitted the commission of
such act which another transit mixer driver could attest to. Gimena recommended further investigation to include the
security guards on duty at the time of the incident.  Respondent was asked to explain why no disciplinary action
should be meted on him for such violation, and to attend the formal investigation on April 26, 2001

In his written explanation, respondent denied the accusation against him and claimed that he could not had driven the
transit mixer out of the company's premises without passing through the guard house; hence, it was impossible to
steal the plastic drum without the knowledge of the guard. He personally delivered his letter of explanation to the
company, but was refused entry by the security guards. Respondent was placed under preventive suspension from
April 20, 2001 to April 27, 2001 pending investigation of his case.

The administrative investigation which was scheduled on April 26, 2001 was postponed to May 4, 2001 and
respondent's preventive suspension was extended up to May 5, 2001. Respondent alleged that after the investigation
on May 4, 2001, he and his counsel had asked for the result of the investigation and were waiting for such result.

While petitioners were deliberating on the violation committed by respondent, they went over the latter's 201 file and
discovered that he appeared not to be registered with the Social Security System as the SSS number he submitted
was that of another person in the name of Alex Cabusas. Thus, petitioners needed clarifications from respondent, but
the latter had been absent since May 6, 2001. On May 25, 2001, petitioners sent respondent a telegram,  to wit: "You
have been absent without official leave since May [6], 2001. Please notify CSI as soon as possible."

On June 12, 2001, petitioners, thru Manager Ardiente, sent respondent a termination letter.

Petitioners submitted to the Department of Labor and Employment an Establishment Termination Report indicating
that the project where respondent was assigned was already completed and also that respondent was terminated for
being absent without leave (AWOL).

Earlier, however, on May 30, 2001, respondent had filed with Regional Arbitration Branch No. VII of Cebu City a
Complaint for unfair labor practice, illegal dismissal, non-payment of holiday pay, premium pay for holiday, rest day,
night shift premium, separation pay and moral damages against petitioners. In his position paper, respondent alleged
among others: that it was not true that he went on AWOL. He alleged that when the administrative investigation on
his alleged theft of company property was conducted and terminated on May 4, 2001, his counsel asked to be
furnished a copy of the result of the investigation; that since then, they eagerly waited for such result, thus they were
surprised to receive a telegram on May 26, 2001 where he was said to have been AWOL since May 5, 2001; that
immediately upon receipt of the telegram, respondent went to petitioners' office, but he was refused entry for the
reason that he was AWOL; that there was no valid cause for his dismissal and petitioners found the lame excuse of
declaring him AWOL if only to create a semblance of justification for his unlawful termination; that he had previously
tendered a follow-up letter for a copy of the resolution of the administrative investigation that was terminated on May
4, 2001, however, petitioners unceremoniously refused to receive a copy of the letter he personally delivered, thus his
counsel was compelled to send the letter by way of registered mail on May 29, 2001;  that petitioners did not reply to
his letter and did not even furnish his counsel with a copy of the suspension letter; that petitioners' imputation that he
committed dishonest acts was founded on falsehood and fabrications as no evidence was presented during the so-
called administrative hearing, except the self-serving and perjured statements of petitioners' employees who were
merely cajoled into making unfounded stories. Respondent had prayed for his reinstatement, among others.

Issue
1. Whether or not respondent deliberately abandoned his work which is a just cause for his dismissal.
2. Whether or not there was illegal dismissal.

Ruling
It is well settled that in termination cases, the burden of proof rests upon the employer to show that the dismissal was
for a just and valid cause, and failure to discharge the same would mean that the dismissal is not justified and,
therefore, illegal.  In this case, petitioners claim that respondent was validly dismissed as he abandoned his work as
shown by the following circumstances, to wit: He did not go back to work on May 6, 2001, i.e., after his preventive
suspension expired on May 5, 2001; he did not report to work despite receipt of the telegram on May 25, 2001 stating
that "he was absent without official leave since May 5, 2001, and to notify CSI as soon as possible," but instead,
through his lawyer, sent a letter asking for a copy of the result of the investigation; despite not being given the result
of the investigation, respondent still did not bother to report back to work; and the complaint he filed with the LA did
not pray for reinstatement.

To constitute abandonment, two elements must concur, to wit: (1) the failure to report for work or absence without
valid or justifiable reason; and (2) a clear intention to sever the employer-employee relationship, with the second
element as the more determinative factor and being manifested by some overt acts.  Abandonment is a matter of
intention and cannot lightly be presumed from certain equivocal acts. To be a valid cause for dismissal for
abandonment, there must be clear proof of deliberate and unjustified intent to sever the employer-employee
relationship.  Clearly, the operative act is still the employee's ultimate act of putting an end to his employment. 

We find that the elements of abandonment are lacking. The CA did not commit any reversible error in affirming the
NLRC's decision that respondent was illegally dismissed for petitioners' failure to substantiate their claim that the
former abandoned his work. The circumstances obtaining in this case do not indicate abandonment. 

Respondent explained that his absence from work was due to the fact that he and his counsel had asked and were
waiting for a copy of result of the investigation on his alleged act of theft or dishonesty conducted on May 4, 2001 but
were not given at all. We find his absence from work not sufficient to establish that he already had intention of
abandoning his job. Besides, settled is the rule that mere absence or failure to report for work is not tantamount to
abandonment of work.  Even the failure to report for work after a notice to return to work has been served does not
necessarily constitute abandonment.  In fact, when respondent received petitioners' telegram on May 25, 2001
stating that "he was absent without official leave since May 5, 2001, and to notify CSI as soon as possible", he went
to petitioners premises but was refused entry for reason that he was AWOL. He also tried to give them a letter dated
May 26, 2001 from his counsel requesting for a copy of the resolution of the investigation conducted on May 4, 2001
but petitioners refused to receive the same which prompted respondent's counsel to send the letter dated May 26,
2001 to petitioners by registered mail on May 29, 2001. The fact of petitioners' refusal to receive the letter was stated
in that letter but they never refuted the same which in effect, negates petitioners' claim that respondent did not
comply with the telegram sent to him.

There is no showing of respondent's intent to sever the employer-employee relationship. It is also notable that when
respondent was refused entry to petitioners' premises and the letter of former's counsel was refused acceptance by
the latter, there is already constructive dismissal which led respondent to seek recourse by filing an illegal dismissal
case against petitioners on May 30, 2001. The proximity of respondent's filing of the complaint from the time he
received the telegram and was refused entry to petitioners' premises showed that he had the least intention of
abandoning his job. Well-settled that the filing by an employee of a complaint for illegal dismissal with a prayer for
reinstatement is proof enough of his desire to return to work, thus, negating the employer's charge of abandonment. 

There is also no merit to petitioners' claim that respondent did not ask for reinstatement. While in his complaint filed
with the LA, respondent failed to ask for reinstatement however, in his position paper, he specifically prayed for
reinstatement. 39 which showed that he had no intention of abandoning his work.
Petitioners' claim that respondent's violations of company rules also warranted his termination on account of loss of
trust and confidence deserves scant consideration since the latter's dismissal was not due to those alleged dishonest
acts but due to abandonment.

Petitioners contend that respondent was a project employee and the project to which he was hired was already
completed, thus he could not be reinstated anymore.

Project employee is one whose employment has been fixed for a specific project or undertaking the completion or
termination of which has been determined at the time of the engagement of the employee or where the work or
services to be performed is seasonal in nature and the employment is for the duration of the season.  We held that
the length of service of a project employee is not the controlling test of employment tenure but whether or not the
employment has been fixed for a specific project or undertaking the completion or termination of which has been
determined at the time of the engagement of the employee. 

We rule that respondent is a project employee. His appointment letter showed that he was hired as transit mixer
driver for the Concrete Solutions, Inc. (CSI) — Batching Plant Project for the period from June 28, 2000 until June 23,
2001. The same letter provided that he was a project employee whose employment was co-terminus with the
completion of the project or any phase thereof and upon completion of the particular project or phase, he was free to
seek other employment of his choice. There is no evidence showing that respondent did not sign the conforme part of
the appointment letter voluntarily. Hence, respondent was bound by the provisions in the appointment letter.
Moreover, there is also no showing that the period fixed in the appointment letter was imposed to preclude acquisition
of tenurial security by the employee and should be disregarded for being contrary to public policy as ruled by the
NLRC since no evidence exists on the record to support such conclusion.

Considering that respondent was dismissed prior to the expiration of the duration of his employment and without a
valid or just cause, his termination was therefore illegal. However, respondent could no longer be reinstated since the
project he was assigned to was already completely finished. However, we find that he is entitled to the salary
corresponding to the unexpired portion of his employment.  Respondent is entitled to the payment of his salary from
the time he was not admitted back to work on May 26, 2001 up to June 23, 2001, the expiration of his employment
contract. 
25. D.M. Consunji vs. Bello, G.R. No. 159371, July 29, 2013

Facts:
Bello brought a complaint for illegal dismissal and damages against DMCI and/or Rachel Consunji. In his position
paper, he claimed that DMCI had employed him as a mason without any interruption from February 1, 1990 until
October 10, 1997 at an hourly rate of P25.081; that he had been a very diligent and devoted worker and had
served DMCI as best as he could and without any complaints; that he had never violated any company rules; that
his job as a mason had been necessary and desirable in the usual business or trade of DMCI; that he had been
diagnosed to be suffering from pulmonary tuberculosis, thereby necessitating his leave of absence; that upon his
recovery, he had reported back to work, but DMCI had refused to accept him and had instead handed to him a
termination paper; that he had been terminated due to "RSD" effective November 5, 1997; that he did not know the
meaning of "RSD" as the cause of his termination; that the cause had not been explained to him; that he had not
been given prior notice of his termination; that he had not been paid separation pay as mandated by law; that at
that time of his dismissal, DMCI's projects had not yet been completed; and that even if he had been terminated
due to an authorized cause, he should have been given at least one month pay or at least one-half month pay for
every year of service he had rendered, whichever was higher.

In its position paper submitted on March 6, 2000,  DMCI contended that Bello had only been a project employee, as
borne out by his contract of employment and appointment papers; that after his termination from employment, it had
complied with the reportorial requirements of the Department of Labor and Employment (DOLE) pursuant to the
mandates of Policy Instruction No. 20, as revised by Department Order No. 19, series of 1993; and that although his
last project employment contract had been set to expire on October 7, 1997, he had tendered his voluntary
resignation on October 4, 1997 for health reasons that had rendered him incapable of performing his job, per his
resignation letter.

Issues

I. WHETHER OR NOT PRIVATE RESPONDENT WAS A REGULAR EMPLOYEE; AND

II.WHETHER OR NOT PRIVATE RESPONDENT WAS DISMISSED OR VOLUNTARILY RESIGNED.

Ruling
The provision that governs the first issue is Article 280 of the Labor Code, which is quoted hereunder as to its
relevant part, viz.:

Article 280.Regular and Casual Employment. — The provisions of written agreement to the contrary notwithstanding
and regardless of the oral agreement of the parties, an employment shall be deemed to be regular where the
employee has been engaged to perform activities which are usually necessary and desirable to the usual business or
trade of the employer, except where the employment has been fixed for a specific project or undertaking the
completion or termination of which has been determined at the time of the engagement of the employee or
where the work or service to be performed is seasonal in nature and the employment is for the duration of the
season. (Emphasis supplied) 
xxx xxx xxx

A project employee is, therefore, one who is hired for a specific project or undertaking, and the completion or
termination of such project or undertaking has been determined at the time of engagement of the employee.   In the
context of the law, Bello was a project employee of DMCI at the beginning of their employer-employee relationship.
The project employment contract they then entered into clearly gave notice to him at the time of his engagement
about his employment being for a specific project or phase of work. He was also thereby notified of the duration of the
project, and the determinable completion date of the project.

However, the history of Bello's appointment and employment showed that he performed his tasks as a mason in
DMCI's various constructions projects.

We affirm the CA's conclusion that Bello acquired in time the status of a regular employee by virtue of his continuous
work as a mason of DMCI. The work of a mason like him — a skilled workman working with stone or similar
material — was really related to building or constructing, and was undoubtedly a function necessary and desirable to
the business or trade of one engaged in the construction industry like DMCI. His being hired as a mason by DMCI in
not one, but several of its projects revealed his necessity and desirability to its construction business. 

It is settled that the extension of the employment of a project employee long after the supposed project has been
completed removes the employee from the scope of a project employee and makes him a regular employee.  In this
regard, the length of time of the employee's service, while not a controlling determinant of project employment, is a
strong factor in determining whether he was hired for a specific undertaking or in fact tasked to perform functions
vital, necessary and indispensable to the usual business or trade of the employer.  On the other hand, how DMCI
chose to categorize the employment status of Bello was not decisive of his employment status. What were of
consequence in that respect were his actual functions and the length of his stay with DMCI. Verily, the principal test
for determining whether an employee is a project employee, as distinguished from a regular employee, is whether or
not he is assigned to carry out a specific project or undertaking, the duration and scope of which are specified at the
time he is engaged for the project. 

Still, DMCI contends that Bello's services as a mason were deemed necessary and desirable in its usual business
only for the period of time it had taken it to complete the project.
The contention may be correct if each engagement of Bello as a mason over the span of eight years was to be
treated separately. The contention cannot be upheld, however, simply because his successive re-engagement in
order to perform the same kind of work as a mason firmly manifested the necessity and desirability of his work in
DMCI's usual business of construction. 

Lastly, DMCI claims that Bello voluntarily resigned from work. It presented his supposed handwritten resignation letter
to support the claim. However, Bello denied having resigned, explaining that he had signed the letter because DMCI
had made him believe that the letter was for the purpose of extending his sick leave.

In resolving the matter against DMCI, the CA relied on the conclusion by ELA Panganiban-Ortiguerra that she could
not give credence to the voluntary resignation for health reasons in the face of Bello's declaration that he had been
led to sign the letter to obtain the extension of his leave of absence due to illness, and on her observation that "the
handwriting in the supposed resignation letter is undeniably different from that of complainant," something that she
said DMCI had not rebutted.

The CA's reliance on the conclusion and finding by ELA Panganiban-Ortiguerra was warranted. Her observation that
the handwriting in the resignation letter was "undeniably different" from that of Bello could not be ignored or shunted
aside simply because she had no expertise to make such a determination, as the NLRC tersely stated in its decision.
To begin with, her supposed lack of expertise did not appear in the records, rendering the NLRC's statement
speculative and whimsical. If we were now to outrightly discount her competence to make that observation, we would
disturb the time-honored practice of according respect to the findings of the first-line trier of facts in order to prefer the
speculative and whimsical statement of an appellate forum like the NLRC. Yet, even had the letter been actually
signed by him, the voluntariness of the resignation could not be assumed from such fact alone. His claim that he had
been led to believe that the letter would serve only as the means of extending his sick leave from work should have
alerted DMCI to the task of proving the voluntariness of the resignation. It was obvious that, if his claim was true, then
he did not fully comprehend the import of the letter, rendering the resignation farcical. The doubt would then be
justifiably raised against the letter being at all intended to end his employment. Under the circumstances, DMCI
became burdened with the obligation to prove the due execution and genuineness of the document as a letter of
resignation. 

We reiterate that it is axiomatic in labor law that the employer who interposes the defense of voluntary resignation of
the employee in an illegal dismissal case must prove by clear, positive and convincing evidence that the resignation
was voluntary; and that the employer cannot rely on the weakness of the defense of the employee. The requirement
rests on the need to resolve any doubt in favor of the working man.
26. Colegio Del Santisimo Rosario et al., vs. Rojo, G.R. No. 170388, Sept. 4, 2013

Facts:
Petitioner Colegio del Santisimo Rosario (CSR) hired respondent as a high school teacher on probationary basis for
the school years 1992-1993, 1993-1994 and 1994-1995. 

On April 5, 1995, CSR, through petitioner Sr. Zenaida S. Mofada, OP (Mofada), decided not to renew respondent's
services. 
Thus, on July 13, 1995, respondent filed a Complaint for illegal dismissal. He alleged that since he had served three
consecutive school years which is the maximum number of terms allowed for probationary employment, he should be
extended permanent employment. Citing paragraph 75 of the 1970 Manual of Regulations for Private Schools (1970
Manual), respondent asserted that "full-time teachers who have rendered three (3) consecutive years of satisfactory
services shall be considered permanent."

On the other hand, petitioners argued that respondent knew that his Teacher's Contract for school year 1994-1995
with CSR would expire on March 31, 1995. Accordingly, respondent was not dismissed but his probationary contract
merely expired and was not renewed.  Petitioners also claimed that the "three years" mentioned in paragraph 75 of
the 1970 Manual refer to "36 months," not three school years.  And since respondent served for only three school
years of 10 months each or 30 months, then he had not yet served the "three years" or 36 months mentioned in
paragraph 75 of the 1970 Manual.

Issue

WILL A BASIC EDUCATION (ELEMENTARY) TEACHER HIRED FOR THREE (3) CONSECUTIVE SCHOOL
YEARS AS A PROBATIONARY EMPLOYEE  AUTOMATICALLY AND/OR BY LAW  BECOMES A PERMANENT
EMPLOYEE UPON COMPLETION OF HIS THIRD YEAR OF PROBATION NOTWITHSTANDING [A] THE
PRONOUNCEMENT OF THIS HONORABLE COURT IN COLEGIO SAN AGUSTIN V. NLRC, 201 SCRA 398 [1991]
THAT A PROBATIONARY TEACHER ACQUIRES PERMANENT STATUS "ONLY WHEN HE IS ALLOWED TO
WORK AFTER THE PROBATIONARY PERIOD" AND [B] DOLE-DECS-CHED-TESDA ORDER NO. 01, S. 1996
WHICH PROVIDE THAT TEACHERS WHO HAVE SERVED THE PROBATIONARY PERIOD "SHALL BE MADE
REGULAR OR PERMANENT IF ALLOWED TO WORK AFTER SUCH PROBATIONARY PERIOD?" 

Ruling

Cases dealing with employment on probationary status of teaching personnel are not governed solely by the Labor
Code as the law is supplemented, with respect to the period of probation, by special rules found in the Manual of
Regulations for Private Schools (the Manual). With regard to the probationary period, Section 92 of the 1992
Manual  provides:

Section 92.Probationary Period. — Subject in all instances to compliance with the Department and school
requirements, the probationary period for academic personnel shall not be more than three (3) consecutive
years of satisfactory service for those in the elementary and secondary levels, six (6) consecutive regular
semesters of satisfactory service for those in the tertiary level, and nine (9) consecutive trimesters of satisfactory
service for those in the tertiary level where collegiate courses are offered on a trimester basis. (Emphasis supplied)

In this case, petitioners' teachers who were on probationary employment were made to enter into a contract effective
for one school year. Thereafter, it may be renewed for another school year, and the probationary employment
continues. At the end of the second fixed period of probationary employment, the contract may again be renewed for
the last time.

Such employment for fixed terms during the teachers' probationary period is an accepted practice in the teaching
profession
However, this scheme "of fixed-term contract is a system that operates during the probationary period and for this
reason is subject to Article 281 of the Labor Code,"  which provides:

. . . The services of an employee who has been engaged on a probationary basis may be terminated  for a just
cause or when he fails to qualify as a regular employee in accordance with reasonable standards made
known by the employer to the employee at the time of his engagement. An employee who is allowed to work
after a probationary period shall be considered a regular employee. [Emphasis supplied]

That teachers on probationary employment also enjoy the protection afforded by Article 281 of the Labor Code is
supported by Section 93 of the 1992 Manual which provides: 
Sec. 93.Regular or Permanent Status. — Those who have served the probationary period shall be made regular or
permanent. Full-time teachers who have satisfactorily completed their probationary period shall be
considered regular or permanent. (Emphasis supplied)

The above provision clearly provides that full-time teachers become regular or permanent employees once they
have satisfactorily  completed the probationary period of three school years. 37 The use of the
term satisfactorily necessarily connotes the requirement for schools to set reasonable standards to be followed by
teachers on probationary employment. For how else can one determine if probationary teachers have satisfactorily
completed the probationary period if standards therefore are not provided? 

As such, "no vested right to a permanent appointment shall accrue until the employee has completed the prerequisite
three-year period necessary for the acquisition of a permanent status. [However, it must be emphasized that] mere
rendition of service for three consecutive years does not automatically ripen into a permanent appointment. It is also
necessary that the employee be a full-time teacher, and that the services he rendered are satisfactory." 

In this case, glaringly absent from petitioners' evidence are the reasonable standards that respondent was expected
to meet that could have served as proper guidelines for purposes of evaluating his performance. Nowhere in the
Teacher's Contract could such standards be found. Neither was it mentioned that the same were ever conveyed to
respondent. Even assuming that respondent failed to meet the standards set forth by CSR and made known to the
former at the time he was engaged as a teacher on probationary status, still, the termination was flawed for failure to
give the required notice to respondent. This is because Book VI, Rule I, Section 2 of the IRR of the Labor Code
provides:
Section 2.Security of Tenure. — (a) In cases of regular employment, the employer shall not terminate the services of
an employee except for just or authorized causes as provided by law, and subject to the requirements of due
process.

(b)The foregoing shall also apply in cases of probationary employment; provided, however, that in such cases,
termination of employment due to failure of the employee to qualify in accordance with the standards of the employer
made known to the former at the time of engagement may also be a ground for termination of employment.

xxx xxx xxx


(d)In all cases of termination of employment, the following standards of due process shall be substantially observed:

xxx xxx xxx

If the termination is brought about by the completion of a contract or phase thereof, or by failure of an employee
to meet the standards of the employer in the case of probationary employment, it shall be sufficient that a
written notice is served the employee, within a reasonable time from the effective date of termination.
(Emphasis supplied) ATHCac

Curiously, despite the absence of standards, Mofada mentioned the existence of alleged performance
evaluations 47 in respondent's case. We are, however, in a quandary as to what could have been the basis of such
evaluation, as no evidence were adduced to show the reasonable standards with which respondent's performance
was to be assessed or that he was informed thereof. Notably too, none of the supposed performance evaluations
were presented. These flaws violated respondent's right to due process. As such, his dismissal is, for all intents and
purposes, illegal.

As a matter of due process, teachers on probationary employment, just like all probationary employees, have the
right to know whether they have met the standards against which their performance was evaluated. Should they fail,
they also have the right to know the reasons therefore.

It should be pointed out that absent any showing of unsatisfactory performance on the part of respondent, it can be
presumed that his performance was satisfactory, especially taking into consideration the fact that even while he was
still more than a year into his probationary employment, he was already designated Prefect of Discipline. In such
capacity, he was able to uncover the existence of a drug syndicate within the school and lessen the incidence of drug
use therein. Yet despite respondent's substantial contribution to the school, petitioners chose to disregard the same
and instead terminated his services; while most of those who were involved in drug activities within the school were
punished with a slap on the wrist as they were merely made to write letters promising that the incident will not happen
again. 

Mofada would also have us believe that respondent chose to resign as he feared for his life, thus, the school's
decision not to renew his contract. However, no resignation letter was presented. Besides, this is contrary to
respondent's act of immediately filing the instant case against petitioners.
27. Philippine Spring Water Resources Inc vs. Court of Appeals, GR No. 205278, June 11, 2014

Facts:
Mahilum was hired by petitioner as Vice President for Sales and Marketing. Sometime in Nov. 2004, the inauguration
of the company's Bulacan plant was to be celebrated, and he was designated to oversee the affair. However on the
inaugural day, he was not able to supervise the program due to tending to some visitors of the company, and
according to him, he delegated the task to Evangelista, the VP of Admin and Finance.
Mahilum's attention was called however when the CEO became furious because his presence was not acknowledged
nor was he called to deliver his speech during the event. According to the MCs Lua's speech was labelled optional in
the program because Lua declined participation in the program due to being busy. Mahilum was placed on preventive
suspension the following day. An investigation was conducted. Upon expiration of the suspension, he tried to go back
to work but was prevented from entering. He received a Memorandum terminating his services, and was made to
sign a Quitclaim after receiving an amount. He then filed for illegal dismissal.

Ruling:

Mahilum was a regular employee.


Based on the agreement of the parties, he was to be on probationary status for 6 months and may be extended
permanent appointment if he could satisfactorily perform his duties. It was the theory of the company that the
probationary status of Mahilum did not end for failing to perform his duties. Mahilum insists that he was a regular
employee since at the time of termination he had been serving for 8 months. Pursuant to Article 281 of the Labor
Code, "an employee who is allowed to work after the probationary period shall be considered a regular employee."
Mahilum was therefore a regular employee.
A probationary employee, like a regular employee, enjoys security of tenure. In cases of probationary employment,
however, aside from just or authorized causes of termination, an additional ground is provided under Article 281 of
the Labor Code, that is, the probationary employee may also be terminated for failure to qualify as a regular
employee in accordance with reasonable standards made known by the employer to the employee at the time of the
engagement. Thus, the services of an employee who has been engaged on probationary basis may be terminated for
any of the following: (1) a just or (2) an authorized cause and (3) when he fails to qualify as a regular employee in
accordance with reasonable standards prescribed by the employer.

Mahilum was illegally dismissed

According to the petitioners, Mahilum’s behavior during the inauguration/party was allegedly tantamount to: 1] serious
misconduct, as displayed by a drinking binge with his own visitors causing the shame and humiliation of Lua; and 2]
willful disobedience, as shown by his refusal to carry out legitimate orders. His designation as the chairman of the
whole affair did not form part of his duty as a supervisor. Mahilum was engaged to supervise the sales and marketing
aspects of PSWRI’s Bulacan Plant. Verily, the charge of loss of trust and confidence had no leg to stand on, as the
act complained of was not work-related.
Accordingly, Philippine Spring Water Resources Inc. is hereby ordered to pay Juvenstein B. Mahilum, his separation
pay, full backwages inclusive of his basic salary, proportionate 13th month pay, and unused leave credits, to be
computed based on his salary at the time of his illegal termination and attorney’s fees.
28. Concrete Solutions Inc. et al., vs. Cabusas, G.R. No. 177812, June 19, 2013

Facts:
Respondent Arthur Cabusas (respondent) was hired by petitioner Primary Structures Corporation (PSC) as transit
mixer driver for petitioner Concrete Solutions Inc. (CSI) – Batching Plant Project. The appointment letter3 dated June
27, 2000, which was signed by petitioner PSC's Human Resource Division Assistant with respondent's conformity,
provided, among others: that respondent was hired for the period from June 28, 2000 until June 23, 2001; the status
of his employment was that of a project employee and, as such, his employment was co-terminus with the completion
of the project or any phase thereof; that upon completion of the particular project or phase, he was free to seek other
employment of his choice; and, that within the duration of the work, petitioners shall have the right to terminate his
employment without any liability on their part if his performance did not meet the company standards, or if he violated
petitioners' rules and regulations.

On February 16, 2001, a report reached petitioners that at around 5 o’clock in the afternoon of that day, respondent,
as the driver of Transit Mixer 13, unloaded less than a cubic meter of concrete mix at Cabancalan, Mandaue City,
more than two kilometers away from its project site located at Wireless, Mandaue City, instead of returning the
excess concrete mix to the plant; and that respondent sold the excess concrete mix to the residents of the place
where he unloaded the same.

Petitioners' Manager required respondent to explain in writing why he should not be meted with a disciplinary action
for the alleged act of theft or dishonesty under the company’s Code of Conduct and Discipline. In his explanation,
respondent stated that he threw away the concrete mix at Cabancalan, Mandaue City, instead of turning them over to
the plant as he will wash the transit mixer at A.S. Fortuna, Mandaue City. Respondent was meted a three (3)-day
suspension effective March 20, 2001 to March 22, 2001.

While petitioners were deliberating on the violation committed by respondent, they went over the latter's 201 file and
discovered that he appeared not to be registered with the Social Security System as the SSS number he submitted
was that of another person in the name of Alex Cabusas. Thus, petitioners needed clarifications from respondent, but
the latter had been absent since May 6, 2001. On May 25, 2001, petitioners sent respondent a telegram,14 to wit:
"You have been absent without official leave since May [6], 2001. Please notify CSI as soon as possible." On June
12, 2001, petitioners, thru Manager Ardiente, sent respondent a termination letter. Earlier, however, on May 30, 2001,
respondent had filed with Regional Arbitration Branch No. VII of Cebu City a Complaint for unfair labor practice, illegal
dismissal, non-payment of holiday pay, premium pay for holiday, rest day, night shift premium, separation pay and
moral damages against petitioners.

Ruling:

1st issue: Whether respondent deliberately abandoned his work which is a just cause for his dismissal or
whether he was illegally dismissed by petitioners.
In this case, petitioners claim that respondent was validly dismissed as he abandoned his work. To constitute
abandonment, two elements must concur, to wit: (1) the failure to report for work or absence without valid or
justifiable reason; and (2) a clear intention to sever the employer-employee relationship, with the second element as
the more determinative factor and being manifested by some overt acts. Abandonment is a matter of intention and
cannot lightly be presumed from certain equivocal acts. To be a valid cause for dismissal for abandonment, there
must be clear proof of deliberate and unjustified intent to sever the employer-employee relationship. Clearly, the
operative act is still the employee's ultimate act of putting an end to his employment.
Respondent explained that his absence from work was due to the fact that he and his counsel had asked and were
waiting for a copy of result of the investigation on his alleged act of theft or dishonesty conducted on May 4, 2001 but
were not given at all. We find his absence from work not sufficient to establish that he already had intention of
abandoning his job. Besides, settled is the rule that mere absence or failure to report for work is not tantamount to
abandonment of work.
Petitioners' claim that respondent's violations of company rules also warranted his termination on account of loss of
trust and confidence deserves scant consideration since the latter's dismissal was not due to those alleged dishonest
acts but due to abandonment.

2nd Issue: whether or not he was a project employee.

Petitioners contend that respondent was a project employee and the project to which he was hired was already
completed, thus he could not be reinstated anymore.

Project employee is one whose employment has been fixed for a specific project or undertaking the completion or
termination of which has been determined at the time of the engagement of the employee or where the work or
services to be performed is seasonal in nature and the employment is for the duration of the season. We held that the
length of service of a project employee is not the controlling test of employment tenure but whether or not the
employment has been fixed for a specific project or undertaking the completion or termination of which has been
determined at the time of the engagement of the employee.
We rule that respondent is a project employee. His appointment letter showed that he was hired as transit mixer
driver for the Concrete Solutions Inc. (CSI) – Batching Plant Project for the period from June 28, 2000 until June 23,
2001. The same letter provided that he was a project employee whose employment was co-terminus with the
completion of the project or any phase thereof and upon completion of the particular project or phase, he was free to
seek other employment of his choice. There is no evidence showing that respondent did not sign the conforme part of
the appointment letter voluntarily. Hence, respondent was bound by the provisions in the appointment letter.
Moreover, there is also no showing that the period fixed in the appointment letter was imposed to preclude acquisition
of tenurial security by the employee and should be disregarded for being contrary to public policy as ruled by the
NLRC since no evidence exists on the record to support such conclusion.

Considering that respondent was dismissed prior to the expiration of the duration of his employment and without a
valid or just cause, his termination was therefore illegal. However, respondent could no longer be reinstated since the
project he was assigned to was already completely finished. However, we find that he is entitled to the salary
corresponding to the unexpired portion of his employment. Respondent is entitled to the payment of his salary from
the time he was not admitted back to work on May 26, 2001 up to June 23, 2001, the expiration of his employment
contract.
29. D.M. Consunji vs. Bello, G.R. No. 159371, July 29, 2013

FACTS:

Bello brought a complaint for illegal dismissal and damages against DMCI. Bello claimed that DMCI had employed
him as a mason without any interruption from February 1, 1990 until October 10, 1997; that his job as a mason had
been necessary and desirable in the usual business or trade of DMCI; that he had been diagnosed to be suffering
from pulmonary tuberculosis, thereby necessitating his leave of absence; that upon his recovery, he had reported
back to work, but DMCI had refused to accept him and had instead handed to him a termination paper effective
November 5, 1997.

DMCI contended that Bello had only been a project employee, as borne out by his contract of employment and
appointment papers; and that although his last project employment contract had been set to expire on October 7,
1997, he had tendered his voluntary resignation on October 4, 1997 for health reasons that had rendered him
incapable of performing his job, per his resignation letter.

ISSUE:

WHETHER OR NOT BELLO WAS A REGULAR EMPLOYEE OF D.M. CONSUNJI

RULING:

A project employee is one who is hired for a specific project or undertaking, and the completion or termination of such
project or undertaking has been determined at the time of engagement of the employee. In the context of the law,
Bello was a project employee of DMCI at the beginning of their employer-employee relationship. The project
employment contract they then entered into clearly gave notice to him at the time of his engagement about his
employment being for a specific project or phase of work. He was also thereby notified of the duration of the project,
and the determinable completion date of the project.

However, the history of Bello’s appointment and employment showed that he performed his tasks as a mason in
DMCI’s various constructions projects. Based on the foregoing, we affirm the CA’s conclusion that Bello acquired in
time the status of a regular employee by virtue of his continuous work as a mason of DMCI. The work of a mason
like him – a skilled workman working with stone or similar material – was really related to building or constructing, and
was undoubtedly a function necessary and desirable to the business or trade of one engaged in the construction
industry like DMCI. His being hired as a mason by DMCI in not one, but several of its projects revealed his necessity
and desirability to its construction business.

It is settled that the extension of the employment of a project employee long after the supposed project has been
completed removes the employee from the scope of a project employee and makes him a regular employee. In this
regard, the length of time of the employee’s service, while not a controlling determinant of project employment, is a
strong factor in determining whether he was hired for a specific undertaking or in fact tasked to perform functions
vital, necessary and indispensable to the usual business or trade of the employer. On the other hand, how DMCI
chose to categorize the employment status of Bello was not decisive of his employment status. What were of
consequence in that respect were his actual functions and the length of his stay with DMCI. Verily, the principal test
for determining whether an employee is a project employee, as distinguished from a regular employee, is
whether or not he is assigned to carry out a specific project or undertaking, the duration and scope of which are
specified at the time he is engaged for the project.

Still, DMCI contends that Bello’s services as a mason were deemed necessary and desirable in its usual business
only for the period of time it had taken it to complete the project. The contention may be correct if each engagement
of Bello as a mason over the span of eight years was to be treated separately. The contention cannot be upheld,
however, simply because his successive reengagement in order to perform the same kind of work as a mason
firmly manifested the necessity and desirability of his work in DMCI’s usual business of construction.
30. Colegio Del Santisimo Rosario et al., vs. Rojo, G.R. No. 170388, Sept. 4, 2013

FACTS:

Colegio del Santisimo Rosario (CSR) hired Rojo as a high school teacher on probationary basis for the school years
1992-1993, 1993-1994 and 1994-1995. On April 5, 1995, CSR, through Sr. Zenaida S. Mofada, OP (Mofada),
decided not to renew respondent’s services.

Rojo filed a Complaint for illegal dismissal. He alleged that since he had served three consecutive school years which
is the maximum number of terms allowed for probationary employment, he should be extended permanent
employment. Citing paragraph 75 of the 1970 Manual of Regulations for Private Schools (1970 Manual), Rojo
asserted that "full- time teachers who have rendered three (3) consecutive years of satisfactory services shall be
considered permanent."

On the other hand, CSR argued that respondent knew that his Teacher’s Contract for school year 1994-1995 with
CSR would expire on March 31, 1995. Accordingly, Rojo was not dismissed but his probationary contract merely
expired and was not renewed. Petitioners maintain that upon the expiration of the probationary period, both the
school and the respondent were free to renew the contract or let it lapse. Petitioners insist that a teacher hired for
three consecutive years as a probationary employee does not automatically become a regular employee upon
completion of his third year of probation. It is the positive act of the school – the hiring of the teacher who has just
completed three consecutive years of employment on probation for the next school year – that makes the teacher a
regular employee of the school.

ISSUE:

WHETHER OR NOT ROJO BECAME A PERMANENT EMPLOYEE UPON COMPLETION OF HIS THIRD YEAR
PROBATION.

RULING:

That teachers on probationary employment also enjoy the protection afforded by Article 281 of the Labor Code is
supported by Section 93 of the 1992 Manual which provides:

Sec. 93. Regular or Permanent Status. - Those who have served the probationary period shall be made regular or
permanent. Full-time teachers who have satisfactorily completed their probationary period shall be considered regular
or permanent. (Emphasis supplied)

The above provision clearly provides that full-time teachers become regular or permanent employees once
they have satisfactorily completed the probationary period of three school years. The use of the term
satisfactorily necessarily connotes the requirement for schools to set reasonable standards to be followed by
teachers on probationary employment. For how else can one determine if probationary teachers have satisfactorily
completed the probationary period if standards therefor are not provided?

As such, "no vested right to a permanent appointment shall accrue until the employee has completed the prerequisite
three-year period necessary for the acquisition of a permanent status. [However, it must be emphasized that] mere
rendition of service for three consecutive years does not automatically ripen into a permanent appointment.
It is also necessary that the employee be a full-time teacher, and that the services he rendered are
satisfactory."

In Mercado, this Court, speaking through J. Brion, held that:

xxx

When fixed-term employment is brought into play under the above probationary period rules, the situation – as in the
present case – may at first blush look muddled as fixed-term employment is in itself a valid employment mode under
Philippine law and jurisprudence. The conflict, however, is more apparent than real when the respective nature of
fixed-term employment and of employment on probationary status are closely examined.

The fixed-term character of employment essentially refers to the period agreed upon between the employer and the
employee; employment exists only for the duration of the term and ends on its own when the term expires. In a
sense, employment on probationary status also refers to a period because of the technical meaning "probation"
carries in Philippine labor law – a maximum period of six months, or in the academe, a period of three years for those
engaged in teaching jobs. Their similarity ends there, however, because of the overriding meaning that being
"on probation" connotes, i.e., a process of testing and observing the character or abilities of a person who is
new to a role or job.

Understood in the above sense, the essentially protective character of probationary status for management can
readily be appreciated. But this same protective character gives rise to the countervailing but equally protective rule
that the probationary period can only last for a specific maximum period and under reasonable, well-laid and properly
communicated standards. Otherwise stated, within the period of the probation, any employer move based on the
probationary standards and affecting the continuity of the employment must strictly conform to the probationary
rules.

x x x If we pierce the veil, so to speak, of the parties’ so-called fixed-term employment contracts, what undeniably
comes out at the core is a fixed-term contract conveniently used by the school to define and regulate its relations with
its teachers during their probationary period.

In the same case, this Court has definitively pronounced that "in a situation where the probationary status overlaps
with a fixed-term contract not specifically used for the fixed term it offers, Article 281 should assume primacy and the
fixed-period character of the contract must give way."

An example given of a fixed-term contract specifically used for the fixed term it offers is a replacement teacher or a
reliever contracted for a period of one year to temporarily take the place of a permanent teacher who is on leave. The
expiration of the reliever’s fixed-term contract does not have probationary status implications as he or she was never
employed on probationary basis. This is because his or her employment is for a specific purpose with particular focus
on the term. There exists an intent to end his or her employment with the school upon expiration of this term.

However, for teachers on probationary employment, in which case a fixed term contract is not specifically used for the
fixed term it offers, it is incumbent upon the school to have not only set reasonable standards to be followed by said
teachers in determining qualification for regular employment, the same must have also been communicated to the
teachers at the start of the probationary period, or at the very least, at the start of the period when they were to be
applied. These terms, in addition to those expressly provided by the Labor Code, would serve as the just cause for
the termination of the probationary contract. The specific details of this finding of just cause must be communicated to
the affected teachers as a matter of due process. Corollarily, should the teachers not have been apprised of
such reasonable standards at the time specified above, they shall be deemed regular employees.

In Tamson’s Enterprises, Inc. v. Court of Appeals,we held that "[t]he law is clear that in all cases of probationary
employment, the employer shall [convey] to the employee the standards under which he will qualify as a regular
employee at the time of his engagement. Where no standards are made known to the employee at that time, he shall
be deemed a regular employee.

In this case, glaringly absent from petitioners’ evidence are the reasonable standards that respondent was expected
to meet that could have served as proper guidelines for purposes of evaluating his performance. Nowhere in the
Teacher’s Contract could such standards be found. Neither was it mentioned that the same were ever conveyed to
respondent.
31. Herrera-Manaois vs. St. Scholasticas College, GR No. 188914, December 11, 2013

Facts:

SSC, situated in the City of Manila, is a private educational institution offering elementary, secondary, and tertiary
education. Manaois graduated from SSC in October 1992 with a degree in Bachelor of Arts in English. In 1994, she
returned to her alma mater as a part-time English teacher. After taking a leave of absence for one year, she was
again rehired by SSC for the same position. Four years into the service, she was later on recommended by her
Department Chairperson to become a full-time faculty member of the English Department.

Manaois thus applied for a position as full-time instructor for school year 2000-2001.
She mentioned in her application letter that she had been taking the course Master of Arts in English Studies, Major
in Creative Writing, at the University of the Philippines, Diliman (UP); that she was completing her master's thesis;
and that her oral defense was scheduled for June 2000. In a reply letter 4 dated 17 April 2000, the Dean of Arts and
Sciences informed her of the SSC Administrative Council's approval of her application. SSC hired her as a
probationary full-time faculty member with the assigned task of instructor for the school year 2000-2001. Her
probationary employment continued for a total of three consecutive years.
Because of the forthcoming completion of her third year of probationary employment, Manaois wrote the Dean of Arts
and Sciences requesting an extension of her teaching load for the school year 2003-2004. Manaois eventually
received a letter from the Dean of College and Chairperson of the Promotions and Permanency Board officially
informing her of the board's decision not to renew her contract.

Issue:

Whether the completion of a master's degree is required in order for a tertiary level educator to earn the status of
permanency in a private educational institution.

Ruling:

Probationary employment refers to the trial stage or period during which the employer examines the competency and
qualifications of job applicants, and determines whether they are qualified to be extended permanent employment
status. Such an arrangement affords an employer the opportunity — before the full force of the guarantee of security
of tenure comes into play — to fully scrutinize and observe the fitness and worth of probationers while on the job and
to determine whether they would become proper and efficient employees. It also gives the probationers the chance to
prove to the employer that they possess the necessary qualities and qualifications to meet reasonable standards for
permanent employment.

Viewed next to the statements and actions of Manaois — i.e., the references to obtaining a master's degree in her
application letter, in the subsequent correspondences between her and SSC, and in the letter seeking the extension
of a teaching load for the school year 2003-2004; and her submission of certifications from UP and from her thesis
adviser — we find that there is indeed substantial evidence proving that she knew about the necessary academic
qualifications to obtain the status of permanency.

At this juncture, we reiterate the rule that mere completion of the three-year probation, even with an above-average
performance, does not guarantee that the employee will automatically acquire a permanent employment status. It is
settled jurisprudence that the probationer can only qualify upon fulfillment of the reasonable standards set for
permanent employment as a member of the teaching personnel.

Thus, pursuant to the 1992 Manual, private educational institutions in the tertiary level may extend "full-time faculty"
status only to those who possess, inter alia, a master's degree in the field of study that will be taught. This minimum
requirement is neither subject to the prerogative of the school nor to the agreement between the parties. For all
intents and purposes, this qualification must be deemed impliedly written in the employment contracts between
private educational institutions and prospective faculty members. The issue of whether probationers were informed of
this academic requirement before they were engaged as probationary employees is thus no longer material, as those
who are seeking to be educators are presumed to know these mandated qualifications. In the light of the failure of
Manaois to satisfy the academic requirements for the position, she may only be considered as a part-time instructor
pursuant to Section 45 of the 1992 Manual.
32. Universal Robina Sugar Milling Corp., vs. Acibo et al., GR No. 186439, January 15, 2014

Facts:

URSUMCO is a domestic corporation engaged in the sugar cane milling business; Cabati is URSUMCO's Business
Unit General Manager. The complainants were employees of URSUMCO. They were hired on various dates
(between February 1988 and April 1996) and on different capacities, 8 i.e., drivers, crane operators, bucket hookers,
welders, mechanics, laboratory attendants and aides, steel workers, laborers, carpenters and masons, among others.
At the start of their respective engagements, the complainants signed contracts of employment for a period of one (1)
month or for a given season. URSUMCO repeatedly hired the complainants to perform the same duties and, for every
engagement, required the latter to sign new employment contracts for the same duration of one month or a given
season.
The complainants filed before the LA complaints for regularization, entitlement to the benefits under the existing
Collective Bargaining Agreement (CBA), and attorney's fees.

Issue:

Whether or not respondents are regular employees of URSUMCO

Ruling:

We find the respondents to be regular seasonal employees of URSUMCO.


Seasonal employment operates much in the same way as project employment, albeit it involves work or service that
is seasonal in nature or lasting for the duration of the season. As with project employment, although the seasonal
employment arrangement involves work that is seasonal or periodic in nature, the employment itself is not
automatically considered seasonal so as to prevent the employee from attaining regular status. To exclude the
asserted "seasonal" employee from those classified as regular employees, the employer must show that: (1) the
employee must be performing work or services that are seasonal in nature; and (2) he had been employed for the
duration of the season. Hence, when the "seasonal" workers are continuously and repeatedly hired to perform the
same tasks or activities for several seasons or even after the cessation of the season, this length of time may likewise
serve as badge of regular employment. In fact, even though denominated as "seasonal workers," if these workers are
called to work from time to time and are only temporarily laid off during the off-season, the law does not consider
them separated from the service during the off-season period. The law simply considers these seasonal workers on
leave until re-employed.

The nature of the employment depends on the nature of the activities to be performed by the employee, considering
the nature of the employer's business, the duration and scope to be done, and, in some cases, even the length of
time of the performance and its continued existence. In light of the above legal parameters laid down by the law and
applicable jurisprudence, the respondents are neither project, seasonal nor fixed-term employees, but regular
seasonal workers of URSUMCO.

(1) The respondents were made to perform various tasks that did not at all pertain to any specific phase of
URSUMCO's strict milling operations that would ultimately cease upon completion of a particular phase in
the milling of sugar; rather, they were tasked to perform duties regularly and habitually needed in
URSUMCO's operations during the milling season.
(2) The respondents were regularly and repeatedly hired to perform the same tasks year after year.
33. DIONARTO Q. NOBLEJAS,  v. ITALIAN MARITIME ACADEMY PHILS., INC., CAPT. NICOLO S. TERREI,
RACELI B. FERREZ AND MA. TERESA R. MENDOZA. G.R. No. 207888, June 09, 2014.

FACTS:

Petitioner Noblejas filed a complaint for illegal dismissal, tax refund, moral and exemplary damages, non-payment of
13th month pay, food, gasoline and schooling allowances, health insurance, monetized leave, and attorney’s fees,
against Italian Maritime Academy Phils., Inc. (IMAPI) and its officers.

Record shows that Procerfina SA. Terrei, IMAPI President, wrote a letter  to Noblejas informing him that he had been
appointed as training instructor/assessor of the company on a contractual basis for a period of three (3) months.
After the expiration of the 3-month period, IMAPI hired Noblejas anew as training instructor/assessor with the same
salary rate, but no written contract was drawn for his rehiring.

The absence of a written contract to cover the renewal of his employment became Noblejas’ major concern. To
address all his apprehensions, he wrote Capt. Terrei a letter requesting that a new contract be executed.

Noblejas averred that the company did not act on his letter-request, so he sought an audience with Capt. Terrei.
During the meeting, an altercation between them ensued. He claimed that after that incident, Capt. Terrei instructed
Ferrez, his secretary, to dismiss him from employment. He claimed that when he asked from Ferrez for a copy of his
old contract, she allegedly replied, “No, you better pack up all your things now and go, you are now dismissed and
you are no longer part in this office – clearly, you are terminated from this day on.”

Respondents submitted that they could not be adjudged guilty of illegal dismissal because there was no positive and
overt act of dismissing Noblejas from employment.

Respondents presented a different version of what took place. According to respondents, Noblejas got angry, hurled
invectives against Ferrez and even threatened to file a case against them after she had relayed to him the response
of Capt. Terrei to his letter to the effect that there was no previous agreement to grant him tax refund, health
insurance and food, schooling and gasoline allowances and that he had to render at least one year of service before
the company could decide whether to accord him the status of a regular employee. The following day, March 17,
2010, he did not report for work anymore and filed the complaint against them.

The Labor Arbiter found that Noblejas was illegally dismissed from his employment, and awarded him limited
backwages.
NLRC reversed the LA decision.

ISSUES:

1. WON petitioner is a contractual employee.


2. WON petitioner was illegally dismissed.
3. WON petitioner is entitled to his money claims.

RULING:
As to the first issue, petitioner Noblejas is a regular employee of IMAPI.

Pursuant to Article 280 of the Labor Code, there are two kinds of regular employees, namely: (1) those who are
engaged to perform activities which are usually necessary or desirable in the usual business or trade of the employer;
and (2) those who have rendered at least one year of service, whether continuous or broken, with respect to the
activities in which they are employed.

Regular employees are further classified into (1) regular employees - by nature of work and (2) regular employees -
by years of service. The former refers to those employees who perform a particular function which is necessary or
desirable in the usual business or trade of the employer, regardless of their length of service; while the latter refers to
those employees who have been performing the job, regardless of its nature thereof, for at least a year.

In the case at bench, Noblejas was employed by IMAPI as a training instructor/assessor for a period of three (3)
months effective May 20, 2009. After the end of the 3-month period, he was rehired by IMAPI for the same position
and continued to work as such until March 16, 2010. There is no dispute that the work of Noblejas was necessary or
desirable in the business or trade of IMAPI, a training and assessment center for seamen and officers of vessels.
Moreover, such continuing need for his services is sufficient evidence of the necessity and indispensability of his
services to IMAPI’s business. Taken in this light, Noblejas had indeed attained the status of a regular employee at the
time he ceased to report for work on March 17, 2010.
As to the second issue, petitioner was not illegally dismissed.
The rule in labor cases is that the employer has the burden of proving that the termination was for a valid or
authorized cause. It is likewise incumbent upon the employees, however, that they should first establish by
competent evidence the fact of their dismissal from employment.  It is an age-old rule that the one who alleges a fact
has the burden of proving it and the proof should be clear, positive and convincing.  Mere allegation is not evidence. 

Aside from his mere assertion, no corroborative and competent evidence was adduced by Noblejas to substantiate
his claim that he was dismissed from employment. The record is bereft of any indication that he was prevented from
returning to work or otherwise deprived of any work assignment. It is also noted that no evidence was submitted to
show that respondent Ferrez, the secretary of Capt. Terrei, was actually authorized by IMAPI to terminate the
employment of the company’s employees or that Ferrez was indeed instructed by Capt. Terrei to dismiss him from
employment.
Respondents’ refusal to grant complainant’s demands does not constitute an overt act of dismissal. On the contrary,
it is rather the apparent disinterest of complainant to continue his employment with respondent company that may be
considered a covert act that severed his employment when the latter did not grant the litany of his demands.
As to the third issue, the SC sustains the LA in granting Noblejas proportionate 13th month pay covering the period of
January 1, 2010 to March 15, 2010 in the aggregate amount of P15,625.00.
Furthermore, the respondents should accept him back and reinstate him to his former position.  However, there can
be no payment of backwages under the principle of “no work, no pay.”
34. OMNI HAULING SERVICES, INC., v. BERNARDO BON, ROBERTO TORTOLES, et al. G.R. No. 199388,
September 03, 2014

FACTS:

Petitioner Omni was awarded a one (1) year service contract by the local government of Quezon City to provide
garbage hauling services for the period July 1, 2002 to June 30, 2003. For this purpose, Omni hired respondents as
garbage truck drivers and paleros who were then paid on a per trip basis.

When the service contract was renewed for another year, or for the period July 1, 2003 to June 30, 2004, petitioners
required each of the respondents to sign employment contracts which provided that they will be “re-hired” only for the
duration of the same period. However, respondents refused to sign the employment contracts, claiming that they
were regular employees since they were engaged to perform activities which were necessary and desirable to Omni’s
usual business or trade.

For this reason, Omni terminated the employment of respondents which, in turn, resulted in the filing of cases for
illegal dismissal, nonpayment of Emergency Cost of Living Allowance (ECOLA) and 13th month pay, and actual,
moral, and exemplary damages.

LA ruled in favor of petitioners, finding that respondents were not illegally dismissed.
The LA found that respondents were informed that their employment will be limited for a specific period of one year
and was co-terminus with the service contract with the Quezon City government. Thus, respondents were not regular
but merely project employees whose hiring was solely dependent on the aforesaid service contract.

The NLRC affirmed in toto the decision of the Labor Arbiter.


The Court of Appeals reversed and set aside the decision of the NLRC and held that the respondents are regular
employees and that NLRC failed to consider the glaring fact that no contract of employment exists to support
petitioners’ allegation that respondents are fixed-term or project employees.

ISSUES:

WON the respondents are project employees.

RULING:

The respondents are not project employees. They are regular employees.
Article 280 of the Labor Code distinguishes a “project employee” from a “regular employee” in this wise:
Art. 280. Regular and casual employment. The provisions of written agreement to the contrary notwithstanding and
regardless of the oral agreement of the parties, an employment shall be deemed to be regular where the employee
has been engaged to perform activities which are usually necessary or desirable in the usual business or trade of the
employer, except where the employment has been fixed for a specificproject or undertaking the completion or
termination of which has been determined at the time of the engagement of the employee or where the work or
service to be performed is seasonal in nature and the employment is for the duration of the season.

A project employee is assigned to a project which begins and ends at determined or determinable times.  Unlike
regular employees who may only be dismissed for just and/or authorized causes under the Labor Code, the services
of employees who are hired as “project employees” may be lawfully terminated at the completion of the project.

According to jurisprudence, the principal test for determining whether particular employees are properly characterized
as “project employees” as distinguished from “regular employees,” is whether or not the employees were assigned to
carry out a “specific project or undertaking,” the duration (and scope) of which were specified at the time they were
engaged for that project. The project could either be (1) a particular job or undertaking that is within the regular or
usual business of the employer company, but which is distinct and separate, and identifiable as such, from the other
undertakings of the company; or (2) a particular job or undertaking that is not within the regular business of the
corporation.  In order to safeguard the rights of workers against the arbitrary use of the word “project” to prevent
employees from attaining a regular status, employers claiming that their workers are project employees should
not only prove that the duration and scope of the employment was specified at the time they were engaged,
but also that there was indeed a project.cr

Hence, even though the absence of a written contract does not by itself grant regular status to respondents, such a
contract is evidence that respondents were informed of the duration and scope of their work and their status as
project employees. In this case, where no other evidence was offered, the absence of an employment contract
puts into serious question whether the employees were properly informed at the onset of their employment
status as project employees. It is doctrinally entrenched that in illegal dismissal cases, the employer has the burden
of proving with clear, accurate, consistent and convincing evidence that a dismissal was valid.
In this case, records are bereft of any evidence to show that respondents were made to sign employment contracts
explicitly stating that they were going to be hired as project employees, with the period of their employment to be co-
terminus with the original period of Omni’s service contract with the Quezon City government. Neither is petitioners’
allegation that respondents were duly apprised of the project-based nature of their employment supported by any
other evidentiary proof.Thus, the logical conclusion is that respondents were not clearly and knowingly informed of
their employment status as mere project employees, with the duration and scope of the project specified at the time
they were engaged. As such, the presumption of regular employment should be accorded in their favor pursuant to
Article 280 of the Labor Code which provides that “[employees] who have rendered at least one year of service,
whether such service is continuous or broken [– as respondents in this case –] shall be considered as [regular
employees] with respect to the activity in which [they] are employed and [their] employment shall continue while such
activity actually exists.” Add to this the obvious fact that respondents have been engaged to perform activities which
are usually necessary or desirable in the usual business or trade of Omni, i.e., garbage hauling, thereby confirming
the strength of the aforesaid conclusion.

The determination that respondents are regular and not merely project employees resultantly means that their
services could not have been validly terminated at the expiration of the project, or, in this case, the service contract of
Omni with the Quezon City government. As regular employees, it is incumbent upon petitioners to establish that
respondents had been dismissed for a just and/or authorized cause. However, petitioners failed in this respect;
hence, respondents were illegally dismissed.
35. Hacienda Ledd vs. Villegas, GR No. 179654, Sept. 22, 2014

Facts:
Villegas is an employee at the Hacienda Leddy as early as 1960, when it was still named Hacienda Teresa. Later on
named Hacienda Leddy owned by Ricardo Gamboa Sr., the same was succeeded by his son Ricardo Gamboa, Jr.
During his employment up to the time of his dismissal, Villegas performed sugar farming job 8 hours a day, 6 days a
week work, continuously for not less than 302 days a year, and for which services he was paid P45.00 per day. He
likewise worked in petitioner's coconut lumber business where he was paid P34.00 a day for 8 hours work.

On June 9, 1993, Gamboa went to Villegas' house and told him that his services were no longer needed without prior
notice or valid reason. Hence, Villegas filed the instant complaint for illegal dismissal.

Gamboa, on the other hand, denied having dismissed Villegas but admitted in his earlier position paper that Villegas
indeed worked with the said farm owned by his father, doing casual and odd jobs until the latter's death in 1993.  He
was even given the benefit of occupying a small portion of the land where his house was erected. He, however,
maintained that Villegas ceased working at the farm as early as 1992, contrary to his allegation that he was
dismissed. 

However, later, Gamboa apparently retracted and instead insisted that the farm records reveal that the only time
Villegas rendered service for the hacienda was only in the year 1993, specifically February 9, 1993 and February 11,
1993 when he was contracted by the farm to cut coconut lumber which were given to regular workers for the repairs
of their houses.  Gamboa added that they informed Villegas that they need the property, hence, they requested that
he vacate it, but he refused. Thus, Gamboa surmised that Villegas filed the instant complaint to gain leverage so he
would not be evicted from the land he is occupying. He further argued that during his employment, Villegas was paid
in accordance with the rate mandated by law and that his claim for illegal dismissal was merely a fabrication as he
was the one who opted not to work.

Issue

1. Whether or nor there was an ER-EE relationship.


2. Whether or not there was illegal dismissal.

Ruling

A perusal of the records would show that respondent, having been employed in the subject Hacienda while the same
was still being managed by petitioner's father until the latter's death in 1993, is undisputed as the same was even
admitted by Gamboa in his earlier pleadings.  While refuting that Villegas was a regular employee, petitioner however
failed to categorically deny that Villegas was indeed employed in their hacienda albeit he insisted that Villegas was
merely a casual employee doing odd jobs.

The rule is long and well settled that, in illegal dismissal cases like the one at bench, the burden of proof is upon the
employer to show that the employee's termination from service is for a just and valid cause. The employer's case
succeeds or fails on the strength of its evidence and not the weakness of that adduced by the employee, in keeping
with the principle that the scales of justice should be tilted in favor of the latter in case of doubt in the evidence
presented by them. Often described as more than a mere scintilla, the quantum of proof is substantial evidence which
is understood as such relevant evidence as a reasonable mind might accept as adequate to support a conclusion,
even if other equally reasonable minds might conceivably opine otherwise. 

In the instant case, if we are to follow the length of time that Villegas had worked with the Gamboas, it should be
more than 20 years of service. Even Gamboa admitted that by act of generosity and compassion, Villegas was given
a privilege of erecting his house inside the hacienda during his employment.  While it may indeed be an act of good
will on the part of the Gamboas, still, such act is usually done by the employer either out of gratitude for the
employee's service or for the employer's convenience as the nature of the work calls for it. Indeed, petitioner's length
of service is an indication of the regularity of his employment. Even assuming that he was doing odd jobs around the
farm, such long period of doing said odd jobs is indicative that the same was either necessary or desirable to
petitioner's trade or business. Owing to the length of service alone, he became a regular employee, by operation of
law, one year after he was employed.

Article 280 of the Labor Code, describes a regular employee as one who is either (1) engaged to perform activities
which are necessary or desirable in the usual business or trade of the employer; and (2) those casual employees who
have rendered at least one year of service, whether continuous or broken, with respect to the activity in which he is
employed.
While length of time may not be the controlling test to determine if Villegas is indeed a regular employee, it is vital in
establishing if he was hired to perform tasks which are necessary and indispensable to the usual business or trade of
the employer. If it was true that Villegas worked in the hacienda only in the year 1993, specifically February 9, 1993
and February 11, 1993, why would then he be given the benefit to construct his house in the hacienda? More
significantly, petitioner admitted that Villegas had worked in the hacienda until his father's demise. Clearly, even
assuming that Villegas' employment was only for a specific duration, the fact that he was repeatedly re-hired over a
long period of time shows that his job is necessary and indispensable to the usual business or trade of the employer.

Gamboa likewise argued that Villegas was paid on a piece-rate basis.  However, payment on a piece-rate basis does
not negate regular employment. "The term 'wage' is broadly defined in Article 97 of the Labor Code as remuneration
or earnings, capable of being expressed in terms of money whether fixed or ascertained on a time, task, piece or
commission basis. Payment by the piece is just a method of compensation and does not define the essence of the
relations." 

We are likewise unconvinced that it was Villegas who suddenly stopped working. Considering that he was employed
with the Gamboas for more than 20 years and was even given a place to call his home, it does not make sense why
Villegas would suddenly stop working therein for no apparent reason. To justify a finding of abandonment of work,
there must be proof of a deliberate and unjustified refusal on the part of an employee to resume his employment. The
burden of proof is on the employer to show an unequivocal intent on the part of the employee to discontinue
employment. Mere absence is not sufficient. It must be accompanied by manifest acts unerringly pointing to the fact
that the employee simply does not want to work anymore. 

Petitioner failed to discharge this burden. Other than the self-serving declarations in the affidavit of his employee,
petitioner did not adduce proof of overt acts of Villegas showing his intention to abandon his work. Abandonment is a
matter of intention; it cannot be inferred or presumed from equivocal acts. On the contrary, the filing of the instant
illegal dismissal complaint negates any intention on his part to sever their employment relationship. The delay of
more than 1 year in filing the instant illegal dismissal case likewise is non-issue considering that the complaint was
filed within a reasonable period during the three-year period provided under Article 291 of the Labor Code.  As aptly
observed by the appellate court, Villegas appeared to be without educational attainment. He could not have known
that he has rights as a regular employee that is protected by law.

Thus, notwithstanding any agreements to the contrary, what determines whether a certain employment is regular or
casual is not the will and word of the employer, to which the desperate worker often accedes, much less the
procedure of hiring the employee or the manner of paying his salary. It is the nature of the activities performed in
relation to the particular business or trades considering all circumstances, and in some cases the length of time of its
performance and its continued existence. 

All these having discussed, as a regular worker, Villegas is entitled to security of tenure under Article 279 of the Labor
Code and can only be removed for cause. We found no valid cause attending to his dismissal and found also that his
dismissal was without due process.

Article 277 (b) of the Labor Code provides that:


. . . Subject to the constitutional right of workers to security of tenure and their right to be protected against dismissal
except for a just and authorized cause and without prejudice to the requirement of notice under Article 283 of this
Code, the employer shall furnish the worker whose employment is sought to be terminated a written notice containing
a statement of the causes for termination and shall afford the latter ample opportunity to be heard and to defend
himself with the assistance of his representative if he so desires in accordance with company rules and regulations
promulgated pursuant to guidelines set by the Department of Labor and Employment. . . .

The failure of the petitioner to comply with these procedural guidelines renders its dismissal of Villegas illegal. An
illegally dismissed employee should be entitled to either reinstatement — if viable, or separation pay if reinstatement
is no longer viable, plus backwages in either instance.  Considering that reinstatement is no longer feasible because
of strained relations between the employee and the employer, separation pay should be granted. The basis for
computing separation pay is usually the length of the employee's past service, while that for backwages is the actual
period when the employee was unlawfully prevented from working. 25 It should be emphasized, however, that the
finality of the illegal dismissal decision becomes the reckoning point. In allowing separation pay, the final decision
effectively declares that the employment relationship ended so that separation pay and backwages are to be
computed up to that point. The decision also becomes a judgment for money from which another consequence flows
— the payment of interest in case of delay. 
36. FVR Skills & Services Exponents Inc. et al., vs. Seva, et al., GR No. 200857, Oct. 22, 2014

Facts:
The twenty-eight (28) respondents in this case were employees of petitioner FVR Skills and Services Exponents,
Inc. (petitioner), an independent contractor engaged in the business of providing janitorial and other manpower
services to its clients. As early as 1998, some of the respondents had already been under the petitioner's
employ.

On April 21, 2008, the petitioner entered into a Contract of Janitorial Service  (service contract) with Robinsons Land
Corporation (Robinsons). Both agreed that the petitioner shall supply janitorial, manpower and sanitation services to
Robinsons Place Ermita Mall for a period of one year — from January 1, 2008 to December 31, 2008. Pursuant to
this, the respondents were deployed to Robinsons.

Halfway through the service contract, the petitioner asked the respondents to execute individual contracts which
stipulated that their respective employments shall end on December 31, 2008, unless earlier terminated. 

The petitioner and Robinsons no longer extended their contract of janitorial services. Consequently, the petitioner
dismissed the respondents as they were project employees whose duration of employment was dependent on the
petitioner's service contract with Robinsons. 

The respondents responded to the termination of their employment by filing a complaint for illegal dismissal with the
NLRC. They argued that they were not project employees; they were regular employees who may only be dismissed
for just or authorized causes. The respondents also asked for payment of their unpaid wage differential, 13th month
pay differential, service incentive leave pay, holiday pay and separation pay. 

Issues:

1. Whether or not the respondents regular or project employees.


2. Whether or not the employment contract they belatedly sign valid.
3. Whether or not there was valid dismissal.

Ruling:

The respondents are regular


employees, not project employees.

Article 280 (now Article 294)  of the Labor Code governs the determination of whether an employee is a regular or a
project employee. 

Under this provision, there are two kinds of regular employees, namely: (1) those who were engaged to perform
activities which are usually necessary or desirable in the usual business or trade of the employer; and (2) those
casual employees who became regular after one year of service, whether continuous or broken, but only with respect
to the activity for which they have been hired.

We distinguish these two types of regular employees from a project employee, or one whose employment was fixed
for a specific project or undertaking, whose completion or termination had been determined at the time of
engagement.
A careful look at the factual circumstances of this case leads us to the legal conclusion that the respondents are
regular and not project employees.

The primary standard in determining regular employment is the reasonable connection between the particular
activity performed by the employee and the employer's business or trade. This connection can be ascertained by
considering the nature of the work performed and its relation to the scheme of the particular business, or the trade in
its entirety. 25

Guided by this test, we conclude that the respondents' work as janitors, service crews and sanitation aides, are
necessary or desirable to the petitioner's business of providing janitorial and manpower services to its clients as
an independent contractor. 

Also, the respondents had already been working for the petitioner as early as 1998. Even before the service
contract with Robinsons, the respondents were already under the petitioner's employ. 26 They had been
doing the same type of work and occupying the same positions from the time they were hired and until they
were dismissed in January 2009. The petitioner did not present any evidence to refute the respondents' claim that
from the time of their hiring until the time of their dismissal, there was no gap in between the projects where they were
assigned to. The petitioner continuously availed of their services by constantly deploying them to its clients.

Lastly, under Department Order (DO) 18-02,  the applicable labor issuance to the petitioner's case, the contractor or
subcontractor is considered as the employer of the contractual employee for purposes of enforcing the provisions of
the Labor Code and other social legislation. 

DO 18-02 grants contractual employees all the rights and privileges due a regular employee, including the following:
(a) safe and healthful working conditions; (b) labor standards such as service incentive leave, rest days,
overtime pay, holiday pay, 13th month pay and separation pay; (c) social security and welfare benefits; (d) self-
organization, collective bargaining and peaceful concerted action; and (e) security of tenure. 

In this light, we thus conclude that although the respondents were assigned as contractual employees to the
petitioner's various clients, under the law, they remain to be the petitioner's regular employees, who are entitled to all
the rights and benefits of regular employment.

The respondents' employment


contracts, which were belatedly
signed, are voidable.

The records show that at the time of the respondents' dismissal, they had already been continuously working for the
petitioner for more than a year. Despite this, they never signed any employment contracts with the petitioner, except
the contracts they belatedly signed when the petitioner's own contract of janitorial services with Robinsons neared
expiration.

As already discussed, for an employee to be validly categorized as a project employee, it is necessary that the
specific project or undertaking had been identified and its period and completion date determined and made
known to the employee at the time of his engagement. This provision ensures that the employee is completely
apprised of the terms of his hiring and the corresponding rights and obligations arising from his undertaking. Notably,
the petitioner's service contract with Robinsons was from January 1 to December 31, 2008. The respondents were
only asked to sign their employment contracts for their deployment with Robinsons halfway through 2008, when the
petitioner's service contract was about to expire.

We find the timing of the execution of the respondents' respective employment contracts to be indicative of the
petitioner's calculated plan to evade the respondents' right to security of tenure, to ensure their easy dismissal as
soon as the Robinsons' contract expired. The attendant circumstances cannot but raise doubts as to the petitioner's
good faith.

If the petitioner really intended the respondents to be project employees, then the contracts should have
been executed right from the time of hiring, or when the respondents were first assigned to Robinsons, not
when the petitioner's service contract was winding up. The terms and conditions of the respondents'
engagement should have been disclosed and explained to them from the commencement of their employment. The
petitioner's failure to do so supports the conclusion that it had been in bad faith in evading the respondents' right to
security of tenure.

Moreover, under Article 1390 of the Civil Code, contracts where the consent of a party was vitiated by  mistake,
violence, intimidation, undue influence or fraud, are voidable or annullable. The petitioner's threat of non-payment of
the respondents' salaries clearly amounted to intimidation. Under this situation, and the suspect timing when these
contracts were executed, we rule that these employment contracts were voidable and were effectively questioned
when the respondents filed their illegal dismissal complaint. 

The respondents were illegally


dismissed.

To be valid, an employee's dismissal must comply with the substantive and procedural requirements of due
process. Substantively, a dismissal should be supported by a just or authorized cause.  Procedurally, the employer
must observe the twin notice and hearing requirements in carrying out an employee's dismissal.

The petitioner argues that these substantive and procedural requisites do not apply to the respondents' case since
they were employed under fixed term contracts. According to the petitioner, the respondents' employment contracts
lapsed by operation of law as the necessary consequence of the termination and non-renewal of its service contract
with Robinsons. Because of this, there was no illegal dismissal to speak of, only contract expiration.

We do not agree with the petitioner.


Having already determined that the respondents are regular employees and not project employees, and that the
respondents' belated employment contracts could not be given any binding effect for being signed under duress, we
hold that illegal dismissal took place when the petitioner failed to comply with the substantive and procedural due
process requirements of the law.

The petitioner also asserts that the respondents' subsequent absorption by Robinsons' new contractors — Fieldmen
Janitorial Service Corporation and Altaserv — negates their illegal dismissal. This reasoning is patently erroneous.
The charge of illegal dismissal was made only against the petitioner which is a separate juridical entity from
Robinsons' new contractors; it cannot escape liability by riding on the goodwill of others.

By law, the petitioner must bear the legal consequences of its violation of the respondents' right to security of tenure.
The facts of this case show that since the respondents' hiring, they had been under the petitioner's employ as
janitors, service crews and sanitation aides. Their services had been continuously provided to the petitioner
without any gap. Notably, the petitioner never refuted this allegation of the respondents. Further, there was
no allegation that the petitioner went out of business after the non-renewal of the Robinsons' service
contract. Thus, had it not been for the respondents' dismissal, they would have been deployed to the petitioner's
other existing clients.

With regard to the award of separation pay, we agree with the CA's finding that this litigation resulted to strained
relations between the petitioner and the respondents. Thus, we also affirm the CA's ruling that instead of
reinstatement, the respondents should be paid their respective separation pays equivalent to one (1) month pay for
every year of service. 

We cannot give credence to the petitioner's assertion that under Section 10 of DO 18-02,  the respondents are not
entitled to separation pay because their employment was terminated due to the completion of the project where they
had been engaged. This provision must be construed with the rest of DO 18-02's other provisions.

As earlier pointed out, Section 7 of DO 18-02 treats contractual employees as the independent contractor's regular
employees for purposes of enforcing the Labor Code and other social legislation laws. Consequently, a finding of
regular employment entitles them to the rights granted to regular employees, particularly the right to security of tenure
and to separation pay.

Thus, a holistic reading of DO 18-02,  guides us to the conclusion that Section 10 only pertains to contractual
employees who are really project employees. They are not entitled to separation pay since the end of the project for
which they had been hired necessarily results to the termination of their employment. On the other hand, we already
found that the respondents are the petitioner's regular employees. Thus, their illegal dismissal entitles them to
backwages and reinstatement or separation pay, in case reinstatement is no longer feasible.
13. MANAGEMENT PREROGATIVE

37. Barba vs Liceo De Cagayan University, GR No. 193857, November 28, 2012

Facts

Petitioner Dr. Ma. Mercedes L. Barba was the Dean of the College of Physical Therapy of respondent Liceo de
Cagayan University, Inc., a private educational institution with school campus located at Carmen, Cagayan de Oro
City.

Petitioner started working for respondent on July 8, 1993 as medical officer/school physician for a period of one
school year or until March 31, 1994. In July 1994, she was chosen by respondent to be the recipient of a scholarship
grant to pursue a three-year residency training in Rehabilitation Medicine at the Veterans Memorial Medical Center
(VMMC).

After completing her residency training with VMMC in June 1997, petitioner returned to continue working for
respondent. She was appointed as Acting Dean of the College of Physical Therapy and at the same time designated
as Doctor-In-Charge of the Rehabilitation Clinic of the Rodolfo N. Pelaez Hall, City Memorial Hospital. 

On June 19, 2002, petitioner's appointment as Doctor-In-Charge of the Rehabilitation Clinic was renewed and she
was appointed as Dean of the College of Physical Therapy by respondent's President, Dr. Jose Ma. R. Golez.

Petitioner accepted her appointment and assumed the position of Dean of the College of Physical Therapy. In the
school year 2003 to 2004, the College of Physical Therapy suffered a dramatic decline in the number of enrollees
from a total of 1,121 students in the school year 1995 to 1996 to only 29 students in the first semester of school year
2003 to 2004. This worsened in the next year or in school year 2004 to 2005 where a total of only 20 students
enrolled. 

Due to the low number of enrollees, respondent decided to freeze the operation of the College of Physical Therapy
indefinitely. Respondent's President Dr. Rafaelita Pelaez-Golez wrote petitioner a letter dated March 16, 2005
informing her that her services as dean of the said college will end at the close of the school year. Thereafter, the
College of Physical Therapy ceased operations on March 31, 2005, and petitioner went on leave without pay starting
on April 9, 2005. Subsequently, respondent's Executive Vice President, Dr. Marian M. Lerin, through Dr. Glory S.
Magdale, respondent's Vice President for Academic Affairs, sent petitioner a letter dated April 27, 2005 instructing
petitioner to return to work on June 1, 2005 and report to Ma. Chona Palomares, the Acting Dean of the College of
Nursing, to receive her teaching load and assignment as a full-time faculty member in that department for the school
year 2005-2006. TIEHSA

In reply, petitioner informed Dr. Lerin that she had not committed to teach in the College of Nursing and that as far as
she can recall, her employment is not dependent on any teaching load. She then requested for the processing of her
separation benefits in view of the closure of the College of Physical Therapy. She did not report to Palomares on
June 1, 2005.

On June 8, 2005, petitioner followed up her request for separation pay and other benefits but Dr. Lerin insisted that
she report to Palomares; otherwise, sanctions will be imposed on her. Thus, petitioner through counsel wrote Dr.
Golez directly, asking for her separation pay and other benefits.

On June 21, 2005, Dr. Magdale wrote petitioner a letter directing her to report for work and to teach her assigned
subjects on or before June 23, 2005. Otherwise, she will be dismissed from employment on the ground of
abandonment. Petitioner, through counsel, replied that teaching in the College of Nursing is in no way related to her
scholarship and training in the field of rehabilitation medicine. Petitioner added that coercing her to become a faculty
member from her position as College Dean is a great demotion which amounts to constructive dismissal.

Dr. Magdale sent another letter to petitioner on June 24, 2005 ordering her to report for work as she was still bound
by the Scholarship Contract to serve respondent for two more years. But petitioner did not do so. Hence, on June 28,
2005, Dr. Magdale sent petitioner a notice terminating her services on the ground of abandonment.

Meanwhile, on June 22, 2005, prior to the termination of her services, petitioner filed a complaint before the Labor
Arbiter for illegal dismissal, payment of separation pay and retirement benefits against respondent, Dr. Magdale and
Dr. Golez. She alleged that her transfer to the College of Nursing as a faculty member is a demotion amounting to
constructive dismissal.

Respondent claimed that petitioner was not terminated and that it was only petitioner's appointment as College Dean
in the College of Physical Therapy that expired as a necessary consequence of the eventual closure of the said
college. Respondent further averred that petitioner's transfer as full-time professor in the College of Nursing does not
amount to constructive dismissal since the transfer was without loss of seniority rights and without diminution of pay.
Also, respondent added that pursuant to the Scholarship Contract, petitioner was still duty bound to serve respondent
until 2007 in whatever position related to her studies the school desires. 
Issus

The decisive issue in the present petition is whether petitioner was an employee or a corporate officer of respondent
university. Resolution of this issue resolves the question of whether the appellate court was correct in ruling that the
Labor Arbiter and the NLRC had no jurisdiction over petitioner's complaint for constructive dismissal against
respondent.

Ruling

After a careful review and examination of the records, we find that the CA's previous ruling that petitioner was
respondent's employee and not a corporate officer is supported by the totality of the evidence and more in accord
with law and prevailing jurisprudence.

Corporate officers are elected or appointed by the directors or stockholders, and are those who are given that
character either by the Corporation Code or by the corporation's by-laws. Section 25 of the Corporation
Code enumerates corporate officers as the president, the secretary, the treasurer and such other officers as may be
provided for in the by-laws.

In declaring petitioner a corporate officer, the CA considered respondent's by-laws and gave weight to the
certifications of respondent's secretary attesting to the resolutions of the board of directors appointing the various
academic deans for the School Years 1991-2002 and 2002-2005, including petitioner. However, an assiduous
perusal of these documents does not convince us that petitioner occupies a corporate office position in respondent
university.

On the other hand, there were two board resolutions that were presented appointed the various academic deans in
the university which included petitioner.

In respondent's by-laws, there are four officers specifically mentioned, namely, a president, a vice president, a
secretary and a treasurer. In addition, it is provided that there shall be other appointive officials, a College Director
and heads of departments whose appointments, compensations, powers and duties shall be determined by the board
of directors. It is worthy to note that a College Dean is not among the corporate officers mentioned in respondent's
by-laws. Petitioner, being an academic dean, also held an administrative post in the university but not a corporate
office as contemplated by law. Petitioner was not directly elected nor appointed by the board of directors to any
corporate office but her appointment was merely approved by the board together with the other academic deans of
respondent university in accordance with the procedure prescribed in respondent's Administrative Manual.  The act of
the board of directors in approving the appointment of petitioner as Dean of the College of Therapy did not make her
a corporate officer of the corporation. CIAHDT

Moreover, the CA, in its amended decision erroneously equated the position of a College Director to that of a College
Dean thereby concluding that petitioner is an officer of respondent.

It bears stressing that the appointive officials mentioned in Article V of respondent's by-laws are not corporate officers
under the contemplation of the law. Though the board of directors may create appointive positions other than the
positions of corporate officers, the persons occupying such positions cannot be deemed as corporate officers as
contemplated by Section 25 of the Corporation Code. On this point, the SEC Opinion dated November 25, 1993 is
instructive:

Thus, pursuant to the above provision (Section 25 of the Corporation Code), whoever are the corporate officers
enumerated in the by-laws are the exclusive Officers of the corporation and the Board has no power to create other
Offices without amending first the corporate By-laws. However, the Board may create appointive positions other
than the positions of corporate Officers, but the persons occupying such positions are not considered as
corporate officers within the meaning of Section 25 of the Corporation Code and are not empowered to
exercise the functions of the corporate Officers, except those functions lawfully delegated to them. Their functions
and duties are to be determined by the Board of Directors/Trustees.

But even assuming that a College Director may be considered a corporate officer of respondent, a review of the
records as well as the other documents submitted by the parties fails to persuade that petitioner was the "College
Director" mentioned in the by-laws of respondent. Nowhere in petitioner's appointment letter was it stated that
petitioner was designated as the College Director or that petitioner was to assume the functions and duties of a
College Director. Neither can it be inferred in respondent's by-laws that a dean of a college is the same as a College
Director of respondent. Respondent's lone surviving incorporating director Yolanda Rollo even admitted that no
College Director has ever been appointed by respondent.

Undoubtedly, petitioner is not a College Director and she is not a corporate officer but an employee of respondent.
Applying the four-fold test concerning (1) the selection and engagement of the employee; (2) the payment of wages;
(3) the power of dismissal; (4) the employer's power to control the employee with respect to the means and methods
by which the work is to be accomplished, it is clear that there exists an employer-employee relationship between
petitioner and respondent. Records show that petitioner was appointed to her position as Dean by Dr. Golez, the
university president and was paid a salary of P32,500 plus transportation allowance. It was evident that respondent
had the power of control over petitioner as one of its deans. It was also the university president who informed
petitioner that her services as Dean of the College of Physical Therapy was terminated effective March 31, 2005 and
she was subsequently directed to report to the Acting Dean of the College of Nursing for assignment of teaching
load. ETHaDC

Thus, petitioner, being an employee of respondent, her complaint for illegal/constructive dismissal against respondent
was properly within the jurisdiction of the Labor Arbiter and the NLRC. Article 217 of the Labor Code provides:

ART. 217.Jurisdiction of Labor Arbiters and the Commission. — (a) Except as otherwise provided under this
Code,

the Arbiters shall have original and exclusive jurisdiction to hear and decide . . . the following cases involving all
workers, whether agricultural or non-agricultural:

1.Unfair labor practice cases;

2.Termination disputes;

3.If accompanied with a claim for reinstatement, those cases that workers may file involving wage, rates of pay, hours
of work and other terms and conditions of employment;

4.Claims for actual, moral, exemplary and other forms of damages arising from the employer-employee relations;

5.Cases arising from any violation of Article 264 of this Code, including questions involving the legality of strikes and
lockouts; and

6.Except claims for Employees Compensation, Social Security, Medicare and maternity benefits, all other claims
arising from employer-employee relations, including those of persons in domestic or household service, involving an
amount exceeding five thousand pesos (P5,000.00) regardless of whether accompanied with a claim for
reinstatement.

(b)The Commission shall have exclusive appellate jurisdiction over all cases decided by Labor Arbiters. 

xxx xxx xxx

Moreover, we agree with the CA's earlier pronouncement that since respondent actively participated in the
proceedings before the Labor Arbiter and the NLRC, it is already estopped from belatedly raising the issue of lack of
jurisdiction. In this case, respondent filed position papers and other supporting documents to bolster its defense
before the labor tribunals but in all these pleadings, the issue of lack of jurisdiction was never raised. It was only in its
Supplemental Petition filed before the CA that respondent first brought the issue of lack of jurisdiction. We have
consistently held that while jurisdiction may be assailed at any stage, a party's active participation in the proceedings
will estop such party from assailing its jurisdiction. It is an undesirable practice of a party participating in the
proceedings and submitting his case for decision and then accepting the judgment, only if favorable, and attacking it
for lack of jurisdiction, when adverse. 

Under Section 6, Rule 10 of the 1997 Rules of Civil Procedure, as amended, governing supplemental pleadings, the
court "may" admit supplemental pleadings, such as the supplemental petition filed by respondent before the appellate
court, but the admission of these pleadings remains in the sound discretion of the court. Nevertheless, we have
already found no credence in respondent's claim that petitioner is a corporate officer, consequently, the alleged lack
of jurisdiction asserted by respondent in the supplemental petition is bereft of merit.

On the issue of constructive dismissal, we agree with the Labor Arbiter and the appellate court's earlier ruling that
petitioner was not constructively dismissed. Petitioner's letter of appointment specifically appointed her as Dean of
the College of Physical Therapy and Doctor-in-Charge of the Rehabilitation Clinic "for a period of three years
effective July 1, 2002 unless sooner revoked for valid cause or causes."  Evidently, petitioner's appointment as
College Dean was for a fixed term, subject to reappointment and revocation or termination for a valid cause. When
respondent decided to close its College of Physical Therapy due to drastic decrease in enrollees, petitioner's
appointment as its College Dean was validly revoked and her subsequent assignment to teach in the College of
Nursing was justified as it is still related to her scholarship studies in Physical Therapy.

In constructive dismissal cases, the employer has the burden of proving that its conduct and action or the transfer of
an employee are for valid and legitimate grounds such as genuine business necessity. Particularly, for a transfer not
to be considered a constructive dismissal, the employer must be able to show that such transfer is not unreasonable,
inconvenient, or prejudicial to the employee. In this case, petitioner's transfer was not unreasonable, inconvenient or
prejudicial to her. On the contrary, the assignment of a teaching load in the College of Nursing was undertaken by
respondent to accommodate petitioner following the closure of the College of Physical Therapy. Respondent further
considered the fact that petitioner still has two years to serve the university under the Scholarship Contract.

Petitioner's subsequent transfer to another department or college is not tantamount to demotion as it was a valid
transfer. There is therefore no constructive dismissal to speak of. That petitioner ceased to enjoy the compensation,
privileges and benefits as College Dean was but a logical consequence of the valid revocation or termination of such
fixed-term position. Indeed, it would be absurd and unjust for respondent to maintain a deanship position in a college
or department that has ceased to exist. Under the circumstances, giving petitioner a teaching load in another
College/Department that is related to Physical Therapy — thus enabling her to serve and complete her remaining two
years under the Scholarship Contract — is a valid exercise of management prerogative on the part of respondent. 
38. Best Wear Garments vs De Lemos, GR No. 191281, November 28, 2012

Facts

Petitioner Best Wear Garments is a sole proprietorship represented by its General Manager Alex Sitosta.
Respondents Cecile M. Ocubillo and Adelaida B. De Lemos were hired as sewers on piece-rate basis by petitioners
on October 27, 1993 and July 12, 1994, respectively.

On May 20, 2004, De Lemos filed a complaint for illegal dismissal with prayer for backwages and other accrued
benefits, separation pay, service incentive leave pay and attorney's fees. A similar complaint was filed by Ocubillo on
June 10, 2004. Both alleged in their position paper that in August 2003, Sitosta arbitrarily transferred them to other
areas of operation of petitioner's garments company, which they said amounted to constructive dismissal as it
resulted in less earnings for them.

De Lemos claimed that after two months in her new assignment, she was able to adjust but Sitosta again transferred
her to a "different operation where she could not earn [as] much as before because by-products require long period of
time to finish." She averred that the reason for her transfer was her refusal "to render [overtime work] up to 7:00 p.m."
Her request to be returned to her previous assignment was rejected and she was "constrained not to report for work
as Sitosta had become indifferent to her since said transfer of operation." She further alleged that her last salary was
withheld by petitioner company. 

On her part, Ocubillo alleged that her transfer was precipitated by her having "incurred excessive absences since
2001." Her absences were due to the fact that her father became very sick since 2001 until his untimely demise on
November 9, 2003; aside from this, she herself became very sickly. She claimed that from September to October
2003, Sitosta assigned her to different machines "whichever is available" and that "there were times, she could not
earn for a day because there was no available machine to work for [sic]." Sitosta also allegedly required her to render
overtime work up to 7:00 p.m. which she refused "because she was only paid up to 6:25 p.m." 

Petitioners denied having terminated the employment of respondents who supposedly committed numerous
absences without leave (AWOL). They claimed that sometime in February 2004, De Lemos informed Sitosta that due
to personal problem, she intends to resign from the company. She then demanded the payment of separation pay. In
March 2004, Ocubillo likewise intimated her intention to resign and demanded separation pay. Sitosta explained to
both De Lemos and Ocubillo that the company had no existing policy on granting separation pay, and hence he could
not act on their request. De Lemos never reported back to work since March 2004, while Ocubillo failed to report for
work from October 2004 to the present.

As to the allegation of respondents that the reason for their transfer was their refusal to render overtime work until
7:00 p.m., petitioners asserted that respondents are piece-rate workers and hence they are not paid according to the
number of hours worked.

Issue

Were the respondents constructively dismissed or were their transfers of assignment an exercise of management
prerogative?

Ruling

The right of employees to security of tenure does not give them vested rights to their positions to the extent of
depriving management of its prerogative to change their assignments or to transfer them. Thus, an employer may
transfer or assign employees from one office or area of operation to another, provided there is no demotion in rank or
diminution of salary, benefits, and other privileges, and the action is not motivated by discrimination, made in bad
faith, or effected as a form of punishment or demotion without sufficient cause. 

We rule that respondents were not constructively dismissed.

Being piece-rate workers assigned to individual sewing machines, respondents' earnings depended on the quality
and quantity of finished products. That their work output might have been affected by the change in their specific work
assignments does not necessarily imply that any resulting reduction in pay is tantamount to constructive dismissal.
Workers under piece-rate employment have no fixed salaries and their compensation is computed on the basis of
accomplished tasks. As admitted by respondent De Lemos, some garments or by-products took a longer time to
finish so they could not earn as much as before. Also, the type of sewing jobs available would depend on the
specifications made by the clients of petitioner company. Under these circumstances, it cannot be said that the
transfer was unreasonable, inconvenient or prejudicial to the respondents. Such deployment of sewers to work on
different types of garments as dictated by present business necessity is within the ambit of management prerogative
which, in the absence of bad faith, ill motive or discrimination, should not be interfered with by the courts. 
The records are bereft of any showing of clear discrimination, insensibility or disdain on the part of petitioners in
transferring respondents to perform a different type of sewing job. It is unfair to charge petitioners with constructive
dismissal simply because the respondents insist that their transfer to a new work assignment was against their will.
We have long stated that "the objection to the transfer being grounded on solely upon the personal inconvenience or
hardship that will be caused to the employee by reason of the transfer is not a valid reason to disobey an order of
transfer." That respondents eventually discontinued reporting for work after their plea to be returned to their former
work assignment was their personal decision, for which the petitioners should not be held liable particularly as the
latter did not, in fact, dismiss them.

Indeed, there was no evidence that respondents were dismissed from employment. In fact, petitioners expressed
willingness to accept them back to work. There being no termination of employment by the employer, the award of
backwages cannot be sustained. It is well settled that backwages may be granted only when there is a finding of
illegal dismissal. In cases where there is no evidence of dismissal, the remedy is reinstatement but without
backwages.

The constitutional policy of providing full protection to labor is not intended to oppress or destroy management. While
the Constitution is committed to the policy of social justice and the protection of the working class, it should not be
supposed that every labor dispute will be automatically decided in favor of labor. Management also has its rights
which are entitled to respect and enforcement in the interest of simple fair play. Thus, where management
prerogative to transfer employees is validly exercised, as in this case, courts will decline to interfere.
39. Royal Plant Workers Union vs. Coca-Cola Bottlers Phils Inc. -Cebu Plant, G.R. No. 198783, April 15, 2013

Facts:

Under the employ of each bottling plant are bottling operators. In the case of the plant in Cebu City, there are 20
bottling operators who work for its Bottling Line 1 while there are 12-14 bottling operators who man its Bottling Line 2.
All of them are male and they are members of herein respondent Royal Plant Workers Union (ROPWU).

The bottling operators work in two shifts. The first shift is from 8 a.m. to 5 p.m. and the second shift is from 5 p.m. up
to the time production operations is finished. Thus, the second shift varies and may end beyond eight (8) hours.
However, the bottling operators are compensated with overtime pay if the shift extends beyond eight (8) hours. For
Bottling Line 1, 10 bottling operators work for each shift while 6 to 7 bottling operators work for each shift for Bottling
Line 2.

Each shift has rotations of work time and break time. Prior to September 2008, the rotation is this: after two and a half
(2 1/2) hours of work, the bottling operators are given a 30-minute break and this goes on until the shift ends. In
September 2008 and up to the present, the rotation has changed and bottling operators are now given a 30-minute
break after one and one half (1 1/2) hours of work.

In 1974, the bottling operators of then Bottling Line 2 were provided with chairs upon their request. In 1988, the
bottling operators of then Bottling Line 1 followed suit and asked to be provided also with chairs. Their request was
likewise granted. Sometime in September 2008, the chairs provided for the operators were removed pursuant to a
national directive of petitioner. With this task of moving constantly to check on the machinery and equipment
assigned to him, a bottling operator does not need a chair anymore, hence, petitioner's directive to remove them.
Furthermore, CCBPI rationalized that the removal of the chairs is implemented so that the bottling operators will avoid
sleeping, thus, prevent injuries to their persons.

The bottling operators took issue with the removal of the chairs.

Issue:

Whether or not the removal of the bottling operators' chairs from CCBPI's production/manufacturing lines a valid
exercise of a management prerogative

Ruling:

Yes.
The Court has held that management is free to regulate, according to its own discretion and judgment, all aspects of
employment, including hiring, work assignments, working methods, time, place, and manner of work, processes to be
followed, supervision of workers, working regulations, transfer of employees, work supervision, lay-off of workers, and
discipline, dismissal and recall of workers. The exercise of management prerogative, however, is not absolute as it
must be exercised in good faith and with due regard to the rights of labor.

In the present controversy, it cannot be denied that CCBPI removed the operators' chairs pursuant to a national
directive and in line with its "I Operate, I Maintain, I Clean" program, launched to enable the Union to perform their
duties and responsibilities more efficiently. The chairs were not removed indiscriminately. They were carefully studied
with due regard to the welfare of the members of the Union. The removal of the chairs was compensated
by: a) a reduction of the operating hours of the bottling operators from a two-and-one-half (2 1/2)-hour rotation
period to a one-and-a-half (1 1/2) hour rotation period; and b) anincrease of the break period from 15 to 30 minutes
between rotations.

Apparently, the decision to remove the chairs was done with good intentions as CCBPI wanted to avoid instances of
operators sleeping on the job while in the performance of their duties and responsibilities and because of the fact that
the chairs were not necessary considering that the operators constantly move about while working. In short, the
removal of the chairs was designed to increase work efficiency. Hence, CCBPI's exercise of its management
prerogative was made in good faith without doing any harm to the workers' rights.  The rights of the Union under any
labor law were not violated. There is no law that requires employers to provide chairs for bottling operators. Further,
The operators' chairs cannot be considered as one of the employee benefits covered in Article 100 of the Labor
Code. In the Court's view, the term "benefits" mentioned in the non-diminution rule refers to monetary benefits or
privileges given to the employee with monetary equivalents. Such benefits or privileges form part of the employees'
wage, salary or compensation making them enforceable obligations
40. Peckson vs. Robinsons Supermarket Corp. G.R. No. 198534, July 3, 2013

Facts:
The petitioner first joined the Robinsons Supermarket Corporation (RSC) as a Sales Clerk on November 3, 1987. On
October 26, 2006, she was holding the position of Category Buyer when respondent Roena Sarte (Sarte), RSC's
Assistant Vice-President for Merchandising, reassigned her to the position of Provincial Coordinator, effective
November 1, 2006. Claiming that her new assignment was a demotion because it was non-supervisory and clerical in
nature, the petitioner refused to turn over her responsibilities to the new Category Buyer, or to accept her new
responsibilities as Provincial Coordinator.

Issue: Whether or not petitioner’s lateral transfer from Category Buyer to Provincial Coordinator is considered a
demotion amounting to constructive dismissal

Ruling:
Under the doctrine of management prerogative, every employer has the inherent right to regulate, according to his
own discretion and judgment, all aspects of employment, including hiring, work assignments, working methods, the
time, place and manner of work, work supervision, transfer of employees, lay-off of workers, and discipline, dismissal,
and recall of employees. The only limitations to the exercise of this prerogative are those imposed by labor laws and
the principles of equity and substantial justice.

Concerning the transfer of employees, these are the following jurisprudential guidelines: (a) a transfer is a movement
from one position to another of equivalent rank, level or salary without break in the service or a lateral movement from
one position to another of equivalent rank or salary; (b) the employer has the inherent right to transfer or reassign an
employee for legitimate business purposes; (c) a transfer becomes unlawful where it is motivated by discrimination or
bad faith or is effected as a form of punishment or is a demotion without sufficient cause; (d) the employer must be
able to show that the transfer is not unreasonable, inconvenient, or prejudicial to the employee.

If the transfer of an employee is not  unreasonable, or inconvenient, or  prejudicial to him, and it does not  involve a
demotion in rank or a  diminution of his salaries, benefits and other privileges, the employee  may not complain that it
amounts  to a constructive dismissal. The respondents had the burden of proof that the transfer of the petitioner was
not tantamount to constructive dismissal. The respondents have discharged the burden of proof that the transfer of
the petitioner was not  tantamount to constructive dismissal.
 
In the case at bar, we agree with the appellate court that there is substantial showing that the transfer of the petitioner
from Category Buyer to Provincial Coordinator was not unreasonable, inconvenient, or prejudicial to her. The
petitioner failed to dispute that the job classifications of Category Buyer and Provincial Coordinator are similar, or that
they command a similar salary structure and responsibilities. We agree with the NLRC that the Provincial
Coordinator's position does not involve mere clerical functions but requires the exercise of discretion from time to
time, as well as independent judgment, since the Provincial Coordinator gives appropriate recommendations to
management and ensures the faithful implementation of policies and programs of the company. It even has influence
over a Category Buyer because of its recommendatory function that enables the Category Buyer to make right
decisions on assortment, price and quantity of the items to be sold by the store.
14. TERMINATION OF EMPLOYMENT

41. Ramirez et al., vs. Mar Fishing Co Inc. et al., G.R. No. 168208, June 13, 2012

Facts:

Mar Fishing sold its principal assets to Miramar Fishing Co through public bidding. In view of that transfer, Mar
Fishing issued a Memorandum informing all its workers that the company would cease to operate by the end of the
month. two days prior to the month’s end, it notified (DOLE) of the closure of its business operations.

Thereafter, Mar Fishing’s labor union, Mar Fishing Workers Union entered into a Memorandum of Agreement. The
Agreement provided that the acquiring company, Miramar, shall absorb Mar Fishing’s regular rank and file employees
whose performance was satisfactory, without loss of seniority rights and privileges previously enjoyed.

Unfortunately, petitioners, who worked as rank and file employees, were not hired or given separation pay by
Miramar. Thus, petitioners filed Complaints for illegal dismissal with money claims before the Arbitration Branch of
(NLRC).

Issue:
Whether or not dismissal based on the closure of business is valid.
Ruling:
YES.
3 requirements for closure of business to be valid , to wit:
1) the cessation of or withdrawal from business operations must be bona fide in character;
2) there must be payment to the employees of termination pay amounting to at least one-half (1/2) month
pay for each year of service, or one (1) month pay, whichever is higher;
3) the company must serve a written notice on the employees and on the DOLE at least one (1) month
before the intended termination.
Petitioners did not dispute the conclusion of the LA and the NLRC that Mar Fishing had an authorized cause to
dismiss its workers. Neither did petitioners challenge the computation of their separation pay. Petitioners only
questioned the holding that only Mar Fishing was liable for their monetary claims.
42. Prudential Guarantee & Assurance Employee Labor Union vs. NLRC et al., G.R. No. 185335, June 13, 2012
Facts:

Vallota commenced his employment with Prudential Guarantee and Assurance, Inc. (PGAI) on 1995 as a Junior
Programmer assigned to the Electronic Data Processing (EDP) . He reported directly to Gerald Dy Victory, then head
of the EDP, until his replacement by respondent Jocelyn Retizos  in 1997.

PGAI’s Human Resource Manager, invited Union President, Mike Apostol to his office. Atty. Rillo informed Apostol
that PGAI was going to conduct an on-the-spot security check in the Information and Technology  (IT) Department.
Atty. Rillo also requested that Union representatives witness the inspection to which Apostol agreed.

The inspection team proceeded to the IT Department, PGAI network administrator Angelo
Gutierrez (Gutierrez), initiated the spot check beginning with the one assigned to Vallota. Gutierrez apparently did not
find anything unusual with Vallota’s computer and said "Wala naman, saan dito?" Retizos insisted, "Nandyan yan,"
and took over the inspection until she found a folder named "MAA." She then exclaimed, "Heto oh! Ano to? Bakit may
MAA dito?" Retizos asked Vallota, "Are you working for MAA?" Vallota replied, "Hindi po, MAA mutual life po yan na
makikita po sa internet." Gutierrez saved a copy of the contents of the MAA folder in a floppy disk.

Vallota received a memorandum directing him to explain within 72 hours why highly confidential files were stored in
his computer. The same memorandum also informed him that he was being placed under preventive suspension for
30 days effective upon receipt of the said notice. A second memorandum , notified Vallota of the extension of his
preventive suspension for another 30 days, in view of the fact that the management needed more time to evaluate
the administrative case against him.

PGAI sent him another memorandum requesting further details on some of the matters he raised in his response.
Vallota requested a conference, to be attended by a Union representative and counsel. In reply, PGAI sent Vallota
another memorandum which, among others, set a new deadline for Vallota to submit his reply and evidence in his
defense.

Meanwhile, the Union sent a letter to PGAI President Philip K. Rico (Rico) requesting that a grievance committee be
convened and that the contents of the computers of other IT personnel be similarly produced. The request for the
convening of a grievance committee was ignored. Vallota was given a notice of termination of his employment
effective January 10, 2006 on the ground of loss of trust and confidence. Thus, the petitioners filed a complaint for
illegal dismissal with claims for full backwages, moral and exemplary damages, and attorney’s fees.
Issues:
1. Whether or not the petitioner was validly dismissed on the ground of loss of trust and confidence.
2. Whether or not the procedural due process requirements for termination were observed.

Ruling:
1. No.
Loss of confidence as a just cause for dismissal was never intended to provide employers with a blank check for
terminating their employees. Such a vague, all-encompassing pretext as loss of confidence, if unqualifiedly given the
seal of approval by this Court, could readily reduce to barren form the words of the constitutional guarantee of
security of tenure.

Having this in mind, loss of confidence should ideally apply only to cases involving employees occupying positions of
trust and confidence or to those situations where the employee is routinely charged with the care and custody of the
employer's money or property. To the first class belong managerial employees, i.e., those vested with the powers or
prerogatives to lay down management policies and/or to hire, transfer, suspend, lay-off, recall, discharge, assign or
discipline employees or effectively recommend such managerial actions; and to the second class belong cashiers,
auditors, property custodians, etc., or those who, in the normal and routine exercise of their functions, regularly
handle significant amounts of money or property.

The first question to be addressed is whether Vallota held a position of trust and confidence. Vallota was employed
by PGAI as a Junior Programmer assigned to the EDP Department. Based on the standards set by previous
jurisprudence, Vallota’s position as Junior Programmer is analogous to the second class of positions of trust and
confidence. Though he did not physically handle money or property, he became privy to confidential data or
information by the nature of his functions. At a time when the most sensitive of information is found not printed on
paper but stored on hard drives and servers, an employee who handles or has access to data in electronic form
naturally becomes the unwilling recipient of confidential information.

Having addressed the nature of his position, the next question is whether the act complained of justified the loss of
trust and confidence of Vallota’s employer so as to constitute a valid cause for dismissal. It must, thus, be determined
whether the alleged basis for dismissal was based on clearly established facts.
The act alleged to have caused the loss of trust and confidence of PGAI in Vallota was the presence in his
computer’s hard drive of a folder named "MAA" allegedly containing files with information on MAA Mutual Life
Philippines.

In this case, there was no other evidence presented to prove fraud in the manner of securing or obtaining the files
found in Vallota’s computer. In fact, aside from the presence of these files in Vallota’s hard drive, there was no other
evidence to prove any gross misconduct on his part. There was no proof either that the presence of such files was
part of an attempt to defraud his employer or to use the files for a purpose other than that for which they were
intended. If anything, the presence of the files reveals some degree of carelessness or neglect in his failure to delete
them, but it is an extremely farfetched conclusion bordering on paranoia to state that it is part of a larger conspiracy
involving corporate espionage.

Moreover, contrary to the respondents’ allegations, the MAA files found in Vallota’s computer, the prospectus and
corporate profile, are not sensitive corporate documents. These are documents routinely made available to the public.

2. Yes.

The following are the guiding principles in connection with the hearing requirement in dismissal cases:
1. “Ample opportunity to be heard” means any meaningful opportunity (verbal or written) given to the employee
to answer the charges against him and submit evidence in support of his defense, whether in a hearing,
conference or some other fair, just and reasonable way.
2. A formal hearing or conference becomes mandatory only when requested by the employee in writing or
substantial evidentiary disputes exist or a company rule or practice requires it, or when similar
circumstances justify it.
3. The “ample opportunity to be heard” standard in the Labor Code prevails over the “hearing or conference”
requirement in the implementing rules and regulations.

In this case, the two-notice requirement was complied with. By the petitioners’ own admission, PGAI issued to Valota
a written Notice of Charges & Preventive Suspension (Ref. No. AC-05-02) dated November 14, 2005. After an
exchange of memoranda, PGAI then informed Vallota of his dismissal in its decision dated December 21, 2005.

But the petitioners expressly requested a conference or a convening of a grievance committee, such formal hearing
became mandatory. After PGAI failed to affirmatively respond to such request, it follows that the hearing requirement
was not complied with and, therefore, Vallota was denied his right to procedural due process.
43. Romeo Paulino v. NLRC, PLDT. 13 JUNE 2012

Loss of trust and confidence, termination for just cause

Facts:

Involved here is a Paulino, a Cable Splicer III who was terminated from employment because of serious misconduct
(theft of company policy). Undisputed are the following series of facts:

1. Paulino surrendered his service vehicle to PLDT’s motor pool for body repairs. Because of this, he unloaded
several company-issued plant materials contained in the vehicle and stored them in his residence. He kept
these properties in his residence for 1 month and 11 days, thus on February, armed with a Search Warrant,
PNP searched his house and found many plant materials. He was thereafter charged with Theft.
2. During this interim, PLDT likewise received security report that Paulino was engaged in illicit disposal of
company properties.
3. Following this, PLDT issued a Memo requiring Paulino to explain why he should not be terminated. The
Memo likewise gave him the option to ask for a formal hearing. Paulino, in his reply, merely requested that
the proceedings be held in abeyance until the criminal case for theft had been concluded.
4. Thereafter, PLDT informed Paulino that since his reply did not provide any clarification, his employment is
terminated.
5. Three years later, after the criminal case for Theft had been dismissed for failure to prove guilt beyond
reasonable doubt, Paulino filed a complaint for Illegal Dismissal.

Issue:

Whether or not Paulino’s employment was illegally terminated

Ruling:

Dismissal was valid, petition is denied.

The Labor Code recognizes that an employer, for just cause, may validly terminate the services of an employee for
serious misconduct or willful disobedience of the lawful orders of the employer or representative in connection with
the employee’s work. Fraud or willful breach by the employee of the trust reposed by the employer in the former, or
simply loss of confidence, also justifies an employee’s dismissal from employment.

1. The LA, the NLRC and the CA all acknowledged that, notwithstanding petitioner’s acquittal in the criminal
case for qualified theft, respondent PLDT had adequately established the basis for the company’s loss of
confidence as a just cause to terminate petitioner. This Court finds that approach to be correct, since proof
beyond reasonable doubt of an employee’s misconduct is not required in dismissing an employee. Rather,
as opposed to the “proof beyond reasonable doubt” standard of evidence required in criminal cases, labor
suits require only substantial evidence to prove the validity of the dismissal.

2. Willful breach of trust or loss of confidence requires that the employee (1) occupied a position of trust or (2)
was routinely charged with the care of the employer’s property. As correctly appreciated by the CA,
petitioner was charged with the care and custody of PLDT’s property.

3. To warrant dismissal based on loss of confidence, there must be some basis for the loss of trust or the
employer must have reasonable grounds to believe that the employee is responsible for misconduct that
renders the latter unworthy of the trust and confidence demanded by his or her position. Here, petitioner
disputes the sufficiency of PLDT’s basis for loss of trust and confidence. He alleges that he did not steal the
plant materials, considering that he had lawful possession.
4. However, assuming that he lawfully possessed the materials, PLDT still had ample reason or basis to
already distrust petitioner. For more than a month, he did not even inform PLDT of the whereabouts of the
plant materials. Instead, he stocked these materials at his residence even if they were needed in the daily
operations of the company. In keeping with the honesty and integrity demanded by his position, he should
have turned over these materials to the plant’s warehouse. The fact that petitioner did not present any
documents or requisition slips at the time that the PNP took the plant materials logically excites suspicion. In
addition, PLDT received a security report stating that petitioner had engaged in the illicit disposal of its plant
materials, which were recovered during the search conducted at his residence.

5. In a final effort to impugn his dismissal, petitioner claims that he could only be faulted for breaching PLDT’s
rules and regulations which prohibited the employees from bringing home company materials.
a. In this regard, petitioner exacerbates his position. By admitting that he breached company rules, he
buttressed his employer’s claim that he committed serious misconduct.
b. Employees cannot take company rules for granted, especially in this case where petitioner’s breach
involved various plant materials that may cause major disruption in the company’s
operations. Indeed, an employer may discharge an employee for refusal to obey a reasonable
company rule. As a rule, although this Court leans over backwards to help workers and employees
continue with their employment, acts of dishonesty in the handling of company property are a
different matter.
c. Given these circumstances, it would have been unfair for PLDT to keep petitioner in its employ.
Petitioner displayed actions that made him untrustworthy. Thus, as a measure of self-protection,
PLDT validly terminated his services for serious misconduct and loss of confidence.
44. Ever Electrical Mfg. (EEMI) and Vicente Go v. Samahang Mangagawas ng Ever Electrical, et al. 13 June
2012

Serious business losses, termination of employment, prevented from entering work by virtue of writ of execution
against employer, solidary liability of corporate officers

Facts: EEMI is a corporation engaged in manufacturing electrical parts/supplies. Respondents are employees of said
corporation, whose employment was terminated. EEMI reasoned that said termination was due to serious business
losses resulting in closure of its business operations which progressed as follows:
EEMI invested in Orient Commercial Banking Corporation a sum of P500 000 000. During the Asian Currency Crisis,
EEMI was one of those who suffered huge losses. It further suffered losses due to the continued entry of cheaper
goods from China. EEMI had to obtain a loan from UCPB in the amount of P121 000 000, secured by a mortgage on
some of its properties. To cap it all off, Orient Bank, where most of EEMI’s resources were invested, closed.
As a result, EEMI was not able to meet its loan obligations with UCPB. EEMI and UCPB then entered into a dacion
en pago, where EEMI transferred ownership of the company’s property to UCPB. EEMI then, through an affiliate
company, leased the transferred property so that it can continue its business operations. However, this lease
agreement came to a halt after the affiliate company (and ultimately EEMI) was evicted by virtue of an unlawful
detainer case. A writ of execution ordered the affiliate company to vacate the premises. Hence, when said writ was
implemented, herein employee-respondents were prevented from entering the factory, causing said employees to file
a Complaint for Illegal Dismissal, payment of 13th month pay and separation pay.
Vicente Go, an officer-stockholder of the corporation was also joined as defendant in the Complaint.

Issues:

1. Whether or not the closure of EEMI’s operation was due to serious business losses.
2. Whether or not Vicente Go is solidarily liable.

Ruling:

Partially granted. EEMI is liable for 13th month and separation pay. Vicente go is absolved.

1. Article 283 of the Labor Code provides:


 
Art. 283. Closure of establishment and reduction of personnel. — The employer may also terminate the
employment of any employee due to the installation of labor saving devices, redundancy, retrenchment to
prevent losses or the closing or cessation of operation of the establishment or undertaking unless the
closing is for the purpose of circumventing the provisions of this Title, by serving a written notice on the
workers and the Ministry of Labor and Employment at least one (1) month before the intended date thereof.
In case of termination due to the installation of labor saving devices or redundancy, the worker affected
thereby shall be entitled to a separation pay equivalent to at least his one (1) month pay or to at least one (1)
month pay for every year of service, whichever is higher. In case of retrenchment to prevent losses and in
cases of closures or cessation of operations of establishment or under taking not due to serious business
losses or financial reverses, the separation pay shall be equivalent to one (1) month pay or at least one-half
(1/2) month pay for every year of service, whichever is higher. A fraction of at least six (6) months shall be
considered one (1) whole year.
 
In this case, EEMI failed to establish that the main reason for its closure was business reverses.  As aptly observed
by the CA, the cessation of EEMI’s business was not directly brought about by serious business losses or financial
reverses, but by reason of the enforcement of a judgment against it.  Thus, EEMI should be required to pay
separation pay to its affected employees.     

2. As to whether or not Go should be held solidarily liable with EEMI, the Court agrees with the petitioner.

 The SC rejected the view of the Labor Arbiter, to wit: the LA was of the view that Go, as President of the corporation,
actively participated in the management of EEMI’s corporate obligations, and, accordingly, rendered judgment
ordering EEMI and Go “in solidum to pay the complainants” [18] their due. He explained that “[r]espondent Go’s
negligence in not paying the lease rental of the plant in behalf of the lessee EGO Electrical Supply, Inc., where EEMI
was operating and reimburse expenses of UCPB for real estate taxes and the like, prompted the bank to file an
unlawful detainer case against the lessee, EGO Electrical Supply Co.  This evasion of an existing obligation, made
respondent Go as liable as respondent EEMI, for complainants’ money awards.” [19]  Added the LA, “being the
President and the one actively representing respondent EEMI, in major contracts i.e. Real Estate Mortgage,
loans,dacion en pago, respondent Go has to be liable in the case.” [20]  As earlier stated, the CA affirmed the LA
decision citing the case of Restaurante Las Conchas v. Llego,[21] where it was held that “when the employer
corporation is no longer existing and unable to satisfy the judgment in favor of the employees, the officers should be
held liable for acting on behalf of the corporation.”
As a general rule, corporate officers should not be held solidarily liable with the corporation for separation pay for it is
settled that a corporation is invested by law with a personality separate and distinct from those of the persons
composing it as well as from that of any other legal entity to which it may be related.  Mere ownership by a single
stockholder or by another corporation of all or nearly all of the capital stock of a corporation is not of itself sufficient
ground for disregarding the separate corporate personality.[17]

In the case at bench, the records do not warrant an application of the exception. The rule, which requires the
presence of malice or bad faith, must still prevail. In the recent case of Wensha Spa Center and/or Xu Zhi Jie v.
Yung, the Court absolved the corporation’s president from liability in the absence of bad faith or malice.  In the said
case, the Court stated:
 
In labor cases, corporate directors and officers may be held solidarily liable with the corporation for the
termination of employment only if done with malice or in bad faith Bad faith does not connote bad judgment
or negligence; it imports a dishonest purpose or some moral obliquity and conscious doing of wrong; it
means breach of a known duty through some motive or interest or ill will; it partakes of the nature of fraud
 
In the present case, Go may have acted in behalf of EEMI but the company’s failure to operate cannot be equated to
bad faith. Cessation of business operation is brought about by various causes like mismanagement, lack of demand,
negligence, or lack of business foresight. Unless it can be shown that the closure was deliberate, malicious and in
bad faith, the Court must apply the general rule that a corporation has, by law, a personality separate and distinct
from that of its owners.  As there is no evidence that Go, as EEMI’s President, acted maliciously or in bad faith in
handling their business affairs and in eventually implementing the closure of its business, he cannot be held jointly
and solidarily liable with EEMI.
45. WATERFRONT CEBU CITY HOTEL VS. MA. MELANIE JIMENEZ ET AL. GR NO. 174214, JUNE 13, 2012

FACTS:

The closure of a department or division of a company constitutes retrenchment by, and not closure of, the company
itself.         

It is upon this principle that both parties to this case, the employer Hotel and the complaining employees repose their
argument.

Respondents Ma. Melanie P. Jimenez et were hired for Club Waterfront (the Club), a division under petitioner
Waterfront Cebu City Hotel (the Hotel) which catered to foreign high stakes gamblers.

 On 12 May 2003, respondents received identical letters of termination from petitioner’s Director of Human Resources
informing them of the temporary suspension of business of the Club.  A total of 45 employees were notified of the
imminent closure. Petitioner averred that since April 2002, the Club has been incurring losses that it had to
temporarily cease its operations effective 15 June 2003.  To support the allegations of losses, petitioner presented
financial statements of Waterfront Promotion, Ltd.  Petitioner argued that pursuant to Article 286 of the Labor Code,
the temporary suspension of business operations does not terminate employment.  Thus, respondents have no cause
of action against them. 

The dismissed employees were offered separation pay equivalent to half-month pay for every year of service. 

On 26 June 2003, respondents filed a complaint before the Labor Arbiter for illegal dismissal, illegal suspension, and
non-payment of salaries and other monetary benefits. They refused to believe that the Club was suffering from losses
because they knew exactly the number of arrivals as well as junket clients of the Club.  They presented documents to
show the arrival of foreign guests at the Club. 

The Labor Arbiter ruled in favor of petitioner and upheld the closure of the Club’s business operations as a
management prerogative.  

The NLRC also ruled in favor of petitioner.

After the denial of respondents’ motion for reconsideration, they elevated the case to the Court of Appeals which
reversed the decision of the NLRC.

ISSUE:

WHETHER OR NOT THERE WAS A VALID RETRENCHMENT

RULING: 

Retrenchment is the termination of employment initiated by the employer through no fault of and without prejudice to
the employees. It is resorted to during periods of business recession, industrial depression, or seasonal fluctuations
or during lulls occasioned by lack of orders, shortage of materials, conversion of the plant for a new production
program or the introduction of new methods or more efficient machinery or of automation. It is an act of the employer
of dismissing employees because of losses in the operation of a business, lack of work, and considerable reduction
on the volume of his business.

In case of retrenchment, proof of financial losses becomes the determining factor in proving its legitimacy.  In
establishing a unilateral claim of actual or potential losses, financial statements audited by independent external
auditors constitute the normal method of proof of profit and loss performance of a company.  The condition of
business losses justifying retrenchment is normally shown by audited financial documents like yearly balance sheets
and profit and loss statements as well as annual income tax returns. 

Retrenchment is subject to faithful compliance with the substantive and procedural requirements laid down by law
and jurisprudence. For a valid retrenchment, the following elements must be present:

(1)    That retrenchment is reasonably necessary and likely to prevent business losses which, if already
incurred, are not merely de minimis, but substantial, serious, actual and real, or if only expected, are
reasonably imminent as perceived objectively and in good faith by the employer 

(2)    That the employer served written notice both to the employees and to the Department of Labor and
Employment at least one month prior to the intended date of retrenchment;

(3)    That the employer pays the retrenched employees separation pay equivalent to one (1) month pay
or at least ½ month pay for every year of service, whichever is higher;

(4)    That the employer exercises its prerogative to retrench employees in good faith for the
advancement of its interest and not to defeat or circumvent the employees’ right to security of tenure; and

(5)    That the employer used fair and reasonable criteria in ascertaining who would be dismissed and
who would be retained among the employees, such as status, efficiency, seniority, physical fitness, age,
and financial hardship for certain workers.

All these elements were successfully proven by petitioner.  First, the huge losses suffered by the Club for the past
two years had forced petitioner to close it down to avert further losses which would eventually affect the operations of 
petitioner.  Second, all 45 employees working under the Club were served with notice of termination. The
corresponding notice was likewise served to the DOLE one month prior to retrenchment. Third, the employees were
offered separation pay, most of whom have accepted and opted not to join in this complaint.  Fourth, cessation of or
withdrawal from business operations was bona fide in character and not impelled by a motive to defeat or circumvent
the tenurial rights of employees. As a matter of fact, as of this writing, the Club has not resumed operations.  Neither
is there a showing that petitioner carried out the closure of the business in bad faith.  No labor dispute existed
between management and the employees when the latter were terminated.    
46. Manila Electric Co., vs Dejan, GR No. 194106, June 18, 2012

FACTS:

Respondent Herminigildo Dejan commenced employment with the Manila Electric Company (Meralco) on July 7,
1992. He was then Meralco's branch representative in its San Pedro, Laguna branch, with a monthly salary of
P30,500.00. His work consisted of accepting payments of the required fees from applicants for electric service
installation and issuing the corresponding meter sockets/bases after payment of a deposit, preceded by an inspection
of the premises to be energized by a Meralco field personnel.

In the mid-afternoon of March 18, 2005, the security guard on duty at the branch, Warlito Silverio, noticed a certain
Estanislao Gozarin a.k.a. Mang Islao, a private electrician, take out from the branch premises 20 pieces of meter
sockets which were then loaded into a parked Meralco contracted jeep belonging to one Cesar Reyes. Reyes brought
the meter sockets to his house. The meter sockets were thereafter allegedly picked up by Gil Duenas, a Meralco field
representative. Dejan was asked to explain the incident. aCITEH

In his letter-explanation, dated March 23, 2005, to a certain Emilia SJ Reaso,  Dejan admitted that he released the
meter sockets in question because the deposit fees had already been paid. The payor, a certain Antonio A. Depante
a.k.a. Bruce, also an electrician, asked for the release of the items. Allegedly, he had several contracts for service
installation with the branch. Dejan indicated the list of contracts covering the released meter sockets. Sometime in
September, October and November 2005, Meralco asked Gozarin,  Dejan,  and Reyes  to give their sworn
statements on the incident.

On February 10, 2006, Dejan received a letter from Marcelino Rosario, head of Meralco's Investigation-Paralegal
Services, charging him with the unauthorized taking of 20 meter sockets, in violation of Section 7, paragraphs 4 and
11 of the Company Code of Employee Discipline, in relation to Article 282 of the Labor Code. On February 17, 2006,
Meralco conducted a formal investigation where Dejan admitted issuing the meter sockets without the authorization of
the applicants for electric connection. He alleged that he released the items even without authorization as it had been
the accepted practice in the office, provided the deposit fee had been paid. He claimed that he talked with Depante,
through the cell phone of Duenas, about it, after Duenas himself requested him (Dejan) to release the meter sockets
to Gozarin. When Dejan released the meter sockets, Duenas instructed Gozarin to take them out of the Meralco
premises and load them in Reyes' jeep.

Also testifying at the investigation, Depante corroborated Dejan's account of the incident. He alleged that he made
the request for the release of the meter sockets due to his inability to pick up the items himself as he was busy with
another project at the time. He and Duenas retrieved the meter sockets from Reyes' house the next day. 

Unconvinced with Dejan's explanation, Meralco served Dejan a letter on April 6, 2006, 9 terminating his employment
effective the following day, with forfeiture of all rights and privileges. On April 20, 2006, Dejan filed his complaint with
the National Labor Relations Commission (NLRC).

ISSUE:

The validity of dejan’s termination.

RULING:

We found for petitioner.


Dejan is liable as charged.  More specifically, he is liable for violation of Section 7, paragraphs 4 and 11 of the
Company Code of Employee Discipline, constituting serious misconduct, fraud and willful breach of trust of the
employer, just causes for termination of employment under the law. The facts and the evidence on record clearly bear
this out and we wonder how the CA could have missed the seriousness or gravity of Dejan’s transgressions.

There is no dispute about the release of the meter sockets. Also, the persons involved were clearly identified – Dejan;
Gozarin or Mang Islao, a private electrician who received the meter sockets; Reyes, the owner of the jeep where the
meter sockets were loaded by Gozarin; Duenas, a Meralco field representative; and Depante, another private
electrician who purportedly owned the meter sockets.

There is also no question that Dejan released the meter sockets to Gozarin without the written authority or SPA from
the customer or customers who applied for electric connection (as a matter of company policy).  Dejan released the
meter sockets to Gozarin on the mere say-so of Depante, as he claimed, through a call to Duenas’ cell phoneand
justified his act to be in accord with accepted company practice.
We cannot blame Meralco for losing its trust and confidence in Dejan. He is no ordinary employee. As branch
representative, "he was principally charged with the function and responsibility to accept payment of fees required for
the installation of electric service and facilitate issuance of meter sockets."  The duties of his position require him to
always act with the highest degree of honesty, integrity and sincerity, as the company puts it. In light of his fraudulent
act, Meralco, an enterprise imbued with public interest, cannot be compelled to continue Dejan's employment, as it
would be inimical to its interest.  Needless to say, "[t]he law, in protecting the rights of the laborer, authorizes neither
oppression nor self-destruction of the employer."  For sure, Dejan was validly dismissed for serious misconduct,
and loss of trust and confidence.
47. Apo Cement vs Baptisma, GR 176671

Principle: The act of a power plant manager in receiving commissions or kickbacks from the company’s suppliers constitutes
breach of the trust and confidence reposed in him, justifying his dismissal. For managerial employees, the mere existence of a
basis for believing that such employee has breached the trust of his employer would suffice for his dismissal.

Facts:
On June 16, 1998, respondent Zaldy E. Baptisma was employed by petitioner Apo Cement Corporation. Sometime in September
2003, petitioner received information from one of its employees, Armando Moralda (Moralda), that some of its personnel, including
respondent who was then the manager of petitioner’s Power Plant Department, were receiving commissions or “kickbacks” from
suppliers.[5] To ascertain the veracity of the information given by Moralda, the top management of petitioner conducted an
investigation during which Jerome Lobitaña (Lobitaña), one of petitioner’s accredited suppliers, doing business under the name
and style “Precision Process,” came forward to corroborate the statement of Moralda.

Having been implicated in the irregularities, respondent, on November 3, 2003, received a Show Cause Letter with Notice of
Preventive Suspension from Plant Director Ariel Mendoza. On November 5, 2003, respondent submitted his written
explanation denying the accusations hurled against him. To further afford respondent ample opportunity to defend himself,
petitioner conducted a series of administrative investigation hearings during which respondent was able to face his accusers.

On March 22, 2004, respondent received the Notice of Termination dated March 19, 2004 informing him of his dismissal from
employment effective immediately on the ground of loss of trust and confidence. [ At the time of his termination, respondent was a
Power Plant Manager earning a monthly salary of P71,100.00. On March 31, 2004, respondent filed with the Regional Arbitration
Branch VII of the National Labor Relations Commission (NLRC) in Cebu City a complaint for illegal dismissal

Issue:

WON respondent was validly dismissed

Ruling:

Yes.

To validly dismiss an employee on the ground of loss of trust and confidence under Article 282 (c) of the Labor Code of the
Philippines, the following guidelines must be observed: “1) loss of confidence should not be simulated; 2) it should not be used as
subterfuge for causes which are improper, illegal or unjustified; 3) it may not be arbitrarily asserted in the face of overwhelming
evidence to the contrary; and 4) it must be genuine, not a mere afterthought to justify earlier action taken in bad faith.” More
important, it “must be based on a willful breach of trust and founded on clearly established facts.
 
In this case, we agree with the NLRC that the termination of respondent on the ground of loss of trust and confidence was
justified.  Unlike the Labor Arbiter and the CA, we find the testimony of Lobitaña credible and truthful. There are no inconsistensies
between the two affidavits executed by Lobitaña, and there appears to be no ill-motive on the part of Lobitaña to falsely accuse
respondent of accepting commissions and/or “kickbacks.”  

All told, we find that the testimony of Lobitaña constitutes substantial evidence to prove that respondent, as the then Power Plant
Manager, accepted commissions and/or “kickbacks” from suppliers, which is a clear violation of Section 2.04 of petitioner’s
Company Rules and Regulations. Jurisprudence consistently holds that for managerial employees “the mere existence of a basis
for believing that such employee has breached the trust of his employer would suffice for his dismissal.”  [60] As we then see it,
respondent’s termination was for a just and valid cause.
48. Cosmos Bottling vs Fermin, GR 193676

Principle: Theft committed against a co-employee is considered as a case analogous to serious


misconduct, and an employee terminated for this cause is considered dismissed for a just cause. Such
employee is not entitled to payment of his full retirement benefits or backwages. The award of financial
compensation or assistance to an employee validly dismissed from service due to serious misconduct has
no basis in law.

Facts:

Wilson B. Fermin (Fermin) was a forklift operator at Cosmos Bottling Corporation (COSMOS), where he started his
employment on 27 August 1976.[4] On 16 December 2002, he was accused of stealing the cellphone of his fellow
employee, Luis Braga (Braga).[5] Fermin was then given a Show Cause Memorandum, requiring him to explain why
the cellphone was found inside his locker.[6] In compliance therewith, he submitted an affidavit the following day,
explaining that he only hid the phone as a practical joke and had every intention of returning it to Braga.
After conducting an investigation, COSMOS found Fermin guilty of stealing Braga’s phone in violation of company
rules and regulations.[9] Consequently, on 2 October 2003,[10] the company terminated Fermin from employment after
27 years of service,[11] effective on 6 October 2003.[12] [
Fermin filed a Complaint for Illegal Dismissal.

Issue:

WON Fermin was validly dismissed

Ruling:

Yes.
It must be noted that in the case at bar, all the lower tribunals were in agreement that Fermin’s act of taking
Braga’s cellphone amounted to theft. Factual findings made by administrative agencies, if established by
substantial evidence as borne out by the records, are final and binding on this Court.
Theft committed against a co-employee is considered as a case analogous to serious misconduct, for which the
penalty of dismissal from service may be meted out to the erring employee.

Finally, it must be emphasized that the award of financial compensation or assistance to an employee validly
dismissed from service has no basis in law. Therefore, considering that Fermin’s act of taking the cellphone of his co-
employee is a case analogous to serious misconduct, this Court is constrained to reverse the CA’s ruling as regards
the payment of his full retirement benefits. In the same breath, neither can this Court grant his prayer for backwages. 
49. Reyes-Ravel vs. Philippine Luen Thai Holdings Corp., G.R. No. 174893, July 11, 2012
50. Reyes-Rayel vs. Philippine Luen Thai Holdings Corp., G.R. No. 174893, July 11, 2012

Facts:

In February 2000, PLTHC hired petitioner as Corporate Human Resources (CHR) Director for Manufacturing for its
subsidiary/affiliate company, L&T. In the employment contract, 5 petitioner was tasked to perform functions in relation
to administration, recruitment, benefits, audit/compliance, policy development/structure, project plan, and such other
works as may be assigned by her immediate superior, Frank Sauceda (Sauceda), PLTHC's Corporate Director for
Human Resources.

On September 6, 2001, petitioner received a Prerequisite Notice from Sauceda and the Corporate Legal Counsel of
PLTHC, Ma. Lorelie T. Edles with reference to her failure to perform in accordance with management directives in
various instances, which collectively have resulted in loss of confidence because on numerous occasions. Also, in the
presence of colleagues and subordinates, she made statements that serve to undermine the Company's efforts at
pursuing the HR2 Program of which the other colleagues complained about her.

She explained that her alleged failure to perform management directives could be attributed to the lack of effective
communication with her superiors due to malfunctioning email system. This caused her to miss certain directives
coming from her superiors and likewise, for her superiors to overlook the reports she was submitting. She denied
uttering negative comments about the HR2 Program and instead claimed to have intimated her support for it.

Petitioner was dismissed from the service for loss of confidence on her ability to promote the interests of the
company. This led petitioner to file a Complaint for illegal dismissal, payment of separation pay, 13th month pay,
moral and exemplary damages, attorney's fees, and other unpaid company benefits against respondents and its
officers.

Issue:

Whether or not there was a valid dismissal.

Ruling:

There exists a valid ground for petitioner's termination for employment.

Jurisprudence provides that an employer has a distinct prerogative and wider latitude of discretion in dismissing a
managerial personnel who performs functions which by their nature require the employer's full trust and confidence.
As distinguished from a rank and file personnel, mere existence of a basis for believing that a managerial employee
has breached the trust of the employer justifies dismissal. Loss of confidence as a ground for dismissal does not
require proof beyond reasonable doubt as the law requires only that there be at least some basis to justify it.

Petitioner was L&T's CHR Director for Manufacturing. As such, she was directly responsible for managing her own
departmental staff. It is therefore without question that the CHR Director for Manufacturing is a managerial position
saddled with great responsibility. Because of this, petitioner must enjoy the full trust and confidence of her superiors.
Not only that, she ought to know that she is "bound by more exacting work ethics" and should live up to this high
standard of responsibility.

Records show that petitioner indeed unreasonably failed to effectively communicate with her immediate superior.
There was an apparent neglect in her obligation to maintain constant communication with Sauceda in order to ensure
that her work is up to par. Second, the affidavits of petitioner's co-workers revealed her negative attitude and
unprofessional behavior towards them and the company. The third and most important is petitioner's display of
inefficiency and ineptitude in her job.

An employer has the right to regulate, according to its discretion and best judgment, all aspects of employment,
including work assignment, working methods, processes to be followed, working regulations, transfer of employees,
work supervision, lay-off of workers and the discipline, dismissal and recall of workers soo long as they are exercised
in good faith.
51. JOMAR S. VERDADERO, Petitioner, vs. BARNEY AUTOLINES GROUP OF COMPANIES TRANSPORT, INC.,
and/or BARNEY D. CHITO, ROSELA F. CHITO and GEIURDO GIMENEZ, G.R. No. 195428 . 29 August 2012

Facts:

Respondent Barney Autolines Group of Companies Transport, Inc. (BALGCO) hired Verdadero as bus conductor and
paid him a salary on commission basis. an altercation took place between Verdadero and respondent Atty. Gerardo
Gimenez (Gimenez), BALGCO’s Disciplinary Officer. Gimenez was on board BALGCO Bus together with his wife and
four other companions. Verdadero was then the assigned bus conductor. BALGCO has a company policy of granting
free rides to company employees and their wives. The story started when Verdadero began issuing fare tickets to
passengers, including the wife of Gimenez.

Gimenez filed an unverified complaint for serious misconduct against Verdadero before the BALGCO Management.
He requested Barney D. Chito (Barney) and Rosela F. Chito (Rosela), owners of BALGCO, to preside over the
conciliation proceedings. Verdadero, accompanied by his father, appeared at the BALGCO Office. Verdadero was
said to have shown willingness to be penalized for his misconduct provided no record of the proceedings would be
made. Gimenez, on the other hand, was willing to waive the imposition of any penalty if Verdadero would give a
simple letter of apology, which the latter supposedly agreed with his father guaranteeing the same. Verdadero,
instead, submitted his counter-affidavit refuting all allegations in the written complaint against him. Verdadero furtively
reported for work for fear of having another confrontation with Gimenez. Rosela sent Verdadero a letter requiring him
to immediately report for work and finish the pending disciplinary proceedings against him. Verdadero submitted his
Letter-Reply, explaining that he had been receiving threats. He likewise believed he was already illegally dismissed
as he was not given any work assignment.

Verdadero filed a complaint for illegal dismissal before the Labor Arbiter (LA), claiming, as well, non-payment of
holiday pay, premium on holiday, 13th month pay, separation pay, retirement benefits, moral and exemplary
damages, and reinstatement plus backwages. LA rendered a Decision dismissing Verdadero's complaint and
declaring that no dismissal took place but merely an administrative investigation. The LA reasoned that Verdadero
made it impossible for BALGCO to give him any trip assignment as he reported for work only when the respondents
were not around. Verdadero filed an appeal before the NLRC. The sworn statement of BALGCO Electrician, who
witnessed the incident, was given weight by the NLRC. It apparently found the sworn affidavit to be corroborative of
Verdadero's testimonies. For said reason, the NLRC partially granted the appeal. It ruled that Verdadero was illegally
dismissed and awarded him backwages and separation pay. The Court of Appeals found no constructive dismissal.
Hence this petition.

Issue:

Whether or not petitioner Verdadero was constructively dismissed.

Ruling:

None.

Verdadero alleges that he was employed as bus conductor of BALGCO from September 10, 2004 until January 28,
2008 when he was no longer allowed to report for work. He claims that he was not given any trip assignment since
the January 27, 2008 incident. He argues that when Gimenez committed the verbal abuse against him in the
presence of the bus passengers and threatened him with physical harm, there was termination by the employee of
his employment under the doctrine of constructive dismissal. BALGCO contends that Verdadero was not given any
trip assignment because he was surreptitiously reporting for work and would come to the office only when Gimenez
was not around. This was confirmed in the letterreply by Rosela to Verdadero. Verdadero admitted not reporting for
work after the incident “because of his mortal fear of being harmed by the Disciplinary Officer and his friends.

Constructive dismissal exists where there is cessation of work, because "continued employment is rendered
impossible, unreasonable or unlikely, as an offer involving a demotion in rank or a diminution in pay" and other
benefits. Aptly called a dismissal in disguise or an act amounting to dismissal but made to appear as if it were not,
constructive dismissal may, likewise, exist if an act of clear discrimination, insensibility, or disdain by an employer
becomes so unbearable on the part of the employee that it could foreclose any choice by him except to forego his
continued employment. In this case, Verdadero cannot be deemed constructively dismissed. Records do not show
any demotion in rank or a diminution in pay made against him. Neither was there any act of clear discrimination,
insensibility or disdain committed by BALGCO against Verdadero which would justify or force him to terminate his
employment from the company. To support his contention of constructive dismissal, Verdadero considers the verbal
abuse by Gimenez against him as an act which rendered his continued employment impossible, unreasonable or
unlikely. The claimed abuse was corroborated by the sworn written statement executed by Mascariña, which was
given credence by the NLRC and the CA. With the alleged threats of Gimenez, Verdadero believed that he could no
longer stay and work for BALGCO. It is to be emphasized that the abovementioned acts should have been committed
by the employer against the employee. Unlawful acts committed by a co-employee will not bring the matter within the
ambit of constructive dismissal.

Assuming arguendo that, Gimenez did commit the alleged unlawful acts, still, this fact will not suffice to conclude that
constructive dismissal was proper. Contrary to the arguments of Verdadero, Gimenez is not the employer. He may be
the “disciplinary officer,” but his functions as such, as can be gleaned from the BALGCO Rules and Regulations, do
not involve the power or authority to dismiss or even suspend an employee. Such power is exclusively lodged in the
BALGCO management. Gimenez remains to be a mere employee of BALGCO and, thus, cannot cause the dismissal
or even the constructive dismissal of Verdadero. The employers are BALGCO and its owners, Barney and Rosela.
Moreover, it was not established that BALGCO itself or its owners had been, in any way, forcing Verdadero to resign
from his employment. In fact, records show that the management had been urging him to report back to work, not
only to face the administrative charge against him, but also because of the scarcity and necessity of bus conductors
in the company. Verdadero, however, failed to present himself before the management, more specifically, to Rosela.
This situation provided no opportunity for BALGCO to give him any trip assignment. The abovementioned act of
BALGCO was even misinterpreted by Verdadero as yet another means of harassment. Furthermore, records are
bereft of any showing that Verdadero was no longer allowed to report for work starting January 28, 2008 when
Gimenez lodged a complaint for serious misconduct against him before the BALGCO management. It was Verdadero
himself who terminated his employment.

In the case at bench, considering that there has been no dismissal at all, there can be no reinstatement. In the same
vein, no separation pay can be awarded as it is given only in lieu of reinstatement. Consequently, there is likewise no
justification for the award of backwages.
52. ALEX Q. NARANJO, DONNALYN DE GUZMAN, RONALD V. CRUZ, ROSEMARIE P. PIMENTEL, and
ROWENA B. BARDAJE, Petitioners, vs BIOMEDICA HEALTH CARE, INC. and CARINA "KAREN" J. MOTOL,
Respondents. G.R. No. 193789. 19 September 2012

Facts:

Respondent Biomedica Health Care, Inc. (Biomedica) was, during the material period, engaged in the distribution of
medical equipment. Respondent Carina “Karen” J. Motol (Motol) was then its President. Petitioners were former
employees of Biomedica. On November 7, 2006, which happened to be Motol’s birthday, petitioners with two (2)
other employees, were all absent for various personal reasons. Notably, these are the same employees who filed a
letter-complaint addressed to Director Lourdes M. Transmonte, National Director, National Capital Region-
Department of Labor and Employment (DOLE) against Biomedica for lack of salary increases, failure to remit Social
Security System and Pag-IBIG contributions, and violation of the minimum wage law, among other grievances. Per
available records, the complaint has not been acted upon.

Later that day, petitioners reported for work after receiving text messages for them to proceed to Biomedica. They
were, however, refused entry and told to start looking for another workplace. The next day, petitioners allegedly came
in for work but were not allowed to enter the premises. Motol purportedly informed petitioners, using foul language, to
just find other employment. Biomedica issued a notice of preventive suspension and notices to explain within 24
hours (Notices) to petitioners. In the Notices, Biomedica accused the petitioners of having conducted an illegal strike
and were accordingly directed to explain why they should not be held guilty of and dismissed for violating the
company policy against illegal strikes. Petitioners filed a Complaint with the NLRC for constructive dismissal and
nonpayment of salaries, overtime pay, 13th month pay as well as non-remittance of SSS, Pag-IBIG and Philhealth
contributions as well as loan payments. Thereafter, Biomedica served Notices of Termination on petitioners.

The Labor Arbiter dismissed the complaint for lack of merit but ordered respondents to pay. The Labor Arbiter found
that, indeed, petitioners engaged in a mass leave akin to a strike. He added that, assuming that petitioners were not
aware of the company policies on illegal strikes, such mass leave can sufficiently be deemed as serious misconduct
under Art. 282 of the Labor Code. Thus, the Labor Arbiter concluded that petitioners were validly dismissed. Unlike
the Labor Arbiter, the NLRC found and so declared petitioners to have been illegally dismissed. The Court of
Appeals, however, reversed the decision of the NLRC and reinstated the decision of the Labor Arbiter.

Issue:

Whether or not petitioners were illegally dismissed.

Ruling:
Petitioners were illegally dismissed.

The fundamental law of the land guarantees security of tenure, thus:

Sec. 3. The State shall afford full protection to labor x x x.


x x x They shall be entitled to security of tenure, humane conditions of work and a living wage. x x x

On the other hand, the Labor Code promotes the right of the worker to security of tenure protecting them against
illegal dismissal under ARTICLE 279.

It bears pointing out that in the dismissal of an employee, the law requires that due process be observed. Such due
process requirement is twofold, procedural and substantive, that is, “the termination of employment must be based on
a just or authorized cause of dismissal and the dismissal must be effected after due notice and hearing.” In the instant
case, petitioners were not afforded both procedural and substantive due process.

Petitioners were not afforded procedural due process

Art. 277(b) of the Labor Code contains the procedural due process requirements in the dismissal of an employee. On
the other hand, Rule XIII, Book V, Sec. 2 I (a) of the Implementing Rules and Regulations of the Labor Code states:
SEC. 2. Standards of due process; requirements of notice.––In all cases of termination of employment, the following
standards of due process shall be substantially observed:

I. For termination of employment based on just causes as defined in Article 282 of the Code:

(a) A written notice served on the employee specifying the ground or grounds for termination, and giving
said employee reasonable opportunity within which to explain his side.
xxx
The Court elaborated in King of Kings Transport, Inc. v. Mamac that a mere general description of the charges
against an employee by the employer is insufficient to comply with the provisions of the law:
x x x Moreover, in order to enable the employees to intelligently prepare their explanation and defenses, the notice
should contain a detailed narration of the facts and circumstances that will serve as basis for the charge
against the employees. A general description of the charge will not suffice. Lastly, the notice should specifically
mention which company rules, if any, are violated and/or which among the grounds under Art. 282 is being charged
against the employees.

In the instant case, the notice specifying the grounds for termination dated November 9, 2006 states:

Effective upon receipt hereof, you are placed under preventive suspension for willfully organizing and/or engaging in
illegal strike on November 7, 2006. Your said illegal act-in conspiracy with your other co-employees, paralyzed the
company operation on that day and resulted to undue damage and prejudice to the company and is direct
violation of Article XI, Category Four Section 6, 8, 12, 18 & 25 of our Company Policy, which if found guilty,
you will be meted a penalty of dismissal. Please explain in writing within 24 hours from receipt hereof why you
should not be held guilty of violating the company policy considering further that you committed and timed such act
during the birthday of our Company president.

Clearly, petitioners were charged with conducting an illegal strike, not a mass leave, without specifying the exact acts
that the company considers as constituting an illegal strike or violative of company policies. Such allegation falls short
of the requirement in King of Kings Transport, Inc. of “a detailed narration of the facts and circumstances that will
serve as basis for the charge against the employees.” A bare mention of an “illegal strike” will not suffice.

Further, while Biomedica cites the provisions of the company policy which petitioners purportedly violated, it failed to
quote said provisions in the notice so petitioners can be adequately informed of the nature of the charges against
them and intelligently file their explanation and defenses to said accusations. The notice is bare of such description of
the company policies. Moreover, it is incumbent upon respondent company to show that petitioners were duly
informed of said company policies at the time of their employment and were given copies of these policies. No such
proof was presented by respondents. There was even no mention at all that such requirement was met. Worse,
respondent Biomedica did not even quote or reproduce the company policies referred to in the notice.

Moreover, the period of 24 hours allotted to petitioners to answer the notice was severely insufficient and in violation
of the implementing rules of the Labor Code. Under the implementing rule of Art. 277, an employee should be given
“reasonable opportunity” to file a response to the notice. “Reasonable opportunity” under the Omnibus Rules means
every kind of assistance that management must accord to the employees to enable them to prepare adequately for
their defense. This should be construed as a period of at least five (5) calendar days from receipt of the notice
to give the employees an opportunity to study the accusation against them, consult a union official or
lawyer, gather data and evidence, and decide on the defenses they will raise against the complaint.

In addition, Biomedica did not set the charges against petitioners for hearing or conference in accordance with Sec.
2, Book V, Rule XIII of the Implementing Rules and Regulations of the Labor Code. While petitioners did not submit
any written explanation to the charges, it is incumbent for Biomedica to set the matter for hearing or conference to
hear the defenses and receive evidence of the employees. More importantly, Biomedica is duty-bound to exert
efforts, during said hearing or conference, to hammer out a settlement of its differences with petitioners. These
prescriptions Biomedica failed to satisfy.

Lastly, Biomedica again deviated from the dictated contents of a written notice of termination as laid down in Sec. 2,
Book V, Rule XIII of the Implementing Rules that it should embody the facts and circumstances to support the
grounds justifying the termination. The Notice of Termination issued by Biomedica miserably failed to satisfy the
requisite contents of a valid notice of termination, as it simply mentioned the failure of petitioners to submit their
respective written explanations without discussing the facts and circumstances to support the alleged violations of
Secs. 6, 8, 12, 18 and 25 of Category Four, Art. XI of the alleged company rules.

Petitioners were denied substantive due process

In any event, petitioners were also not afforded substantive due process, that is, they were illegally dismissed. The
just causes for the dismissal of an employee are exclusively found in Art. 282(a) of the Labor Code. It was on the
ground of serious misconduct that the CA upheld the dismissal of petitioners from their employment.

Misconduct has been defined as improper or wrong conduct; the transgression of some established and
definite rule of action, a forbidden act, a dereliction of duty, unlawful in character implying wrongful intent
and not mere error of judgment. The misconduct to be serious must be of such grave and aggravated character
and not merely trivial and unimportant. In the instant case, Biomedica failed to even establish that petitioners indeed
violated company rules, failing to even present a copy of the rules and to prove that petitioners were made aware of
such regulations. In fact, from the records of the case, Biomedica has failed to prove that petitioners are guilty of a
wrongdoing that is punishable with termination from employment. Art. 277(b) of the Labor Code states, “The burden
of proving that the termination was for a valid or authorized cause shall rest on the employer.” In the instant case,
Biomedica failed to overcome such burden.

Petitioners did not stage a mass leave

The accusation is for engaging in a mass leave tantamount to an illegal strike. The term “Mass Leave” has been left
undefined by the Labor Code. Plainly, the legislature intended that the term’s ordinary sense be used. “Mass” is
defined as “participated in, attended by, or affecting a large number of individuals; having a large-scale character.”
While the term “Leave” is defined as “an authorized absence or vacation from duty or employment usually with pay.”
Thus, the phrase “mass leave” may refer to a simultaneous availment of authorized leave benefits by a large number
of employees in a company. It is undeniable that going on leave or absenting one’s self from work for personal
reasons when they have leave benefits available is an employee’s right. In addition to sick leave, the company, as a
policy or practice or as agreed to in a CBA, grants vacation leave to employees. Lastly, even the Labor Code grants a
service incentive leave of 5 days to employees.

In the factual milieu at bar, Biomedica did not submit a copy of the CBA or a company memorandum or circular
showing the authorized sick or vacation leaves which petitioners can avail of. Neither is there any document to show
the procedure by which such leaves can be enjoyed. Absent such pertinent documentary evidence, the Court can
only conclude that the availment of petitioners of their respective leaves on November 7, 2006 was authorized, valid
and in accordance with the company or CBA rules on entitlement to and availment of such leaves.

Moreover, a mass leave involves a large number of people or in this case, workers. Here, the five (5) petitioners were
absent on November 7, 2006. The records are bereft of any evidence to establish how many workers are employed in
Biomedica. There is no evidence on record that 5 employees constitute a substantial number of employees of
Biomedica. Having failed to show that there was a mass leave, the Court concludes that there were only individual
availment of their leaves by petitioners and they cannot be held guilty of any wrongdoing, much less anything to
justify their dismissal from employment. On this ground alone, the petition must be granted.

Petitioners did not go on strike

Granting for the sake of argument that the absence of the 5 petitioners on November 7, 2006 is considered a mass
leave, still, their actions cannot be considered a strike. Art. 212(o) of the Labor Code defines a strike as “any
temporary stoppage of work by the concerted action of employees as a result of any industrial or labor dispute.”
“Concerted” is defined as “mutually contrived or planned” or “performed in unison.”38 In the case at bar, the 5
petitioners went on leave for various reasons. Biomedica did not prove that the individual absences can be
considered as “temporary stoppage of work.” Biomedica’s allegation that the mass leave “paralyzed the company
operation on that day” has remained unproved.

Dismissal is not the proper penalty

But setting aside from the nonce the facts established above, the most pivotal argument against the dismissal of
petitioners is that the penalty of dismissal from employment cannot be imposed even if we assume that petitioners
went on an illegal strike. It has not been shown that petitioners are officers of the Union. In the instant case,
Biomedica has not alleged, let alone, proved the commission by petitioners of any illegal act during the alleged mass
leave. There being none, the mere fact that petitioners conducted an illegal strike cannot be a legal basis for their
dismissal.

Petitioners are entitled to separation pay in lieu of reinstatement, backwages and nominal damages.
53. THE NEW PHILIPPINE SKYLANDERS, INC. and/or JENNIFER M. ENANO-BOTE , vs. FRANCISCO N.
DAKILA,  G.R. No. 199547. September 24, 2012

FACTS:

Respondent Dakila was employed by petitioner corporation as early as 1987 and terminated for cause in April 1997
when the corporation was sold. In May 1997, he was rehired as consultant by the petitioners under a Contract for
Consultancy Services dated April 30, 1997.

Thereafter, in a letter dated April 19, 2007, respondent Dakila informed petitioners of his compulsory retirement
effective May 2, 2007 and sought for the payment of his retirement benefits pursuant to the Collective Bargaining
Agreement. His request, however, was not acted upon. Instead, he was terminated from service effective May 1,
2007.

Consequently, respondent Dakila filed a complaint for constructive illegal dismissal, non-payment of retirement
benefits, under/non-payment of wages and other benefits of a regular employee, and damages against petitioners
before the NLRC. He averred, among others, that the consultancy contract was a scheme to deprive him of the
benefits of regularization, claiming to have assumed tasks necessary and desirable in the trade or business of
petitioners and under their direct control and supervision.

On the other hand, petitioners, in their position paper, asserted that respondent Dakila was a consultant and not their
regular employee. The latter was not included in petitioners' payroll and paid a fixed amount under the consultancy
contract. He was not required to observe regular working hours and was free to adopt means and methods to
accomplish his task except as to the results of the work required of him. Hence, no employer-employee relationship
existed between them. Moreover, respondent Dakila terminated his contract in a letter dated April 19, 2007, thus,
negating his dismissal.

ISSUES:

Whether or not respondent Dakila is a regular employee; and whether or not he has been illegally dismissed.

RULING:

The issue of illegal dismissal is premised on the existence of an employer-employee relationship between the parties
herein. Records reveal that both the LA and the NLRC, as affirmed by the CA, have found substantial evidence to
show that respondent Dakila was a regular employee who was dismissed without cause. Respondent Dakila is found
to be a regular employee on the basis of the unrebutted documentary evidence showing that he was under the
petitioners' direct control and supervision and performed tasks that were either incidental or usually desirable and
necessary in the trade or business of petitioner corporation for a period of ten years.

Following Article 279 of the Labor Code, an employee who is unjustly dismissed from work is entitled to reinstatement
without loss of seniority rights and other privileges and to his full backwages computed from the time he was illegally
dismissed. However, considering that respondent Dakila was terminated on May 1, 2007, or one (1) day prior to his
compulsory retirement on May 2, 2007, his reinstatement is no longer feasible. Accordingly, the NLRC correctly held
him entitled to the payment of his retirement benefits pursuant to the CBA. On the other hand, his backwages should
be computed only for days prior to his compulsory retirement which in this case is only a day. Consequently, the
award of reinstatement wages pending appeal must be deleted for lack of basis.
54. LENN MORALES vs. METROPOLITAN BANK AND TRUST COMPANY, G.R. No. 182475               November
21, 2012

FACTS:

Petitioner Lenn Morales was hired by Solidbank as Teller for its Rizal Avenue Branch in Tacloban City. With said
bank’s merger with respondent Metrobank, the latter, as surviving entity, absorbed Morales and assigned him to its
Customer Service Relations-Reserve Pool (CSR-RP) which was composed of employees who, with no permanent
places of assignment, acted as relievers whenever temporary vacancies arise in other branches. Designated as
reliever for Metrobank’s Main Branch in Tacloban City, Morales was likewise assigned to work in the same capacity
for the bank’s other Visayas Region III branches. From a job with a grade four rank, Morales was subsequently
promoted  to the position of Customer Service Representative (CSR), with a job grade 6 rank. It was while occupying
the latter position that Morales was informed by Federico Mariano, the Senior Manager of Metrobank’s Tacloban City
Main Branch, that he was covered by the bank’s Special Separation Program (SSP) and that, in accordance
therewith, his employment was going to be terminated on the ground of redundancy.

Having signed a form on the same day signifying his unqualified and unconditional acceptance of Metrobank’s
decision to terminate his employment, Morales execute a Release, Waiver and Quitclaim acknowledging receipt of
the sum of P158,496.95 as full payment of his monetary entitlements.

On 20 February 2004, Morales filed against Metrobank a complaint for illegal dismissal, separation pay, backwages,
moral and exemplary damages as well as attorney’s fees.

ISSUE:

Whether or not the dismissal of petitioner was based on authorized cause of redundancy.

RULING:

One of the authorized causes for the dismissal of an employee, redundancy exists when the service capability of the
workforce is in excess of what is reasonably needed to meet the demands of the business enterprise. A position is
redundant when it is superfluous, and superfluity of a position or positions could be the result of a number of factors,
such as the overhiring of workers, a decrease in the volume of business or the dropping of a particular line or service
previously manufactured or undertaken by the enterprise. Time and again, it has been ruled that an employer has no
legal obligation to keep more employees than are necessary for the operation of its business. For the implementation
of a redundancy program to be valid, however, the employer must comply with the following requisites: (1) written
notice served on both the employees and the DOLE at least one month prior to the intended date of termination of
employment; (2) payment of separation pay equivalent to at least one month pay for every year of service; (3) good
faith in abolishing the redundant positions; and (4) fair and reasonable criteria in ascertaining what positions are to be
declared redundant and accordingly abolished.

In implementing a redundancy program, it has been ruled that the employer is required to adopt a fair and reasonable
criteria, taking into consideration such factors as (a) preferred status; (b) efficiency; and (c) seniority, among others.
Consistent with this principle, Metrobank established that, as a direct result of the adoption of the HRP, it was
determined that the volume of transactions in Visayas Region III required the further reduction of its eight-man
reserve pool by two employees. As these employees had no permanent place of assignment and merely acted as
relievers whenever temporary vacancies arise in other branches, they were the most logical candidates for inclusion
in the SSP. Already lacking preferred status in Metrobank’s hierarchy of positions, Morales was included in the SSP
because of his poor work performance which reportedly caused complaints from the branches where he was
temporarily assigned as reliever. To our mind, the foregoing circumstances contradict Morales’ claim that he was
arbitrarily singled out for termination by Metrobank which, having validly determined the surplus in its manpower
complement, appears to have appropriately identified him as a candidate for the SSP on account of his work attitude.
55. Mirant vs. Sario G.R. No. 197598, 21 Nov 2012

FACTS:

Danilo A. Sario (Sario) filed a complaint for illegal dismissal, backwages, damages and attorney’s fees against the
petitioner,Mirant (Philippines) Corporation (company), and its officers, namely: Harris (President); Sliman(Executive
Vice-President forOperations); and Aprieto (Officer-in-Charge, MMD). The company owns shares in Mirant Sual
Corporation and Mirant PagbilaoCorporation which operate power stations in the provinces of Pangasinan and
Quezon.Sario worked for the company as procurement officer from March 1998 to October 2005. Allegedly, the
companydiscovered that some of its employees had been involved in the rampant practice of favoring certain
suppliers, therebyseriously impairing transparency in its procurement process and compromising the quality of
purchased materials. To curbthe practice, the company issued the 2002 MMD Policies and Procedures Manual (2002
Procurement Manual) for theguidance of its employees and officers in soliciting bid quotations and proposals from
vendors, suppliers and contractors. The2002 Procurement Manual was duly disseminated through seminars and
it became effective in January 2002.The 2002 Procurement Manual was replaced by the 2004 Procurement Policies
and Procedures Manual (2004Procurement Manual) which was disseminated and which became effective on August
31, 2004. Again, seminars wereconducted and a proficiency examination was administered to familiarize the
company buyers/procurement officers and theteam leaders with the 2004 Procurement Manual. Sario took the
proficiency examination on September 28, 2004.On September 8, 2005, Sario received a Show Cause Notice from
the company, advising him that based on an internalaudit, he was found to have committed the following
violations:1. Non-compliance with the Minimum Bid/Quotation Requirements;2. Non-compliance with the Single
Tender Justification Requirement;3. No Evidence of Independent Approval of the PRF;4. No Evidence of Authorized
Recommendation or Approval of the PO;5. PO not Awarded to the lowest Bidder; and6. No TAS Attached.Sario was
given ten (10) days, or until September 18, 2005, to explain why no disciplinary action should be taken against himfor
the violations. He was also notified that an investigation would be conducted on the matter. He was placed on
preventivesuspension pending the investigation.As a result of the administrative hearing, Sliman sent Sario a letter
informing him of the termination of hisemployment for his failure to comply with the standard operating
procedures/instructions; for his serious misconduct orwillful disobedience of the lawful orders of the company in
connection with his work; and for his gross and habitual neglect of his duties. The company found Sario liable for his
failure to comply with the 2002 and 2004 Procurement Manuals, especiallyhis unabated practice of sending Requests
for Quotation (RFQs) to suppliers who have a history of not responding to requestsor of not sending quotes. The
practice, the company lamented, resulted in the issuance of purchase orders to the lone bidders.Labor Arbiter (LA)
Anni declared Sario to have been illegally dismissed. Consequently, he ordered: (1) Sario’s immediate reinstatement
without loss of seniority rights and other privileges; and (2) the company, Sliman and Aprieto,jointly and severally, to
pay Sario back wages, moral damages, exemplary damages and attorney’s fees. Labor Arbiter Anniabsolved Harris
from liability.The respondents appealed to the NLRC, which reversed the labor arbiter’s ruling and dismissed the
complaint forlack of merit. Sario moved for reconsideration, but the NLRC denied the motion. He then sought relief
from the CA, through apetition for certiorari under Rule 65 of the Rules of Court.In its decision, the CA granted the
petition. It set aside the NLRC rulings and reinstated the Labor Arbiter’s decision, with modifications. It deleted the
award of moral and exemplary damages, and absolved Harris, Sliman and Aprieto fromliability in the case. Like
the Labor Arbiter, it found the penalty of dismissal meted on Sario too harsh. The company moved forpartial
reconsideration, but the CA denied the motion; hence, the present recourse.

ISSUE:

Whether Sario was legally dismissed.

RULING:

We find the petition meritorious. The Supreme Court held in favor of Mirant, finding that Sario was legally dismissed.
Under the law, the burden of proving that the termination of a worker’s employment was for a valid or authorized
cause rests on the employer. In this case, the company was able to prove that Sario’s dismissal was for a valid
cause. Throughhis repeated violations of the company’s 2002 and 2004 Procurement Manuals, Sario committed a
serious misconduct or willful disobedience of the lawful directives or orders of his employer, constituting a just cause
for termination of employment.  Sario was not an ordinary rank-and-file employee. He was a procurement officer. While
he did not occupy a highposition in the company hierarchy, the nature of his work made him, as the company avers, a
vital cog in its procurement program.

The effectiveness of the program depended in no small measure on the people running it, i.e., from the lowliest
employee to the highest official. Sario was one of these people and he was occupying, not a lowly but, a middle
position.

This position carries with it responsibilities which only he can, and should, answer for.As the records show, Sario
failed to faithfully discharge his duties as procurement officer. Over a span of almost one-and-a-half years, from
January 2004 to May 2005 (not two years as the company claims), Sario committed 27violations of the2002 and 2004
Procurement Manuals in critical areas of the procurement process, in particular, non-compliance with theminimum
bid/quotation requirements, non-compliance with the single tender justification requirement, failure to provideproof of
approval of the purchase requisition form, failure to provide proof of authorized recommendation of the purchaseorder, failure to award
purchase order to the lowest bidder, and no tender analysis summary. Sario’s transgressions cannot be mitigated by
the supposed approval of his actions by his superiors. Sario has toaccount for his own actions.

The circumstance that his recommendations were approved by his superiors does not erase thefact that he
repeatedly violated the 2002 and 2004 Procurement Manuals. He was well aware of his duties and theirparameters,
based on the 2002 and 2004 Procurement Manuals. He committed the violations for one-and-a-half years.
Theserepeated violations can only indicate a willful disobedience to reasonable company rules and regulations.Based
on the facts, the law and jurisprudence, Sario deserves to be dismissed for willful disobedience.

In Gold City Integrated Port Services, Inc. v. NLRC  the Court stressed that willful disobedience of an employee
contemplates the concurrence of at least two requisites: the employee’s assailed conduct must have been willful or
intentional, the willfulnes sbeing characterized by a "wrongful and perverse attitude"; and the order violated must
have been reasonable, lawful andmade known to the employee, and must pertain to the duties which he had been
engaged to discharge. We find the tworequisites present in this case. Sario’s repeated violations of the company’s
2002 and 2004 Procurement Manuals lawful orders in themselves as they provide the dos and, necessarily, the
don’ts of a procurement officer constitute willful disobedience. He committed therepeated violations because he knew
or was confident that he would not get caught since his actions were being approved, ashe claims, by his superiors,
evidencing wrongful or perverse intent. While the Constitution urges the moderation of thesanction that may be applied to an
employee where a penalty less punitive would suffice, as the Court pronounced in MarivalTrading, Inc. v. NLRC, cited
by the CA, we do not believe that such a moderation is proper in this case. Sario has become unfit to remain in
employment. A contrary view would be oppressive to the employer. "The law, in protecting the rights of thelaborer, authorizes neither
oppression nor self-destruction of the employer."
56. Barba vs. Liceo De Cagayan University

Facts:

Petitioner Dr. Ma. Mercedes L. Barba was the Dean of the College of Physical Therapy of respondent Liceo de
CagayanUniversity, Inc., a private educational institution with school campus located at Carmen, Cagayan de Oro
City.Petitioner started working for respondent on July 8, 1993 as medical officer/school physician for a period of
oneschool year or until March 31, 1994. In July 1994, she was chosen by respondent to be the recipient of a
scholarship grant topursue a three-year residency training in Rehabilitation Medicine at the Veterans Memorial
Medical Center (VMMC). TheScholarship Contract provides:

That the SCHOLAR after the duration of her study and training shall serve the SCHOOL in whatever positionthe SCHOOL desires related to
the SCHOLAR’s studies for a period of not less than ten (10) years;
 
After completing her residency training with VMMC in June 1997, petitioner returned to continue working for respondent. Shewas appointed
as Acting Dean of the College of Physical Therapy and at the same time designated as Doctor-In-Charge of
theRehabilitation Clinic of the Rodolfo N. Pelaez Hall, City Memorial Hospital.

On June 19, 2002, petitioner’s appointment as Doctor-In-Charge of the Rehabilitation Clinic was renewed and she
wasappointed as Dean of the College of Physical Therapy by respondent’s President, Dr. Jose Ma. R. Golez.
Petitioner accepted herappointment and assumed the position of Dean of the College of Physical Therapy.In the
school year 2003 to 2004, the College of Physical Therapy suffered a dramatic decline in the number of  enrollees
from a total of 1,121 students in the school year 1995 to 1996 to only 29 students in the first semester of school
year2003 to 2004. This worsened in the next year or in school year 2004 to 2005 where a total of only 20 students
enrolled.

Due to the low number of enrollees, respondent decided to freeze the operation of the College of Physical Therapy
indefinitely. Respondent’s President wrote petitioner a letter informing her that her services as dean of the said
college willend at the close of the school year. Thereafter, the College of Physical Therapy ceased operations on
March 31, 2005, and petitioner went on leave without pay starting on April 9, 2005. Subsequently, respondent’s
Executive Vice President, Dr.Lerin,sent petitioner a letter instructing petitioner to return to work on June 1, 2005 and
report to Palomares, the Acting Dean of the College of Nursing, to receive her teaching load and assignment as a full-
time faculty member in that department for theschool year 2005-2006.In reply, petitioner informed Dr. Lerin that she
had not committed to teach in the College of Nursing and that as far asshe can recall, her employment is not
dependent on any teaching load. She then requested for the processing of her separationbenefits in view of
the closure of the College of Physical Therapy. Dr. Magdale, Vice-President for Academic Affairs, sent another letter
to petitioner ordering her to report for work asshe was still bound by the Scholarship Contract to serve respondent for two more years.
But petitioner did not do so.

Hence,on June 28, 2005, Dr. Magdale sent petitioner a notice terminating her services on the ground
of abandonment.Meanwhile, on June 22, 2005, prior to the termination of her services, petitioner filed a complaint
before the LaborArbiter for illegal dismissal, payment of separation pay and retirement benefits against respondent,
Dr. Magdale and Dr. Golez.She alleged that her transfer to the College of Nursing as a faculty member is a demotion
amounting to constructive dismissal.Labor Arbiter (LA) found that respondent did not constructively dismiss petitioner;
therefore, she was not entitled to separation pay. The Labor Arbiter held that petitioner’s assignment as full-time
professor in the College of Nursing was not ademotion tantamount to constructive dismissal.Petitioner appealed the
above decision to the NLRC. NLRC reversed the LA’s decision and held that petitioner wasconstructively dismissed.
The NLRC held that petitioner was demoted when she was assigned as a professor in the College of Nursing
because there are functions and obligations and certain allowances and benefits given to a College Dean but not to
anordinary professor.Respondent went to the CA on a petition for certiorari alleging that the NLRC committed grave abuse of
discretion when it declared that petitioner’s transfer to the College of Nursing as full-time professor but without
diminution of salariesand without loss of seniority rights amounted to constructive dismissal because there was
a demotion involved in the transferand because petitioner was compelled to accept her new assignment.The CA
reversed and set aside the NLRC resolutions and reinstated the decision of the Labor Arbiter. The CA did not find
merit in respondent’s assertion in its Supplemental Petition that the position of petitioner as College Dean was a
corporate office. Instead, the appellate court held that petitioner was respondent’s employee

ISSUE:

Whether petitioner was constructively dismissed.

RULING:

We grant the petition. The Supreme Court ruled in favor of the University, finding that Barba was not
constructivelydismissed, but was instead validly transferred.On the issue of constructive dismissal, we agree with the
Labor Arbiter and the appellate court’s earlier ruling that petitioner was not constructively dismissed. Petitioner’s letter
of appointment specifically appointed her as Dean of the College of Physical Therapy and Doctor-in-Charge of the
Rehabilitation Clinic “ for a period of three years effective July 1, 2002 unless sooner revoked for valid cause or causes.”
Evidently, petitioner’s appointment as College Dean was for a fixed term, subject to reappointment and revocation or
termination for a valid cause. When respondent decided to close its College of Physical Therapy due to drastic
decrease in enrollees, petitioner’s appointment as its College Dean was validly revoked andher subsequent
assignment to teach in the College of Nursing was justified as it is still related to her scholarship studies inPhysical
Therapy.As we observed in Brent School, Inc. v. Zamora also cited by the CA, it is common practice in
educational institutionsto have fixed-term contracts in administrative positions, thus:

Some familiar examples may be cited of employment contracts which may be neither for seasonal work nor forspecific projects, but to which a
fixed term is an essential and natural appurtenance: overseas employment contracts, for one, to which, whatever the nature of the
engagement, the concept of regular employment with allthat it implies does not appear ever to have been applied, Article 280 of the Labor Code
notwithstanding; alsoappointments to the positions of dean, assistant dean, college secretary, principal, and
otheradministrative offices in educational institutions, which are by practice or tradition rotated among thefaculty
members, and where fixed terms are a necessity without which no reasonable rotation would bepossible.x x x

In constructive dismissal cases, the employer has the burden of proving that its conduct and action or the transfer of
anemployee are for valid and legitimate grounds such as genuine business necessity.

Particularly, for a transfer not to beconsidered a constructive dismissal, the employer must be able to show that such
transfer is not unreasonable, inconvenient, or prejudicial to the employee. In this case, petitioner’s transfer was not
unreasonable, inconvenient or prejudicial to her. Onthe contrary, the assignment of a teaching load in the College of
Nursing was undertaken by respondent to accommodatepetitioner following the closure of the College of Physical
Therapy. Respondent further considered the fact that petitioner stillhas two years to serve the university under the
Scholarship Contract. Petitioner’s subsequent transfer to another department or college is not tantamount to
demotion as it was a valid transfer. There is therefore no constructive dismissal to speak of. That petitioner ceased to
enjoy the compensation, privilegesand benefits as College Dean was but a logical consequence of the valid
revocation or termination of such fixed-term position.Indeed, it would be absurd and unjust for respondent to maintain
a deanship position in a college or department that hasceased to exist. Under the circumstances, giving petitioner a
teaching load in another College/Department that is related toPhysical Therapy thus enabling her to serve and
complete her remaining two years under the Scholarship Contract is avalid exercise of management prerogative on
the part of respondent.

CA held that contrary to theallegation of petitioner, the position of Dean does not appear to be the same as that of a
College Director.Aside from the obvious disparity in name, the By-Laws of LDCU provides for onlyone College
Director. But as shownby LDCU itself,

Numerous persons have been appointed as Deans. If it is indeed the intention of LDCU to give its manyDeans the rank of College Director,
then it exceeded the authority given to it by its By-Laws because only one CollegeDirector is authorized to be appointed. It must amend
its By-Laws.Petitioner filed a motion for reconsideration from the above decision, but her motion was denied by the
CA. Hence, petitioner filed the present petition.
57. SAMPAGUITA AUTO TRANSPORT CORPORATION vs. NLRC, G.R. No. 197384. January 30,
2013

Facts:

Sagad alleged that on May 14, 2006, the company hired him as a regular bus driver, not as a probationary employee
as the company claimed. He disowned his purported signature on the contract of probationary employment 5
submitted in evidence by the company. He maintained that his signature was forged. He further alleged that on
November 5, 2006, he was dismissed by the company for allegedly conniving with conductor Vitola in issuing tickets
outside their assigned route. EScIAa

The company countered that it employed Sagad as a probationary bus driver 6 from May 14, 2006 to October 14,
2006; he was duly informed of his corresponding duties and responsibilities. 7 He was further informed that during the
probationary period, his attendance, performance and work attitude shall be evaluated to determine whether he
would qualify for regular employment. For this purpose and as a matter of company policy, an evaluator was
deployed on a company bus (in the guise of a passenger) to observe the driver's work performance and attitude.

Allegedly, on September 21, 2006, an evaluator boarded Sagad's bus. The evaluator described Sagad's manner of
driving as "reckless driver, nakikipaggitgitan, nakikipaghabulan, nagsasakay sa gitna ng kalsada, sumusubsob ang
pasahero[.]"8 Sagad disputed the evaluator's observations. In an explanation (rendered in Filipino), 9 he claimed that
he could not have been driving as reported because his wife (who was pregnant) and one of his children were with
him on the bus. He admitted though that at one time, he chased an "Everlasting" bus to serve warning on its driver
not to block his bus when he was overtaking. He also admitted that once in a while, he sped up to make up for lost
time in making trips.

The company further alleged that on October 13, 2006, it conducted a thorough evaluation of Sagad's performance. It
requested conductors who had worked with Sagad to comment on his work. Conductors A. Hemoroz and Israel
Lucero revealed that Sagad proposed that they cheat on the company by way of an unreported early bus trip. 10 The
company also cited Sagad's involvement in a hit-and-run accident on September 9, 2006 along Commonwealth
Avenue in Quezon City while on a trip. Allegedly, Sagad did not report the accident to the company.

On October 15, 2006, upon conclusion of the evaluation, the company terminated Sagad's employment for his failure
to qualify as a regular employee.

Issue:

Whether or not Saga was dismissed illegally.

Ruling:

Sagad was dismissed, not as a probationary employee, but as one who had attained regular status. The company's
evidence on Sagad's purported hiring as a probationary employee is inconclusive.
The NLRC resolved the doubt in Sagad's favor, ruling that Sagad's signature on the contract was not his, because it
was a forgery. It declared that his signature on the contract "is extremely different from those in his pleadings and
from the other documents on record[,]"33 without explaining how and why the two sets of signatures were vastly
different. Lending further support to the NLRC conclusion, which the CA upheld, is its finding that the company failed
to refute Sagad's denial of his signature in the contract, which the labor tribunal considered as an admission of the
veracity of Sagad's statement, pursuant to the Rules of Court.

Independently of the above discussion and even if we were to consider that Sagad went through a probationary
period, the records indicate that he was retained even beyond the expiration of his supposed probationary
employment on October 14, 2006. As the NLRC noted, Sagad claimed that he was dismissed by the company on
November 5, 2006, after he was accused of conniving with conductor Vitola in issuing tickets outside their assigned
route.

The company never refuted this particular assertion of Sagad and its silence can only mean that Sagad remained in
employment until November 4, 2006, thereby attaining regular status as of that date. Under the law, "an employee
who is allowed to work after a probationary period shall be considered a regular employee."

Is Sagad's dismissal illegal?

We disagree with the finding that Sagad's dismissal had no basis.

First. It is not disputed that the company called Sagad's attention to his negative actuations as a bus driver, which
were reported by a company evaluator 38 who boarded his bus on September 21, 2006. The evaluator reported that
he was driving recklessly, racing and jostling for position on the road, thereby jarring the passengers on their seats,
and picking up passengers on the middle of the road. He disputed the evaluator's observations, 39 claiming that he
could not have been driving as reported because his pregnant wife and one of his children were with him on the bus
at the time. He admitted, however, that on one occasion, he chased an "Everlasting" bus to warn its driver not to
block him. He also admitted that once in a while, he sped up to compensate for lost time in his trips.

Sagad's explanation reveals more than what it stated. During his brief employment with the company, he
exhibited the tendency to speed up when he finds the need for it, very obviously in violation of traffic rules, regulations
and company policy. Instead of negating the evaluator's observations, his admissions make them credible.

Second. He was also asked to react to the comments of conductors who had worked with him (Hemoroz and Lucero)
to the effect that he proposed to them that they cheat on the company by making early (but not to be reported) bus
trips. Further, there was Castillo's evaluation dated October 13, 2006, rating Sagad's work performance as poor on
account of: (1) the low revenue of Sagad's bus; (2) his inability to make all his scheduled trips; and (3) his habit of
bringing his wife with him on his trips. Castillo also heard of talks of Sagad's orders to the conductors to earn money
in a questionable way.

During the arbitration, Sagad disputed the conductors' comments, maintaining that they were not under oath and that
the fraudulent proposal they mentioned could only be committed by conductors. With respect to Castillo's evaluation,
Sagad invoked the CA's pronouncement that the infractions mentioned in the report are not just causes for the
termination of his employment.

Sagad's position fails to convince us. We find no evidence that Hemoroz and Lucero had an ax to grind against
Sagad so that they would lie about their impression of him as a bus driver. Significantly, their statements validate
Castillo's own observation that he heard talks of Sagad's orders to the conductors for them to cheat on the company.
The scheme, contrary to Sagad's explanation, can only be committed with the cooperation, or even at the behest, of
the driver, as the proposed scheme is for the bus to make unscheduled, but unreported, early trips.

Lastly, the company cites Sagad's involvement in a hit-and-run incident on September 9, 2006 while driving his
assigned bus (with Plate No. NYK-216 and Body No. 3094). Once more, he denies the charge, claiming that it was
not his bus, but two other vehicles, a Honda City and an Elf truck, which figured in the incident. To prove his point, he
submitted the "SALAYSAY" of his replacement driver, Carlito Laude, for September 10, 2006, saying that there was
no dents or scratches on the bus.

Again, Sagad's stance fails to persuade us. Sagad's statements vis-à-vis the incident, as well as those of Laude,
are belied by the Traffic Accident Investigation Report which mentioned the "Unidentified driver of Public Utility Bus
with plate No. NYK-216 and Body No. 3094." The report was corroborated by the sworn statements of Ronald Apura,
driver of the Elf truck, UFF-597, the second party in the incident, and Bibiana Fuentes, driver of the White Honda City,
WDV-422 (owned by Purefoods Hormel Co.), the first party in the vehicular accident. There was also the letter to the
company of Standard Insurance Co., Inc. dated February 14, 2007 47 demanding the reimbursement of P24,667.54 it
paid to Purefoods Hormel Co. by way of damages sustained by the Honda City.

Third. The CA misappreciated the law when it declared that the grounds relied upon by the company in terminating
Sagad's employment are not among those enumerated under Article 282 of the Labor Code as just causes for
employee dismissals. Article 282 of the Code provides:

Art. 282. Termination by employer. — An employer may terminate an employment for any of the
following causes:
(a) Serious misconduct or willful disobedience by the employee of the lawful orders of his employer or
representative in connection with his work;
(b) Gross and habitual neglect by the employee of his duties;
(c) Fraud or willful breach by the employee of the trust reposed in him by his employer or duly authorized
representative;
(d) Commission of a crime or offense by the employee against the person of his employer or any
immediate member of his family or his duly authorized representative; and
(e) Other causes analogous to the foregoing.

The irregularities or infractions committed by Sagad in connection with his work as a bus driver constitute a
serious misconduct or, at the very least, conduct analogous to serious misconduct, under the above-cited
Article 282 of the Labor Code. To be sure, his tendency to speed up during his trips, his reckless driving, his picking
up passengers in the middle of the road, his racing with other buses and his jostling for vantage positions do not
speak well of him as a bus driver. While he denies being informed, when he was hired, of the duties and
responsibilities of a driver — contained in a document submitted in evidence by the company — the
requirement ". to obey traffic rules and regulations as well as the company policies. . to ensure the safety of
the riding public as well as the other vehicles and motorist (sic)" is so fundamental and so universal that
any bus driver is expected to satisfy the requirement whether or not he has been so informed.

Sagad tries to minimize the adverse effect of the evaluator's report of September 21, 2006 about his conduct as a
driver with the argument that he had already been penalized with a five-day suspension for chasing an "Everlasting"
bus at one time. The suspension is of no moment. He was penalized for one reckless driving incident, but it does not
erase all the other infractions he committed. The conductors' comments and the dispatcher's evaluation, together with
the earlier on-board evaluation, all paint a picture of a reckless driver who endangers the safety of his passengers,
other motorists and the general public. With this record, it is not surprising that he figured in a hit-and-run accident on
September 9, 2006.
Under the circumstances, Sagad has become a liability rather than an asset to his employer, more so when we
consider that he attempted to cheat on the company or could have, in fact, defrauded the company during his brief
tenure as a bus driver. This calls to mind Castillo's report on the low revenue of Sagad's bus, an observation which is
validated by the company's Daily Operation Reports from June to October 2006.

All told, we find substantial evidence supporting Sagad's removal as a bus driver. Through his reckless
driving and his schemes to defraud the company, Sagad committed serious misconduct and breach of the
trust and confidence of his employer, which, without doubt, are just causes for his separation from the
service. It is well to stress, at this point, an earlier pronouncement of the Court "that justice is in every case for the
deserving, to be dispensed in the light of the established facts and applicable law and doctrine."

The twin-notice requirement


Even as we find a just cause for Sagad's dismissal, we agree with the CA that the company failed to comply with the
two-notice rule. It failed to serve notice of: (1) the particular acts for which Sagad was being dismissed on November
5, 2006 and (2) his actual dismissal. Consistent with our ruling in Agabon v. NLRC, 52 we hold that the violation of
Sagad's right to procedural due process entitles him to an indemnity in the form of nominal damages. Considering the
circumstances in the present case, we deem it appropriate to award Sagad P30,000.00.
58. Sampaguita Auto Transport Corp. vs. NLRC, et al., G.R. No. 197384, Jan. 30, 2013

Facts:

On May 14, 2006, Sagad was hired by the company hired him as a regular bus driver until his dismissal on November
5, 2006 for allegedly conniving with conductor Vitola in issuing tickets outside their assigned route
However, the company claimed that it employed Sagad as a probationary bus driver (evidenced by a probationary
employment contract) from May 14, 2006 to October 14, 2006; he was duly informed of his corresponding duties and
responsibilities. He was further informed that during the probationary period, his attendance, performance and work
attitude shall be evaluated to determine whether he would qualify for regular employment. For this purpose and as a
matter of company policy, an evaluator was deployed on a company bus (in the guise of a passenger) to observe the
driver's work performance and attitude.

On September 21, 2006, an evaluator boarded Sagad's bus. The evaluator described Sagad's manner of driving as
"reckless driver, nakikipaggitgitan, nakikipaghabulan, nagsasakay sa gitna ng kalsada, sumusubsob ang pasahero[.]"
Sagad disputed the evaluator's observations. In an explanation, he claimed that he could not have been driving as
reported because his wife (who was pregnant) and one of his children were with him on the bus. He admitted though
that at one time, he chased an "Everlasting" bus to serve warning on its driver not to block his bus when he was
overtaking. He also admitted that once in a while, he sped up to make up for lost time in making trips.

The company further alleged that on October 13, 2006, it conducted a thorough evaluation of Sagad's performance. It
requested conductors who had worked with Sagad to comment on his work. Conductors A. Hemoroz and Israel
Lucero revealed that Sagad proposed that they cheat on the company by way of an unreported early bus trip.
Dispatcher E. Castillo likewise submitted a negative report and even recommended the termination of Sagad's
employment. The company also cited Sagad's involvement in a hit-and-run accident on September 9, 2006 along
Commonwealth Avenue in Quezon City while on a trip (bus with Plate No. NYK-216 and Body No. 3094). Allegedly,
Sagad did not report the accident to the company.

On October 15, 2006, upon conclusion of the evaluation, the company terminated Sagad's employment for his failure
to qualify as a regular employee.

Issues:

I. WON the employee was dismissed as a regular employee, and not as a probationary.
II. WON the employee was dismissed for just cause.
III. WON the employer failed to observe procedural due process. Hence, the employee is entitled to nominal
damages.

Ruling:

I. The employee was dismissed as a regular employee, and not as a probationary.

The employer was not able to clearly establish that the employee was a probationary. The employee denied having
signed any probationary contract. Moreover, the employer had retained the employee beyond the supposed
probationary period of employment. The payslips presented by the employee evidences such extension.

In labor law, it is an elementary rule that "an employee who is allowed to work after a probationary period shall be
considered a regular employee."

II. The employee was dismissed for just cause.

The employee was evidently dismissed for just cause.


First, it was established that the employer called the employee's attention to his "negative actuations as a bus driver".
These were reported by a company evaluator who boarded the employee's bus. The evaluator reported that the
employee was "driving recklessly, racing and jostling for position on the road, thereby jarring the passengers on their
seats, and picking up passengers on the middle of the road."

By way of reply, the employee claimed that he could not have been driving recklessly as his wife and one of his
children were with him on the bus at the time of evaluation. However, he admitted to two things: (a) that, on one
occasion, he chased on "Everlasting" bus in order "to warn the driver not to block him", and (b) that, once in a while,
"he sped up to compensate for lost time in his trips".

"Sagad's explanation reveals more than what is stated. During his brief employment with the company, he exhibited
the tendency to speed up when he finds the need for it, very obviously in violation of traffic rules, regulations and
company policy. Instead of negating the evaluator's observations, his admissions make them credible."

Second, the two conductors who had worked with the employee commented that the employee "proposed to them
that they cheat on the company by making early trips (but not to be reported) bus trips". The two conductors had no
ax to grind against the employee. Furthermore, the evaluator's evaluation report rated the employee's work
performance as poor on account of: "(1) the low revenue of Sagad's bus; (2) his inability to make all his scheduled
trips; and (3) his habit of bringing his wife with him on trips". He likewise heard of talks of the employee's orders to the
conductors "to earn money in a questionable way".

As the two conductors have no ax to grind with the employee, their testimony is credible. In addition, their testimony
validates the evaluator's observation that he heard talks of the employee's orders to the conductors "for them to cheat
on the company".

"x x x The scheme, contrary to Sagad's explanation, can only be committed with the corporation, or even at the
behest of the driver, as the proposed scheme is for the bus to make unscheduled, but unreported, early trips.

Third, the employee's involvement in a hit-and-run incident while driving his assigned bus was clearly established.
The following were the substantial evidence presented: (1) the Traffic Accident Investigation Report; (2) Sworn
Statement of the driver of the Elf truck, which was hit; (3) Sworn Statement of the driver of the White Honda City
(owned by Purefoods Hormel Co.), the first party to the vehicular accident; and, (4) Demand Letter by the insurance
company demanding reimbursement it paid to Purefoods.

In view of the above-mentioned grounds, the "irregularities or infractions committed by Sagad in connection with his
work as a bus driver constitutes a serious misconduct under Article 282 of the Labor Code.
"The irregularities or infractions committed by Sagad in connection with his work as a bus driver constitutes serious
misconduct or, at the very least, conduct analogous to serious misconduct, under the above-cited Article 282 of the
Labor Code. To be sure his tendency to speed up during his trips, his reckless driving, his picking up passengers in
the middle of the road, his racing with other buses and his jostling for vantage positions do not speak well of him as a
bus driver. While he denies being informed when he was hired, of the duties and responsibilities of a driver contained
in a document submitted in evidence by the company - the requirement "3. to obey traffic rules and regulations as
well as the company policies. 4. to ensure the safety of the riding public as well as the other vehicles and motorist
(sic)" is so fundamental and so universal that any bus driver is expected to satisfy the requirement whether or not he
has been so informed." (Citations omitted.)

While the employee tried to minimize the adverse effect of the evaluator's report by claiming that he had already been
penalized by a 5-day suspension, the same is of no moment. "He was penalized for one reckless driving incident, but
it does not erase all other infractions he committed."

III. The employer failed to observe procedural due process. Hence, the employee is entitled to nominal damages.

While the employee was dismissed based on a just cause, the employer failed to comply with the two-notice rule.That
is to say, the employer failed to serve: (1) the initial notice - stating the particular acts on which the employee is being
dismissed on 05 November 2006, and (2) the termination notice.

Following Agabon v. NLRC, the violation of the employee's right to procedural due process entitles him to nominal
damages in the amount of Php30,000.00.
59. GENERAL MILLING CORPORATION, vs. VIOLETA L. VIAJAR. G.R. No. 181738. January 30, 2013.

Facts:

GMC is a domestic corporation with principal office in Makati City and a manufacturing plant in Lapu-Lapu City.

In October 2003, GMC terminated the services of thirteen (13) employees for redundancy, including herein respondent, Violeta
Viajar (Viajar). GMC alleged that it has been gradually downsizing its Vismin (Visayas-Mindanao) Operations in Cebu where a
sizeable number of positions became redundant over a period of time.

When Viajar reported for work on October 31, 2003, almost a month before the effectivity of her severance from the company,
the guard on duty barred her from entering GMC's premises. She was also denied access to her office computer and was
restricted from punching her daily time record in the bundy clock.

On November 7, 2003, Viajar was invited to the HRD Cebu Office where she was asked to sign certain documents, which
turned out to be an "Application for Retirement and Benefits." The respondent refused to sign and sought clarification because
she did not apply for retirement and instead asserted that her services were terminated for alleged redundancy. Almocera told
her that her signature on the Application for Retirement and Benefits was needed to process her separation pay. The
respondent also claimed that between the period of July 4, 2003 and October 13, 2003, GMC hired fifteen (15) new employees
which aroused her suspicion that her dismissal was not necessary. At the time of her termination, the respondent was
receiving the salary rate of P19,651.41 per month.

GMC presented the following as proof of validity of its redundancy plan, the notification letter to respondent Viajar; the
"Establishment Termination Report" it submitted to the DOLE Office; the two (2) checks issued in the respondent's name
amounting to P440,253.02 and P21,211.35; and the list of terminated employees as of June 6, 2006. 

Issue:

Whether or not the dismissal of Viajar was valid on the ground of redundancy.

Ruling:

No. Viajar was illegally dismissed.

Article 283 of the Labor Code provides that redundancy is one of the authorized causes for dismissal. It reads:
Article 283.Closure of establishment and reduction of personnel. — The employer may also terminate the employment of any
employee due to the installment of labor-saving devices, redundancy, retrenchment to prevent losses or the closing or
cessation of operation of the establishment or undertaking unless the closing is for the purpose of circumventing the provisions
of this Title, by serving a written notice on the worker and the Ministry of Labor and Employment at least one (1)
month before the intended date thereof. In case of termination due to the installation of labor-saving devices or
redundancy, the worker affected thereby shall be entitled to a separation pay equivalent to at least his one (1) month
pay or to at least one (1) month pay for every year of service, whichever is higher . In case of retrenchment to prevent
losses and in cases of closures or cessation of operations of establishment or undertaking not due to serious business losses
or reverses, the separation pay shall be equivalent to one (1) month pay or at least one-half (1/2) month pay for every year of
service, whichever is higher. A fraction of at least six (6) months shall be considered one (1) whole year.

From the above provision, it is imperative that the employer must comply with the requirements for a valid implementation of
the company's redundancy program, to wit: (a) the employer must serve a written notice to the affected employees and the
DOLE at least one (1) month before the intended date of retrenchment; (b) the employer must pay the employees a separation
pay equivalent to at least one month pay or at least one month pay for every year of service, whichever is higher; (c) the
employer must abolish the redundant positions in good faith; and (d) the employer must set fair and reasonable criteria in
ascertaining which positions are redundant and may be abolished.

While it is true that the "characterization of an employee's services as superfluous or no longer necessary and, therefore,
properly terminable, is an exercise of business judgment on the part of the employer," the exercise of such judgment,
however, must not be in violation of the law, and must not be arbitrary or malicious. The Court has always stressed that a
company cannot simply declare redundancy without basis. To exhibit its good faith and that there was a fair and reasonable
criteria in ascertaining redundant positions, a company claiming to be over manned must produce adequate proof of the same.

The letter-memorandum which contains general allegations is not enough to convince this Court that Viajar's termination of
employment due to redundancy was warranted under the circumstances. There is no showing that GMC made an evaluation
of the existing positions and their effect to the company. Neither did GMC exert efforts to present tangible proof that it was
experiencing business slow down or over hiring. The "Establishment Termination Report" it submitted to the DOLE Office did
not account for anything to justify declaring the positions redundant. The Court notes that the list of terminated employees
presented by GMC was a list taken as of June 6, 2006 or almost three years after the respondent was illegally dismissed and
almost a year after the LA promulgated its decision. While the petitioner had been harping that it was on a "reduction mode" of
its employees, it has not presented any evidence (such as new staffing pattern, feasibility studies or proposal, viability of newly
created positions, job description and the approval of the management of the restructuring, audited financial documents like
balance sheets, annual income tax returns and others) which could readily show that the company's declaration of redundant
positions was justified. Such proofs, if presented, would suffice to show the good faith on the part of the employer or that this
business prerogative was not whimsically exercised in terminating respondent's employment on the ground of redundancy.
Unfortunately, these are wanting in the instant case. The petitioner only advanced a self-serving general claim that it was
experiencing business reverses and that there was a need to reduce its manpower complement.

On the other hand, the respondent presented proof that the petitioner had been hiring new employees while it was firing the
old ones, negating the claim of redundancy. It must, however, be pointed out that in termination cases, like the one before us,
the burden of proving that the dismissal of the employees was for a valid and authorized cause rests on the employer. It was
incumbent upon the petitioner to show by substantial evidence that the termination of the employment of the respondent was
validly made and failure to discharge that duty would mean that the dismissal is not justified and therefore illegal. 
60. JONATHAN I. SANG-AN, vs. EQUATOR KNIGHTS DETECTIVE AND SECURITY AGENCY, G.R. No. 173189.
February 13, 2013.

Facts:

Jonathan was the Assistant Operation Manager of respondent Equator Knights Detective and Security Agency, Inc. (Equator).
He was tasked, among others, with the duty of assisting in the operations of the security services; he was also  in charge of
safekeeping Equator's firearms.
On April 21, 2001, Equator discovered that two firearms were missing from its inventory. The investigation revealed that it was
Jonathan who might have been responsible for the loss. He was then suspended.

While Jonathan was under suspension, a security guard from Equator was apprehended by policemen for violating the
Commission on Elections' gun ban rule. The security guard stated in his affidavit that the unlicensed firearm had been issued
to him by Jonathan.

He then filed a complaint before the NLRC for illegal suspension. He treated his case as one for illegal dismissal and alleged
that he was denied due process when he was dismissed.

Equator argued that Jonathan was dismissed for just cause and making his dismissal valid.

Issue:

Whether or not there was illegal dismissal.

Ruling:

Equator complied with the substantive requirements for the dismissal of Jonathan on the ground of serious misconduct.
Misconduct is improper or wrongful conduct; it is the transgression of some established and definite rule of action, a forbidden
act, a dereliction of duty, willful in character, and implies wrongful intent and not mere error of judgment. The misconduct, to be
serious within the meaning of the Labor Code, must be of such grave and aggravated character and not merely trivial or
unimportant. It is also important that the misconduct be in connection with the employee's work to constitute just cause for his
separation.

By losing two firearms and issuing an unlicensed firearm, Jonathan committed serious misconduct. He did not merely violate a
company policy; he violated the law itself (Presidential Decree No. 1866 or Codifying the Laws on Illegal/Unlawful Possession,
Manufacture, Dealing in, Acquisition or Disposition, of Firearms, Ammunition or Explosives or Instruments Used in the
Manufacture of Firearms, Ammunition or Explosives, and Imposing Stiffer Penalties for Certain Violations Thereof and for
Relevant Purposes), and placed Equator and its employees at risk of being made legally liable. Thus, Equator had a valid
reason that warranted Jonathan's dismissal from employment as Assistant Operation Manager.

However, Equator failed to observe the proper procedure in dismissing Jonathan.

Jurisprudence has expounded on the guarantee of due process, requiring the employer to furnish the employee with two
written notices before termination of employment can be effected: a first written notice that informs the employee of the
particular acts or omissions for which his or her dismissal is sought, and a second written notice which informs the employee
of the employer's decision to dismiss him. In considering whether the charge in the first notice is sufficient to warrant dismissal
under the second notice, the employer must afford the employee ample opportunity to be heard.

A review of the records shows that Jonathan was not furnished with any written notice that informed him of the acts he
committed justifying his dismissal from employment. The notice of suspension given to Jonathan only pertained to the first
offense, i.e., the loss of Equator's firearms under Jonathan's watch. With respect to his second offense (i.e., the issuance of an
unlicensed firearm to Equator's security guard — that became the basis for his dismissal), Jonathan was never given any
notice that allowed him to air his side and to avail of the guaranteed opportunity to be heard. That Equator brought the second
offense before the LA does not serve as notice because by then, Jonathan had already been dismissed.

In order to validly dismiss an employee, the observance of both substantive and procedural due process by the employer is a
condition sine qua non. Procedural due process requires that the employee be given a notice of the charge against him, an
ample opportunity to be heard, and a notice of termination.
61. Leopard Security & Investigation Agency vs. Quitoy, G.R. No. 186344, Feb. 20, 2013

Facts:

Respondents were hired as security guards by Petitioner (LSIA). They were assigned by the petitioner to the different
branches of Union Bank in Cebu city. On April 1, 2005, Union Bank served notice to LSIA terminating the security
service contract effective at the end of business hours of April 30, 2005. However, the respondents were only
informed of the termination of the contract with Union Bank on April 29, 2005. The respondents went to union bank on
April 30, 2005 for the turnover of their service firearms to Cortes, Union Bank Chief security officer. Respondents filed
a complaint for illegal dismissal against LSIA. CA sustained the award of separation pay of NLRC to respondents on
the ground that the parties' relationship had already been strained.

Issues:

WON there was illegal dismissal.


WON the award of separation pay was proper.

Ruling:

NO. Applying Article 286 of the  Labor Code of the Philippines by analogy, this Court has repeatedly recognized that
security guards may be temporarily sidelined by their security agency as their assignments primarily depend on the
contracts entered into by the latter with third parties.  Temporary "off-detail" or "floating status" is the period of time
when security guards are in between assignments or when they are made to wait after being relieved from a previous
post until they are transferred to a new one. It takes place when, as here, the security agency's clients decide not to
renew their contracts with the agency, resulting in a situation where the available posts under its existing contracts
are less than the number of guards in its roster.  For as long as such temporary inactivity does not continue for a
period exceeding six months, it has been ruled that placing an employee on temporary "off-detail" or "floating status"
is not equivalent to dismissal.

Considering that a security guard is only considered illegally dismissed from service when he is sidelined from duty
for a period exceeding six months, respondents were not illegally dismissed by LSIA. Under Article 279 of the Labor
Code, an illegally dismissed  employee is entitled to the twin reliefs of full backwages and reinstatement without loss
of seniority rights. Aside from the instances provided under Articles 283 and 284 of the Labor Code, separation pay
is, however, granted when reinstatement is no longer feasible because of strained relations between the employer
and the employee. In cases of illegal dismissal, the accepted doctrine is that separation pay is available in lieu of
reinstatement when the latter recourse is no longer practical or in the best interest of the parties.

As a relief granted in lieu of reinstatement, however, it consequently goes without saying that an award of separation
pay is inconsistent with a finding that there was no illegal dismissal. Even in cases of illegal dismissal, the doctrine of
strained relations is not applied indiscriminately as to bar reinstatement, especially when the employee has not
indicated an aversion to returning to work or does not occupy a position of trust and confidence in  or has no say in
the operation of the employer's business. Although litigation may also engender a certain degree of hostility, it has
likewise been ruled that the understandable strain in the parties' relations would not necessarily rule out
reinstatement which would, otherwise, become the rule rather than the exception in illegal dismissal cases.

Our perusal of the position paper they filed a quo shows that, despite erroneously believing themselves to have been
illegally dismissed, respondents had alleged no circumstance indicating the strained relations between them and
LSIA and had even alternatively prayed for reinstatement alongside the payment of separation pay. Since application
of the doctrine of strained relations presupposes a question of fact which must be demonstrated and adequately
supported by evidence, the CA clearly erred in ruling that the parties' relations had already soured and that an award
of separation pay in favor of respondents is proper.
62. Philippine Plaza Holdings Inc. vs. Episcope, G.R. No. 192826, Feb. 27, 2013

Facts:

Petitioner Philippine Plaza Holdings, Inc. (PPHI) is the owner and operator of the Westin Philippine Plaza Hotel
(Hotel). Respondent Ma. Flora M. Episcope (Episcope) was employed by PPHI since July 24, 1984 until she was
terminated on November 4, 2004 for dishonesty, willful disobedience and serious misconduct amounting to loss of
trust and confidence.

In order to check the performance of the employees and the services in the different outlets of the Hotel, PPHI
regularly employed the services of independent auditors and/or professional shoppers. For this purpose, Sycip,
Gorres and Velayo auditors dined at the Hotel's Café Plaza on August 28, 2004. After dining, the auditors were billed
the total amount of P2,306.65, representing the cost of the food and drinks they had ordered under Check No.
565938. 4 Based on the audit report 5 submitted to PPHI, Episcope was one of those who attended to the auditors
and was the one who handed the check and received the payment of P2,400.00. She thereafter returned Check No.
565938, which was stamp marked "paid," together with the change.

Upon verification of the foregoing check receipt with the sales report of Café Plaza, it was discovered that the Hotel's
copy of the receipt bore a discount of P906.45 6 on account of the use of a Starwood Privilege Discount Card
registered in the name of Peter A. Pamintuan, while the receipt issued by Episcope to the auditors reflected the
undiscounted amount of P2,306.65 considering that none of the auditors had such discount card. In view of the
foregoing, the amount actually remitted to the Hotel was only P1,400.20 thus, leaving a shortage of P906.45.

On September 30, 2004, the Hotel issued a Show-Cause Memo 7 directing Episcope to explain in writing why no
disciplinary action should be taken against her for the questionable and invalid discount application on the settlement
check issued to the auditors on August 28, 2004.

Finding Episcope to have failed to sufficiently explain the questionable discount application on the settlement bill of
the auditors, her employment was terminated for committing acts of dishonesty, which was classified as a Class D
offense under the Hotel's Code of Discipline, as well as for willful disobedience, serious misconduct and loss of trust
and confidence.

Aggrieved, Episcope filed a complaint for illegal dismissal with prayer for payment of damages and attorney's fees
against PPHI before the NLRC docketed as NLRC-NCR Case No. 00-12-13621-04.

Issue:

WON there was illegal dismissal.

Ruling:

Article 293 (formerly Article 279) of the Labor Code 25 provides that the employer shall not terminate the services of
an employee except only for a just or authorized cause. If an employer terminates the employment without a just or
authorized cause, then the employee is considered to have been illegally dismissed and is thus, entitled to
reinstatement or in certain instances, separation pay in lieu thereof, as well as the payment of backwages.

Among the just causes for termination is the employer's loss of trust and confidence in its employee. Article 296 (c)
(formerly Article 282 [c]) of the Labor Code provides that an employer may terminate the services of an employee for
fraud or willful breach of the trust reposed in him. But in order for the said cause to be properly invoked, certain
requirements must be complied with namely, (1) the employee concerned must be holding a position of trust and
confidence and (2) there must be an act that would justify the loss of trust and confidence.

It is noteworthy to mention that there are two classes of positions of trust: on the one hand, there are managerial
employees whose primary duty consists of the management of the establishment in which they are employed or of a
department or a subdivision thereof, and to other officers or members of the managerial staff; on the other hand,
there are fiduciary rank-and-file employees, such as cashiers, auditors, property custodians, or those who, in the
normal exercise of their functions, regularly handle significant amounts of money or property. These employees,
though rank-and-file, are routinely charged with the care and custody of the employer's money or property, and are
thus classified as occupying positions of trust and confidence. 27 Episcope belongs to this latter class and therefore,
occupies a position of trust and confidence.
As may be readily gleaned from the records, Episcope was employed by PPHI as a service attendant in its Café
Plaza. In this regard, she was tasked to attend to dining guests, handle their bills and receive their payments for
transmittal to the cashier. It is also apparent that whenever discount cards are presented, she maintained the
responsibility to take them to the cashier for the application of discounts. Being therefore involved in the handling of
company funds, Episcope is undeniably considered an employee occupying a position of trust and confidence and as
such, was expected to act with utmost honesty and fidelity.
Anent the second requisite, records likewise reveal that Episcope committed an act which justified her employer's
(PPHI's) loss of trust and confidence in her.

Primarily, it is apt to point out that proof beyond reasonable doubt is not required in dismissing an employee on the
ground of loss of trust and confidence; it is sufficient that there lies some basis to believe that the employee
concerned is responsible for the misconduct and that the nature of the employee's participation therein rendered him
absolutely unworthy of trust and confidence demanded by his position.

Perforce, having substantially established the actual breach of duty committed by Episcope and the due observance
of due process, no grave abuse of discretion can be imputed against the NLRC in sustaining the finding of the LA that
her dismissal was proper under the circumstances.

Finally, with respect to Episcope's other monetary claims, namely, service incentive leave credits and 13th month
pay, the Court finds no error on the part of the LA when it denied the foregoing claims considering that Episcope
failed to proffer any legitimate basis to substantiate her entitlement to the same
63. TORRES vs. RURAL BANK OF SAN JUAN, INC., et. al. G.R. No. 184520. March 13, 2013

FACTS:

The petitioner was hired by RBSJI. On September 24, 1996, the petitioner was temporarily assigned as the manager
of RBSJI's N. Domingo branch in view of the resignation of Jacinto Figueroa (Jacinto). On the same month, Jacinto
requested the petitioner to sign a standard employment clearance pertaining to his accountabilities with RBSJI. When
the petitioner declined his request, Jacinto threw a fit and shouted foul invectives. To pacify him, the petitioner
bargained to issue a clearance but only for Jacinto's paid cash advances and salary loan. About seven months later,
respondent issued a memorandum to the petitioner requiring him to explain why no administrative action should be
imposed on him for his unauthorized issuance of a clearance to Jacinto whose accountabilities were yet to be
audited. The petitioner submitted his explanation on the same day clarifying that the clearance was limited only to
Jacinto's paid cash advances and salary loan based on the receipts presented by the cashier. He emphasized that he
had no foreknowledge nor was he forewarned of Jacinto's unliquidated cash advances and questionable transactions
and that the clearance did not extend to those matters.

After conducting an investigation, RBSJI's Human Resources Department recommended the petitioner's termination
from employment for the following reasons, to wit: the issuance of clearance to Mr. Jacinto Figueroa have been
prejudicial to the Bank; the petitioner is not in any authority to issue said clearance which is a violation of the
Company Code of Conduct and Discipline under Category B Grave Offense No. 1 (falsifying or misrepresenting
persons or other company records, documents or papers); and the nature of his participation in the issuance of the
said clearance could be a reasonable ground for the Management to believe that he is unworthy of the trust and
confidence demanded by his position which is also a ground for termination under Article [282] of the Labor Code.
Feeling aggrieved, the petitioner filed the herein complaint for illegal dismissal. The petitioner averred that the
supposed loss of trust and confidence on him was a sham as it is in fact the calculated result of the respondents'
dubious plot to conveniently oust him from RBSJI.

The LA sustained the claims of the petitioner as against the factually unsubstantiated allegation of loss of trust and
confidence propounded by the respondents. The NLRC disagreed with the LA's conclusion. But on motion for
reconsideration, the NLRC reversed its earlier ruling and reinstated the LA's Decision. The CA reversed and set aside
the NLRC Decision and ruled that the petitioner was dismissed for a just cause.

ISSUE:

Whether or not petitioner was illegally dismissed.

RULING:

YES. The respondents failed to prove that the petitioner was dismissed for a just cause. The law mandates that
before validity can be accorded to a dismissal premised on loss of trust and confidence, two requisites must concur,
viz.: (1) the employee concerned must be holding a position of trust; and (2) the loss of trust must be based on willful
breach of trust founded on clearly established facts.

The presence of the first requisite is certain. Ever since the petitioner was employed, he has occupied positions that
entail the power or prerogative to dictate management policies. Anent the second requisite, the Court finds that the
respondents failed to meet their burden of proving that the petitioner's dismissal was for a just cause.

The unsubstantiated claims of the respondents fall short of the standard proof required for valid termination of
employment. They failed to clearly and convincingly establish that the petitioner's act of issuing a clearance to Jacinto
rendered him unfit to continue working for RBSJI. Furthermore, the Court has repeatedly emphasized that the act that
breached the trust must be willful such that it was done intentionally, knowingly, and purposely, without justifiable
excuse, as distinguished from an act done carelessly, thoughtlessly, heedlessly or inadvertently. The conditions
under which the clearance was issued exclude any finding of deliberate or conscious effort on the part of the
petitioner to prejudice his employer.

Also, the petitioner did not commit an irregular or prohibited act. He did not falsify or misrepresent any company
record as it was officially confirmed by the cashier that the items covered by the clearance were truly settled by
Jacinto. Hence, the respondents had no factual basis in declaring that the petitioner violated Category B Grave
Offense No. 1 of the Company Code of Conduct and Discipline. All told, the petitioner was illegally dismissed from
employment and is entitled to back wages, to be computed from the date he was illegally dismissed until the finality of
this decision.

In addition to his back wages, the petitioner is also entitled to separation pay. It cannot be gainsaid that animosity and
antagonism have been brewing between the parties since the petitioner was gradually eased out of key positions in
RBSJI and to reinstate him will only intensify their hostile working atmosphere. Thus, based on strained relations,
separation pay equivalent to one (1) month salary for every year of service, with a fraction of a year of at least six (6)
months to be considered as one (1) whole year, should be awarded in lieu of reinstatement, to be computed from
date of his engagement by RBSJI up to the finality of this decision.

The award of separation pay in case of strained relations is more beneficial to both parties in that it liberates the
employee from what could be a highly oppressive work environment in as much as it releases the employer from the
grossly unpalatable obligation of maintaining in its employ a worker it could no longer trust.

Moral damages are recoverable only where the dismissal of the employee was attended by bad faith or fraud, or
constituted an act oppressive to labor, or was done in a manner contrary to morals, good customs or public policy.
Such an award cannot be justified solely upon the premise that the employer fired his employee without just cause or
due process. Here, the petitioner failed to prove that his dismissal was attended by explicit oppressive, humiliating or
demeaning acts. Since no moral damages can be granted under the facts of the case, exemplary damages cannot
also be awarded.
64. THE ORCHARD GOLF AND COUNTRY CLUB vs. FRANCISCO G.R. No. 178125. March 18, 2013

FACTS:

Respondent Francisco was employed by petitioner as Club Accountant. The respondent reports directly to the Club's
Financial Comptroller, Jose Ernilo P. Famy. On May 18, 2000, Famy directed Francisco to draft a letter to SGV & Co.
(SGV), the Club's external auditor, inquiring about the accounting treatment that should be accorded property that will
be sold or donated to the Club. Francisco failed to prepare the letter, even after Famy's repeated verbal and written
reminders. For this reason, Famy issued a memorandum suspending Francisco without pay for a period of 15 days.

Francisco filed an amended Complaint for illegal suspension. Meanwhile, she continued to report for work. A day
after serving her suspension, Francisco again received a memorandum placing her on forced leave with pay. After
the expiration of her forced leave, Francisco reported back to work. This time she was handed a memorandum
informing her that, due to strained relations between her and Famy and the pending evaluation of her betrayal of
company trust charge, she has been permanently transferred, without diminution of benefits, to the Club's Cost
Accounting Section.

On account of her transfer, Francisco once more amended her Complaint to include illegal/constructive dismissal.
And in her prayer, she sought to be reinstated to her former position as Club Accountant.

The Labor Arbiter dismissed Francisco's Complaint for lack of merit. The NLRC reversed the ruling of LA and the CA
sustained the NLRC ruling.

ISSUE:

Whether or not the respondent was constructively dismissed.

RULING:

YES. There was constructive dismissal when Francisco was transferred to the Cost Accounting Section. We agree
with the NLRC and the CA that Francisco's transfer to the position of Cost Controller was without valid basis and that
it amounted to a demotion in rank. Hence, there was constructive dismissal.

For one, there was no valid reason to temporarily transfer Francisco to Cost Accounting Section. She had already
served her penalty for her failure to draft the SGV letter, through the 15-day suspension period which she just
completed. Secondly, the transfer was not even rooted in any new infraction she is accused of committing. There was
thus an absolute lack of basis for her temporary transfer.

As for her permanent transfer, the same is null and void for lack of just cause. Also, the transfer is a penalty imposed
on a charge that has not yet been resolved. Definitely, to punish one for an offense that has not been proved is truly
unfair; this is deprivation without due process. Finally, the Court sees no necessity for Francisco's transfer; on the
contrary, such transfer is outweighed by the need to secure her office and documents from Famy's possible
intervention on account of the complaint she filed against him.

We also agree with the findings of the NLRC, as affirmed by the CA, that Francisco's transfer constituted a demotion,
It is obvious, therefore, that Complainant's position of Club Accountant is higher in level/rank than that of Cost
Controller/Accountant. Patently, Complainant's transfer from the position of Club Accountant to the position of Cost
Accountant resulted to her demotion in level/rank. Complainant's transfer resulting to her demotion is, therefore,
tantamount to constructive dismissal. The fact that Francisco continued to report for work does not necessarily
suggest that constructive dismissal has not occurred, nor does it operate as a waiver. Constructive dismissal occurs
not when the employee ceases to report for work, but when the unwarranted acts of the employer are committed to
the end that the employee's continued employment shall become so intolerable. In these difficult times, an employee
may be left with no choice but to continue with his employment despite abuses committed against him by the
employer, and even during the pendency of a labor dispute between them. This should not be taken against the
employee.
65. Banares vs. Tabaco Womens Transport Service Cooperative, G.R. No. 197353, April 1, 2013

Facts:

PETITIONER Alexander B. Bañares worked for some time as general manager of respondent Tabaco Women’s
Transport Service Cooperative (TAWTRASCO). He filed a complaint for illegal dismissal and payment of monetary
claims against TAWTRASCO. Among others, the Labor Arbiter ordered TAWTRASCO to immediately reinstate
petitioner to his former position.

In compliance with the decision, TAWTRASCO directed the petitioner to report at the company’s Virac, Catanduanes
terminal. Petitioner asked for a lodging allowance, which he used to enjoy in his previous assignment but was told to
just stay at the Virac office.

He, however, found the Virac office very dilapidated and empty of an office table, chairs, filing cabinet and other office
supplies. He asked for expenses for renovation, which respondents turned deaf ears to. Hence, he stopped reporting
to work and filed a complaint for non-payment of salaries.

Issue:

WON the complaint will prosper.

Ruling:
Yes.
Under Article 223 of the Labor Code, an employee entitled to reinstatement “shall either be admitted back to work
under the same terms and conditions prevailing prior to his dismissal or separation x x x.” An illegally dismissed
employee is entitled to reinstatement without loss of seniority rights and to other established employment privileges,
and to his full back wages. The boarding house privilege, being an established perk accorded to petitioner, ought to
have been granted him if a real and authentic reinstatement to his former position as general manager is to be
posited.

It cannot be stressed enough that TAWTRASCO withheld petitioner’s salaries for and after his purported refusal to
report for work at the Virac terminal. The reality, however, is that TAWTRASCO directed petitioner to work under
terms and conditions prejudicial to him, the most hurtful cut being that he was required to work without a decent
office, partly performing a checker’s job. This embarrassing work arrangement is what doubtless triggered the refusal
to work, which under the premises appears justified.

Generally, employees have a demandable right over benefits voluntarily granted to them by their employers. And if
the grant or benefit is founded on an express policy or has, for a considerable period, been given regularly and
deliberately, then the grant ripens into a vested right that the employer cannot unilaterally diminish, discontinue or
eliminate. So it must be here with respect, at the minimum, to the lodging accommodation which TAWTRASCO, as
found by the National Labor Relations Commission, appears to have regularly extended for free for some time to
petitioner.
66. Reyes, et al., vs. RP Guardians security Agency Inc. G.R. No. 193756, April 10, 2013

Facts:

Petitioners Venancio S. Reyes, Edgardo C. Dabbay, Walter A. Vigilia, Nemesio M. Calanno, Rogelio A. Supe, Jr.,
Roland R. Trinidad, and Aurelio A. Duldulao (petitioners) were hired by respondent RP Guardians Security Agency,
Inc. (respondent) as security guards. They were deployed to various clients of respondent, the last of which were the
different branches of Banco Filipino Savings and Mortgage Bank (Banco Filipino).
In September 2006, respondent’s security contract with Banco Filipino was terminated. In separate letters,
petitioners were individually informed of the termination of the security contract with Banco de Oro. In two (2)
memoranda, dated September 21, 2006 and September 29, 2006, petitioners were directed to turnover their duties
and responsibilities to the incoming security agency and were advised that they would be placed on floating status
while waiting for available post. Petitioners waited for their next assignment, but several months lapsed and they were
not given new assignments.

Consequently, on April 10, 2007, petitioners filed a complaint7 for constructive dismissal.
In its position paper, respondent claimed that there was no dismissal, of petitioners, constructive or otherwise, and
asserted that their termination was due to the expiration of the service contract which was coterminus with their
contract of employment.

On August 20, 2007, the Labor Arbiter (LA) rendered a decision9 in favor of petitioners ordering respondent to pay
petitioners separation pay, backwages, refund of trust fund, moral and exemplary damages, and attorney’s fees.
Aggrieved, respondent appealed to the NLRC.

On April 9, 2008, the NLRC promulgated its decision sustaining the finding of constructive dismissal by the LA, and
the awards she made in the decision. The award of moral and exemplary damages, however, were deleted.

Upon denial of its motion for reconsideration, respondent filed a petition for certiorari before the CA.
On February 26, 2010, the CA rendered a decision12 dismissing the petition and affirming the assailed NLRC
decision and resolution.

On motion for reconsideration, the CA issued the Amended Decision13 dated May 18, 2010, modifying its earlier
decision. Citing Section 6.5 (4) of Department Order No. 14 of the Department of Labor and Employment (DOLE D.O.
No. 14), otherwise known as Guidelines Governing the Employment and Working Conditions of Security Guards and
Similar Personnel in the Private Security Industry, the CA reduced the computation of the separation pay from one
month pay per year of service to one-half month pay for every year of service; reduced the refund of trust fund
contribution from Sixty (P60.00) Pesos to Thirty (P30.00)Pesos; and deleted the award of backwages and attorney’s
fees.

Issue:

WON the first decision of the CA is correct in affirming the decision of the NLRC awarding both backwages and
separation pay of one month pay for every year of service

WON petitioners are entitled to backwages for the period covered from the time the Labor Arbiter rendered the
decision in their favor on August 20, 2007 until said decision was reversed by the Court of Appeals in its Amended
Decision promulgated on May 18, 2010.
Ruling:

There is no doubt that petitioners were constructively dismissed. The LA, the NLRC and the CA were one in their
conclusion that respondent was guilty of illegal dismissal when it placed petitioners on floating status beyond the
reasonable six-month period after the termination of their service contract with Banco de Oro. Temporary
displacement or temporary off-detail of security guard is, generally, allowed in a situation where a security agency’s
client decided not to renew their service contract with the agency and no post is available for the relieved security
guard. Such situation does not normally result in a constructive dismissal. Nonetheless, when the floating status lasts
for more than six (6) months, the employee may be considered to have been constructively dismissed. No less than
the Constitution guarantees the right of workers to security of tenure, thus, employees can only be dismissed for just
or authorized causes and after they have been afforded the due process of law.

Settled is the rule that that an employee who is unjustly dismissed from work shall be entitled to reinstatement without
loss of seniority rights and other privileges, and to his full backwages, inclusive of allowances and to his other
benefits or their monetary equivalent computed from the time his compensation was withheld up to the time of actual
reinstatement. If reinstatement is not possible, however, the award of separation pay is proper.

Backwages and reinstatement are separate and distinct reliefs given to an illegally dismissed employee in order to
alleviate the economic damage brought about by the employee’s dismissal.21 “Reinstatement is a restoration to a
state from which one has been removed or separated” while “the payment of backwages is a form of relief that
restores the income that was lost by reason of the unlawful dismissal.” Therefore, the award of one does not bar the
other.

In the case of Aliling v. Feliciano, citing Golden Ace Builders v. Talde, the Court explained:
Thus, an illegally dismissed employee is entitled to two reliefs: backwages and reinstatement. The two reliefs
provided are separate and distinct. In instances where reinstatement is no longer feasible because of strained
relations between the employee and the employer, separation pay is granted. In effect, an illegally dismissed
employee is entitled to either reinstatement, if viable, or separation pay if reinstatement is no longer viable, and
backwages.

The normal consequences of respondents’ illegal dismissal, then, are reinstatement without loss of seniority rights,
and payment of backwages computed from the time compensation was withheld up to the date of actual
reinstatement. Where reinstatement is no longer viable as an option, separation pay equivalent to one (1) month
salary for every year of service should be awarded as an alternative. The payment of separation pay is in addition to
payment of backwages.

Furthermore, the entitlement of the dismissed employee to separation pay of one month for every year of service
should not be confused with Section 6.5 (4) of DOLE D.O. No. 14 which grants a separation pay of one-half month
for every year service, to wit:

6.5 Other Mandatory Benefits. In appropriate cases, security guards/similar personnel are entitled to the mandatory
benefits as listed below, although the same may not be included in the monthly cost distribution in the contracts,
except the required premiums for their coverage:
a. Maternity benefit as provided under the SSS Law;
b. Separation pay if the termination of employment is for authorized cause as provided by law and as enumerated
below:

Half-Month Pay Per Year of Service, but in no case less than One Month Pay, if separation is due to:

1. Retrenchment or reduction of personnel effected by management to prevent serious losses;


2. Closure or cessation of operation of an establishment not due to serious losses or financial reverses;
3. Illness or disease not curable within a period of 6 months and continued employment is prohibited by law or
prejudicial to the employee's health or that of co-employees; or
4. Lack of service assignment for a continuous period of 6 months.

The said provision contemplates a situation where a security guard is removed for authorized causes such as when
the security agency experiences a surplus of security guards brought about by lack of clients. In such a case, the
security agency has the option to resort to retrenchment upon compliance with the procedural requirements of “two-
notice rule” set forth in the Labor Code and to pay separation pay of one-half month for every year of service.

In this case, respondent would have been liable for reinstatement and payment of backwages. Reinstatement,
however, was no longer feasible because, as found by the LA, respondent had already ceased operation of its
business. Thus, backwages and separation pay, in the amount of one month for every year of service, should be paid
in lieu of reinstatement.

As to their claim of attorney’s fees, petitioners were compelled to file an action for the recovery of their lawful wages
and other benefits and, in the process, incurred expenses. Hence, petitioners are entitled to attorney’s fees
equivalent to ten percent (10%) of the monetary award.
67. Celdran vs. Forza Integrated Services et al., G.R. No. 189460, June 5, 2013, Res.

Facts:

Petitioner Leo Mario C. Celdran was working for [(Philamlife)] before he was hired by respondent City Service
Corporation [(City Service)] as Vice-President in Cebu City.

Respondents City Service Corporation and [(Peerless Integrated)] are affiliate companies of respondent Forza
Integrated)]. CELDRAN was paid monthly basis. Celdran alleges that his compensation package included a car
benefit which to him, was a car plan wherein respondent City Service Corporation would assume 50% of the
purchase price while Celdran pays the other 50% thereof.

On the contrary, respondents City Service, states that it was a car lease arrangement whereby an agreed
percentage of the [amount paid to Philamlife] was to be set as residual value and the remaining amount was to be
spread over a certain period as monthly car rental with an option to buy the vehicle at its residual value after the end
of the lease period.

Deductions were then made from Celdran's salary with the monthly deductions in place and the vehicle being already
in the possession and use of Celdran, he was required to sign a motor vehicle lease contract in order to formalize the
arrangement. Celdran consistently refused to sign the said lease contract as it was contrary to what he allegedly
agreed with respondent Valentin B. Prieto, Jr. [(V. Prieto)] during his employment interview, since Celdran avers it
was a car plan not a car lease arrangement.

Thereafter, Santiago, the Chief Operating Officer of City Service, accused Celdran of dishonesty for allegedly
charging a personal lunch to the company.

Respondent Santiago then demanded Celdran's resignation. Eight days after, Celdran received a termination notice
signed by private respondent Santiago which led the former to file a complaint for illegal dismissal before the LA.
However, the said case was settled as he was reinstated to his position.

Upon his return, Celdran was told to occupy the last open cubicle at the ground floor and not to his former office. He
was given a new copy of the motor vehicle lease contract for his signature which he refused to sign. Subsequently,
he was relieved as Mancom Chairman for no reason at all. He was subjected to check and inspection by the security
guard and his transportation and cellular phone allowances were subjected to new guidelines. City Service
Corporation gave Celdran the option to buy the Honda CRV at its residual value, otherwise, the former would recover
the vehicle from the latter.

Then, Celdran filed a complaint again with the LA charging private respondents with violation of the terms of the car
plan. Celdran filed an injunction case with a prayer for the issuance of a TRO with the NLRC, to prevent City Service
from taking the car. NLRC granted the prayed-for TRO as well as the writ of preliminary injunction.

Soon, Celdran was placed under preventive suspension for 30 days due to his belligerent attitude and required to
explain why he should not be terminated. He was not asked to return to work after his suspension for which reason
he amended his complaint, to include charges of illegal suspension, constructive dismissal and unpaid money claims.

Afterwards, private respondent City Service Corporation informed all its employees, including Celdran, that by virtue
of a board resolution, the company decided to replace the Visayas Regional Office with a small Liaison Office.
Consequently, Celdran made a second amendment of his complaint to include charges of illegal lay-off/downsizing.

The Executive Labor Arbiter Ruling, found: 1) the long-standing policy is that of granting Car Lease Agreement with
option to buy at residual value after end of the lease term and NOT car plan. 2) That because Celdran amended his
complaint to subsequently include illegal dismissal due to retrenchment (when his original complaint alleges
constructive dismissal, earlier), this has the effect of Celdran waiving his claim for constructive dismissal, because
employment can only be terminated once. 3) That the suspension was not made illegal by the fact that it continued
after 30 days since City Service reinstated Celdran in the payroll. 4) That the downsizing or retrenchment was legal
justified by management prerogative.

On appeal NLRC affirmed LA, but found that it was a carplan not car lease agreement. CA affirm NLRC.

Hence the petition

Issue:

Whether or not there is constructive dismissal

Ruling:

Petition of CELDRAN is denied.

There is constructive dismissal when employees resort to involuntary resignation, because continued employment is
rendered impossible, unreasonable, or unlikely; when there is a demotion in rank and/or a diminution in pay; or when
a clear discrimination, insensibility, or disdain by the employer becomes unbearable to them. That is, they find
themselves in a situation in which there is a belief that personal reasons cannot be sacrificed in favor of the exigency
of the service, and that they have no other choice but to disassociate themselves from their employment.
We rule that there was no constructive dismissal in the case at bar. According to petitioner, he had been experiencing
a kind of treatment that rendered "employment impossible and unreasonable" as early as in the last quarter of 2005.
However, he never resigned. In fact, when he filed a complaint in March 2006 regarding his car plan benefit, he did
not make any allegation concerning his inability to continue working for respondents due to an alleged ill working
environment. We thus find that he was still willing and able to continue his employment despite any alleged ill
treatment. To repeat, for there to be constructive dismissal, the employer must be shown to have committed an act of
clear discrimination, insensibility, or disdain, which had become so unbearable on the part of the employee that it
foreclosed any choice other than for the latter to forego continued employment.

In this case downsizing or retrenchment was valid (since city service had been experiencing a downtrend in their
Visayas operations since three years before they decided to downsize). Pursuant to Article 283 of the Labor Code, an
employer may reduce the number of its employees based on economic grounds in order to protect and preserve the
employer's viability and ensure its survival. 5 Consequently, employers are given the management prerogative to
implement a retrenchment program for the purpose of preventing losses or cessation of business operations due to
business recession, industrial depression, seasonal fluctuations, lack of work, or considerable reduction in the volume
of their business.
68. Surigao Del Norte Electric Cooperative Inc. vs. Gonzaga, G.R. No. 187722, June 10, 2013

Facts:

Petitioner (SURNECO) hired Gonzaga as its lineman. Thereafter he was assigned as Temporary Teller at
SURNECO's sub-office.

On June 26, 2001, petitioner (Escalante), General Manager of SURNECO, issued Memorandum Order with attached
report of SURNECO's Internal Auditor, (Collection Report) and two (2) sets of summaries of collections and
remittances (Summaries), seeking an explanation from Gonzaga regarding his remittance shortages in the total
amount of P314,252.23,

Later, Gonzaga asked for an extension of three (3) weeks within which to submit his explanation since he needed to
go over the voluminous receipts of collections and remittances with the assistance of an accountant. On the same
day, he sent another letter, denying any unremitted amount on his part and thereby, requesting that the charges
against him be lifted. Attached to the same letter is an Audit Opinion prepared by one (Laluna), a certified public
accountant (CPA), stating that the Internal Auditor's Report cannot accurately establish any remittance shortage on
Gonzaga's part since the amount of collections stated in the Summaries was not supported by any bills or official
receipts.

Meanwhile, SURNECO formed an Investigation Committee to investigate Gonzaga's alleged remittance shortages.
Soon, the Committee sent Gonzaga an invitation to attend the investigation proceedings, in which he participated.
Pending investigation, Gonzaga was placed under preventive suspension.

Afterwards, the Committee tendered its report, finding Gonzaga guilty of (a) gross and habitual neglect of duty); (b)
misappropriation of REC funds and (c) failure to remit collections/monies under the Code of Ethics and Discipline for
Rural Electric Cooperative (REC) Employees (Code of Ethics. Thereafter, a notice of termination was served on
Gonzaga. Gonzaga sought reconsideration before SURNECO's Board of Directors but the latter denied the same
after he presented his case. Another notice of termination (Final Notice of Termination) was then served on Gonzaga.
Consequently, he was dismissed from the service.

In view thereof, Gonzaga filed a complaint with the LA for illegal dismissal claiming that he was denied due process
and dismissed without just cause. He alleged that while he was asked in the Memorandum to explain the remittance
shortage, he was nonetheless denied due process since the actual grounds for his dismissal, i.e., gross and habitual
neglect of duties and responsibilities, misappropriation of REC funds and failure to remit collections/monies, were not
indicated in the said memorandum. 16 He also claimed that petitioners' evidence failed to show any missing
collection since (a) the attached Summary of Collections and Remittances did not bear any receipt numbers, both
with respect to collections and remittances and (b) the other Summary of Collections and Remittances only contained
receipt numbers for the remittances and none for the collections.

The LA's Ruling: 1) Gonzaga’s dismissal was illegal since it was not founded on just cause. Hence he should be
reinstated. 2) Shortages in remittances not proved since actual receipts not presented in evidence. 3) Gonzaga not
afforded due process under the said code of ethics.

Thereafter, petitioner elevated matter to NLRC. Petitioner also submitted additional audit report by a CPA to
corroborate the findings of shortage.

NLRC reversed LA decision. NLRC found that the dismissal was valid. CA reversed NLRC decision, finding that
dismissal was illegal. Hence the petition.

Issue:

WON the dismissal was based on just and valid cause

Ruling:

Yes, the dismissal was valid.

In termination cases, the burden of proof rests on the employer to show that the dismissal is for a valid cause. Failing
in which, the law considers the matter a case of illegal dismissal.

The quantum of proof which the employer must discharge is substantial evidence which, as defined in case law,
means that amount of relevant evidence as a reasonable mind might accept as adequate to support a conclusion,
even if other minds, equally reasonable, might conceivably opine otherwise.

Petitioners were able to prove, by substantial evidence, that there lies a valid cause to terminate Gonzaga's
employment.

The Court concurs with the NLRC's finding that petitioners' evidence — which consists of the Collection Report, the
Summaries, and the September 15, 2003 Audit Report with attached Cash Flow Summary — adequately supports
the conclusion that Gonzaga misappropriated the funds of the cooperative.

Labor tribunals, such as the NLRC, are not precluded from receiving evidence submitted on appeal as technical rules
are not binding in cases submitted before them. In fact, labor officials should use every and reasonable means to
ascertain the facts in each case speedily and objectively, without regard to technicalities of law or procedure, all in the
interest of due process.
The evidence presented is sufficient to constitute substantial evidence to prove that he committed serious misconduct
and gross and habitual neglect of duty to warrant his dismissal from employment. Such are just causes for
termination which are explicitly enumerated under Article 296 of the Labor Code:

Article 296.Termination by Employer. — An employer may terminate an employment for any of the following causes:

(a)Serious Misconduct or wilful disobedience by the employee of the lawful orders of his employer or representative
in connection with his work;

(b)Gross and habitual neglect by the employee of his duties;

xxx xxx xxx

B. Termination procedure; statutory compliance.

The statutory procedure for terminating an employee is found in Section 2 (III), Rule XXIII, Book V of the Omnibus
Rules Implementing the Labor Code (Omnibus Rules) which states:

SEC. 2.Standards of due process: requirements of notice. — In all cases of termination of employment, the following
standards of due process shall be substantially observed:

For termination of employment based on just causes as defined in Article 282 of the Labor Code:

(i)A written notice served on the employee specifying the ground or grounds for termination, and giving said
employee reasonable opportunity within which to explain his side.

(ii)A hearing or conference during which the employee concerned, with the assistance of counsel if he so desires is
given opportunity to respond to the charge, present his evidence, or rebut the evidence presented against him.

(iii)A written notice of termination served on the employee, indicating that upon due consideration of all the
circumstances, grounds have been established to justify his termination.

Succinctly put, the foregoing procedure consists of (a) a first written notice stating the intended grounds for
termination; (b) a hearing or conference where the employee is given the opportunity to explain his side; and (c) a
second written notice informing the employee of his termination and the grounds therefor. Records disclose that
petitioners were able to prove that they sufficiently complied with these procedural requirements:

At this juncture, it must be pointed out that while petitioners have complied with the procedure laid down in the
Omnibus Rules, they, however, failed to show that the established company policy in investigating employees was
adhered to. In this regard, SURNECO's breach of its company procedure necessitates the payment of nominal
damages as will be discussed below.

C. Company procedure;

Consequences of breach.

Jurisprudence dictates that it is not enough that the employee is given an "ample opportunity to be heard" if company
rules or practices require a formal hearing or conference. In such instance, the requirement of a formal hearing and
conference becomes mandatory. In Perez v. Philippine Telegraph and Telephone Company, 56 the Court laid down
the following principles in dismissing employees:

(a)"ample opportunity to be heard" means any meaningful opportunity (verbal or written) given to the employee to
answer the charges against him and submit evidence in support of his defense, whether in a hearing, conference or
some other fair, just and reasonable way.

(b) A formal hearing or conference becomes mandatory only when requested by the employee in writing or
substantial evidentiary disputes exists or a company rule or practice requires it, or when similar circumstances justify
it.

(c) The "ample opportunity to be heard" standard in the Labor Code prevails over the "hearing and conference"
requirement in the implementing rules and regulations. The rationale behind this mandatory characterization is
premised on the fact that company rules and regulations which regulate the procedure and requirements for
termination, are generally binding on the employer. Thus, as pronounced in Suico v. NLRC, et al.:

Company policies or practices are binding on the parties. Some can ripen into an obligation on the part of the
employer, such as those which confer benefits on employees or regulate the procedures and requirements for their
termination.

Records reveal that while Gonzaga was given an ample opportunity to be heard within the purview of the foregoing
principles, SURNECO, however, failed to show that it followed its own rules which mandate that the employee who is
sought to be terminated be afforded a formal hearing or conference.

Accordingly, since only an informal inquiry was conducted in investigating Gonzaga's alleged cash shortages,
SURNECO failed to comply with its own company policy, violating the proper termination procedure altogether.

In this relation, case law states that an employer who terminates an employee for a valid cause but does so through
invalid procedure is liable to pay the latter nominal damages.
In Agabon v. NLRC (Agabon), the Court pronounced that where the dismissal is for a just cause, the lack of statutory
due process should not nullify the dismissal, or render it illegal, or ineffectual. However, the employer should
indemnify the employee for the violation of his statutory rights. Thus, in Agabon, the employer was ordered to pay
the employee nominal damages in the amount of P30, 000.00.

By analogy, the Court finds that the same principle should apply to the case at bar for the reason that an employer's
breach of its own company procedure is equally violative of the laborer's rights, albeit not statutory in source. Hence,
although the dismissal stands, the Court deems it appropriate to award Gonzaga nominal damages in the amount of
P30, 000.00.
69. Univac Developments Inc. vs. Soriano, G.R. No. 182072, June 19, 2013

Facts:

Soriano was hired by Univac Development, Inc., as legal assistant on probationary basis with a monthly salary of
P15, 000.00. Eight days prior to the completion of his probationary period, he was informed that he was being
terminated due to cost-cutting measures. He asked for 30 day notice but the termination was effective immediately.
He then filed for illegal dismissal against Univac Development, Inc. It was found out in this case that Soriano was not
apprised of the standards prescribed y the company for him to qualify as regular employee at the time of his
engagement. It was not even shown that performance evaluation was conducted to determine whether his
performance was unsatisfactory.

Issue:

Whether or not there was illegal dismissal.

Ruling:

YES. It is undisputed that respondent was hired as a probationary employee. As such, he did not enjoy a permanent
status. Nevertheless, he is accorded the constitutional protection of security of tenure which means that he can only
be dismissed from employment for a just cause or when he fails to qualify as a regular employee in accordance with
reasonable standards made known to him by the employer at the time of his engagement.

It is primordial that at the start of the probationary period, the standards for regularization be made known to the
probationary employee. Equally important is the requirement that in order to invoke "failure to meet the probationary
standards" as a justification for dismissal, the employer must show how these standards have been applied to the
subject employee.

Indeed, the power of the employer to terminate a probationary employee is subject to three limitations, namely: (1) it
must be exercised in accordance with the specific requirements of the contract; (2) the dissatisfaction on the part of
the employer must be real and in good faith, not feigned so as to circumvent the contract or the law; and (3) there
must be no unlawful discrimination in the dismissal.  In this case, not only did petitioner fail to show that respondent
was apprised of the standards for regularization but it was likewise not shown how these standards had been applied
in his case.

Pursuant to well-settled doctrine, petitioner's failure to specify the reasonable standards by which respondent's
alleged poor performance was evaluated as well as to prove that such standards were made known to him at the
start of his employment, makes respondent a regular employee. In other words, because of this omission on the part
of petitioner, respondent is deemed to have been hired from day one as a regular employee.

To justify the dismissal of an employee, the employer must, as a rule, prove that the dismissal was for a just cause
and that the employee was afforded due process prior to dismissal. Respondent's termination from employment is
without just and valid ground. Neither was due process observed, making his termination illegal.
70. Unilever Phils vs. Rivera, G.R. No. 201701, June 3, 2013

Facts:

Rivera was hired by Unilever as its Area Activation Executive in the cities of Cotabato and Davao. She was primarily
tasked with managing the sales, distribution and promotional activities in her area and supervising Ventureslink
International, Inc., a third party service provider for the company's activation projects. It was the policy of the
company that the allocated budget for specific activity must be used for such activity only. However, sometime in
2007, a random audit was conducted and found out that there were fictitious billings and fabricated receipts
supposedly coming from Ventureslink amounting to P11, 200,000.00. It was also discovered that some funds were
diverted from the original intended projects, and the deviation was upon the instruction of Rivera. She was made to
explain the incident by Unilever where she admitted the diversion of funds. She explained that diversion was made
because of the difficulty in procuring budget from the head office, and that the funds were utilized in the company’s
promotional venture in her area of coverage. Unilever found Rivera guilty of serious breach of the company's Code of
Business Principles compelling it to sever their professional relations. When the case reached the Court of appeals, it
upheld the validity of the dismissal but awarded Rivera a separation pay as a measure of social justice.

Issue:

Whether or not a validly dismissed employee, like Rivera, is entitled to an award of separation pay.

Ruling:

As a general rule, an employee who has been dismissed for any of the just causes enumerated under Article 282 of
the Labor Code is not entitled to a separation pay. In exceptional cases, however, the Court has granted separation
pay to a legally dismissed employee as an act of "social justice" or on "equitable grounds." In both instances, it is
required that the dismissal (1) was not for serious misconduct; and (2) did not reflect on the moral character of the
employee.

Separation pay is only warranted when the cause for termination is not attributable to the employee's fault, such as
those provided in Articles 283 and 284 of the Labor Code, as well as in cases of illegal dismissal in which
reinstatement is no longer feasible. It is not allowed when an employee is dismissed for just cause.

In this case, Rivera was dismissed from work because she intentionally circumvented a strict company policy,
manipulated another entity to carry out her instructions without the company's knowledge and approval, and directed
the diversion of funds, which she even admitted doing under the guise of shortening the laborious process of securing
funds for promotional activities from the head office. These transgressions were serious offenses that warranted her
dismissal from employment and proved that her termination from work was for a just cause. Hence, she is not entitled
to a separation pay.
71. Samr-med Distribution vs. NLRC, et al., G.R. No. 162385, July 15, 2013

Facts:

Gutang was hired by Samar-Med and had the task of supervising the company’s sales personnel and sales agents.
Later, Gutang filed a complaint for money claims against Samar-Med claiming that he had been illegally dismissed by
the later. Samar-Med on the other hand alleged that there was no dismissal to speak of but abandonment on
Gutang’s part; and that this is merely in retaliation to the criminal case filed by Samar-Med againt Gutang for
misappropriating its funds which, during the preliminary investigation thereof, was ruled that there is probable cause
that Gutang is guilty of estafa.

Issue:

WON Gutang was illegally dismissed.

Ruling:

The petition is partly meritorious.

The onus of proving that an employee was not dismissed or, if dismissed, his dismissal was not illegal fully rests on
the employer, and the failure to discharge the onus would mean that the dismissal was not justified and was illegal. In
Gutang’s case, Roleda tendered no showing outside of his mere allegations to substantiate his averment of
abandonment by Gutang. Moreover, although Gutang had undoubtedly stopped working for Samar Med, his doing so
had been for a justifiable reason, consisting in the non-payment of his salary since November 1995 and his being
forced to stop working for Samar Med to enable him to seek employment elsewhere, albeit temporarily, in order to
survive.

The dismissal, however, of Gutang was for a just cause. We note that Gutang was a managerial employee whom
Roleda had vested with confidence on delicate matters, such as the custody, handling, care and protection of Samar
Med’s properties and funds, as well as its operations and transactions in Region VIII. Gutang was shown to have
failed to account for and to turn over his sales collections. In that regard, Roleda’s filing of the criminal case against
Gutang and the public prosecutor’s finding of a prima facie case for the offense charged after preliminary
investigation amounted to substantial evidence of Gutang’s breach of the trust and confidence reposed in him, a just
cause to terminate the employment based on loss of trust and confidence.

Under Article 282(c) of the Labor Code, an employer may terminate an employee’s employment on the ground of the
latter’s fraud or willful breach of the trust and confidence reposed in him. For loss of trust and confidence to constitute
a sufficient ground for termination, the employer must have a reasonable ground to believe, if not to entertain the
moral conviction, that the employee was responsible for the misconduct, and that the nature of his participation
therein rendered him absolutely unworthy of the trust and confidence demanded by his position. Those requirements
were undeniably met in Gutang’s case.

The finding of a just cause to dismiss Gutang notwithstanding, we also find that he was not accorded due process.
Roleda as the employer had the obligation to send to him two written notices before finally dismissing him.

The first written notice would inform Gutang of the particular acts or omissions for which his dismissal was being
sought. The second written notice would notify him of the employer’s decision to dismiss him. But the second written
notice must not be made until after he was given a reasonable period after receiving the first written notice within
which to answer the charge, and after he was given the ample opportunity to be heard and to defend himself with the
assistance of his representative, if he so desired. The requirement was mandatory.

Gutang’s receipt of the demand letter from Samar-Med to return the amount of P3,302,000.71 was certainly not even
a substantial compliance with the twin-notice requirement, because the purpose of the demand letter was different
from those defined for the sending of the required notices. Nor was he thereby allowed a meaningful opportunity to be
heard or to be notified of his impending termination.

the lack of statutory due process would not nullify the dismissal or render it illegal or ineffectual when the dismissal
was for just cause. But the violation of Gutang’s right to statutory due process clearly warranted the payment of
indemnity in the form of nominal damages, whose amount is addressed to the sound discretion of the Court taking
into account the relevant circumstances. Accordingly, the Court deems the amount of P30,000.00 as nominal
damages sufficient vindication of Gutang's right to due process under the circumstances.
72. Naranjo et al, vs. Biomedica Health Care Inc. G.R. No. 193789, Sept. 19, 2012

Facts:

Petitioners are employees of respondent Biomedica. During the birthday of Biomedica’s President (Motol), petitioners
were all absent for various reasons. De Guzman was allegedly absent due to loose bowel movement, Pimentel for an
ophthalmology check-up, Bardaje due to migraine, Cruz for not feeling well, and Naranjo because he had to attend a
meeting at his child’s school. Notably, these are the same employees who filed a letter-complaint against Biomedica
for lack of salary increases, failure to remit Social Security System and Pag-IBIG contributions, and violation of the
minimum wage law, among other grievances.

Later that day, the petitioners were texted to report to work, but upon arriving, they were refused entry and told to
start looking for another job.

Biomedica issued a notice of preventive suspension to petitioners, to wit:

Subject: Notice of Preventive Suspension

& Notice to explain within 24 hours

Effective upon receipt hereof, you are placed under preventive suspension for willfully organizing and/or engaging in
illegal strike on November 7, 2006. Your said illegal act-in conspiracy with your other coemployees, paralyzed the
company operation on that day and resulted to undue damage and prejudice to the company and is direct violation of
Article XI, Category Four Section 6, 8, 12, 18 & 25 of our Company Policy, which if found guilty, you will be meted a
penalty of dismissal.

Please explain in writing within 24 hours from receipt hereof why you should not be held guilty of violating the
company policy considering further that you committed and timed such act during the birthday of our Company
president.

Thereafter, petitioners filed a complaint for constructive dismissal with NLRC.

Subsequently, Biomedica seved Notices of Termination on petitioners, to wit:

We regret to inform you that since you did not submit the written letter of explanation as requested in your preventive
suspension notice dated November 9, 2006, under Article XI, Category Four, Section 6, 8, 12, 18 and 25 you are
hereby dismissed from service effective immediately.

Issue:

WON petitoners were illegally dismissed

Held:

The petition is meritorious.

Petitioners were illegally dismissed

It bears pointing out that in the dismissal of an employee, the law requires that due process be observed. Such due
process requirement is twofold, procedural and substantive, that is, “the termination of employment must be based on
a just or authorized cause of dismissal and the dismissal must be effected after due notice and hearing.” In the instant
case, petitioners were not afforded both procedural and substantive due process.

Petitioners were not afforded Procedural due process

Art. 277(b) of the Labor Code contains the procedural due process requirements in the dismissal of an employee.

On the other hand, Rule XIII, Book V, Sec. 2 I (a) of the

Implementing Rules and Regulations of the Labor Code states:

SEC. 2. Standards of due process; requirements of notice.––In all cases of termination of employment, the following
standards of due process shall be substantially observed:

I. For termination of employment based on just causes as defined in Article 282 of the Code:

(a) A written notice served on the employee specifying the ground or grounds for termination, and giving said
employee reasonable opportunity within which to explain his side.

(b) A hearing or conference during which the employee concerned, with the assistance of counsel if he so desires is
given opportunity to respond to the charge, present his evidence, or rebut the evidence presented against him.

(c) A written notice of termination served on the employee, indicating that upon due consideration of all the
circumstances, grounds have been established to justify his termination.

Thus, the Court elaborated in King of Kings Transport, Inc. v.Mama:

x x x Moreover, in order to enable the employees to intelligently prepare their explanation and defenses, the notice
should contain a detailed narration of the facts and circumstances that will serve as basis for the charge against the
employees. A general description of the charge will not suffice. Lastly, the notice should specifically mention which
company rules, if any, are violated and/or which among the grounds under Art. 282 is being charged against the
employees.

Reading from the first notice sent by Biomedica, clearly, petitioners were charged with conducting an illegal strike, not
a mass leave, without specifying the exact acts that the company considers as constituting an illegal strike or violative
of company policies. Such allegation falls short of the requirement in King of Kings Transport, Inc. of “a detailed
narration of the facts and circumstances that will serve as basis for the charge against the employees.” A bare
mention of an “illegal strike” will not suffice.

Further, while Biomedica cites the provisions of the company policy which petitioners purportedly violated, it failed to
quote said provisions in the notice so petitioners can be adequately informed of the nature of the charges against
them and intelligently file their explanation and defenses to said accusations. The notice is bare of such description of
the company policies. Moreover, it is incumbent upon respondent company to show that petitioners were duly
informed of said company policies at the time of their employment and were given copies of these policies. No such
proof was presented by respondents.

Without a copy of the company policy being presented in the CA or the contents of the pertinent policies being quoted
in the pleadings, there is no way by which one can determine whether or not there was, indeed, a violation of said
company policies.

Moreover, the period of 24 hours allotted to petitioners to answer the notice was severely insufficient and in violation
of the implementing rules of the Labor Code. Under the implementing rule of Art. 277, an employee should be given
“reasonable opportunity” to file a response to the notice. King of Kings Transport, Inc. elucidates in this wise:

“Reasonable opportunity”under the Omnibus Rules means every kind of assistance that management must accord to
the employees to enable them to prepare adequately for their defense. This should be construed as a period of at
least five (5) calendar days from receipt of the notice to give the employees an opportunity to study the accusation
against them, consult a union official or lawyer, gather data and evidence, and decide on the defenses they will raise
against the complaint.

Following King of Kings Transport, Inc., the notice sent out by Biomedica in an attempt to comply with the first notice
of the due process requirements of the law was severely deficient.

In addition, Biomedica did not set the charges against petitioners for hearing or conference in accordance with Sec.
2, Book V, Rule XIII of the Implementing Rules and Regulations of the Labor Code and in line with ruling in King of
Kings Transport, Inc., where the Court explained:

(2) After serving the first notice, the employers should schedule and conduct a hearing or conference wherein the
employees will be given the opportunity to: (1) explain and clarify their defenses to the charge against them; (2)
present evidence in support of their defenses; and (3) rebut the evidence presented against them by the
management. During the hearing or conference, the employees are given the chance to defend themselves
personally, with the assistance of a representative or counsel of their choice. Moreover, this conference or hearing
could be used by the parties as an opportunity to come to an amicable settlement.

It is incumbent for Biomedica to set the matter for hearing or conference to hear the defenses and receive evidence
of the employees.

Lastly, Biomedica again deviated from the dictated contents of a written notice of termination as laid down in Sec. 2,
Book V, Rule XIII of the Implementing Rules that it should embody the facts and circumstances to support the
grounds justifying the termination. As amplified in King of Kings Transport, Inc.:

(3) After determining that termination of employment is justified, the employers shall serve the employees a written
notice of termination indicating that: (1) all circumstances involving the charge against the employees have been
considered; and (2) grounds have been established to justify the severance of their employment.

The November 26, 2006 Notice of Termination issued by Biomedica miserably failed to satisfy the requisite contents
of a valid notice of termination, as it simply mentioned the failure of petitioners to submit their respective written
explanations without discussing the facts and circumstances to support the alleged violations of Secs. 6, 8, 12, 18
and 25 of Category Four, Art. XI of the alleged company rules.

Petitioners were denied Substantive due process

In any event, petitioners were dismissed without just or authotized cause.The CA upheld the dismissal of petitioners
on the ground of serious misconduct.

Misconduct has been defined as improper or wrong conduct; the transgression of some established and definite rule
of action, a forbidden act, a dereliction of duty, unlawful in character implying wrongful intent and not mere error of
judgment. The misconduct to be serious must be of such grave and aggravated character and not merely trivial and
unimportant. To be a just cause for dismissal, such misconduct (a) must be serious; (b) must relate to the
performance of the employee’s duties; and (c) must show that the employee has become unfit to continue working for
the employer.

Clearly, to justify the dismissal of an employee on the ground of serious misconduct, the employer must first establish
that the employee is guilty of improper conduct, that the employee violated an existing and valid company rule or
regulation, or that the employee is guilty of a wrongdoing. In the instant case, Biomedica failed to even establish that
petitioners indeed violated company rules, failing to even present a copy of the rules and to prove that petitioners
were made aware of such regulations.

Petitioners did not stage a mass leave

The accusation is for engaging in a mass leave tantamount to an illegal strike.

The term “Mass Leave” has been left undefined by the Labor Code. Plainly, the legislature intended that the term’s
ordinary sense be used. “Mass” is defined as “participated in, attended by, or affecting a large number of individuals;
having a large-scale character.” While the term “Leave” is defined as “an authorized absence or vacation from duty or
employment usually with pay.”

Thus, the phrase “mass leave” may refer to a simultaneous availment of authorized leave benefits by a large number
of employees in a company.

It is undeniable that going on leave or absenting one’s self from work for personal reasons when they have leave
benefits available is an employee’s right.

In the factual milieu at bar, Biomedica did not submit a copy of the CBA or a company memorandum or circular
showing the authorized sick or vacation leaves which petitioners can avail of. Neither is there any document to show
the procedure by which such leaves can be enjoyed. Absent such pertinent documentary evidence, the Court can
only conclude that the availment of petitioners of their respective leaves was authorized, valid and in accordance with
the company or CBA rules on entitlement to and availment of such leaves. The contention of Biomedica that the
enjoyment of said leaves is in reality an illegal strike does not hold water in the absence of strong controverting proof
to overturn the presumption that “a person is innocent of x x x wrong.” Thus, the individual leaves of absence taken
by the petitioners are not such absences that can be regarded as an illegal mass action.

Moreover, a mass leave involves a large number of people or in this case, workers.

Here, the five (5) petitioners were absent that day. The records are bereft of any evidence to establish how many
workers are employed in Biomedica. There is no evidence on record that 5 employees constitute a substantial
number of employees of Biomedica. And, as earlier stated, it is incumbent upon Biomedica to prove that petitioners
were dismissed for just causes, this includes the duty to prove that the leave was large-scale in character and
unauthorized. This, Biomedica failed to prove.

Petitioners did not go on strike

Granting for the sake of argument that the absence of the 5 petitioners is considered a mass leave, still, their actions
cannot be considered a strike.

Art. 212(o) of the Labor Code defines a strike as “any temporary stoppage of work by the concerted action of
employees as a result of any industrial or labor dispute.”

“Concerted” is defined as “mutually contrived or planned” or “performed in unison.” In the case at bar, the 5
petitioners went on leave for various reasons. Petitioners were in different places on that day to attend to their
personal needs or affairs. They did not go to the company premises to petition Biomedica for their grievance. To
demonstrate their good faith in availing their leaves, petitions reported for work and were at the company premises in
the afternoon after they received text messages asking them to do so. This shows that there was NO intent to go on
strike.

Dismissal is not the proper penalty

Even setting aside the facts established above, petitioners still cannot be validly dismissed.

An ordinary striking worker cannot be terminated for mere participation in an illegal strike. There must be proof that
he committed illegal acts during a strike.

In the instant case, Biomedica has not alleged, let alone, proved the commission by petitioners of any illegal act
during the alleged mass leave. There being none, the mere fact that petitioners conducted an illegal strike cannot be
a legal basis for their dismissal.

Petitioners are entitled to separation pay in lieu of

Reinstatement, backwages and nominal damages

The normal consequences of respondents’ illegal dismissal, then, are reinstatement without loss of seniority rights,
and payment of backwages computed from the time compensation was withheld up to the date of actual
reinstatement. Where reinstatement is no longer viable as an option, separation pay equivalent to one (1) month
salary for every year of service should be awarded as an alternative. The payment of separation pay is in addition to
payment of backwages.
Petitioners were absent from work on Motol’s birthday. Respondent Motol, in the course of denying entry to them
uttered harsh, degrading and bad words. Petitioners were terminated in swift fashion and in gross violation of their
right to due process revealing that they are no longer wanted in the company. The convergence of these facts
coupled with the filing by petitioners of their complaint with the DOLE shows a relationship governed by antipathy and
antagonism as to justify the award of separation pay in lieu of reinstatement. Thus, in addition to backwages, owing to
the strained relations between the parties, separation pay in lieu of reinstatement would be proper. In Golden Ace
Builders, We explained why:

Under the doctrine of strained relations, the payment of separation pay is considered an acceptable alternative to
reinstatement when the latter option is no longer desirable or viable. On one hand, such payment liberates the
employee from what could be a highly oppressive work environment. On the other hand, it releases the employer
from the grossly unpalatable obligation of maintaining in its employ a worker it could no longer trust.

Strained relations must be demonstrated as a fact, however, to be adequately supported by evidence — substantial
evidence to show that the relationship between the employer and the employee is indeed strained as a necessary
consequence of the judicial controversy.

Petitioners are entitled tonominal damages in the amount of PhP 30,000 each for Biomedica’s violation of procedural
due process.
73. Manila Jockey Club Inc. vs. Trajano, G.R. No. 160982, June 26, 2013

Facts

MJCI had employed Trajano as a selling teller of betting tickets since November 1989. On April 25, 1998, she
reported for work. At around 7:15 p.m., two regular bettors gave her their respective lists of bets (rota) and money for
the bets for Race 14. Although the bettors suddenly left her, she entered their bets in the selling machine and
segregated the tickets for pick up by the two bettors upon their return. Before closing time, one of the bettors
(requesting bettor) returned and asked her to cancel one of his bets worth P2,000.00. Since she was also operating
the negative machine on that day, she obliged and immediately cancelled the bet as requested. She gave the
remaining tickets and the P2,000.00 to the requesting bettor, the money pertaining to the canceled bet. When Race
14 was completed, she counted the bets received and the sold tickets. She found that the bets and the tickets
balanced. But then she saw in her drawer the receipt for the canceled ticket, but the canceled ticket was not inside
the drawer. Thinking she could have given the canceled ticket to the requesting bettor, she immediately looked for
him but could not find him. It was only then that she remembered that there were two bettors who had earlier left their
bets with her. Thus, she went to look for the other bettor (second bettor) to ask if the canceled ticket was with him.
When she located the second bettor, she showed him the receipt of the canceled ticket to counter-check the serial
number with his tickets.

Thereafter, the second bettor returned to Trajano and told her that it was one of his bets that had been canceled,
instead of that of the requesting bettor. To complicate things, it was also the same bet that had won Race 14.
Considering that the bet was for a daily double, the second bettor only needed to win Race 15 in order to claim
dividends. At that point, she realized her mistake, and explained to the second bettor that the cancellation of his ticket
had not been intentional, but the result of an honest mistake on her part. She offered to personally pay the dividends
should the second bettor win Race 15, which the latter accepted. When Race 15 was completed, the second bettor
lost. She was thus relieved of the obligation to pay any winnings to the second bettor. 

To her surprise, the reliever-supervisor later approached Trajano and told her to submit a written explanation about
the ticket cancellation incident. The next day (April 26, 1998), she submitted the handwritten explanation to Atty. Joey
R. Galit, Assistant Racing Supervisor. She then resumed her work as a selling teller, until later that day, when she
received an inter-office correspondence signed by Atty. Galit informing her that she was being placed under
preventive suspension effective April 28, 1998, for an unstated period of time. At the end of thirty days of her
suspension, Trajano reported for work. But she was no longer admitted.  She then learned that she had been
dismissed when she read a copy of an inter-office correspondence about her termination posted in a selling station of
MJCI. 

Trajano instituted a complaint  for illegal dismissal against MJCI in the Department of Labor and Employment
(DOLE). She claimed that her dismissal was not based on any of the grounds enumerated under Article 282 of
the Labor Code; that her dismissal on the ground of unauthorized cancellation of ticket had no basis because she
was also the operator of the negative machine on the day in question with the authority to cancel tickets as
requested; that the cancellation was not intentional on her part but resulted from an honest mistake that did not
amount to dishonesty; that her dismissal was without due process of law because she was not aware of any
justifiable cause of her termination; that she was not notified about or furnished a copy of the notice of dismissal; that
instead, MJCI simply posted copies of the notice in all its selling stations, an act intended to embarrass and humiliate
her by imputing an allegedly unauthorized cancellation of ticket against her; and that MCJI's acts were tainted with
evident bad faith and malice.

Trajano prayed that she be reinstated to her former position without loss of seniority rights; that she be paid
backwages until she would be fully reinstated; and that she be paid moral and exemplary damages amounting to
P180,000.00 and attorney's fees of 10% of the total award. 

On its part, MJCI averred that on April 25, 1998, it received a letter from Jun Carpio, the Field Officer of the Games
and Amusement Board, calling its attention to a complaint against Trajano brought by a certain bettor named "Tito"
who had reported the cancellation of his ticket that had already won the first leg (Race 14) of the daily double bet; that
it acted on the complaint by placing her under preventive suspension  upon her submission of a written
explanation  and after the conduct of preliminary investigation on the matter; that on June 5, 1998, it invited her to a
clarificatory meeting in the presence of MJCI Raceday Union President Miguel Altonaga; and that it terminated her
services on the next day "for cause due to unauthorized cancellation of ticket." 

MJCI maintained that Trajano's dismissal was justified because the unauthorized cancellation of the ticket had
constituted a serious violation of company policy amounting to dishonesty; that her action had also constituted a just
cause for terminating her employment under Article 282 of the Labor Code, particularly paragraph(a) on serious
misconduct or willful disobedience and paragraph (b) on gross and habitual neglect of duty; that the admissions made
in her written explanation left no doubt as to her participation in the unauthorized cancellation of the ticket; that she
was afforded her right to due process by being given the chance to submit her written explanation and being
appraised of the charges against her; that she was accompanied by the union leaders during the preliminary
investigation of her case; and that the non-appeal of the decision to terminate her indicated that she and the union
leaders believed in the merit of the decision to terminate her. 

Issues

1.Whether or not there was just cause when Petitioner (MJCI) dismissed Respondent Aimee O. Trajano from the
service;  and
2.Whether or not Petitioner MJCI complied with the due process requirement when it effected the dismissal of
Respondent Trajano. 
Ruling

MJCI posits that Trajano held a position of trust and confidence; that the act of canceling the ticket was unauthorized
because it was done without the consent of the bettor; that the CA thus erred in construing the phrase unauthorized
cancellation of ticket  as referring to whether or not she was authorized to cancel the ticket pursuant to company
rules; that under the same premise, the loss of trust and confidence was established because the unauthorized
cancellation of the ticket was a serious misconduct on her part considering that had the bet of P2,000.00 won the
daily double race, the dividend to be paid could have been such a big amount that she would be unable to pay on her
own; that the repercussions of her act to MJCI would have been disastrous had the bet won, with MJCI being sued by
the bettor and being scandalized in the media; that MJCI would have suffered great loss in both income and
reputation due to such unauthorized cancellation of ticket; and that, consequently, MJCI had the just cause to dismiss
her. 

We cannot sustain the position of MJCI.

The valid termination of an employee may either be for just causes under Article 282  or for authorized causes under
Article 283 and Article 284,  all of the Labor Code.

Specifically, loss of the employer's trust and confidence is a just cause under Article 282 (c), a provision that ideally
applies only to cases involving an employee occupying a position of trust and confidence, or to a situation where the
employee has been routinely charged with the care and custody of the employer's money or property.  But the loss of
trust and confidence, to be a valid ground for dismissal, must be based on a willful breach of trust and confidence
founded on clearly established facts. "A breach is willful, if it is done intentionally, knowingly and purposely, without
justifiable excuse, as distinguished from an act done carelessly, thoughtlessly, heedlessly or inadvertently. It must
rest on substantial grounds and not on the employer's arbitrariness, whims, caprices or suspicion; otherwise, the
employee would eternally remain at the mercy of the employer."  An ordinary breach is not enough. 

Moreover, the loss of trust and confidence must be related to the employee's performance of duties.

Loss of confidence, as a just cause for termination of employment, is premised on the fact that the employee
concerned holds a position of responsibility, trust and confidence. He must be invested with confidence on delicate
matters such as the custody, handling, care and protection of the employer's property and/or funds. But in order to
constitute a just cause for dismissal, the act complained of must be "work-related" such as would show the employee
concerned to be unfit to continue working for the employer.

As a selling teller, Trajano held a position of trust and confidence. The nature of her employment required her to
handle and keep in custody the tickets issued and the bets made in her assigned selling station. The bets were funds
belonging to her employer. Although the act complained of — the unauthorized cancellation of the ticket
(i.e., unauthorized because it was done without the consent of the bettor) — was related to her work as a selling
teller, MJCI did not establish that the cancellation of the ticket was intentional, knowing and purposeful on her part in
order for her to have breached the trust and confidence reposed in her by MJCI, instead of being only out of an
honest mistake.

Still, to justify the supposed loss of its trust and confidence in Trajano, MJCI contends that the unauthorized
cancellation of the ticket could have greatly prejudiced MJCI for causing damage to both its income and
reputation. CASTDI

We consider the contention of MJCI unwarranted. As the records indicate, MJCI's prejudice remained speculative
and unrealized. To dismiss an employee based on speculation as to the damage the employer could have suffered
would be an injustice. The injustice in the case of Trajano would be greater if the supposed just cause for her
dismissal was not even sufficiently established. While MJCI as the employer understandably had its own interests to
protect, and could validly terminate any employee for a just cause, its exercise of the power to dismiss should always
be tempered with compassion and imbued with understanding, avoiding its abuse. 

In this regard, we have to stress that the loss of trust and confidence as a ground for the dismissal of an employee
must also be shown to be genuine, for, “the loss of confidence should not be simulated in order to justify what would
otherwise be, under the provisions of law, an illegal dismissal. It should not be used as a subterfuge for causes which
are illegal, improper and unjustified. It must be genuine, not a mere afterthought to justify an earlier action taken in
bad faith."

The foregoing notwithstanding, the Court unavoidably notes that the invocation of loss of trust and confidence as a
ground for dismissing Trajano was made belatedly. In its position paper dated September 2, 1998,  MJCI invoked the
grounds under Article 282 (a) and (b) of the Labor Code to support its dismissal of her, submitting then that the
unauthorized cancellation of the ticket constituted a serious violation of company policy amounting to dishonesty. The
first time that MJCI invoked breach of trust was in its motion for the reconsideration of the decision of the
NLRC.  MJCI also thereafter urged the ground of breach of trust in its petition for certiorari  in the CA.  Such a belated
invocation of loss of confidence broadly hints the ground as a mere afterthought to buttress an otherwise baseless
dismissal of the employee.

Anent compliance with due process, MJCI argues that Trajano's notification of her termination through the posting in
the selling stations should be deemed a substantial if not full compliance with the due process requirement,
considering that she herself even presented a copy of the posting as evidence;  that the rule on giving notice of
termination to an employee did not expressly require the personal service of the notice to the dismissed worker; and
that what mattered was that she was notified in writing of MJCI's decision to terminate her through the posting in its
selling stations. 
The argument is bereft of worth and substance. 

The procedure to be followed in the termination of employment based on just causes is laid down in Section 2 (d),
Rule I of the Implementing Rules of Book VI of the Labor Code, to wit:

Section 2.Security of Tenure. —

xxx xxx xxx

(d)In all cases of termination of employment, the following standards of due process shall be substantially
observed:
For termination of employment based on just causes as defined in Article 282 of the Labor Code:

(i)A written notice served on the employee specifying the ground or grounds for termination, and giving said
employee reasonable opportunity within which to explain his side.

(ii)A hearing or conference during which the employee concerned, with the assistance of counsel if he so
desires is given opportunity to respond to the charge, present his evidence, or rebut the evidence presented
against him.

(iii)A written notice of termination served on the employee, indicating that upon due consideration of all the
circumstances, grounds have been established to justify his termination. In case of termination, the
foregoing notices shall be served on the employee's last known address.

A review of the records warrants a finding that MJCI did not comply with the prescribed procedure.

There is no question that an illegally dismissed employee is entitled to her reinstatement without loss of seniority
rights and other privileges, and to full backwages, inclusive of allowances and other benefits or their monetary
equivalent. 

In case the reinstatement is no longer possible, however, an award of separation pay, in lieu of reinstatement, will be
justified.  The Court has ruled that reinstatement is no longer possible: (a) when the former position of the illegally
dismissed employee no longer exists;  or (b) when the employer's business has closed down;  or (c)when the
employer-employee relationship has already been strained as to render the reinstatement impossible.  The Court
likewise considered reinstatement to be non-feasible because a "considerable time" has lapsed between the
dismissal and the resolution of the case.  In that regard, a lag of eight years or ten years is sufficient to justify an
award of separation pay in lieu of reinstatement. 

Applying the foregoing to this case, the Court concludes that the reinstatement of Trajano is no longer feasible. More
than 14 years have already passed since she initiated her complaint for illegal dismissal in 1998, filing her position
paper on September 3, 1998,  before the Court could finally resolve her case. The lapse of that long time has
rendered her reinstatement an impractical, if not an impossible, option for both her and MJCI. Consequently, an
award of separation pay has become the practical alternative, computed at one month pay for every year of service. 

Anent backwages, Trajano is entitled to full backwages, inclusive of allowances and other benefits or their monetary
equivalent, computed from the time her actual compensation was withheld on June 6, 1998 up to the finality of this
decision (on account of her reinstatement having meanwhile become non-feasible and impractical). This ruling is
consistent with the legislative intent behind Republic Act No. 6715. 
74. Fianza vs. NLRC et al., G.R. No. 163061, June 26, 2013

Facts

On 3 June 1997, petitioner Fianza was employed as Officer for Social Acceptance of respondent Binga Hydroelectric
Plant, Inc. The details of his employment are embodied in Memorandum 97-10 dated 2 June 1997 issued by Mr.
Catalino Tan, the president and chairperson of the board at that time. 

In February 1999, petitioner did not receive his salary of P15,000 for the first 15 days of the month of February. He
was advised not to report for work until his status was officially clarified by the Manila office. 

After petitioner made several other inquiries concerning his status,  he was told by a supervisor to report for work. 
However, he was also told that the new management committee had to concur in his reappointment before he could
be reinstated in the payroll.  It also wanted an opportunity to determine whether his services would still be necessary
to the company.  Meanwhile, the chief of the rehabilitation department of the company recommended his return. 

As the management committee did not act on his inquiries for several months, on 24 May 1999 petitioner filed a
Complaint for illegal dismissal before the LA. 

Issues

There are, in essence, two important questions to be answered: first, whether petitioner abandoned his work; and
second, whether his employment was regular.

Ruling

In his pleadings, petitioner argues that he was a supervisory employee, as shown by the evidence he presented and
the nature of his work. He further contends that he did not abandon his work, because he always made sure he
followed up the status of his employment, and he was willing to go back to work once he was re-enrolled in the
payroll. 

Respondent company asserts in its Memorandum that petitioner was a confidential consultant of its former president
and chairperson Catalino Tan. As such, petitioner's tenure was therefore co-terminus with that of Mr. Tan.

At the outset, it is clear that the requisites for a judicial declaration of abandonment are absent in this case. Suffice it
to say that abandonment is a fact that must be proven in accordance with the standard set by this Court: 

It is well-settled in our jurisprudence that "For abandonment to constitute a valid cause for termination of
employment, there must be a deliberate, unjustified refusal  of the employee to resume his employment.
This refusal must be clearly shown. Mere absence is not sufficient, it must be accompanied by overt acts
unerringly pointing to the fact that the employee does not want to work anymore" (Emphasis and italics
supplied) 

Abandonment as a fact and a defense can only be claimed as a ground for dismissal if the employer follows the
procedure set by law.  In line with the burden of proof set by law, the employer who alleges abandonment "has the
burden of proof to show a deliberate and unjustified refusal of the employee to resume his employment without any
intention of returning."  As this Court has stated in  Agabon v. National Labor Relations Commission:  ECaTAI

For a valid finding of abandonment, these two factors should be present: (1) the failure to report for work or
absence without valid or justifiable reason; and (2) a clear intention to sever employer-employee
relationship, with the second as the more determinative factor which is manifested by overt acts from which
it may be deduced that the employees has no more intention to work. The intent to discontinue the
employment must be shown by clear proof that it was deliberate and unjustified. 

From the foregoing, it is clear that respondent company failed to prove the necessary elements of abandonment.
Additionally, the NLRC and the CA failed to take into account the strict requirements set by jurisprudence when they
determined the existence of abandonment on the basis of mere allegations that were contradicted by the evidence
shown.

The very act of filing the Complaint for illegal dismissal should have negated any intention on petitioner's part to sever
his employment.  In fact, it should already have been sufficient evidence to declare that there was no abandonment of
work. Moreover, petitioner went back to the company several times to inquire about the status of his
employment.  The fact that his inquiries were not answered does not prejudice this position.

Throughout the entire ordeal, petitioner was vigilant in protecting himself from any claim that he had abandoned his
work. The following circumstances evinced his intent to return to work:

1.His continuous inquiry with respondent about the status of his work. 
2.His willingness to return to work at any time, subject to the approval of respondent, and his visits to the
plant to apply for work. 
3.His filing of an illegal dismissal case. 

Considering all these facts, established by the LA and confirmed by the NLRC and the CA, we conclude that both
appellate bodies were remiss in declaring the existence of abandonment. ECSHAD
Since the first question has been disposed of, the second one now becomes the core issue, because the existence of
an employer-employee relationship in the nature of regular employment will determine whether or not the company
dismissed petitioner illegally.
Respondent company claims that because petitioner was a confidential employee of its former president, his tenure
was co-terminus with that of his employer.  To establish this contention, respondent cites the CA's determination of
the facts, as follows:

1.Petitioner directly reported to Mr. Tan, the hiring authority.


2.The hiring did not pass through the existing procedure.
3.The position of officer for social acceptance was absent from the company's table of organization and
position title.
4.Petitioner did not submit any daily time record.
5.Monthly fees received from Mr. Tan were denominated as retainer fees and subjected to 10% deductions.
6.Petitioner was not included in the payroll.
7.The taxes on the fees were paid by respondent company on behalf of petitioner.
8.Petitioner's name was absent from respondent's records. 

These facts allegedly proved that petitioner was the confidential employee of Mr. Tan, respondent's former
president.  All of this occurred in the context of a rehabilitation receivership conducted by the Securities and
Exchange Commission Management Committee. 

Respondent company failed to realize however that Mr. Tan, being its president, was clothed with authority to hire
employees on its behalf. This was precisely the import of petitioner's appointment papers, which even carried the
letterhead of the company.  There is no indication from the facts that his employment was of a confidential nature.
The wording of his appointment itself does not bear out that conclusion. 

Several things stand out in his appointment paper. First, its letterhead is that of respondent company, indicating the
official nature of the document. Second, there is no indication that the employment is co-terminus with that of the
appointing power, or that the position was a confidential one. In fact, alongside the obligation of petitioner to report to
Mr. Tan, is that of reporting to those whom the latter had designated as well as to the management in case petitioner
had any suggestion. This description evinces a supervisory function, by which the employee will carry out company
policy, but can only give suggestions to management as to the creation or implementation of a new policy. 

Finally, the appointment paper recognizes that the petitioner would initially be on probation status for two months, at
the end of which he would be made a permanent employee should his services be found satisfactory by respondent.
All these circumstances are evident from the appointment paper itself, which belies the claim of respondent that it had
no employer-employee relationship with petitioner.
75. Pasos vs. Phil National Construction Corp. G.R. No. 192394, July 3, 2013

Facts:

Roy D. Pasos (Pasos) was first hired by Philippine National Construction Corporation (PNCC) on 26 April 1996. In his
employment contract, he was designated as a Project Employee from 26 April 1996 to 25 July 1996 and was
assigned to NAIA-II Project as Clerk II (Accounting). His employment however did not end on 25 July 1996 but was
extended until 4 August 1998 (or more than two years later). His employment contract specifically provided that, “If
services are still needed beyond the validity of this contract, the Company shall extend your (Pasos’) services. After
services are terminated, the employee shall be under no obligation to re-employ with the Company nor shall the
Company be obliged to re-employ the employee”.

Pasos was again hired on 11 November 1998, this time as project employee from 11 November 1998 to 11 February
1999 and assigned to PCSO-Q.I. Project as Accounting Clerk (Reliever). However, his employment did not actually
end on 11 February 1999 but was extended until 19 February 1999.

On 23 February 1999, Pasos was again hired as Accounting Clerk and was assigned to SM Project. This time
though, his employment contract did not specify the date when his employment will end but it was stated therein that
it will be co-terminus with the termination of the project. Said employment supposedly ended on 19 August 1999.
However, it appears that said employment was extended as Pasos was again appointed as Accounting Clerk for SM
Project (Package II). Again, the employment contract did not state a specific date up to when his extended
employment will be.

Pasos’ employment was supposedly terminated on 19 October 2000. Pasos however claimed that his superior
instructed him to report for work the following day, intimating to him that he will again be hired in succeeding SM
Projects. For purposes of re-employment, Pasos underwent medical examination which allegedly revealed that he
had pneumonitis. He was advised to take a 14-day sick leave. After his 14-day leave, he again underwent a medical
examination which revealed that he has Koch’s Disease. He was advised to go on a 6-month leave of absence. After
such leave of absence he was declared fit to work, but was denied employment. He was told that his employment
ended on 19 October 2000. PNCC reasoned that it was not obliged to rehire Pasos, as he was a Project Employee
and his employment ended on 19 October 2000.

Hence this case for illegal dismissal with prayer for backwages and reinstatement.

Issues:

1. Is Pasos a regular employee and not a mere project employee and thus can only be dismissed for cause?

2. Is Pasos entitled to reinstatement?

Ruling:

1) Yes.

In the instant case, the appointments issued to petitioner indicated that he was hired for specific projects. This Court
is convinced however that although he started as a project employee, he eventually became a regular employee of
PNCC.

The principal test used to determine whether employees are project employees is whether or not the employees were
assigned to carry out a specific project or undertaking, the duration or scope of which was specified at the time the
employees were engaged for that project.

In the case at bar, petitioner worked continuously for more than two years after the supposed three-month duration of
his project employment for the NAIA II Project. While his appointment for said project allowed such extension since it
specifically provided that in case his "services are still needed beyond the validity of the contract, the Company shall
extend his services," there was no subsequent contract or appointment that specified a particular duration for the
extension.

While for first three months, petitioner can be considered a project employee of PNCC, his employment thereafter,
when his services were extended without any specification of as to the duration, made him a regular employee of
PNCC. And his status as a regular employee was not affected by the fact that he was assigned to several other
projects and there were intervals in between said projects since he enjoys security of tenure.

A regular employee dismissed for a cause other than the just or authorized causes provided by law is illegally
dismissed. Petitioner’s regular employment was terminated by PNCC due to contract expiration or project completion,
which are both not among the just or authorized causes provided in the Labor Code, as amended, for dismissing a
regular employee. Thus, petitioner was illegally dismissed.

2) Yes.

Article 279 of the Labor Code, as amended, provides that an illegally dismissed employee is entitled to reinstatement,
full back wages, inclusive of allowances, and to his other benefits or their monetary equivalent from the time his
compensation was withheld from him up to the time of his actual reinstatement.

We agree with petitioner that there was no basis for the Labor Arbiter’s finding of strained relations and order of
separation pay in lieu of reinstatement. This was neither alleged nor proved. Moreover, it has long been settled that
the doctrine of strained relations should be strictly applied so as not to deprive an illegally dismissed employee of his
right to reinstatement.

As held in Globe-Mackay Cable and Radio Corporation v. NLRC:

Obviously, the principle of "strained relations" cannot be applied indiscriminately. Otherwise, reinstatement can never
be possible simply because some hostility is invariably engendered between the parties as a result of litigation. That
is human nature.

Besides, no strained relations should arise from a valid and legal act of asserting one’s right; otherwise an employee
who shall assert his right could be easily separated from the service, by merely paying his separation pay on the
pretext that his relationship with his employer had already become strained.
76 Universal Robina Corp et al., v. Castillo G.R. No. 189686, July 10, 2013

Facts:

Wilfredo Castillo (Castillo) was hired by Universal Robina Corporation (URC) as a truck salesman on 23 March 1983.
He rose to the ranks and eventually became a Regional Sales Manager until his dismissal on 12 January 2006.

As Regional Sales Manager, he is in tasked to transact, sign and represent the company in all its dealings with key
accounts or customers subject however to his selling expense budget duly approved by URC Management.
Consequently, he is obliged to give an account of all his dealings or transactions with all his customers to URC. One
such customer is Liana’s Supermarket (Liana’s).

Sometime in August 2005, URC’s Credit and Collection Department Analyst noted an outright deduction in the
amount of Php 72,000 tagged as Gift Certificate. This was unusual and so an investigation was conducted, which
investigation led to the conclusion that Castillo received Gift Certificates from Liana’s worth Php 72 000, which
amount remains unresolved in the URC Account Receivables records. The Corporate Internal Audit, who conducted
the investigation, suspected that Castillo may have committed an act of fraud against the company and Liana’s for his
personal gain.

When Liana’s Vice President confirmed that Castillo received Gift Certificate worth Php 70 000 from Liana, Castillo
was asked was asked to explain in writing why the company should not institute the appropriate disciplinary action
against him.

Further investigation likewise revealed that Castillo signed two (2) blank Charge Invoices of Liana’s Supermarket.
Thus, another memo was sent to Castillo directing him to explain why no administrative sanctions should be meted
against him.

Castillo repeatedly denied that he signed two (2) blank Charge Invoices. Clarification inquiries were held and on 9
January 2006, respondent was served with a written notice of termination.

Thus on 30 May 2006, Castillo filed a case for illegal dismissal against URC. The Labor Arbiter rendered a decision in
Castillo’s favor and directed URC to pay Backwages, Separation Pay and Attorney’s Fees. URC appealed to NLRC
which reversed the Labor Arbiter finding the dismissal of Castillo as valid. On Certiorari to CA, the dismissal was
upheld but the award of Separation Pay was reinstated, “as a form of equitable relief”. Hence, this case.

Issues:

Whether Castillo is entitled to Separation Pay

Held:

No.

Why and when separation pay may be awarded or denied, has been the subject of many cases. We pick out the
rulings pertinent to the case at hand.

As the rule now stands, the award of separation pay is authorized in the situations dealt with in Article 283 and 284 of
the Labor Code, but not in terminations of employment based on instances enumerated in Article 282.18Article 282
states that:

ART. 282. Termination by employer. – An employer may terminate an employment for any of the following causes:

(a) Serious misconduct or willful disobedience by the employee of the lawful orders of his employer or representative
in connection with his work;

(b) Gross and habitual neglect by the employee of his duties;

(c) Fraud or willful breach by the employee of the trust reposed in him by his employer or duly authorized
representative;

(d) Commission of a crime or offense by the employee against the person of his employer or any immediate member
of his family or his duly authorized representatives; and

(e) Other causes analogous to the foregoing.

Central Philippines Bandag Retreaders, Inc. cautioned labor tribunals in indiscriminately awarding separation pay as
a measure of social justice, in this wise:

x x x Labor adjudicatory officials and the CA must demur the award of separation pay based on social justice when
an employee’s dismissal is based on serious misconduct or willful disobedience; gross and habitual neglect of duty;
fraud or willful breach of trust; or commission of a crime against the person of the employer or his immediate family—
grounds under Art. 282 of the Labor Code that sanction dismissals of employees. They must be most judicious and
circumspect in awarding separation pay or financial assistance as the constitutional policy to provide full protection to
labor is not meant to be an instrument to oppress the employers. The commitment of the Court to the cause of labor
should not embarrass us from sustaining the employers when they are right, as here. In fine, we should be more
cautious in awarding financial assistance to the undeserving and those who are unworthy of the liberality of the law.
Indeed, respondent has committed acts constituting willful breach of trust and confidence reposed on him by URC
based on the following facts established by the Court of Appeals, thus:

x x x the principal charge against petitioner Castillo was hinged upon "unauthorized arrangements" which he
allegedly entered into. Petitioner Castillo’s unauthorized dealing with respect to the changes in the Account
Development Agreement is exactly the offending cause of the host of infractions he committed, i.e., his neglect in
signing the blank charge invoices and his improper receipt of gift certificates for his personal gain. These acts taken
together constitute a breach of the trust and confidence reposed on petitioner Castillo by private respondent URC. x x
x

Nonetheless, the evidence on record negates petitioner Castillo’s claim of good faith and furnishes sufficient basis for
the breach of trust and loss of confidence reposed on him by private respondent URC. Petitioner Castillo’s receipt of
the gift certificates is categorically confirmed by Peter Sy, the Vice President of Marketing of Liana’s Supermarket.
This piece of evidence, coming from a disinterested party, speaks eloquently of petitioner Castillo’s perfidy. Such an
affirmative statement coupled with petitioner Castillo's signatures on the charge invoices convincingly established the
fact that he indeed received the P72,000.00 worth of gift certificates.

In Bank of the Philippine Islands v. NLRC and Arambulo, we ruled that an employee who has been dismissed for a
just cause under Article 282 of the Labor Code is not entitled to separation pay. The complainant therein was likewise
dismissed on the ground of Joss of trust and confidence. Applying that rule to the instant case, we here hold that
respondent is not entitled to separation pay.
77. Martinez vs. Central Pangasinan Electric Cooperative, G.R. No. 192306, July 15, 2013

Facts:

In 1991, CENPELCO employed Martinez on a contractual basis and in 1993, was subsequently regularized as a
billing clerk at the former's main office in San Carlos City, Pangasinan. On January 7, 2002, CENPELCO gave
Martinez the position of teller at Area VI in Malasiqui, Pangasinan.

On April 26, 2002, CENPELCO’s Internal Audit Department (IAD) conducted a cash count audit at its Area VI.
Josefina Mandapat (Mandapat), the IAD Officer-in-Charge, analyzed the audit results and concluded that there was
an error in the count of Benjamin Madriaga (Madriaga), cashier for Area VI, regarding the breakdown of collection
turned over by Martinez for April 23, 2002. Specifically, Madriaga erroneously recorded that Martinez remitted 390
pieces of P500-bills, instead of the correct number which was just 290, and issued a handwritten temporary receipt
for P406,130.31 instead of P360,447.13. Upon noting that Madriaga issued Official Receipts Nos. 77365-77367 for
the amount of P360,447.13 with corresponding remittance stubs for Martinez’s April 23, 2002 collections, Mandapat
concluded that Martinez’s overage for the same day in the amount of P45,682.58 is questionable. Further, Mandapat
noted that on April 25, 2002, Martinez committed a shortage in the amount ofP44,846.77, considering that the latter’s
total accountability for the said date is in the amount of P212,258.56 but his actual cash count only amounted to
P167,411.79.

In view of such audit, Mandapat recommended that Madriaga and Martinez be made to explain why no disciplinary
action should be taken against them. Thus, on May 15, 2002, Martinez filed his letter-explanation, explaining that he
submitted his collections and remittance stubs to Madriaga who was the one tasked to make the report thereon and
who may have mishandled the proper listing and tallying of the money collected vis-à-vis the collection stubs. He
further admitted the existence of such shortage and tried to offset the same with his alleged overage on April 23,
2002.

On June 30 2002, the Company’s Grievance Committee, which was commissioned to investigate the charges
imputed to Martinez, submitted its report recommending Martinez’s termination from employment as well as the filing
of the appropriate case in court. On November 26, 2002, Martinez was dismissed from service, prompting him to file
a complaint for illegal dismissal with money claims for 13th month pay, service incentive leave pay and allowances,
as well as moral and exemplary damages.

Issue:

The sole issue raised for the Court’s resolution is whether Martinez’s dismissal on the ground of loss of trust and
confidence is valid.

Ruling:

The petition is bereft of merit.

To validly dismiss an employee on the ground of loss of trust and confidence under Article 296(c) (formerly Article
282[c]) of the Labor Code, the following guidelines must be observed: (1) the employee concerned must be holding a
position of trust and confidence; and (2) there must be an act that would justify the loss of trust and confidence.

Anent the first requisite, it is noteworthy to mention that there are two classes of positions of trust, namely: (1)
managerial employees whose primary duty consists of the management of the establishment in which they are
employed or of a department or a subdivision thereof, and to other officers or members of the managerial staff; and
(2) fiduciary rank-and-file employees such as cashiers, auditors, property custodians, or those who, in the normal
exercise of their functions, regularly handle significant amounts of money or property. These employees, though rank-
and-file, are routinely charged with the care and custody of the employer’s money or property, and are thus classified
as occupying positions of trust and confidence. Being an employee tasked to collect payments and remit the same to
CENPELCO, Martinez belongs to the latter class and thus, occupies a position of trust and confidence.

Anent the second requisite, the audit report conducted on Martinez's cash count revealed that he had a shortage in
the amount of P44,846. When asked to explain such shortage, Martinez not only admitted the same but even tried to
exculpate himself from liability by attempting to offset said shortage with his alleged overage on April 23, 2002 in the
amount of P45,682.58. The Court agrees with the CA that this practice should never be countenanced because it
would allow the employees to patch up inaccuracies or even their own wrongdoings and thus, the true revenues or
losses of the company will never be correctly identified. Verily, this irregular practice would be detrimental to the
interests of the employer whose bread and butter depends solely on realized profits. Perforce, Martinez's failure to
properly account for his shortage of such a significant amount is enough reason for CENPELCO to lose trust and
confidence in him.
78. Zuellig Pharma Corp vs. Sibal et al., G.R. No. 173587, July 15, 2013

Facts:

Petitioner Zuellig Pharma Corporation (Zuellig) is a domestic corporation engaged in the manufacture and distribution
of pharmaceutical products. It also distributes pharmaceutical products manufactured by other companies like Syntex
Pharmaceuticals (Syntex). Respondents (36 in all), on the other hand, were the employees of Zuellig at its Syntex
Division.

In 1995, Roche Philippines, Inc. (Roche) purchased Syntex and took over from Zuellig the distribution of Syntex
products. Consequently, Zuellig closed its Syntex Division and terminated the services of respondents due to
redundancy. They were properly notified of their termination and were paid their respective separation pay in
accordance with Section 3(b), Article XIV of the March 21, 1995 Collective Bargaining Agreement (CBA) for which,
respondents individually signed Release and Quitclaim in full settlement of all claims arising from their employment
with Zuellig.

Controversy arose when respondents filed before the Arbitration Branch of the NLRC separate Complaints (which
were later consolidated) for payment of retirement gratuity and monetary equivalent of their unused sick leave on top
of the separation pay already given them. Respondents claimed that they are still entitled to retirement benefits and
that their receipt of separation pay and execution of Release and Quitclaim do not preclude pursuing such claim.

Issues:

THE COURT OF APPEALS COMMITTED GRAVE ERROR WHEN IT HELD THAT UNDER THE TERMS AND
CONDITIONS OF THE CBA AND THE RETIREMENT AND GRATUITY PLAN X X X RESPONDENTS COULD
AVAIL OF BOTH REDUNDANCY PAY AND RETIRMENT BENEFITS.

II

THE COURT OF APPEALS COMMITTED GRAVE ERROR IN FINDING THAT RESPONDENTS ARE ENTITLED TO
THE MONETARY EQUIVALENT OF UNUSED SICK LEAVE.

III

THE COURT OF APPEALS COMMITTED GRAVE ERROR IN FAILING TO HOLD THAT QUITCLAIMS BAR
RESPONDENTS FROM CLAIMING FROM PETITIONER ANY MORE THAN THEY HAVE LAWFULLY RECEIVED.

Ruling:

The Petition is impressed with merit.

I. The CBA does not allow recovery of both separation pay and retirement gratuity.

In the case of Aquino, the petitioner employees were retrenched after their employer Otis Elevator Company (Otis)
adopted cost-cutting measures and streamlined its operations. They were thus given separation pay double the
amount required by the Labor Code. Subsequently, however, the employees filed a claim for retirement benefits,
alleging entitlement thereto by virtue of the Retirement Plan. Otis denied the claim by asserting that separation pay
and retirement benefits are mutually exclusive of each other; hence, acceptance of one bars recovery of the other.
When the case reached its final review, this Court held that in the absence of specific prohibition in the retirement
plan or the CBA, retirement benefits and separation pay are not mutually exclusive of each other and the employees
whose services were terminated without cause are entitled to both separation pay and retirement gratuity.

In the present case, the CBA contains specific provisions which effectively bar the availment of retirement benefits
once the employees have chosen separation pay or vice versa.

Section 2 of Article XIV of the CBA explicitly states that any payment of retirement gratuity shall be chargeable
against separation pay. Clearly, respondents cannot have both retirement gratuity and separation pay, as selecting
one will preclude recovery of the other. To illustrate the mechanics of how Section 2 of Article XIV bars double
recovery, if the employees choose to retire, whatever amount they will receive as retirement gratuity will be charged
against the separation pay they would have received had their separation from employment been for a cause which
would entitle them to severance pay. These causes are enumerated in Section 3, Article XIV of the CBA (i.e.,
retrenchment, closure of business, merger, redundancy, or installation of labor-saving device). However, if the cause
of the termination of their employment was any of the causes enumerated in said Section 3, they could no longer
claim retirement gratuity as the fund from which the same would be taken had already been used in paying their
separation pay. Put differently, employees who were separated from the company cannot have both retirement
gratuity and separation pay as there is only one fund from which said benefits would be taken. Inarguably, Section 2
of Article XIV effectively disallows recovery of both separation pay and retirement gratuity. Consequently,
respondents are entitled only to one. Since they have already chosen and accepted redundancy pay and have
executed the corresponding Release and Quitclaim, they are now barred from claiming retirement gratuity.

In Suarez, Jr. v. National Steel Corporation, the same issue cropped up – whether the retrenched employees are
entitled to retirement gratuity even after they have received their separation pay in accordance with the retrenchment
program of the company. In ruling in the negative, this Court observed that Sections 1 and 3 of Article XIV on
Retirement Benefits of the CBA separately provide for retirement benefits and severance pay for retrenched
employees.

Section 1 thereof states, among others, that those retiring with at least 10 years of service credits are entitled to a
retirement pay equivalent to one and one-half months of basic pay for every year of service, while Section 3 extends
two months base pay for every year of service for laid-off employees pursuant to retrenchment program. This Court
elaborated thus:

A perusal of Article XIV of the parties’ 1994-1996 CBA readily shows that retirement benefits shall be granted only to
those employees who, after rendering at least ten (10) years of continuous services, would retire upon reaching the
mandatory retirement age, or would avail of optional voluntary retirement. Nowhere can it be deduced from the CBA
that those employees whose employment was terminated through one of the authorized causes are entitled to
retirement benefits. In fact, Section 3 of the afore-quoted Article XIV specifically provides that retrenched employees
shall be given two (2) months pay for every year of service. Section 3 shows the intention of the parties to exclude
retrenched employees, like herein petitioners, from receiving retirement benefits under the existing retirement plan as
set forth in Section 1.

Similarly, in this case, there is also nothing in the CBA which would indicate that those employees whose services
were terminated by reason of redundancy are entitled to retirement gratuity. As in Suarez, Sections 1 and 3 of Article
XIV of the CBA of the parties herein separately provide for the amount of benefits to be received by retired
employees on the one hand and those who were terminated due to retrenchment, closure of business, merger,
redundancy, or installation of labor-saving device on the other. In short, Sections 1 and 3 clearly spell out the
difference in the treatment of employees who retired as provided in Section 1 and those who were constrained to
leave the company due to any of the causes enumerated in Section 3. Such difference in the treatment, as well as in
the corresponding pay or gratuity, indicates the parties’ intention to exclude retired employees from receiving
separation pay and vice versa. A contrary construction would distort the clear intent of the parties and render useless
the classification specifically spelled out in the CBA.

II. Respondents are not entitled to the monetary equivalent of their unused sick leave credits.

The CA’s ruling in effect put something into the CBA that is not written in it, contrary to the old and familiar Latin
maxim of expressio unius est exclusio alterius. The express mention of one person, thing, act, or consequence
excludes all others. Put differently, where the terms are expressly limited to certain matters, it may not, by
interpretation or construction, be extended to other matters. In this case, Article VIII of the CBA covers only (1) an
employee who is 60 years old and due for compulsory retirement; (2) an employee who retires prior to attaining the
compulsory retirement age but has served at least 25 years; and, (3) an employee who retires before attaining
compulsory retirement age due to illness or disability. Necessarily, the enumeration cannot be extended to include
those who will be leaving the company due to redundancy, death, merger, installation of labor cost-saving device,
retrenchment, or closure of business as mistakenly ruled by the CA.

As the law between the parties, the CBA must be strictly complied with.

III. The Release and Quitclaim executed by each of the respondents remains valid.

It is true that quitclaims executed by employees are often frowned upon as contrary to public policy. But that is not to
say that all waivers and quitclaims are invalid as against public policy. Quitclaims will be upheld as valid if the
following requisites are present: "(1) the employee executes a deed of quitclaim voluntarily; (2) there is no fraud or
deceit on the part of any of the parties; (3) the consideration of the quitclaim is credible and reasonable; and, (4) the
contract is not contrary to law, public order, public policy, morals or good customs or prejudicial to a third person with
a right recognized by law."

In this case, there is no showing that Zuellig coerced or forced respondents to sign the Release and Quitclaim. In
fact, there is no allegation that Zuellig employed fraud or deceit in making respondents sign the Release and
Quitclaim. On the other hand, respondents declared that they had received the separation pay in full settlement of all
claims arising from their employment with Zuellig. For which reason, they have remised, released and discharged
Zuellig.

Notably, the Release and Quitclaim represents a reasonable and fair settlement of respondents’ claims. Under Article
283 of the Labor Code, the employers are required to pay employees separated from employment by reason of
redundancy at least one (1) month pay or at least one (1) month pay for every year of service, whichever is higher.
Here, respondents received 100% of their one (1) month basic pay for every year of service, plus a premium ranging
from 20% to 85% of such basic pay for every year of service (depending on the number of years in service), as
separation pay. In Goodrich Manufacturing Corporation, v. Ativo, this Court declared that –

It is only where there is clear proof that the waiver was wangled from an unsuspecting or gullible person, or the terms
of settlement are unconscionable on its face, that the law will step in to annul the questionable transaction. But where
it is shown that the person making the waiver did so voluntarily, with full understanding of what he was doing, and the
consideration for the quitclaim is credible and reasonable, the transaction must be recognized as a valid and binding
undertaking.
79. Zuellig Freight & Cargo System vs. NLRC, G.R. No. 157900, July 22, 2013

The mere change in the corporate name is not considered under the law as the creation of a new corporation; hence,
the renamed corporation remains liable for the illegal dismissal of its employee separated under that guise.

Facts:

San Miguel was an employee of Zeta Brokerage Corporation from 1985 until dismissed on 1994. On 1994, Zeta
Brokerage made amendments of the articles of incorporation of to change its corporate name, broadening the
primary functions, and increasing the capital stock. Zeta Brokerage changed its name to Zuellig Freight & Cargo
System. With the change of its corporate name, Zuellig contended that Zeta Brokerage now ceased to exist, and that
there is now cessation or closure of establishment, an authorized cause for dismissal of employees, one of them was
San Miguel.

San Miguel brought a complaint for unfair labor practice, illegal dismissal, non-payment of salaries and moral
damages against petitioner, formerly known as Zeta Brokerage Corporation (Zeta).

On the other hand, petitioner countered that San Miguel's termination from Zeta had been for a cause authorized by
the Labor Code (Article 283 - Closure of establishment and reduction of personnel)

The Labor Arbiter, NLRC, and Court of Appeals all ruled in favor of San Miguel, ruling that there was merely a change
of business name and primary purpose and upgrading of stocks of the corporation. Zuellig and Zeta are therefore
legally the same person and entity.

Issue:

Whether or not a change in corporate name leads to closure of establishment, and consequently, to valid dismissal of
employees.

Ruling:

It is worthy to point out that the Labor Arbiter, the NLRC, and the CA were united in concluding that the cessation of
business by Zeta was not a bona fide closure to be regarded as a valid ground for the termination of employment of
San Miguel within the ambit of Article 283 of the Labor Code. The provision pertinently reads:

Article 283.Closure of establishment and reduction of personnel. — The employer may also terminate the
employment of any employee due to the installation of labor-saving devices, redundancy, retrenchment to prevent
losses or the closing or cessation of operation of the establishment or undertaking unless the closing is for the
purpose of circumventing the provisions of this Title, by serving a written notice on the workers and the Department of
Labor and Employment at least one (1) month before the intended date thereof.

Verily, the amendments of the articles of incorporation of Zeta to change the corporate name to Zuellig Freight and
Cargo Systems, Inc. did not produce the dissolution of the former as a corporation. For sure, the Corporation Code
defined and delineated the different modes of dissolving a corporation, and amendment of the articles of
incorporation was not one of such modes. The effect of the change of name was not a change of the corporate being.
"The changing of the name of a corporation is no more the creation of a corporation than the changing of the name of
a natural person is begetting of a natural person. The act, in both cases, would seem to be what the language which
we use to designate it imports — a change of name, and not a change of being."

A change in the corporate name does not make a new corporation, whether effected by a special act or under a
general law. It has no effect on the identity of the corporation, or on its property, rights, or liabilities. The corporation,
upon such change in its name, is in no sense a new corporation, nor the successor of the original corporation. It is the
same corporation with a different name, and its character is in no respect changed.

In short, Zeta and petitioner remained one and the same corporation. The change of name did not give petitioner the
license to terminate employees of Zeta like San Miguel without just or authorized cause. The situation was not similar
to that of an enterprise buying the business of another company where the purchasing company had no obligation to
rehire terminated employees of the latter. 18 Petitioner, despite its new name, was the mere continuation of Zeta's
corporate being, and still held the obligation to honor all of Zeta's obligations, one of which was to respect San
Miguel's security of tenure. The dismissal of San Miguel from employment on the pretext that petitioner, being a
different corporation, had no obligation to accept him as its employee, was illegal and ineffectual.
80. Abbott Laboratrories Phils et al., Vs. Alcaraz, G.R. No. 192571, July 23, 2013 En banc

Facts:

Alcaraz was hired by Abbott Laboratories as Medical and Regulatory Affairs Manager (Regulatory Affairs Manager)
on a probationary basis. During Alcaraz's pre-employment orientation, petitioner Allan G. Almazar (Almazar),
Hospira's Country Transition Manager, briefed her on her duties and responsibilities as Regulatory Affairs Manager,
stating that: (a) she will handle the staff of Hospira ALSU and will directly report to Almazar on matters regarding
Hopira's local operations, operational budget, and performance evaluation of the Hospira ALSU Staff who are on
probationary status; (b) she must implement Abbott's Code of Good Corporate Conduct (Code of Conduct), office
policies on human resources and finance, and ensure that Abbott will hire people who are fit in the organizational
discipline; (c) petitioner Kelly Walsh (Walsh), Manager of the Literature Drug Surveillance Drug Safety of Hospira, will
be her immediate supervisor; (d) she should always coordinate with Abbott's human resource officers in the
management and discipline of the staff; (e) Hospira ALSU will spin off from Abbott in early 2006 and will be officially
incorporated and known as Hospira, Philippines. In the interim, Hospira ALSU operations will still be under Abbott's
management, excluding the technical aspects of the operations which is under the control and supervision of Walsh;
and (f) the processing of information and/or raw material data subject of Hospira ALSU operations will be strictly
confined and controlled under the computer system and network being maintained and operated from the United
States. For this purpose, all those involved in Hospira ALSU are required to use two identification cards: one, to
identify them as Abbott's employees and another, to identify them as Hospira employees.

On May 23, 2005, Walsh, Almazar, and Bernardo personally handed to Alcaraz a letter stating that her services had
been terminated effective May 19, 2005. The letter detailed the reasons for Alcaraz's termination — particularly, that
Alcaraz: (a) did not manage her time effectively; (b) failed to gain the trust of her staff and to build an effective rapport
with them; (c) failed to train her staff effectively; and (d) was not able to obtain the knowledge and ability to make
sound judgments on case processing and article review which were necessary for the proper performance of her
duties. On May 27, 2005, Alcaraz received another copy of the said termination letter via registered mail.

Alcaraz felt that she was unjustly terminated from her employment and thus, filed a complaint for illegal dismissal and
damages against Abbott and its officers, namely, Misa, Bernardo, Almazar, Walsh, Terrible, and Feist. She claimed
that she should have already been considered as a regular and not a probationary employee given Abbott's failure to
inform her of the reasonable standards for her regularization upon her engagement as required under Article 295 of
the Labor Code. In this relation, she contended that while her employment contract stated that she was to be
engaged on a probationary status, the same did not indicate the standards on which her regularization would be
based.

On the contrary, petitioners maintained that Alcaraz was validly terminated from her probationary employment given
her failure to satisfy the prescribed standards for her regularization which were made known to her at the time of her
engagement.

Issues:

Whether or not Alcaraz was illegally dismissed.

Whether or not damages may be awarded.

Ruling:

A probationary employee, like a regular employee, enjoys security of tenure. However, in cases of probationary
employment, aside from just or authorized causes of termination, an additional ground is provided under Article 295
of the Labor Code, i.e., the probationary employee may also be terminated for failure to qualify as a regular employee
in accordance with the reasonable standards made known by the employer to the employee at the time of the
engagement. Thus, the services of an employee who has been engaged on probationary basis may be terminated for
any of the following: (a) a just or (b) an authorized cause; and (c) when he fails to qualify as a regular employee in
accordance with reasonable standards prescribed by the employer.

Corollary thereto, Section 6 (d), Rule I, Book VI of the Implementing Rules of the Labor Code provides that if the
employer fails to inform the probationary employee of the reasonable standards upon which the regularization would
be based on at the time of the engagement, then the said employee shall be deemed a regular employee, viz.:

(d)In all cases of probationary employment, the employer shall make known to the employee the standards under
which he will qualify as a regular employee at the time of his engagement. Where no standards are made known to
the employee at that time, he shall be deemed a regular employee.

In other words, the employer is made to comply with two (2) requirements when dealing with a probationary
employee: first, the employer must communicate the regularization standards to the probationary employee; and
second, the employer must make such communication at the time of the probationary employee's engagement. If the
employer fails to comply with either, the employee is deemed as a regular and not a probationary employee.

Keeping with these rules, an employer is deemed to have made known the standards that would qualify a
probationary employee to be a regular employee when it has exerted reasonable efforts to apprise the employee of
what he is expected to do or accomplish during the trial period of probation. This goes without saying that the
employee is sufficiently made aware of his probationary status as well as the length of time of the probation.
The exception to the foregoing is when the job is self-descriptive in nature, for instance, in the case of maids, cooks,
drivers, or messengers. Also, in Aberdeen Court, Inc. v. Agustin, it has been held that the rule on notifying a
probationary employee of the standards of regularization should not be used to exculpate an employee who acts in a
manner contrary to basic knowledge and common sense in regard to which there is no need to spell out a policy or
standard to be met. In the same light, an employee's failure to perform the duties and responsibilities which have
been clearly made known to him constitutes a justifiable basis for a probationary employee's non-regularization.

In this case, petitioners contend that Alcaraz was terminated because she failed to qualify as a regular employee
according to Abbott's standards which were made known to her at the time of her engagement. Contrarily, Alcaraz
claims that Abbott never apprised her of these standards and thus, maintains that she is a regular and not a mere
probationary employee.

The Court finds petitioners' assertions to be well-taken.

A punctilious examination of the records reveals that Abbott had indeed complied with the above-stated
requirements. This conclusion is largely impelled by the fact that Abbott clearly conveyed to Alcaraz her duties and
responsibilities as Regulatory Affairs Manager prior to, during the time of her engagement, and the incipient stages of
her employment.

Verily, basic knowledge and common sense dictate that the adequate performance of one's duties is, by and of itself,
an inherent and implied standard for a probationary employee to be regularized; such is a regularization standard
which need not be literally spelled out or mapped into technical indicators in every case. In this regard, it must be
observed that the assessment of adequate duty performance is in the nature of a management prerogative which
when reasonably exercised — as Abbott did in this case — should be respected. This is especially true of a
managerial employee like Alcaraz who was tasked with the vital responsibility of handling the personnel and
important matters of her department.

In fine, the Court rules that Alcaraz's status as a probationary employee and her consequent dismissal must stand.

A different procedure is applied when terminating a probationary employee; the usual two-notice rule does not
govern. 65 Section 2, Rule I, Book VI of the Implementing Rules of the Labor Code states that "if the termination is
brought about by the . . . failure of an employee to meet the standards of the employer in case of probationary
employment, it shall be sufficient that a written notice is served the employee, within a reasonable time from the
effective date of termination."

As the records show, Alcaraz's dismissal was effected through a letter dated May 19, 2005 which she received on
May 23, 2005 and again on May 27, 2005. Stated therein were the reasons for her termination, i.e., that after proper
evaluation, Abbott determined that she failed to meet the reasonable standards for her regularization considering her
lack of time and people management and decision-making skills, which are necessary in the performance of her
functions as Regulatory Affairs Manager. Undeniably, this written notice sufficiently meets the criteria set forth above,
thereby legitimizing the cause and manner of Alcaraz's dismissal as a probationary employee under the parameters
set by the Labor Code.

Nonetheless, despite the existence of a sufficient ground to terminate Alcaraz's employment and Abbott's compliance
with the Labor Code termination procedure, it is readily apparent that Abbott breached its contractual obligation to
Alcaraz when it failed to abide by its own procedure in evaluating the performance of a probationary employee.

Veritably, a company policy partakes of the nature of an implied contract between the employer and employee.

Once an employer establishes an express personnel policy and the employee continues to work while the policy
remains in effect, the policy is deemed an implied contract for so long as it remains in effect. If the employer
unilaterally changes the policy, the terms of the implied contract are also thereby changed.

Records show that Abbott's PPSE procedure mandates, inter alia, that the job performance of a probationary
employee should be formally reviewed and discussed with the employee at least twice: first on the third month and
second on the fifth month from the date of employment. Abbott is also required to come up with a Performance
Improvement Plan during the third month review to bridge the gap between the employee's performance and the
standards set, if any. In addition, a signed copy of the PPSE form should be submitted to Abbott's HRD as the same
would serve as basis for recommending the confirmation or termination of the probationary employment.

In this case, it is apparent that Abbott failed to follow the above-stated procedure in evaluating Alcaraz. For one, there
lies a hiatus of evidence that a signed copy of Alcaraz's PPSE form was submitted to the HRD. It was not even
shown that a PPSE form was completed to formally assess her performance. Neither was the performance evaluation
discussed with her during the third and fifth months of her employment. Nor did Abbott come up with the necessary
Performance Improvement Plan to properly gauge Alcaraz's performance with the set company standards.

While it is Abbott's management prerogative to promulgate its own company rules and even subsequently amend
them, this right equally demands that when it does create its own policies and thereafter notify its employee of the
same, it accords upon itself the obligation to faithfully implement them. Indeed, a contrary interpretation would entail a
disharmonious relationship in the work place for the laborer should never be mired by the uncertainty of flimsy rules in
which the latter's labor rights and duties would, to some extent, depend.

In this light, while there lies due cause to terminate Alcaraz's probationary employment for her failure to meet the
standards required for her regularization, and while it must be further pointed out that Abbott had satisfied its statutory
duty to serve a written notice of termination, the fact that it violated its own company procedure renders the
termination of Alcaraz's employment procedurally infirm, warranting the payment of nominal damages.

It was explained that if the dismissal is based on a just cause under Article 282 of the Labor Code (now Article 296)
but the employer failed to comply with the notice requirement, the sanction to be imposed upon him should be
tempered because the dismissal process was, in effect, initiated by an act imputable to the employee; if the dismissal
is based on an authorized cause under Article 283 (now Article 297) but the employer failed to comply with the notice
requirement, the sanction should bestiffer because the dismissal process was initiated by the employer's exercise of
his management prerogative. Hence, in Jaka, where the employee was dismissed for an authorized cause of
retrenchment as contradistinguished from the employee in Agabon who was dismissed for a just cause of neglect of
duty the Court ordered the employer to pay the employee nominal damages at the higher amount of P50,000.00.

Evidently, the sanctions imposed in both Agabon and Jaka proceed from the necessity to deter employers from future
violations of the statutory due process rights of employees. In similar regard, the Court deems it proper to apply the
same principle to the case at bar for the reason that an employer's contractual breach of its own company procedure
— albeit not statutory in source — has the parallel effect of violating the laborer's rights. Suffice it to state, the
contract is the law between the parties and thus, breaches of the same impel recompense to vindicate a right that has
been violated. Consequently, while the Court is wont to uphold the dismissal of Alcaraz because a valid cause exists,
the payment of nominal damages on account of Abbott's contractual breach is warranted in accordance with Article
2221 of the Civil Code.

Anent the proper amount of damages to be awarded, the Court observes that Alcaraz's dismissal proceeded from her
failure to comply with the standards required for her regularization. As such, it is undeniable that the dismissal
process was, in effect, initiated by an act imputable to the employee, akin to dismissals due to just causes under
Article 296 of the Labor Code. Therefore, the Court deems it appropriate to fix the amount of nominal damages at the
amount of P30,000.00, consistent with its rulings in both Agabon and Jaka.
81. Manila Polo Club Employees Union vs. Manila Polo Club, G.R. No. 172846, July 24, 2013

Principle: Standards for a valid retrenchment and closure or cessation of business operations.

Facts:

Petitioner Manila Polo Club Employees Union (MPCEU), which is affiliated with the Federation of Unions of Rizal
(FUR)-TUCP, is a legitimate labor organization duly registered with DOLE, while respondent Manila Polo Club, Inc. is
a non-profit and proprietary membership organization which provides recreation and sports facilities to its proprietary
members, their dependents, and guests.

The Board of Directors of respondent unanimously resolved to completely terminate the entire operations of its Food
and Beverage (F & B) outlets, except the Last Chukker, and award its operations to a qualified restaurant operator or
caterer. Cited as reasons were as follows:

WHEREAS, the Food and Beverage (F & B) operations has resulted in yearly losses to the Club in six (6) out of the
last eight (8) years with FY 2001 suffering the largest loss at P10,647,981 and that this loss is due mainly to the
exceedingly high manpower cost and other management inefficiencies;

WHEREAS, due to the substantial losses incurred by the Club in both F&B operations and in its recurring operations,
the Board and management had instituted cost and loss-cutting measures;

WHEREAS, the Board recognized the non-viability of the operations of the Food and Beverage Department and that
its continued operations by the Club will result in substantial losses that will seriously impair the Club's financial
health and membership satisfaction;

WHEREAS, the Board recognized the urgent need to act and act decisively and eliminate factors contributing to
substantial losses in the operations of the Club, more particularly the food and beverage operations. Thus, F & B
operations are to cease wholly and totally, subject to observance and requirements of the law and other rules.

Subsequently, respondent's Board approved the implementation of the retrenchment program of employees who are
directly and indirectly involved with the operations of the F & B outlets.

Issue:

Whether or not the retrenchment of the 117 union members is legal.

Ruling:

It is apparent from the records that this case involves a closure of business undertaking, not retrenchment. The legal
requirements and consequences of these two authorized causes in the termination of employment are discernible.

We distinguished, in Alabang Country Club, Inc. v. NLRC:

Retrenchment is the reduction of personnel for the purpose of cutting down on costs of operations in terms of salaries
and wages resorted to by an employer because of losses in operation of a business occasioned by lack of work and
considerable reduction in the volume of business.

Closure of a business or undertaking due to business losses is the reversal of fortune of the employer whereby there
is a complete cessation of business operations to prevent further financial drain upon an employer who cannot pay
anymore his employees since business has already stopped.

One of the prerogatives of management is the decision to close the entire establishment or to close or abolish a
department or section thereof for economic reasons, such as to minimize expenses and reduce capitalization.

While the Labor Code provides for the payment of separation package in case of retrenchment to prevent losses, it
does not obligate the employer for the payment thereof if there is closure of business due to serious losses.

Likewise, the case of Eastridge Golf Club, Inc. v. Eastridge Golf Club, Inc., Labor-Union, Super stressed the
differences:

Retrenchment or lay-off is the termination of employment initiated by the employer, through no fault of the employees
and without prejudice to the latter, during periods of business recession, industrial depression, or seasonal
fluctuations, or during lulls occasioned by lack of orders, shortage of materials, conversion of the plant for a new
production program or the introduction of new methods or more efficient machinery, or of automation. It is an exercise
of management prerogative which the Court upholds if compliant with certain substantive and procedural
requirements, namely:

1. That retrenchment is necessary to prevent losses and it is proven, by sufficient and convincing evidence such as
the employer's financial statements audited by an independent and credible external auditor, that such losses are
substantial and not merely flimsy and actual or reasonably imminent; and that retrenchment is the only effective
measure to prevent such imminent losses;

2. That written notice is served on to the employees and the DOLE at least one (1) month prior to the intended date of
retrenchment; and
3. That the retrenched employees receive separation pay equivalent to one (1) month pay or at least one-half (1/2)
month pay for every year of service, whichever is higher.

The employer must prove compliance with all the foregoing requirements. Failure to prove the first requirement will
render the retrenchment illegal and make the employer liable for the reinstatement of its employees and payment of
full backwages. However, were the retrenchment undertaken by the employer isbona fide, the same will not be
invalidated by the latter's failure to serve prior notice on the employees and the DOLE; the employer will only be liable
in nominal damages, the reasonable rate of which the Court En Banc has set at P50,000.00 for each employee.

Closure or cessation of business is the complete or partial cessation of the operations and/or shut-down of the
establishment of the employer. It is carried out to either stave off the financial ruin or promote the business interest of
the employer.

Unlike retrenchment, closure or cessation of business, as an authorized cause of termination of employment, need
not depend for validity on evidence of actual or imminent reversal of the employer's fortune. Article 283 authorizes
termination of employment due to business closure, regardless of the underlying reasons and motivations therefor, be
it financial losses or not.

To be precise, closure or cessation of an employer's business operations, whether in whole or in part, is governed by
Article 283 of the Labor Code, as amended. It states:

Article 283.Closure of establishment and reduction of personnel. — The employer may also terminate the
employment of any employee due to the installation of labor-saving devices, redundancy, retrenchment to prevent
losses or the closing or cessation of operation of the establishment or undertaking unless the closing is for the
purpose of circumventing the provisions of this Title, by serving a written notice on the workers and the Ministry of
Labor and Employment at least one (1) month before the intended date thereof. In case of termination due to the
installation of labor-saving devices or redundancy, the worker affected thereby shall be entitled to a separation pay
equivalent to at least his one (1) month pay or to at least one (1) month pay for every year of service, whichever is
higher. In case of retrenchment to prevent losses and in cases of closures or cessation of operations of establishment
or undertaking not due to serious business losses or financial reverses, the separation pay shall be equivalent to one
(1) month pay or at least one-half (1/2) month pay for every year of service, whichever is higher. A fraction of at least
six (6) months shall be considered one (1) whole year.

In Industrial Timber Corporation v. Ababon, 27the Court explained the above-quoted provision in this wise:

A reading of the foregoing law shows that a partial or total closure or cessation of operations of establishment or
undertaking may either be due to serious business losses or financial reverses or otherwise. Under the first kind, the
employer must sufficiently and convincingly prove its allegation of substantial losses, while under the second kind, the
employer can lawfully close shop anytime as long as cessation of or withdrawal from business operations was bona
fide in character and not impelled by a motive to defeat or circumvent the tenurial rights of employees, and as long as
he pays his employees their termination pay in the amount corresponding to their length of service. Just as no law
forces anyone to go into business, no law can compel anybody to continue the same. It would be stretching the intent
and spirit of the law if a court interferes with management's prerogative to close or cease its business operations just
because the business is not suffering from any loss or because of the desire to provide the workers continued
employment.

In sum, under Article 283 of the Labor Code, three requirements are necessary for a valid cessation of business
operations: (a) service of a written notice to the employees and to the DOLE at least one month before the intended
date thereof; (b) the cessation of business must be bona fide in character; and (c) payment to the employees of
termination pay amounting to one month pay or at least one-half month pay for every year of service, whichever is
higher.

Based on the above and cases of similar import, We summarize:

1. Closure or cessation of operations of establishment or undertaking may either be partial or total.

2. Closure or cessation of operations of establishment or undertaking may or may not be due to serious business
losses or financial reverses. However, in both instances, proof must be shown that: (1) it was done in good faith to
advance the employer's interest and not for the purpose of defeating or circumventing the rights of employees under
the law or a valid agreement; and (2) a written notice on the affected employees and the DOLE is served at least one
month before the intended date of termination of employment.

3. The employer can lawfully close shop even if not due to serious business losses or financial reverses but
separation pay, which is equivalent to at least one month pay as provided for by Article 283 of the Labor Code, as
amended, must be given to all the affected employees.

4. If the closure or cessation of operations of establishment or undertaking is due to serious business losses or
financial reverses, the employer must prove such allegation in order to avoid the payment of separation pay.
Otherwise, the affected employees are entitled to separation pay.

5. The burden of proving compliance with all the above-stated falls upon the employer.
Guided by the foregoing, the Court shall refuse to dwell on the issue of whether respondent was in sound financial
condition when it resolved to stop the operations of its F & B Department. As stated, an employer can lawfully close
shop anytime even if not due to serious business losses or financial reverses.

The closure of the F & B Department was due to legitimate business considerations, a resolution which the Court has
no business interfering with. We have already resolved that the characterization of the employee's service as no
longer necessary or sustainable, and therefore, properly terminable, is an exercise of business judgment on the part
of the employer; the determination of the continuing necessity of a particular officer or position in a business
corporation is a management prerogative, and the courts will not interfere with the exercise of such so long as no
abuse of discretion or arbitrary or malicious action on the part of the employer is shown.

As recognized by both the VA and the CA, evident proofs of respondent's good faith to arrest the losses which the F
& B Department had been incurring since 1994 are: engagement of an independent consulting firm to conduct
manpower audit/organizational development; institution of cost-saving programs, termination of the services of
probationary employees, substantial reduction of a number of agency staff and personnel, and the retrenchment of
eight (8) managers. After the effective date of the termination of employment relation, respondent even went on to aid
the displaced employees in finding gainful employment by soliciting the assistance of respondent's members, Makati
Skyline, Human Resource Managers of some companies, and the Association of Human Resource Managers.
These were not refuted by petitioner. Only that, it perceives them as inadequate and insists that the operational
losses are very well covered by the other income of respondent and that less drastic measures could have been
resorted to, like increasing the membership dues and the prices of food and beverage. Yet the wisdom or soundness
of the Management decision is not subject to discretionary review of the Court for, even the VA admitted, it enjoys a
pre-eminent role and is presumed to possess all relevant and necessary information to guide its business decisions
and actions.

Further, unlike in the case of Eastridge Golf Club, Inc., there is nothing on record to indicate that the closure of
respondent's F & B Department was made in bad faith. It was not motivated by any specific and clearly determinable
union activity of the employees; rather, it was truly dictated by economic necessity. Despite petitioner's allegations,
no convincing and credible proofs were presented to establish the claim that such closure qualifies as an act of union-
busting and ULP. No evidence was shown that the closure is stirred not by a desire to avoid further losses but to
discourage the workers from organizing themselves into a union for more effective negotiations with the
management. Allegations are not proofs and it is incumbent upon petitioner to substantiate the same. On the
contrary, respondent continued to negotiate with petitioner even after April 30, 2002. In fact, a Memorandum of
Agreement was executed before the NCMB between petitioner and respondent on June 10, 2002 whereby the parties
agreed, among others, to maintain the existing provisions of the CBA, except those pertaining to wage increases and
signing bonus.

Finally, even if the members of petitioner are not considered as illegally dismissed, they are entitled to separation pay
pursuant to Article 283 of the Labor Code, as amended. Per respondent's information, however, the separation
packages of all 117 union members were already paid during the pendency of the case. Petitioner did not oppose
this representation; hence, We shall treat the fact of receipt of separation pay as having been voluntarily entered into,
with a full understanding of its import, and the amount received as credible and reasonable settlement that should be
respected by the Court as the law between the parties are valid and binding between them.
82. Canedo vs. Kampilan Security & Detective Agency Inc. et al., G.R. No. 179326, July 31, 2013

Facts:

Respondent agency hired petitioner as security guard and assigned him at the Naga Power Barge 102 of the National
Power Corporation (NPC) at Lutopan, Toledo City.

For not wearing proper uniform while on duty as per report of Allan Alfafara (Alfafara) of the NPC, petitioner was
suspended for a month.

In a letter, NPC informed respondent agency that it was no longer interested in petitioner’s services and thus
requested for his replacement.

Subsequently, etitioner requested respondent Arquiza to issue a certification in connection with his intended
retirement effective that month. Thus, respondent Arquiza issued the Certification dated June 25, 2003 (June 25,
2003 Certification):

CERTIFICATION

TO WHOM IT MAY CONCERN:

This is to certify that Mr. Luciano ParagosoCañedo whose address is at Lower Bunga, Toledo City was employed by
this agency from November 20, 1996 up to May 7, 2003 as Security Guard assigned at NPC, Sigpit Substation. He
was terminated from his employment by this agency on May 7, 2003 as per client’s request.

Done this 25th day of June 2003 at Cebu City, Philippines.

(Signed)RAMONCITO L. ARQUIZAGeneral ManagerKSDAI

Five days later, petitioner filed before the Labor Arbiter a Complaint for illegal dismissal, illegal suspension and non-
payment of monetary benefits against respondents.

Proceedings before the Labor Arbiter

The Labor Arbiter held that petitioner was illegally dismissed from the service. He also found petitioner’s prior
suspension illegal and granted him all his monetary claims except for underpayment of wages

Proceedings before the National Labor Relations Commission

The NLRC opined that petitioner’s intention to retire as shown by his June 17, 2003 letter negated his claim of
termination. Nevertheless, it maintained that petitioner was suspended without being notified of his infraction. Thus,
he should be paid his salary during the period of his illegal suspension.

Ruling of the Court of Appeals

The CA denied the Petition after it found no grave abuse of discretion on the part of the NLRC.

In view of the above, the CA concluded that petitioner was merely placed on temporary "off-detail" which is not
equivalent to dismissal. However, like the NLRC, the CA found that petitioner was deprived of due process when he
was suspended and thus affirmed his entitlement to his salary during the period of suspension. It also affirmed the
awards for holiday pay and service incentive leave pay as well as the deduction therefrom of P10,000.00
representing petitioner’s cash advance.

Issue:

Whether or not petitioner was dismissed from service.

Ruling:

Petitioner relies on the word "terminated" as used in the June 25, 2003 Certification issued him by respondent
Arquiza and argues that the same is a clear indication that he was dismissed from service. SC held that petitioner
cannot simply rely on this piece of document since the fact of dismissal must be evidenced by positive and overt acts
of an employer indicating an intention to dismiss.Here, aside from this single document, petitioner proffered no other
evidence showing that he was dismissed from employment. While it is true that he was not allowed to report for work
after the period of his suspension expired, the same was due to NPC’s request for his replacement as NPC was no
longer interested in his services. And as correctly argued by respondents, petitioner from that point onward is not
considered dismissed but merely on a floating status. "Such a ‘floating status’ is lawful and not unusual for security
guards employed in security agencies as their assignments primarily depend on the contracts entered into by the
agency with third parties."

A floating status can ripen into constructive dismissal only when it goes beyond the six-month maximum period
allowed by law. In this case, petitioner filed the Complaint for illegal dismissal even before the lapse of the six-month
period. Hence, his claim of illegal dismissal lacks basis. Moreover and as aptly observed by the NLRC, it was in fact
petitioner who intended to terminate his relationship with respondents through his planned retirement. This is further
bolstered by his prayer in his Complaint where he sought for separation pay and not for reinstatement.
83. Ang vs. San Joaquin Jr et al., G.R. No. 185549, Aug. 7, 2013

PRINCIPLE: The employer’s act of tearing to pieces the employee’s time card may be considered an outright – not
only symbolic – termination of the parties’ employment relationship.

Facts:

Petitioner Ang is the proprietor of Virose Furniture and Glass Supply. Respondents San Joaquin and Fernandez were
regular employees of Virose: San Joaquin as helper, while Fernandez as driver. Respondents have been
continuously in Ang’s employ without any derogatory record.

Through the years, San Joaquin became a pahinante or delivery helper, and later on an all-around worker of Virose.

On 1999, respondents attended the court hearing relative to the 41 criminal cases filed by former Virose employee
Abrera against Ang for the latter’s non-remittance of Social Security System (SSS) contributions. During that hearing,
respondents testified against Ang. Previously, respondents joined Abrera in questioning Ang’s procedure in remitting
their SSS contributions. After the said hearing Ang began to treat respondents with hostility and antagonism.

On August 28, 1999, Ang’s wife, Rosa, instructed a Virose salesclerk to find helpers who would transfer monobloc
chairs from the Virose store to her restaurant, Leng-Leng’s Foodshop, located just beside the store. The salesclerk
instructed San Joaquin to help, but the latter refused, saying that he was not an employee of the restaurant but a
glass installer of Virose. A heated argument ensued between San Joaquin on the one hand and Rosa, her son
Jonathan, and the salesclerk on the other. San Joaquin left the store, shouting invectives.

On August 30, 1999, San Joaquin returned to the store, only to find out that Ang had torn his DTR to pieces that day
while the DTR of Fernandez was torn to pieces by Ang immediately after the hearing in which the respondents
testified. On the same day, Fernandez reported for work and received a memorandum of even date issued by Ang
informing him that he was placed on a one-week suspension for insubordination. The memorandum did not specify
the act of insubordination.

Respondents then filed a case for constructive dismissal in the Labor Arbiter and the latter decided against them. The
former then appealed to the NLRC but it just affirmed the decision of the Labor Arbiter. Respondent then filed an
original Petition for Certiorari to the CA which reversed the rulings of the LA and the NLRC.

Issue:

Whether or not employer’s act of tearing to pieces the employee’s time card may be considered constructive
dismissal or termination of the parties’ employment relationship.

Ruling:

There is constructive dismissal in this case. "Constructive dismissal exists where there is cessation of work because
continued employment is rendered impossible, unreasonable or unlikely, as an offer involving a demotion in rank and
a diminution in pay." It is a "dismissal in disguise or an act amounting to dismissal but made to appear as if it were
not." Constructive dismissal may likewise exist if an "act of clear discrimination, insensibility, or disdain by an
employer becomes so unbearable on the part of the employee that it could foreclose any choice by him except to
forego his continued employment." "Constructive dismissal exists when the employee involuntarily resigns due to the
harsh, hostile, and unfavorable conditions set by the employer." "The test of constructive dismissal is whether a
reasonable person in the employee’s position would have felt compelled to give up his position under the
circumstances."

The CA is correct in its pronouncement that respondents were constructively dismissed from work.1âwphi1 Moreover,
by destroying respondents’ time cards, Ang discontinued and severed his relationship with respondents. The purpose
of a time record is to show an employee’s attendance in office for work and to be paid accordingly, taking into
account the policy of "no work, no pay". A daily time record is primarily intended to prevent damage or loss to the
employer, which could result in instances where it pays an employee for no work done; it is a mandatory requirement
for inclusion in the payroll, and in the absence of an employment agreement, it constitutes evidence of employment.
Thus, when Ang tore the respondents’ time cards to pieces, he virtually removed them from Virose’s payroll and
erased all vestiges of respondents’ employment; respondents were effectively dismissed from work. The act may be
considered an outright – not only symbolic – termination of the parties’ employment relationship.
84. Sanoh Fulton Phils Inc. et al., vs. Bernardo et al., G.R. No. 187214, Aug. 14, 2013

PRINCIPLE: That losses as a valid ground for termination must be supported by sufficient and convincing evidence.

Facts:

Sanoh is a domestic corporation engaged in the manufacture of automotive parts and wire condensers for home
appliances. Its Wire Condenser Department employed 61 employees. Respondents belonged to this department.

In view of job order cancellations relating to the manufacture of wire condensers by Matsushita, Sanyo and National
Panasonic, Sanoh decided to phase out the Wire Condenser Department. On 22 December 2003, the Human
Resources Manager of Sanoh informed the 17 employees, 16 of whom belonged to the Wire Condenser Department,
of retrenchment effective 22 January 2004. All 17 employees are union members.

A grievance conference was held where the affected employees were informed of the following grounds for
retrenchment:

1) Lack of local market.

2) Competition from imported products.

3) Phasing out of Wire Condenser Department.

Two succeeding conciliation conferences were likewise held but the parties failed to reach an amicable settlement.
Thus, two (2) separate complaints for illegal dismissal.

The Labor Arbiter dismissed the complaint while the NLRC affirmed the former’s decision in toto.

Respondents appealed to the CA which decided in their favor. The petitioners the appealed to the Supreme Court.

Issues:

Whether or not there was illegal dismissal in this case.

Ruling:

There was illegal dismissal.

Retrenchment to prevent losses and closure not due to serious business losses are two separate authorized causes
for terminating the services of an employee. In J.A.T. General Services v. NLRC, the Court took the occasion to draw
the distinction between retrenchment and closure, to wit:

Closure of business, on one hand, is the reversal of fortune of the employer whereby there is a complete cessation of
business operations and/or an actual locking-up of the doors of establishment, usually due to financial losses.
Closure of business as an authorized cause for termination of employment aims to prevent further financial drain
upon an employer who cannot pay anymore his employees since business has already stopped. On the other hand,
retrenchment is reduction of personnel usually due to poor financial returns so as to cut down on costs of operations
in terms of salaries and wages to prevent bankruptcy of the company. It is sometimes also referred to as down-sizing.
Retrenchment is an authorized cause for termination of employment which the law accords an employer who is not
making good in its operations in order to cut back on expenses for salaries and wages by laying off some employees.
The purpose of retrenchment is to save a financially ailing business establishment from eventually collapsing.

The respective requirements to sustain their validity are likewise different.

For retrenchment, the three (3) basic requirements are:

(a) proof that the retrenchment is necessary to prevent losses or impending losses;

(b) service of written notices to the employees and to the Department of Labor and Employment at least one (1)
month prior to the intended date of retrenchment; and

(c) payment of separation pay equivalent to one (1) month pay, or at least one-half (1/2) month pay for every year of
service, whichever is higher.

In addition, jurisprudence has set the standards for losses which may justify retrenchment, thus:

(1) the losses incurred are substantial and not de minimis;

(2) the losses are actual or reasonably imminent;

(3) the retrenchment is reasonably necessary and is likely to be effective in preventing the expected losses; and

(4) the alleged losses, if already incurred, or the expected imminent losses sought to be forestalled, are proven by
sufficient and convincing evidence.

Upon the other hand, in termination, the law authorizes termination of employment due to business closure,
regardless of the underlying reasons and motivations therefor, be it financial losses or not. However, to put a stamp
to its validity, the closure/cessation of business must be bona fide, i.e., its purpose is to advance the interest of the
employer and not to defeat or circumvent the rights of employees under the law or a valid agreement.
In termination cases either by retrenchment or closure, the burden of proving that the termination of services is for a
valid or authorized cause rests upon the employer. Not every loss incurred or expected to be incurred by an employer
can justify retrenchment. The employer must prove, among others, that the losses are substantial and that the
retrenchment is reasonably necessary to avert such losses. And to repeat, in closures, the bona fides of the employer
must be proven.

In this case, there was no valid retrenchment. Nor was there a closure of business.

We are mindful of the principle that losses in the operation of the enterprise, lack of work, or considerable reduction
on the volume of business may justify an employer to reduce the work force. But a lull caused by lack of orders or
shortage of materials must be of such nature as would severely affect the continued business operations of the
employer to the detriment of all and sundry if not properly addressed.

Sanoh asserts that cancelled orders of wire condensers led to the phasing out of the Wire Condenser Department
which triggered retrenchment. Sanoh presented the letters of cancellation given by Matsushita and Sanyo as
evidence of cancelled orders. The evidence presented by Sanoh barely established the connection between the
cancelled orders and the projected business losses that may be incurred by Sanoh. Sanoh failed to prove that these
cancelled orders would severely impact on their production of wire condensers.

We held in Lambert Pawnbrokers and Jewelry Corporation v. Binamira, that the losses must be supported by
sufficient and convincing evidence and the normal method of discharging this is by the submission of financial
statements duly audited by independent external auditors. It was aptly observed by the appellate court that no
financial statements or documents were presented to substantiate Sanoh’s claim of loss of P7 million per month. And
a business lull caused by lack of orders which could have justified retrenchment was not shown by petitioner. As
observed once more by the Court of Appeals, petitioner failed to present proof of the extent of the reduced order and
its contribution to the sustainability of its business.

As the Wire Condenser Department is still in operation and no business losses were proven by Sanoh, the dismissal
of respondents was unlawful.
85. Daabay vs.Coca-Cola Bottlers Phils G.R. No. 199890, Aug. 19, 2013

Facts:

The employment of Daabay with Coca-Cola as Sales Logistics Checker was terminated by the company in June
2005, following receipt of information from one Cesar Sorin that Daabay was part of a conspiracy that allowed the
pilferage of company property.

The allegations of Sorin were embodied in an affidavit where The losses to the company were also confirmed by an
inventory and audit conducted by Coca-Cola’s Territory Finance Head, Silvia Ang. Such losses comprised of cases of
assorted softdrinks, empty bottles, missing shells and missing pallets valued at P20,860,913.00.

Coca-Cola then served upon Daabay a Notice to Explain with Preventive Suspension, which required him to explain
in writing his participation in the scheme that was reported to involve logistics checkers and gate guards. In
compliance therewith, Daabay submitted an Explanation wherein he denied any participation in the reported.

A formal investigation on the matter ensued. Eventually, Coca-Cola served upon Daabay a Notice of Termination that
cited pilferage, serious misconduct and loss of trust and confidence as grounds. At the time of his dismissal, Daabay
had been a regular employee of Coca-Cola for eight years, and was receiving a monthly pay of P20,861.00, exclusive
of other benefits. Daabay then filed the subject labor complaint against Coca-Cola and the President and Plant
Logistics Managers of Coca-Cola at the time of the dispute.

Labor Arbiter: Daabay was illegally dismissed because his participation in the alleged conspiracy was not proved by
substantial evidence. In lieu of reinstatement and considering the already strained relations between the parties,
ordered the payment to Daabay of backwages and separation pay or retirement benefits, as may be applicable.
([P]750,996.00.)

Dissatisfied, Coca-Cola, appealed to the NLRC. Daabay filed a separate appeal to ask for his reinstatement without
loss of seniority rights, the payment of backwages instead of separation pay or retirement benefits, and an award of
litigation expenses, moral and exemplary damages and attorney’s fees.

NLRC: reversed the finding of illegal dismissal. That there was “reasonable and well-founded basis to dismiss
[Daabay], not only for serious misconduct, but also for breach of trust or loss of confidence arising from such
company losses.” Daabay’s participation in the conspiracy was sufficiently established. Several documents such as
checkers receipts and sales invoices that made the fraudulent scheme possible were signed by Daabay. The NLRC
also found fault in Daabay for his failure to detect the pilferage, considering that the “timely recording and monitoring
as security control for the outgoing [sic] of company products are necessarily connected with the functions, duties and
responsibilities reposed in him as Sales Logistics Checker.” Notwithstanding its ruling on the legality of the dismissal,
the NLRC awarded retirement benefits in favor of Daabay.

Coca-Cola’s partial motion for reconsideration to assail the award of retirement benefits was denied by the NLRC
explaining that there was a need “to humanize the severe effects of dismissal” and “tilt the scales of justice in favor of
labor as a measure of equity and compassionate social justice.” Daabay also moved to reconsider, but his motion
remained unresolved by the NLRC. Undaunted, Coca-Cola appealed to the CA.

CA: agreed with Coca-Cola that the award of retirement benefits lacked basis considering that Daabay was
dismissed for just cause. Daabay’s motion for reconsideration was denied in a Resolution dated December 9, 2011;
hence, this petition.

Issue:

Whether or not Daabay is entitled to retirement benefits.

Ruling:

No.

Daabay was declared by the NLRC to have been lawfully dismissed by Coca-Cola on the grounds of serious
misconduct, breach of trust and loss of confidence. Our pronouncement in Philippine Airlines, Inc. v. NLRC on the
issue of whether an employee who is dismissed for just cause may still claim retirement benefits equally applies to
this case. We held:

At the risk of stating the obvious, private respondent was not separated from petitioner’s employ due to mandatory or
optional retirement but, rather, by termination of employment for a just cause. Thus, any retirement pay provided by
PAL’s “Special Retirement & Separation Program” dated February 15, 1988 or, in the absence or legal inadequacy
thereof, by Article 287 of the Labor Code does not operate nor can be made to operate for the benefit of private
respondent. Even private respondent’s assertion that, at the time of her lawful dismissal, she was already qualified for
retirement does not aid her case because the fact remains that private respondent was already terminated for cause
thereby rendering nugatory any entitlement to mandatory or optional retirement pay that she might have previously
possessed.

In ruling against the grant of the retirement benefits, we also take note of the NLRC’s lone justification for the award.
Being intended as a mere measure of equity and social justice, the NLRC’s award was then akin to a financial
assistance or separation pay that is granted to a dismissed employee notwithstanding the legality of his dismissal.
Jurisprudence on such financial assistance and separation pay then equally apply to this case.
A contrary rule would, as the petitioner correctly argues, have the effect, of rewarding rather than punishing the erring
employee for his offense. And we do not agree that the punishment is his dismissal only and that the separation pay
has nothing to do with the wrong he has committed. Of course it has. Indeed, if the employee who steals from the
company is granted separation pay even as he is validly dismissed, it is not unlikely that he will commit a similar
offense in his next employment because he thinks he can expect a like leniency if he is again found out. This kind of
misplaced compassion is not going to do labor in general any good as it will encourage the infiltration of its ranks by
those who do not deserve the protection and concern of the Constitution.

Clearly, considering that Daabay was dismissed on the grounds of serious misconduct, breach of trust and loss of
confidence, the award based on equity was unwarranted.

Although retirement benefits, where not mandated by law, may still be granted by agreement of the employees and
their employer or as a voluntary act of the employer, there is no proof that any of these incidents attends the instant
case.
86. MZR Industries et al., vs. Colambot, G.R. No. 179001, Aug. 28, 2013

Facts:

Respondent Colambot was a messenger of petitioner MZR Industries whose work responsibilities include field,
messengerial and other liaison work. Colambot’s work performance started to deteriorate after 2 years in the
company. Petitioner served memoranda to Colambot for habitual tardiness, negligence, and violations of office
policies as well as written warnings for insubordination and for negligence caused by careless handling of confidential
office documents. Despite the warnings issued, Colambot failed to mend his ways and thereafter in a Memorandum
dated October 25, 2004 issued by petitioner Lea Timbal, Colambot was given a notice of suspension.

Again, in a Memorandum12 dated November 25, 2004, Colambot was suspended from November 26, 2004 until
December 6, 2004 for insubordination. Petitioners claimed they waited for Colambot to report back for work on
December 7, 2004, but they never heard from him anymore. Later, petitioners were surprised to find out that
Colambot had filed a complaint for illegal suspension, underpayment of salaries, overtime pay, holiday pay, rest day,
service incentive leave and 13th month pay. On December 16, 2004, the complaint was amended to illegal dismissal,
illegal suspension, underpayment of salaries, holiday pay, service incentive pay, 13th month pay and separation pay.

Colambot claimed that he was made to choose between resigning from the company or the company will be the one
to terminate his services. He said he refused to resign. Colambot alleged that Quiroz made him sign a memorandum
for his suspension, from November 26 to December 6, 2004. After affixing his signature, Quiroz told him that effective
December 7, 2004, he is already deemed terminated. Later, on December 2, 2004, respondent went back to the
company to look for Timbal to get his salary. He claimed that Timbal asked him to turn over his company I.D.

Petitioners maintained that they never terminated Colambot's employment and argues that the failure to report to
work constitutes abandonment of work.

Labor Arbiter: ruled that MZR Industries was guilty of illegal dismissal and ordered to reinstate complainant to his
former position with full backwages from date of dismissal until actual reinstatement and moral and exemplary
damages.

There was no abandonment as there was no deliberate intent on the part of Colambot to sever the employer-
employee relationship. The Labor Arbiter likewise noted that Colambot should have been notified to return back to
work, which petitioner failed to do.

NLRC: Judgment of the Labor Arbiter was set aside and the Complaint is dismissed for lack of merit.

The NLRC further believed that Colambot abandoned his work due to his refusal to report for work after his
suspension. The failure of MZR to notify Colambot to return back to work is not tantamount to actual dismissal.

CA: reversed the assailed Decision of the NLRC and ordered the decision of the LA to be reinstated with modification
that in lieu of reinstatement, petitioners were ordered to pay respondent separation pay equivalent to one (1) month
pay for every year of service in addition to full backwages.

CA based its decision on the grounds that: (1) MZR failed to prove abandonment on the part of Colambot, and (2)
MZR failed to serve Colambot with the required written notices of dismissal.2007.

Issues:

1.) W/N there was illegal dismissal.

2.) W/N there was abandonment of work.

3.) W/N Respondent is entitled to backwages.

Ruling:

1.) While the rule is that in illegal dismissal cases, the employer bears the burden of proving that the termination was
for a valid or authorized cause, there must be facts and evidence to establish a prima facie case that the employee
was dismissed from employment. Before the employer must bear the burden of proving that the dismissal was legal,
the employee must first establish by substantial evidence the fact of his dismissal from service.

Colambot's unsubstantiated allegation of having been verbally terminated from his work, there was no evidence
presented to show that he was indeed dismissed from work or was prevented from returning to his work. In the
absence of any showing of an overt or positive act proving that petitioners had dismissed respondent, the latter's
claim of illegal dismissal cannot be sustained.

Neither could the petitioners be blamed for failing to order respondent to return back to work.1âwphi1 Records show
that Colambot immediately filed the complaint for illegal dismissal on December 16, 2004,29 or just a few days when
he was supposed to report back to work on December 7, 2004. For petitioners to order respondent to report back to
work, after the latter had already filed a case for illegal dismissal, would be unsound.

2.) Mere absence or failure to report for work, even after notice to return, is not tantamount to abandonment. The
burden of proof to show that there was unjustified refusal to go back to work rests on the employer. Abandonment is
a matter of intention and cannot lightly be presumed from certain equivocal acts. To constitute abandonment, there
must be clear proof of deliberate and unjustified intent to sever the employer-employee relationship. Clearly, the
operative act is still the employee’s ultimate act of putting an end to his employment.

A complaint for illegal dismissal is inconsistent with abandonment of employment. An employee who takes steps to
protest his dismissal cannot logically be said to have abandoned his work. The filing of such complaint is proof
enough of his desire to return to work, thus negating any suggestion of abandonment

3.) These circumstances, taken together, the lack of evidence of dismissal and the lack of intent on the part of the
respondent to abandon his work, the remedy is reinstatement but without backwages. However, considering that
reinstatement is no longer applicable due to the strained relationship between the parties and that Colambot already
found another employment, each party must bear his or her own loss, thus, placing them on equal footing.
87. Integrated Microelectronics, Inc. v. Pionilla, G.R. No. 200222, August 28, 2013

Facts:

On November 14, 1996, Pionilla was hired by IMI as its production worker. On May 5, 2005, Pionilla received a notice
from IMI requiring him to explain the incident which occurred the day before where he was seen escorting a lady to
board the company shuttle bus at the Alabang Terminal. It was reported by the bus marshall that the lady was
wearing a company identification card (ID) — which serves as a free pass for shuttle bus passengers — even if she
was just a job applicant at IMI. In this regard, Pionilla admitted that he lent his ID to the lady who turned out to be his
relative. He further intimated that he risked lending her his ID to save on their transportation expenses. Nevertheless,
he apologized for his actions.

IMI found Pionilla guilty of violating Article 6.12 of the Company Rules and Regulations (CRR) which prohibits the
lending of one's ID since the same is considered a breach of its security rules and carries the penalty of dismissal.
Subsequently, or on August 17, 2005, Pionilla received a letter dated August 16, 2005 informing him of his dismissal
from service. Three days after, he filed a complaint for illegal dismissal with damages against IMI.

LA, NLRC and CA ruled that Pionilla was illegally dismissed and ordered IMI to reinstate Pionilla to his former
position and to pay him backwages.

Issue:

WON Pionilla was illegally dismissed and if so, is he entitled to reinstatement and full backwages.

Ruling:

Yes. The Court ruled that the imposition of the penalty of dismissal was too harsh and incommensurate to the
infraction he committed, this especially considering his admission of the incident and apologized for it, and his nine
years of unblemished service.

As a general rule, an illegally dismissed employee is entitled to reinstatement (or separation pay, if reinstatement is
not viable) and payment of full backwages. In certain cases, however, the Court has carved out an exception to the
foregoing rule and thereby ordered the reinstatement of the employee without backwages on account of the following:
(a) the fact that dismissal of the employee would be too harsh of a penalty; and (b) that the employer was in good
faith in terminating the employee.

In this case, the Court observes that: (a) the penalty of dismissal was too harsh of a penalty to be imposed against
Pionilla for his infractions; and (b) IMI was in good faith when it dismissed Pionilla as his dereliction of its policy on ID
usage was honestly perceived to be a threat to the company's security. In this respect, since these concurring
circumstances trigger the application of the exception to the rule on backwages as enunciated in the above-cited
cases, the Court finds it proper to accord the same disposition and consequently directs the deletion of the award of
backwages in favor or Pionilla, notwithstanding the illegality of his dismissal.
88. Asia Brewery, Inc. v. Tunay na Pagkakaisa ng mga Manggagawa sa Asia, G.R. Nos. 171594-96, September
18, 2013

Principle: the financial statements of the employer must be properly audited by an external and independent auditor
in order to be admissible in evidence for purposes of determining the proper wage award.

Facts:

[Respondent union] and [petitioner corporation] had been negotiating for a new collective bargaining agreement
(CBA) for the years 2003-2006 since the old CBA expired last July 2003. After about 18 sessions or negotiations, the
parties were still unable to reconcile their differences on their respective positions on most items, particularly on
wages and other economic benefits.

There was a deadlock so the respondent union filed a notice of strike with the NCMB. The union conducted a strike.

On November 20, 2003, [petitioner corporation] then petitioned the Secretary of the Department of Labor and
Employment (DOLE) to assume jurisdiction over the parties' labor dispute, invoking Article 263 (g) of the Labor Code.

On December 19, 2003, the public respondent, through Undersecretary/Acting Secretary Manuel G. Imson, issued an
order assuming jurisdiction over the labor dispute between the [respondent union] and [petitioner corporation].

In the meantime, in a decision dated January 19, 2004, Secretary of Labor Patricia Sto. Tomas resolved the deadlock
between the parties. As summarized in a later resolution, the public respondent granted the following arbitral awards
(which is now in dispute):

(1)WAGE INCREASES as follows:

First Year = P18.00

Second Year = 15.00

Third Year = 12.00

Total = P45.00

=====

(2)HEALTH CARE (HMO)

P1,300 premium to be shouldered by Asia Brewery, Inc., for each covered employee and P1,800 contribution [for
each] Union member-dependent.

In modifying the arbitral award of the Secretary of Labor, the CA ruled that: (1) ...XXX... ; (2) The computation of
wage increase should be remanded to the Secretary of Labor because the computation was based on petitioner
corporation's unaudited financial statements, which have no probative value pursuant to the ruling in Restaurante Las
Conchas v. Llego, 6 and was done in contravention of DOLE Advisory No. 1, Series of 2004, which contained the
guidelines in resolving bargaining deadlocks; and (3) The health benefits should be P1,390.00 per covered employee
because petitioner corporation had already agreed to this amount and the same cannot be altered or reduced by the
Secretary of Labor.

Issue:

Did the Secretary properly consider and appreciate the evidence presented before him?

Petitioner’s contention: Petitioner corporation admits that what it submitted to the Secretary of Labor were unaudited
financial statements which were then used as one of the bases in fixing the wage award. However, petitioner
corporation argues that these financial statements were duly signed and certified by its chief financial officer. These
statements have also been allegedly submitted to various government agencies and should, thus, be considered
official and public documents. Moreover, respondent union did not object to the subject financial statements in the
proceedings before the Secretary of Labor and even used the same in formulating its (the union's) arguments in said
proceedings. Thus, petitioner corporation contends that although the subject financial statements were not audited by
an external and independent auditor, the same should be considered substantial compliance with the order of the
Secretary of Labor to produce the petitioner corporation's complete audited financial statements for the past five
years. Furthermore, the Decision of the Secretary of Labor was not solely based on the subject financial statements
as the CBA history, costing of the proposals, and wages in other similarly situated bargaining units were considered.
Finally, petitioner corporation claims that the demands of respondent union on wage increase are unrealistic and will
cause the former to close shop.

Ruling:

The contention is untenable.

In Restaurante Las Conchas v. Llego, several employees filed a case for illegal dismissal after the employer closed
its restaurant business. The employer sought to justify the closure through unaudited financial statements showing
the alleged losses of the business. We ruled that such financial statements are mere self-serving declarations and
inadmissible in evidence even if the employees did not object to their presentation before the Labor Arbiter. Similarly,
in Uichico v. National Labor Relations Commission, the services of several employees were terminated on the ground
of retrenchment due to alleged serious business losses suffered by the employer. We ruled that by submitting
unaudited financial statements, the employer failed to prove the alleged business losses, viz.:

. . . It is true that administrative and quasi-judicial bodies like the NLRC are not bound by the technical rules of
procedure in the adjudication of cases. However, this procedural rule should not be construed as a license to
disregard certain fundamental evidentiary rules. While the rules of evidence prevailing in the courts of law or equity
are not controlling in proceedings before the NLRC, the evidence presented before it must at least have a modicum
of admissibility for it to be given some probative value. The Statement of Profit and Losses submitted by Crispa, Inc.
to prove its alleged losses, without the accompanying signature of a certified public accountant or audited by an
independent auditor, are nothing but self-serving documents which ought to be treated as a mere scrap of paper
devoid of any probative value. For sure, this is not the kind of sufficient and convincing evidence necessary to
discharge the burden of proof required of petitioners to establish the alleged losses suffered by Crispa, Inc. in the
years immediately preceding 1990 that would justify the retrenchment of respondent employees. . . .

While the above-cited cases involve proof necessary to establish losses in cases of business closure or
retrenchment, we see no reason why this rule should not equally apply to the determination of the proper level of
wage award in cases where the Secretary of Labor assumes jurisdiction in a labor dispute pursuant to Article 263 (g)
of the Labor Code.

We take note of the "middle ground" approach employed by the Secretary in this case which we do not necessarily
find to be the best method of resolving a wage dispute. Merely finding the midway point between the demands of the
company and the union, and "splitting the difference" is a simplistic solution that fails to recognize that the parties
may already be at the limits of the wage levels they can afford. It may lead to the danger too that neither of the parties
will engage in principled bargaining; the company may keep its position artificially low while the union presents an
artificially high position, on the fear that a "Solomonic" solution cannot be avoided. Thus, rather than encourage
agreement, a "middle ground approach" instead promotes a "play safe" attitude that leads to more deadlocks than to
successfully negotiated CBAs.

Thus, we rule that the Secretary of Labor gravely abused her discretion when she relied on the unaudited financial
statements of petitioner corporation in determining the wage award because such evidence is self-serving and
inadmissible. Not only did this violate the December 19, 2003 Order of the Secretary of Labor herself to petitioner
corporation to submit its complete audited financial statements, but this may have resulted to a wage award that is
based on an inaccurate and biased picture of petitioner corporation's capacity to pay — one of the more significant
factors in making a wage award. Petitioner corporation has offered no reason why it failed and/or refused to submit its
audited financial statements for the past five years relevant to this case. This only further casts doubt as to the
veracity and accuracy of the unaudited financial statements it submitted to the Secretary of Labor. Verily, we cannot
countenance this procedure because this could unduly deprive labor of its right to a just share in the fruits of
production and provide employers with a means to understate their profitability in order to defeat the right of labor to a
just wage.

UNION['S] DEMANDS

COMPANY'S OFFERS this Office awards the following wage increases:

For the FIRST YEAR: P36

For the SECOND YEAR: 36

For the THIRD YEAR: 36 For the First 18 months: P18

For the Second 18 months: 18 For the FIRST YEAR: P18

For the SECOND YEAR: 15

For the THIRD YEAR: 12

TOTAL: P108 for three (3) years P36 for 36 months P45 for three (3) years

As can be seen, the Secretary of Labor failed to indicate the actual data upon which the wage award was based. It
even appears that she utilized the "middle ground approach which we precisely warned against in Meralco. Factors
such as the actual and projected net operating income, impact of the wage increase on net operating income, the
company's previous CBAs, and industry trends were not discussed in detail so that the precise bases of the wage
award are not discernible on the face of the Decision. The contending parties are effectively precluded from seeking a
review of the wage award, even if proper under our ruling in Meralco, because of the general but unsubstantiated
statement in the Decision that the wage award was based on factors like the bargaining history, trends of arbitrated
and agreed awards, and industry trends. In fine, there is no way of determining if the Secretary of Labor utilized the
proper evidence, figures or data in arriving at the subject wage award as well as the reasonableness thereof. This
falls short of the requirement of administrative due process obligating the decision-maker to adjudicate the rights of
the parties in such a manner that they can know the various issues involved and the reasons for the decision
rendered.
89. Mcmer Corp., et al., vs. NLRC et al., GR No. 193421, June 4, 2014

Facts:

Private respondent, Libunao is employed as McMer’s legal department head. He has exhibited a strong opposition to
some company practices resulting in a severe marginal distance between him and petitioners, General Manager
Roque and, company President Alvestir at the workplace.

On About noon of July 20, 2007, private responent was summoned by Roque supposedly to discuss administrative
matter. However, private respondent refused to comply with the Roque’s summon. Consequently petitioner Roque
went to private respondent’s office at the height of his anger with threat to inflict physical harm, shouted a command
for private respondent to proceed to petitioner’s office. Private respondent was approached sarcastically with
commanding voice by petitioner Roque even in front of some officers and rank-and-file employees and newly-hired
employees.

As a consequence of the foregoing, private respondent elected to discontinue work that afternoon and immediately
proceeded to the Valenzuela Police Headquarters to report on the incident in the police blotter. Private respondent
did not report for work from July 21, 2007 up to July 30, 2007.

Private respondent filed a complaint for unfair labor practices, constructive illegal dismissal, non-payment of 13th
month pay and separation pay, moral and exemplary damages, as well as attorney’s fees, against petitioners McMer
Corporation, Inc., Roque, and Alvestir. Libunao was informed verbally by petitioner Alvestir that on account of
strained relationship brought about by the institution of a labor case against petitioners, the latter is inclined to dismiss
him from office. Private respondent was, likewise, offered a separation pay in the sum of P55,000.00.

Issues

1: WON private respondent was constructively dismissed

2: WON he is entitled to full backwages, separation pay in lieu of reinstatement, and moral, exemplary and nominal
damages.

Ruling:

1: Yes, there was constructive dismissal.

Constructive dismissal the cessation of work because continued employment is rendered impossible, unreasonable
or unlikely; when there is a demotion in rank or diminution in pay or both; or when a clear discrimination,
insensibility, or disdain by an employer becomes unbearable to the employee.

It is an act amounting to dismissal but made to appear as if it were not, ergo, a dismissal in disguise.

As such, the law recognizes and resolves this situation in favor of employees in order to protect their rights and
interests from the coercive acts of the employer.

The test of constructive dismissal is whether a reasonable person in the employee’s position would have felt
compelled to give up his position under the circumstances. In Aguilar v. Burger Machine Holdings Corporation the
hostile and unreasonable working conditions of petitioner justified the finding of the Labor Arbiter and the NLRC that
petitioner was constructively dismissed.

Petitioner’s performance may not have been exceptional as he ranked 14th in the quality food service control survey
for the 1st quarter of 2002. But he was certainly not grossly inefficient as Burger Machine pictured him to be. In fact,
he received several citations and was able to comply with the directive to reduce his shortages for the month of
November 2001. From all indications, there is really no ground to dismiss petitioner for gross inefficiency. And, as
Burger Machine saw it, the only way to get rid of the latter was to constructively dismiss him.

The employee who is constructively dismissed may be allowed to keep on coming to work.

In Siemens Philippines, Inc. v. Domingo, we have declared that

“An employee who is forced to surrender his position through the employer's unfair or unreasonable acts is deemed
to have been illegally terminated and such termination is deemed to be involuntary.”

Constructive dismissal does not always involve forthright dismissal or diminution in rank, compensation, benefit and
privileges.

There may be constructive dismissal if an act of clear discrimination, insensibility or disdain by an employer becomes
so unbearable on the part of the employee that it could foreclose any choice by him except to forego his continued
employment.

In this case, the private respondent was in a position wherein he would have felt compelled to give up his position
under the circumstances because continued employment was just impossible, unreasonable or unlikely.

The Affidavit executed by Guiao is sufficient to depict the hostile working environment petitioner McMer maintains.
Further, we find her description of petitioner Roque’s disposition adequate to support a conclusion that private
respondent was caught in the state of humiliation and embarrassment in the presence of his co-employees as a
result thereof. It is also corroborated another documentary evidence, i.e., the police blotter.
Aside from what transpired on July 20, 2007, various factors were considered in determining the working environment
of petitioner McMer. As may gleaned from the records, what transpired on July 20, 2007 was not merely an isolated
outburst on the part of petitioner Roque. The latter’s behaviour towards his employees shows a clear insensibility
rendering the working condition of private respondent unbearable.

Private respondent had reason to dawdle and refuse to comply with the summon of petitioner Roque out of severe
fear that he will be physically harmed. In fact, the same was clearly manifested by his immediate reaction to the
situation by going to the Valenzuela Police to report the incident.

Roque has displayed harassment and intimidation towards his employees. The fact that none of the employees
complained or brought this to the attention of the appropriate authority does not validate petitioners’ actions. Private
respondent reasoned that it was difficult for him to look for another employment, considering that at the time he filed
his Position Paper, he was already 58 years old. His eventual decision to leave petitioners due to the agonizing
situation at the workplace cannot, therefore, be discounted.

Private respondent could not have given up a job he has engaged in for eight years unless it has become so
unbearable for him to stay therein. Indeed, private respondent felt compelled to give up his employment.

Private respondent’s professional ethic or moral belief was compromised due to certain business practices of
petitioner McMer that were never exposed due to the employee’s fear of reprisal, as shown in private respondent’s
Position Paper.

As far as private respondent is concerned, how the working place is being run has caused inordinate strain on his
professional work and moral principles, even stretching to desecration of dignity in the workplace. The allegation that
all of private respondent’s staff were removed one by one until finally only the latter was left alone performing
managerial and clerical duties is merely part of the greater scheme brought forth by the insensibility of petitioners in
dealing with the employees.

No employee should be subjected to constant harassment, ridicule and inhumane treatment on the basis of
management prerogative or even for poor performance at work. While we concur with petitioners that raising one’s
voice in the workplace as a result of displeasure in the performance of an employee is not illegal per se, the right to
impose disciplinary sanctions upon an employee for just and valid cause is not without limit. The means does not
justify the end; thus, the same should be in accordance with the norms of due process.

2: Section 279 of the Labor Code explicitly states that an employee who is unjustly dismissed from work shall be
entitled to reinstatement without loss of seniority rights and other privileges and to his full backwages, inclusive of
allowances, and to his other benefits or their monetary equivalent computed from the time his compensation was
withheld from him up to the time of his actual reinstatement.

As early as Santos v. NLRC, the Supreme Court already explained the underlying basis for the foregoing provision, to
wit –

x x x. These twin remedies — reinstatement and payment of backwages — make the dismissed employee whole who
can then look forward to continued employment. Thus, do these two remedies give meaning and substance to the
constitutional right of labor to security of tenure. The two forms of relief are distinct and separate, one from the other.
Though the grant of reinstatement commonly carries with it an award of backwages, the inappropriateness or non-
availability of one does not carry with it the inappropriateness or non-availability of the other. x x x

Generally, a constructively dismissed employee is entitled to the twin remedies of reinstatement and payment of
backwages pursuant to Sec. 279 of the Labor Code. However, strained relations between the employer and
employee is an exception to the rule requiring actual reinstatement for illegally dismissed employees for the practical
reason that the already existing antagonism will only fester and deteriorate, and will only worsen with possible
adverse effects on the parties if we shall compel reinstatement; thus, the use of a viable substitute that protects the
interests of both parties while ensuring that the law is respected.

In this case, considering that reinstatement is no longer feasible due to the strained relations between petitioners and
private respondent, we find that the payment of separation pay of one month’s salary for every year of service is just
and reasonable as an alternative of reinstatement.

Private respondent was unjustly treated in the workplace, and, consequently, bore wounded feelings and suffered
mental anguish during his tenure with petitioner McMer until he was constructively dismissed from service. Therefore,
he is entitled to moral, exemplary and nominal damages in the aggregate amount of P90,000.00 due to the wanton,
oppressive and malevolent manner by which private respondent was illegally and constructively terminated.
90. Philippine Spring Water Resources Inc vs. Court of Appeals, GR No. 205278, June 11, 2014

Facts:

Mahilum was engaged as the VP for Sales and Marketing for the Bulacan-South Luzon Area of petitioner Philippine
Spring Water Resources, Inc. (PSWRI). Petitioner Danilo Lua (Lua) is the President and Chief Executive Officer
(CEO) of PSWRI.

Sometime in November 2004, the inauguration of PSWRI’s Bulacan plant would be celebrated at the same time with
the company’s Christmas party. Mahilum was designated as over-all chairman of the affair to be held on December
19, 2004.

On the inaugural day, Mahilum was not seen around to supervise the program proper as he entertained some visitors
of the company. According to him, he delegated the task to Evangelista.

Mahilum’s attention was, however, called when Lua got furious because he was not recognized during the program.
He was not mentioned in the opening remarks or called to deliver his inaugural speech. Upon inquiry from the
emcees of the program, Mahilum learned that they were not apprised of Lua’s decision to deliver the speech
considering that he previously declined to have a part in the program as he would be very busy during the affair.
Thus, Lua’s speech appeared to be “optional” in the printed program during the affair.

On the following day, Mahilum was required to explain why Lua was not recognized and made to deliver his speech.
At the same time, he was placed under preventive suspension for thirty (30) days. Mahilum submitted his written
explanation. Subsequently, an investigation was conducted.

When his 30-day suspension ended, Mahilum reported for work but was prevented from entering the workplace.
Sometime in the first week of March 2005, he received a copy of the Memorandum, dated January 31, 2005,
terminating his services effective the next day or on February 1, 2005. On February 9, 2005, a clearance certificate
was issued to Mahilum. He received the amount of 43,998.56 and was made to execute the Release, Waiver and
Quitclaim in favor of the company and Lua.

Mahilum filed a complaint for illegal dismissal with prayer for reinstatement, payment of back wages and damages.
He argued that he was illegally suspended and, thereafter, dismissed constructively from the service. He also claimed
that he was forced to sign the waiver.

1st Issue:

WON Mahilum was dismissed for a just cause under the ground of loss of trust and confidence.

Ruling:

Yes, Mahilum was illegally dismissed. The charge of loss of trust and confidence had no leg to stand on, as the act
complained of was not work-related. The petitioners were not able to prove that Mahilum was unfit to continue
working for the company.

Mahilum’s failure to effectively discharge his assignment as the over-all chairman of the festivities was due to mere
inadvertence and the mistaken belief that he had properly delegated the details of the program to another officer.
Further, his designation as the chairman of the whole affair did not form part of his duty as a supervisor. Mahilum was
engaged to supervise the sales and marketing aspects of PSWRI’s Bulacan Plant.

Even as jurisprudence has distinguished the treatment of managerial employees or employees occupying positions
of trust and confidence from that of rank-and-file personnel, insofar as the application of the doctrine of trust and
confidence is concerned, such is inapplicable to the instant case since as above-stated, private respondent’s lapse
was justified, unintentional, without deliberate intent and unrelated to the duty for which he was engaged.

The quitclaim executed by Mahilum did not operate to bar a cause of action for illegal dismissal. That the amounts
received by Mahilum were only those owing to him under the law indeed bolstered the fact that the quitclaim was
executed without consideration. Suffice it to say, the subject quitclaim may not be considered as a valid and binding
undertaking.

2nd Issue:

Mahilum’s entitlement to monetary claims.

Ruling:

Mahilum, as a regular employee at the time of his illegal dismissal, is entitled to separation pay and backwages,
computed from the time of his dismissal up to the finality of the decision. Reinstatement is no longer viable
considering the circumstances of animosity between Mahilum and Lua.

Article 279 of the Labor Code provides that an employee who is unjustly dismissed from work shall be entitled to
reinstatement without loss of seniority rights and other privileges, to full backwages, inclusive of allowances, and to
other benefits or their monetary equivalent computed from the time his compensation was withheld from him up to the
time of his actual reinstatement. Due to the strained relations of the parties, however, the payment of separation pay
has been considered an acceptable alternative, when reinstatement is no longer desirable or viable. On the one
hand, such payment liberates the employee from what could be a highly oppressive work environment. On the other,
the payment releases the employer from the grossly unpalatable obligation of maintaining in its employ a worker it
could no longer trust.17 Thus, as an illegally or constructively dismissed employee, the respondent is entitled to: (1)
either reinstatement, if viable, or separation pay, if reinstatement is no longer viable; and (2) backwages. These two
reliefs are separate and distinct from each other and are awarded conjunctivel

3rd Issue:

Inclusion of commission as part of Mahilum’s backwages.

Ruling:

The 0.25% commission on cash and delivery sales should not be included in his backwages because the alleged
commissions were profit-sharing payments and had no clear, direct or necessary relation to the amount of work he
actually performed.

Back wages are granted on grounds of equity to workers for earnings lost due to their illegal dismissal from work.

They are a reparation for the illegal dismissal of an employee based on earnings which the employee would have
obtained, either by virtue of a lawful decree or order, as in the case of a wage increase under a wage order, or by
rightful expectation, as in the case of one’s salary or wage.

The outstanding feature of backwages is thus the degree of assuredness to an employee that he would have had
them as earnings had he not been illegally terminated from his employment.

It is well-established in jurisprudence that the determination of whether or not a commission forms part of the basic
salary depends upon the circumstances or conditions for its payment.

In Phil Duplicators, Inc. v. NLRC, the Court held that commissions earned by salesmen form part of their basic salary.
The salesmen’s commissions, comprising a pre-determined percentage of the selling price of the goods sold by each
salesman, were properly included in the term basic salary for purposes of computing the 13th month pay. The
salesmen’s commissions are not overtime payments, nor profit-sharing payments nor any other fringe benefit, but a
portion of the salary structure which represents an automatic increment to the monetary value initially assigned to
each unit of work rendered by a salesman.

On the other hand, in Boie-Takeda Chemicals, Inc. v. De la Serna, the so-called commissions paid to or received by
medical representatives were excluded from the term basic salary because these were paid to the medical
representatives and rank-and-file employees as productivity bonuses, which were generally tied to the productivity, or
capacity for revenue production, of a corporation and such bonuses closely resemble profit-sharing payments and
had no clear direct or necessary relation to the amount of work actually done by each individual employee.

In Mahilum’s case, Phil. Duplicator cannot be automatically applied without considering his position as Vice-President
for sales and marketing of the PSWRI’s Bulacan-South Luzon Area. This factor constrains the Court to hold that
Mahilum’s 0.25% commission based on the monthly sales and 0.25% commission for cash payments must be taken
to come in the nature of overriding commission, not sales commission. The latter is not properly includable in the
basic salary as it must be earned by actual market transactions attributable to the claimant. Curiously, Mahilum did
not comment on the petitioners’ objection to the award. Not being a salesman who directly effected any sale of a
product, the commission embodied in the agreement partook of the nature of profit-sharing business based on quota.

4th Issue:

Mahilum’s entitlement to the award of moral and exemplary damages.

Ruling:

Mahilum is only entitled to attorney's fees in the amount of ten percent (10%) of his total monetary award, having
been forced to litigate in order to seek redress of his grievances, as provided in Article 111 of the Labor Code, as
amended, and existing jurisprudence.

He is not entitled to an award of moral or exemplary damages because no evidence presented to prove the
same.Worth reiterating is the rule that moral damages are recoverable where the dismissal of the employee was
attended by bad faith or fraud or constituted an act oppressive to labor, or was done in a manner contrary to morals,
good customs, or public policy. Likewise, exemplary damages may be awarded if the dismissal was effected in a
wanton, oppressive or malevolent manner.
91. Libcap Marketing Corp. et al., vs. Baquial, GR No. 192011, June 30, 2014

Facts:

Private worked as accounting clerk of petitioner company in Cagayan de Oro city. One day, petitioner company
noticed that there was something amiss with one of the transactions entered into by petitioner in that for a single
deposit to a bank, two separate slips were filled out by the latter. She tried to explain that two deposits were made
that day, but upon further investigation, it was discovered that there was only one deposit was made. Shortly after the
incident, respondent’s salary was charged with the payment of the amount involved, as during each payday, her
salary was deducted with staggered amounts to answer for the loss imputed to her. Respondent then gave notice to
petitioner that an investigation would be made in Ilo-ilo City (she was based in Cagayan de Oro), but due to financial
and geographical constraints, she was not able to participate in the proceedings. She was subsequently terminated
from work. In the Labor Arbiter, NLRC and CA, private respondent was awarded nominal damages due to the failure
of petitioner company to give her due notice before she got terminated. Libcap (petitioner company), on the other
hand, claimed that what is merely required is the opportunity to be heard, which it claimed to have extended to private
respondent but that it was the latter who failed to attend in the investigations conducted for her alleged infraction.

Issue:

Whether private respondent is entitled to nominal damages, and corollary to answering this question, whether due
notice was given to private respondent.

Ruling:

1. Private respondent is entitled to nominal damages for not being given due process before she was slapped with an
administrative penalty. The CA, the NLRC and the Labor Arbiter are correct in concluding that respondent was denied
due process, but their reasons for arriving at such conclusion are erroneous. What they seem to have overlooked is
that respondent's case has been pre-judged even prior to the start of the investigation on July 28, 2003. This is
evident from the fact that the amount of P1,437.00 — or the amount which petitioners claim was embezzled — was
peremptorily deducted each payday from respondent's salary on a staggered basis, culminating on June 30, 2003, or
nearly one month prior to the scheduled investigation on July 28, 2003. In doing so, petitioners have made it clear
that they considered respondent as the individual responsible for the embezzlement; thus, in petitioners' eyes,
respondent was adjudged guilty even before she could be tried — the payroll deductions being her penalty and
recompense. By pre-judging respondent's case, petitioners clearly violated her right to due process from the very
beginning, and from then on it could not be expected that she would obtain a fair resolution of her case. In a
democratic system, the infliction of punishment before trial is fundamentally abhorred. What petitioners did was
clearly illegal and improper.

2. There is a difference in separation pay between a just cause (nominal damages) and an authorized cause. Since in
the case of JAKA, the employee was terminated for authorized causes as the employer was suffering from serious
business losses, the Court fixed the indemnity at a higher amount of P50,000.00. In the case at bar, the cause for
termination was abandonment, thus it is due to the employee's fault. It is equitable under these circumstances to
order the petitioner company to pay nominal damages in the amount of P30,000.00, similar to the case of Agabon.

3. Nominal damages is for the vindication of a right, not for compensating hours worked and not paid. Prescinding
from the foregoing, we find it necessary to reduce the amount of nominal damages the CA awarded from
P100,000.00 to P30,000.00. We cannot subscribe to the CA's ratiocination that since respondent rendered overtime
work for four years without receiving any overtime pay, she is entitled to P100,000.00 nominal damages. Nominal
damages are awarded for the purpose of vindicating or recognizing a right and not for indemnifying a loss. Hence, the
CA should have limited the justification of the award of nominal damages to petitioners' violation of respondent's right
to due process in effecting her termination. It should not have considered the claimed unpaid overtime pay.|||
(LIBCAP Marketing Corp. v. Baquial, G.R. No. 192011, June 30, 2014)
92. Ampeloquio vs. Jaka Distribution Inc. GR No. 196936, July 2, 2014

Facts:

Petitioner works as a merchandiser for respondent company. He previously filed a case for illegal dismissal against
respondent company, and obtained a favorable judgment. Sometime in August 2004 (and after the Labor Arbiter
ordered his reinstatement in respondent company), he resumed work and was designated as a regular merchandiser
for JAKA (company). He was thereafter assigned to other branches outside of Manila, but his wage rate remained the
same, as well as his other allowances. Petitioner, in this case, claims that he is underpaid compared to how the other
merchandisers were paid, plus he also avers that they receive allowances which he does not receive. Respondent,
on the other hand, claims that the new merchandisers are casual or contractual employees who were contracted from
other companies who supplied them workers, so that the difference in salary between petitioner and them is justified
since these outsourced employees had different employers which gave them benefits based on their own company
practices.

Issues:

1. When an employee is reinstated, does his employment resume as though there was no interruption or does he
start from scratch?

2. Was the difference in salary and benefits entitlement justified between petitioner and other merchandisers who
were seasonally and/or casually employed?

Ruling:

1. The employee's years of service is deemed continuous and never interrupted. Such is likewise the rationale for
reinstatement's twin relief of full backwages. Ampeloquio is correct in asserting that he is a senior employee
compared to the other merchandisers whom he himself designates as casual or contractual merchandisers. He is
likewise senior to other regular employees subsequently hired by JAKA, specifically two regular messenger
employees which Ampeloquio claims receive wages higher than what he is receiving from JAKA. Attached to the
recognition of seniority rights of a reinstated employee who had been illegally dismissed is the entitlement to wages
appurtenant thereto. As the sole regular merchandiser of JAKA, Ampeloquio's reinstatement entitles him, at the
minimum, to the standard minimum wage at the time of his employment and to the wages he would have received
from JAKA had he not been illegally dismissed, as if there was no cessation of employment. Ampeloquio is likewise
entitled to any increase which JAKA may have given across the board to all its regular employees. To repeat,
Ampeloquio is not entitled to all benefits or privileges received by other employees subsequently hired by JAKA just
by the fact of his seniority in the service with JAKA.|||

2. Ampeloquio cannot likewise compare his wages to that received by "casual or contractual merchandisers" or
merchandisers who are admittedly outsourced from manpower agencies or those who are considered seasonal
employees hired only during peak season when JAKA is in need of extra merchandisers. To say the least, these
merchandisers are not, strictly speaking, employees of JAKA, but of a service provider company which has a service
contract with JAKA. The merchandisers in this case simply perform the work at JAKA's outlets, wearing uniforms
approved by JAKA but provided by the service company who is actually their employer. There is no employer-
employee relationship between JAKA and these merchandisers. Receipt by these merchandisers of a benefit such as
transportation or meal allowance is part of the monies they receive from their employer and embedded in the contract
price of the service agreement the employer has with JAKA.
92. Lim vs. HMR Phils Inc. GR No. 201483, August 4, 2014

Facts:

On February 8, 2001, petitioner Conrado A. Lim (Lim) filed a case for illegal dismissal and money claims against
respondents, HMR Philippines, Inc. (HMR) and its officers, Teresa G. Santos-Castro, Henry G. Bunag and Nelson S.
Camiller. The Labor Arbiter (LA) dismissed the complaint for lack of merit. On April 11, 2003, the National Labor
Relations Commission (NLRC) in NLRC NCR No. 02-00926-01, reversed the LA and declared Lim to have been
illegally dismissed.

Both Lim and HMR filed their respective petitions for certiorari before the CA, docketed as CA-G.R. SP No. 80379
and CA-G.R. SP No. 80630, respectively, which were consolidated. Pending resolution of the petitions, the CA issued
the Temporary Restraining Order (TRO) enjoining the execution of the NLRC decision.

On February 7, 2007, this Court, in G.R. No. 175950-51, dismissed the petition for certiorari4 filed by HMR assailing
the November 15, 2005 CA decision. Entry of judgment was ordered on July 27, 2007.5cralawred

On September 24, 2007, Lim moved for execution.6 On November 28, 2007, the Computation and Research Unit
(CRU) of the NLRC computed the total award to amount to P2,020,053.46,7 which computed the backwages from
February 3, 2001, the date of the illegal dismissal, up to October 31, 2007, the date of actual reinstatement.

HMR opposed the computation arguing that the backwages should be computed until April 11, 2003 only, the date of
promulgation of the NLRC decision, as stated in the dispositive portion of the NLRC decision, which provided that
backwages shall be “reckoned from his dismissal on February 3, 2001 up to the promulgation of this Decision.” It also
noted that the 10% annual increase was computed from 1998 to 2007, instead of only from 1998 to 2000 as
decreed.8cralawred

In his Comment, Lim argued that the body of the NLRC decision explictly stated that he was entitled to full
backwages from the time he was illegally dismissed until his actual reinstatement, which was also in accord with
Article 279 of the Labor Code and all prevailing jurisprudence

On April 21, 2009, the LA issued the order10 granting the motion for execution filed by Lim. Holding that the
backwages should be reckoned until April 11, 2003 only in accordance with the NLRC decision.

Ruling of the NLRC

Lim filed his “Motion Ad Cautelam for Reconsideration or Recomputation and Partial Execution of Monetary Award,”
insisting that his backwages should be computed up to his actual reinstatement.12 On August 28, 2009, the NLRC
treated the motion as an appeal and sustained the computation of the LA, explaining that the dispositive portion was
clear, and that it could not alter or amend the amount based on the final decision of the NLRC which was affirmed by
both the CA and this Court.13 Aggrieved, petitioner filed a petition for certiorari before the CA.

Ruling of the CA

In its assailed March 30, 2012 Decision, the CA dismissed the petition. It emphasized that the April 11, 2003 NLRC
decision had long become final and executory after it was affirmed by the Court and, as such, it may no longer be
amended or corrected. While noting that the body of the NLRC decision stated that petitioner was entitled to
backwages until his actual reinstatement, the CA ruled that when there was a conflict between the dispositive portion
and the body of the decision, the former must prevail as the dispositive portion was the final order, and that it was the
dispositive portion which was the subject of execution. It wrote that the fallo was clear and unequivocal and could,
therefore, be given effect without going to the body of the decision or further interpretation or construction.

The CA found that although the NLRC had recognized that petitioner was entitled to backwages until actual
reinstatement, nonetheless, it expressly limited the computation of backwages to the promulgation date of its
decision. It wrote that the issue of whether such limitation was lawful or improper could no longer be ventilated due to
the finality of the judgment.

Issues:

1. Whether the computation of backwages should be reckoned until the promulgation of the NLRC Decision on April
11, 2003 or until actual reinstatement?

2. Whether the petitioner is entitled to the unpaid 10% annual salary increase from 1998-2000?

3. Whether the petitioner is entitled to the 10% annual salary increase after the year 2000?

4. Whether the petitioner is entitled to holiday pay?

5. Whether the petitioner is entitled to sick leave pay?

6. Whether the respondents should be held jointly and severally liable for additional moral and exemplary damages?

7. Whether the interest in accordance with Eastern Shipping should be awarded?


Ruling:

Backwages

It is beyond question that Lim was illegally dismissed by HMR. All that remains to be settled is the exact amount
owing to petitioner as an illegally dismissed employee.

Article 279 of the Labor Code is clear in providing that an illegally dismissed employee is entitled to his full
backwages computed from the time his compensation was withheld up to the time of his actual reinstatement, to wit:

Art. 279. Security of tenure. In cases of regular employment, the employer shall not terminate the services of an
employee except for a just cause or when authorized by this Title. An employee who is unjustly dismissed from work
shall be entitled to reinstatement without loss of seniority rights and other privileges and to his full backwages,
inclusive of allowances, and to his other benefits or their monetary equivalent computed from the time his
compensation was withheld from him up to the time of his actual reinstatement.

In accordance with this provision, the body of the April 11, 2003 NLRC decision expressly recognizes that Lim is
entitled to his full backwages until his actual reinstatement, as follows:

In fine, the act of complainant-appellant herein, do not constitute a serious misconduct as to justify his dismissal. As
such, he is, thus, entitled to reinstatement to his former position as Assistant Technical Manager, unless such
position no longer exists, in which case, he shall be given a substantially equivalent position without loss of seniority
rights. He is, likewise, entitled to his full backwages from the time he was illegally dismissed until his actual
reinstatement.20cralawred

Nowhere in the body of the NLRC decision was there a discussion restricting the award of backwages. Nonetheless,
the fallo of the said decision limited the computation of the backwages up to its promulgation on April 11, 2003.

Considering that the judgment decreeing the computation of backwages up to the promulgation of the NLRC decision
has long become final and executory, the key question is whether a recomputation of backwages up to the date of the
actual reinstatement of Lim would violate the principle of immutability of judgments.

The rule is that it is the dispositive portion that categorically states the rights and obligations of the parties to the
dispute as against each other. Thus, it is the dispositive portion that must be enforced to ensure the validity of the
execution. That a judgment should be implemented according to the terms of its dispositive portion is a long and well-
established rule. A companion to this rule is the principle of immutability of final judgments. Save for recognized
exceptions, a final judgment may no longer be altered, amended or modified, even if the alteration, amendment or
modification is meant to correct what is perceived to be an erroneous conclusion of fact or law and regardless of what
court renders it. Any attempt to insert, change or add matters not clearly contemplated in the dispositive portion
violates the rule on immutability of judgments.

The cases of Session Delights Ice Cream and Fast Foods v. Court of Appeals (Session Delights) and Nacar v.
Gallery Frames (Nacar) shed much light on the apparent discrepancy in the case at hand. As in the present case,
both involve labor cases finding that the employees therein were illegally dismissed. At the LA level, in awarding
backwages, a precise computation was provided from the time of illegal dismissal up to the promulgation of the LA
decision. Additionally, the dispositive portion of the LA decision in Nacar also made a declaration that separation pay
in lieu of reinstatement be “computed only up to promulgation of this decision.” The LA decisions in these cases were
affirmed by the NLRC and the CA and subsequently became final and executory. At the execution stage, the
computation of backwages came into issue.

Session Delights made clear that a case for illegal dismissal is one that relates to status, where the decision or ruling
is essentially declaratory of the status and of the rights, obligations and monetary consequences that flow from the
declared status, such as, the payment of separation pay and backwages. In execution, what is primarily implemented
is the declaratory finding on the status and the rights and obligations of the parties therein; the arising monetary
consequences from the declaration only follow as component of the parties’ rights and obligations.27 The precise
amount of backwages should ideally be stated in the final decision; otherwise, the matter is for handling and
computation by the LA of origin as the labor official charged with the implementation of decisions before the
NLRC.28cralawred

The Court’s disquisition in Session Delights, also referenced with approval in Nacar, is enlightening:

A source of misunderstanding in implementing the final decision in this case proceeds from the way the original labor
arbiter framed his decision. The decision consists essentially of two parts.

The first is that part of the decision that cannot now be disputed because it has been confirmed with finality. This is
the finding of the illegality of the dismissal and the awards of separation pay in lieu of reinstatement, backwages,
attorney’s fees, and legal interests.

The second part is the computation of the awards made. On its face, the computation the labor arbiter made shows
that it was time-bound as can be seen from the figures used in the computation. This part, being merely a
computation of what the first part of the decision established and declared, can, by its nature, be re-computed. This is
the part, too, that the petitioner now posits should no longer be re-computed because the computation is already in
the labor arbiter’s decision that the CA had affirmed. The public and private respondents, on the other hand, posit that
a re-computation is necessary because the relief in an illegal dismissal decision goes all the way up to reinstatement
if reinstatement is to be made, or up to the finality of the decision, if separation pay is to be given in lieu of
reinstatement.

Clearly implied from this original computation is its currency up to the finality of the labor arbiter’s decision. As we
noted above, this implication is apparent from the terms of the computation itself, and no question would have arisen
had the parties terminated the case and implemented the decision at that point.

However, the petitioner disagreed with the labor arbiter’s findings on all counts – i.e., on the finding of illegality as well
as on all the consequent awards made. Hence, the petitioner appealed the case to the NLRC which, in turn, affirmed
the labor arbiter’s decision. By law, the NLRC decision is final, reviewable only by the CA on jurisdictional grounds.

It was at this point that the present case arose. Focusing on the core illegal dismissal portion of the original labor
arbiter’s decision, the implementing labor arbiter ordered the award re-computed; he apparently read the figures
originally ordered to be paid to be the computation due had the case been terminated and implemented at the labor
arbiter’s level. Thus, the labor arbiter re-computed the award to include the separation pay and the backwages due
up to the finality of the CA decision that fully terminated the case on the merits. Unfortunately, the labor arbiter’s
approved computation went beyond the finality of the CA decision (July 29, 2003) and included as well the payment
for awards the final CA decision had deleted – specifically, the proportionate 13th month pay and the indemnity
awards. Hence, the CA issued the decision now questioned in the present petition.

We see no error in the CA decision confirming that a re-computation is necessary as it essentially considered the
labor arbiter’s original decision in accordance with its basic component parts as we discussed above. To reiterate, the
first part contains the finding of illegality and its monetary consequences; the second part is the computation of the
awards or monetary consequences of the illegal dismissal, computed as of the time of the labor arbiter’s original
decision.

To illustrate these points, had the case involved a pure money claim for a specific sum (e.g. salary for a specific
period) or a specific benefit (e.g. 13th month pay for a specific year) made by a former employee, the labor arbiter’s
computation would admittedly have continuing currency because the sum is specific and any variation may only be
on the interests that may run from the finality of the decision ordering the payment of the specific sum.

In contrast with a ruling on a specific pure money claim, is a claim that relates to status (as in this case, where the
claim is the legality of the termination of the employment relationship). In this type of cases, the decision or ruling is
essentially declaratory of the status and of the rights, obligations and monetary consequences that flow from the
declared status (in this case, the payment of separation pay and backwages and attorney’s fees when illegal
dismissal is found). When this type of decision is executed, what is primarily implemented is the declaratory finding
on the status and the rights and obligations of the parties therein; the arising monetary consequences from the
declaration only follow as component of the parties’ rights and obligations.

In the present case, the CA confirmed that indeed an illegal dismissal had taken place, so that separation pay in lieu
of reinstatement and backwages should be paid. How much that separation pay would be, would ideally be stated in
the final CA decision; if not, the matter is for handling and computation by the labor arbiter of origin as the labor
official charged with the implementation of decisions before the NLRC.

Consistent with what we discussed above, we hold that under the terms of the decision under execution, no essential
change is made by a re-computation as this step is a necessary consequence that flows from the nature of the
illegality of dismissal declared in that decision. A re-computation (or an original computation, if no previous
computation has been made) is a part of the law – specifically, Article 279 of the Labor Code and the established
jurisprudence on this provision – that is read into the decision. By the nature of an illegal dismissal case, the reliefs
continue to add on until full satisfaction, as expressed under Article 279 of the Labor Code. The re-computation of the
consequences of illegal dismissal upon execution of the decision does not constitute an alteration or amendment of
the final decision being implemented. The illegal dismissal ruling stands; only the computation of monetary
consequences of this dismissal is affected and this is not a violation of the principle of immutability of final judgments.

That the amount the petitioner shall now pay has greatly increased is a consequence that it cannot avoid as it is the
risk that it ran when it continued to seek recourses against the labor arbiter’s decision. Article 279 provides for the
consequences of illegal dismissal in no uncertain terms, qualified only by jurisprudence in its interpretation of when
separation pay in lieu of reinstatement is allowed. When that happens, the finality of the illegal dismissal decision
becomes the reckoning point instead of the reinstatement that the law decrees. In allowing separation pay, the final
decision effectively declares that the employment relationship ended so that separation pay and backwages are to be
computed up to that point.

Although the NLRC decision in the present case did not provide a precise computation, the principles enunciated in
Session Delights still equally apply. In Session Delights, the computation of the LA was found to be time-bound,
which implied the currency of the computation up to the finality of the LA decision. In the present case, the NLRC
declared backwages to be reckoned “up to the promulgation” of its decision, which was an express declaration of the
currency of the computation up to the finality of the NLRC decision, especially considering that HMR was “ordered to
reinstate immediately” petitioner Lim. The decisions in both cases are premised on their immediate execution, in that
no question would have arisen had the parties terminated the case and the decision implemented at that point.

As discussed above, no essential change is being made by a recomputation because such is a necessary
consequence which flows from the nature of the illegality of the dismissal. To reiterate, a recomputation, or an original
computation, if no previous computation was made, as in the present case, is a part of the law that is read into the
decision, namely, Article 279 of the Labor Code and established jurisprudence.31 Article 279 provides for the
consequences of illegal dismissal, one of which is the payment of full backwages until actual reinstatement, qualified
only by jurisprudence when separation pay in lieu of reinstatement is allowed, where the finality of the illegal
dismissal decision instead becomes the reckoning point.

The nature of an illegal dismissal case requires that backwages continue to add on until full satisfaction. The
computation required to reflect full satisfaction does not constitute an alteration or amendment of the final decision
being implemented as the illegal dismissal ruling stands. Thus, in the present case, a computation of backwages until
actual reinstatement is not a violation of the principle of immutability of final judgments.

The respondents aver that the recoverable backwages cannot go beyond December 26, 2007, the date HMR offered
to reinstate Lim, who allegedly refused to be reinstated and abandoned his job.

HMR sent the petitioner a letter, dated December 22, 2007, directing him to report for work on December 26, 2007,
with an offer of separation pay in the amount of P150,000.00 in lieu of reinstatement which he could avail of not later
than December 26, 2007. Lim replied in a letter, dated December 24, 2007, requesting for a meeting in January 2008,
considering that his counsel was out of the country; that the NLRC was still in the process of computing the amount of
the award which was necessary to consider the offer of separation pay; and that a writ of execution had not yet been
issued. HMR never responded to the petitioner’s request, and up to the present, the latter has yet to be reinstated.

From the above, it is apparent that the petitioner cannot be deemed to have refused reinstatement or to have
abandoned his job. HMR’s offer of reinstatement appeared superficial and insincere considering that it never replied
to the petitioner’s letter. It did not make any further attempt to reinstate the petitioner either. The recoverable
backwages, thus, continue to run, and must be reckoned up until the petitioner’s actual reinstatement.

10% annual salary increase

Petitioner Lim argues that the LA completely failed to include in its computation the unpaid 10% annual increase in
his salary from 1998 to 2000, as stated in the fallo of the NLRC decision, and the 10% salary increase per annum in
backwages until actual reinstatement.

The pertinent portion of the fallo of the NLRC decision reads:

The Computation and Research Unit (CRU) of this Commission is hereby directed to compute the backwages and
the 10% annual increase from 1998 to 2000.

In awarding the 10% annual salary increase from 1998 to 2000, the body of the NLRC decision explained:

We see no reason, therefore, why complainant-appellant herein, being a regular employee, should be deprived of
what he is entitled to under Company policy. As such, he should be paid his unpaid 10% annual increase for the
years 1998, 1999 and 2000.37cralawred

Lim is, thus, entitled to be paid his unpaid 10% annual salary increase for the years 1998-2000.

A reading of the assailed order of the LA would reveal that it made the following adjustment in connection to the 10%
annual salary increase:

2) that the base rate applicable is his salary as of February 3, 2003 inclusive of the ten percent adjustment due at the
time, or P12,500.00 plus ten percent (10%) or P13,750.00;38chanrobleslaw

This is incorrect on two counts. First, the LA failed to include the actual unpaid 10% annual increase from 1998-2000.
The first computation of the LA, as well as the suggested computation of respondent HMR itself, gave the correct
computation of the unpaid salary increase from 1998-2000
93. Benson Industries Employees Union et al. vs. Benson Industries, Inc. GR No. 200746, August 6, 2014

FACTS:

Respondent Benson Industries, Inc. (Benson) is a domestic corporation engaged in the manufacturing of green coils
with the brand name Lion-Tiger Mosquito Killer. On February 12, 2008, Benson sent its employees, including herein
petitioners, a notice5 informing them of their intended termination from employment, to be effected on March 15,
2008 on the ground of closure and/or cessation of business operations. In consequence, the majority of Benson’s
employees resigned.6 Meanwhile, petitioners, through Benson Industries Employees Union-ALU-TUCP (Union), filed
a notice of strike, claiming that the company’s supposed closure was merely a ploy to replace the union members
with lower paid workers, and, as a result, increase its profit at their expense.7 The strike did not, however, push
through due to the parties’ amicable settlement during the conciliation proceedings before the NCMB, whereby
petitioners accepted Benson’s payment of separation pay, computed at 15 days for every year of service, as per the
parties’ Memorandum of Agreement8 dated April 9, 2008.9cr

This notwithstanding, petitioners proffered a claim for the payment of additional separation pay at the rate of four (4)
days for every year of service. As basis, petitioners invoked Section 1, Article VIII of the existing collective bargaining
agreement (CBA) executed by and between the Union and Benson which states that “[Benson] shall pay to any
employee/laborer who is terminated from the service without any fault attributable to him, a ‘Separation Pay’
equivalent to not less than nineteen (19) days’ pay for every year of service based upon the latest rate of pay of the
employee/laborer concerned.”10 Benson opposed petitioners’ claim, averring that the separation pay already paid to
them was already more than what the law requires. Reaching an impasse on the conflict, the parties referred the
issue to voluntary arbitration, wherein the validity of Benson’s closure was brought up as well.11cralawred

VA Ruled in favor of the employees.

CA reversed the VA.

ISSUE:

Whether or not the employees are entitled to additional separation pay of 4 days as stated in the CBA

RULING

The company was made to pay the 4 day additional separation pay. VA Decision was reinstated.

Closure of business may be considered as a reversal of an employer’s fortune whereby there is a complete cessation
of business operations and/or an actual locking-up of the doors of the establishment, usually due to financial losses.
Under the Labor Code, it is treated as an authorized cause for termination, aimed at preventing further financial drain
upon an employer who cannot anymore pay its employees since business has already stopped. As a form of
recompense, the employer is required to pay its employees separation benefits, except when the closure is due to
serious business losses.

While serious business losses generally exempt the employer from paying separation benefits, it must be pointed that
the exemption only pertains to the obligation of the employer under Article 297 of the Labor Code. This is because of
the law’s express parameter that mandates payment of separation benefits “in case of closures or cessation of
operations of establishment or undertaking not due to serious business losses or financial reverses.” The policy
distinction underlying Article 297 – that is, the distinction between closures due to serious business losses and those
which are not – was deftly discussed by the Court in the case of Cama v. Joni’s Food Services, Inc.,21 as
follows:chanRoblesvirtualLawlibrary

For a similar exemption to obtain against a contract, such as a CBA, the tenor of the parties’ agreement ought to be
similar to the law’s tenor. When the parties, however, agree to deviate therefrom, and unqualifiedly covenant the
payment of separation benefits irrespective of the employer’s financial position, then the obligatory force of that
contract prevails and its terms should be carried out to its full effect. Verily, it is fundamental that obligations arising
from contracts have the force of law between the contracting parties and thus should be complied with in good faith.

As in all contracts, the parties in a CBA may establish such stipulations, clauses, terms and conditions as they may
deem convenient provided these are not contrary to law, morals, good customs, public order or public policy. Thus,
where the CBA is clear and unambiguous, it becomes the law between the parties and compliance therewith is
mandated by the express policy of the law.28

In this case, it is undisputed that a CBA was forged by the employer, Benson, and its employees, through the Union,
to govern their relations effective July 1, 2005 to June 30, 2010. It is equally undisputed that Benson agreed to and
was thus obligated under the CBA to pay its employees who had been terminated without any fault attributable to
them separation benefits at the rate of 19 days for every year of service.
The postulation that Benson had closed its establishment and ceased operations due to serious business losses
cannot be accepted as an excuse to clear itself of any liability since the ground of serious business losses is not,
unlike Article 297 of the Labor Code, considered as an exculpatory parameter under the aforementioned CBA.
94. Montinola vs. Philippine Airlines, GR No. 198656, Sept. 8, 2014

FACTS

Montinola was employed as a flight attendant of Philippine Airlines (PAL) since 1996. On January 29, 2008,
Montinola and other flight crew members were subjected to custom searches in Honolulu, Hawaii, USA. Items from
the airline were recovered from the flight crew by customs officials. Nancy Graham (Graham), US Customs and
Border Protection Supervisor, sent an email to PAL regarding the search. The email7 contained a list of PAL flight
crew members.

PAL conducted an investigation. Montinola was among those implicated because she was mentioned in Graham’s
email.9 On February 1, 2008, PAL’s Cabin Services Sub-Department required Montinola to comment on the incident.
She gave a handwritten explanation three days after, stating that she did not take anything from the aircraft. She also
committed to give her full cooperation should there be any further inquiries on the matter.

During the hearing, Montinola admitted that in Honolulu, US customs personnel conducted a search of her person. At
that time, she had in her possession only the following food items: cooked camote, 3-in-1 coffee packs, and Cadbury
hot chocolate.

PAL, through Senior Assistant Vice President for Cabin Services Sub-Department Sylvia C. Hermosisima, found
Montinola guilty of 11 violations of the company’s Code of Discipline and Government Regulation. She was meted
with suspension for one (1) year without pay. Montinola asked for a reconsideration. Hermosisima, however, denied
her motion for reconsideration a month after.

Labor Arbiter: Suspension was illegal.

In addition, the Labor Arbiter awarded moral damages in the amount of P100,000.00 and exemplary damages
amounting to P100,000.00 for the following reasons:

The office observes that the records are replete with substantial evidence that the circumstances leading to
complainant’s one-year suspension without pay are characterized by arbitrariness and bad faith on the part of
respondents.

The Labor Arbiter also awarded attorney’s fees to Montinola because she was “forced to litigate and incur expenses
to protect [her] rights.”

NLRC Affirmed. CA AFFIRMED. Court of Appeals deleted the moral and exemplary damages and attorney’s fees
stating that: to the award of moral damages, not every employee who is illegally dismissed or suspended is entitled to
damages.

ISSUE:

Whether or not Montinola is entitled to moral and exemplary damages awarded by the LA.

RULING:

Montinola is entitled to moral and exemplary damages. She is also entitled to attorney’s fees.

The Labor Code provides:

Art. 279. Security of Tenure – In cases of regular employment, the employer shall not terminate the services of an
employee except for a just cause or when authorized by this Title. An employee who is unjustly dismissed from work
shall be entitled to reinstatement without loss of seniority rights and other privileges and to full backwages, inclusive
of allowances, and to his other benefits or their monetary equivalent computed from the time his compensation was
withheld from him up to the time of his actual reinstatement.

Security of tenure of workers is not only statutorily protected, it is also a constitutionally guaranteed right.61 Thus, any
deprivation of this right must be attended by due process of law.62 This means that any disciplinary action which
affects employment must pass due process scrutiny in both its substantive and procedural aspects.

Suspension from work is prima facie a deprivation of this right. Thus, termination and suspension from work must be
reasonable to meet the constitutional requirement of due process of law. It will be reasonable if it is based on just or
authorized causes enumerated in the Labor Code.65cralawred

The procedure can be summarized in this manner. First, the employer must furnish the employee with a written notice
containing the cause for termination. Second, the employer must give the employee an opportunity to be heard. This
could be done either through a position paper or through a clarificatory hearing.66 The employee may also be
assisted by a representative or counsel. Finally, the employer must give another written notice apprising the
employee of its findings and the penalty to be imposed against the employee, if any.67 In labor cases, these
requisites meet the constitutional requirement of procedural due process, which “contemplates notice and opportunity
to be heard before judgment is rendered, affecting one’s person or property.”68cralawred
In this case, PAL complied with procedural due process as laid out in Article 277, paragraph (b) of the Labor Code.
PAL issued a written notice of administrative charge, conducted a clarificatory hearing, and rendered a written
decision suspending Montinola. However, we emphasize that the written notice of administrative charge did not serve
the purpose required under due process. PAL did not deny her allegation that there would be a waiver of the
clarificatory hearing if she insisted on a specific notice of administrative charge. With Montinola unable to clarify the
contents of the notice of administrative charge, there were irregularities in the procedural due process accorded to
her.

Moreover, PAL denied Montinola substantial due process.

Just cause has to be supported by substantial evidence. Substantial evidence, or “such relevant evidence as a
reasonable mind might accept as adequate to support a conclusion,”69 is the quantum of evidence required in
administrative bodies such as the National Labor Relations Commission.

PAL, however, merely relied on these pieces of information in finding administrative liability against Montinola:

1) A list of offenses found in PAL’s Code of Discipline that Montinola allegedly violated;

2) A list of flight crew members that were checked at the Honolulu airport; and

3) A list of all items confiscated from all these flight crew members.

The lists are not sufficient to show the participation of any of the flight crew members, least of all Montinola. None of
the evidence presented show that the customs officials confiscated any of these items from her.

The employee is entitled to moral damages when the employer acted a) in bad faith or fraud; b) in a manner
oppressive to labor; or c) in a manner contrary to morals, good customs, or public policy.

Bad faith “implies a conscious and intentional design to do a wrongful act for a dishonest purpose or moral
obliquity.”73Cathay Pacific Airways v. Spouses Vazquez74 established that bad faith must be proven through clear
and convincing evidence.75 This is because “[b]ad faith and fraud . . . are serious accusations that can be so
conveniently and casually invoked, and that is why they are never presumed. They amount to mere slogans or
mudslinging unless convincingly substantiated by whoever is alleging them.”76 Here, there was clear and convincing
evidence of bad faith adduced in the lower tribunals.

PAL’s actions in implicating Montinola and penalizing her for no clear reason show bad faith. PAL’s denial of her
request to clarify the charges against her shows its intent to do a wrongful act for moral obliquity. If it were acting in
good faith, it would have gathered more evidence from its contact in Honolulu or from other employees before it
started pointing fingers. PAL should not have haphazardly implicated Montinola and denied her livelihood even for a
moment.

II.

Montinola is also entitled to exemplary damages.

Under Article 2229 of the Civil Code, “[e]xemplary or corrective damages are imposed, by way of example or
correction for the public good, in addition to the moral, temperate, liquidated or compensatory damages.” As this court
has stated in the past: “Exemplary damages are designed by our civil law to permit the courts to reshape behaviour
that is socially deleterious in its consequence by creating negative incentives or deterrents against such
behaviour.”83cralawred

It is socially deleterious for PAL to suspend Montinola without just cause in the manner suffered by her. Hence,
exemplary damages are necessary to deter future employers from committing the same acts.

III.

Montinola is also entitled to attorney’s fees.

Article 2208 of the Civil Code enumerates the instances when attorney’s fees can be awarded.

This case qualifies for the first, second, and seventh reasons why attorney’s fees are awarded under the Civil Code.

First, considering that we have awarded exemplary damages in this case, attorney’s fees can likewise be awarded.

Second, PAL’s acts and omissions compelled Montinola to incur expenses to protect her rights with the National
Labor Relations Commission and the judicial system. She went through four tribunals, and she was assisted by
counsel. These expenses would have been unnecessary if PAL had sufficient basis for its decision to discipline
Montinola.
Finally, the action included recovery for wages. To bring justice to the illegal suspension of Montinola, she asked for
backwages for her year of suspension.
95. NORTHWEST AIRLINES, INC., vs. MA. CONCEPCION M. DEL ROSARIO, G.R. No. 157633. September 10,
2014

FACTS:

Respondent Del Rosario was one of the Manila-based flight attendants for Northwest Airlines and was assigned at
the Business Class Section of the Airline. Gamboa, one of the flight attendants assigned at the first class section of
the same airline had a blunt wine bottle opener and asked her runner to borrow one from the other flight attendants
who happen to be the respondent. Del Rosario told the runner that a flight attendant who was not ready with a wine
bottle oepener has no business being working at the First Class Section. Unfortunately, this comment reached the
ears of Gamboa and resulted to a heated exchange of words between the two. This happened while the passengers
were already on board the aircraft. Escano, another flight attendant tried to pacify but to no avail, resorted to call the
attention of Morales, the Assistant Base Manager to pacify them. Morales tried to pacify them and ordered that they
take the argument somewhere else where there are no passengers. Morales further told them that they are allowed
to continue to fly on the same flight on the condition that they would have to stay away from each other during the
entire flight. Del Rosario was not willing to commit herself to do so that she decided not to allow both of them on the
same flight and furnished them a Notive of Removal from Service.

An investigation was conducted. Thereafter, Del Rosario was informed of her termination from service on the ground
that although there was no physical contact between her and Gamboa, fighting was strictly prohibited by Northwest
Airline that would entail dismissal from service even committed for the first time. Del Rosario filed a complaint before
the Labor Arbiter for Illegal Dismissal against Nortwest Airline. The decision of the Labor Arbiter was in favor of
Northwest. Del Rosario appealed to the NLRC. The decision of the Labor Arbiter was reversed by the NLRC in it’s
decision declaring that the incident between her and Gamboa could not be considered as synonymous with fighting
as the activity prohibited by Northwest's Rules of Conduct. Northwest elevated the decision to the CA alleging. CA
sustained the decision of the NLRC and ruled that Del Rosario’s conduct does not amount to serious misconduct
because the heated exchange of words between Gamboa and respondend did not come withing the definition of the
word “fighting”. Hence, this petition.

ISSUE:

Whether or not the dismissal of Del Rosario was valid.

HELD:

The Court AFFIRMS the decision of the CA.

As provided in Article 282 of the Labor Code, an employer may terminate an employee for a just cause, to wit:

Art. 282. TERMINATION BY EMPLOYER. —

An employer may terminate an employee for any of the following causes:

(a) Serious misconduct or willful disobedience by the employee of the lawful orders of his employer or representative
in connection with his work;

(b) Gross and habitual neglect by the employee of his duties;

(c) Fraud or willful breach by the employee of the trust reposed in him by his employer or duly authorized
representative;

(d) Commission of a crime or offense by the employee against the person of his employer or any immediate member
of his family or his duly authorized representative; and

(e) Other causes analogous to the foregoing.

Misconduct or improper behavior, to be a just cause for termination of employment, must: (a) be serious; (b) relate to
the performance of the employee's duties; and (c) show that the employee has become unfit to continue working for
the employer. There is no doubt that the last two elements of misconduct were present in the case of Del Rosario.
The cause of her dismissal related to the performance of her duties as a flight attendant, and she became unfit to
continue working for Northwest. However, the incident involving Del Rosario and Gamboa could not be justly
considered as akin to the fight contemplated by Northwest. In the eyes of the NLRC, Del Rosario and Gamboa were
arguing but not fighting. The understanding of fight as one that required physical combat was absent during the
incident. Moreover, even assuming arguendo that the incident was the kind of fight prohibited by Northwest's Rules of
Conduct, the same could not be considered as of such seriousness as to warrant Del Rosario's dismissal from the
service. The gravity of the fight, which was not more than a verbal argument between them, was not enough to
tarnish or diminish Northwest's public image.
96. Mount Carmel College Employees Union et al., vs. Mount Carmel College Inc. GR No. 187621, Sept. 24,
2014

FACTS:

Petitioners are high school academic and non-academic personnel of the Mount Carmel College Inc. They were
informed of their retrenchment due to the closure of the elementary and high school department of the school. The
petitioners contend that such closure was merely a subterfuge of their termination due to their union activities. The
petitioners contend that it was motivated by ill-will since the school reopened the elementary and high school
departments 2 months after with newly hired teachers, thus claiming for the remaining separation pay differentials.
The respondents denied committing any unfair labor practice as the closure of their elementary and high school
department was due to financial losses it suffered due to the decline of its enrollment, coupled with the demand of
increase in salaried from the teachers which gave the school no other choice but to close their elementary and high-
school departments.

The Labor Arbiter ruled that the petitioners were illegally dismissed, as the losses of the respondent were not serious
enough to warrant a closure of business and their financial statements even showed a net surplus, awarding
petitioners with separation pay in lieu of reinstatement and attorney’s fees. It further ruled that the respondents
committed unfair labor practice. The respondent appealed the same to the NLRC. The NLRC ruled that the
petitioners were not illegally dismissed; that the retrenchment of the petitioners was an exercise of management
prerogative as it’s financial status justifies the same. The petitioners questioned the bond posted by the respondents
however the NLRC ruled that the failure to a copy of the appeal bond and other documents to the Appeal
Memorandum furnished to the petitioners is a minor defect, that they acted in good faith when the procured the bond
from CBIC, which was blacklisted but thereafter was accredited again by the SC, making it’s bond good. Alleging
grave abuse of discretion, the CA sustained the ruling of the NLRC, that there is no factual basis in the allegation that
the school closed down for the purpose of union busting and that the school cannot be compelled to continue to
operate at a loss as shown in its financial statements.

ISSUE:

1. Whether or not the CA correctly ruled that the NLRC did not commit any grave abuse of discretion when it allowed
the respondent's appeal despite the blacklisting of CBIC at the time it issued the appeal bond.

2. Whether or not the CA committed grave abuse of discretion when it sustained the NLRC’s ruling of valid
retrenchment.

HELD:

1. In this regard, the Court has ruled that in a judgment involving a monetary award, the appeal shall be perfected
only upon: (1) proof of payment of the required appeal fee; (2) posting of a cash or surety bond issued by a reputable
bonding company; and (3) filing of a memorandum of appeal.

In this case, it was not disputed that at the time CBIC issued the appeal bond, it was already blacklisted by the NLRC.
Good faith, however, is not an excuse for setting aside the mandatory and jurisdictional requirement of the law. The
condition of posting a cash or surety bond is not a meaningless requirement — it is meant to assure the workers that
if they prevail in the case, they will receive the money judgment in their favor upon the dismissal of the former's
appeal. Such aim is defeated if the bond issued turned out to be invalid due to the surety company's expired
accreditation. Much more in this case where the bonding company was blacklisted at the time it issued the appeal
bond.

2. Retrenchment, as an authorized cause for the dismissal of employees, finds basis in Article 283 24 of the Labor
Code, which states:

Art. 283. Closure of establishment and reduction of personnel. — The employer may also terminate the employment
of any employee due to the installation of labor-saving devices, redundancy, retrenchment to prevent losses or the
closing or cessation of operation of the establishment or undertaking unless the closing is for the purpose of
circumventing the provisions of this Title, by serving a written notice on the workers and the Ministry of Labor and
Employment at least one (1) month before the intended date thereof. . . . . In case of retrenchment to prevent losses
and in cases of closures or cessation of operations of establishment or undertaking not due to serious business
losses or financial reverses, the separation pay shall be equivalent to one (1) month pay or at least one-half (1/2)
month pay for every year of service, whichever is higher. A fraction of at least six (6) months shall be considered one
(1) whole year.

Standards have been laid down by the Court in order to prevent its abuse by an employer, to wit:

(1) That retrenchment is reasonably necessary and likely to prevent business losses which, if already incurred, are
not merely de minimis, but substantial, serious, actual and real, or if only expected, are reasonably imminent as
perceived objectively and in good faith by the employer;

(2) That the employer served written notice both to the employees and to the Department of Labor and Employment
at least one month prior to the intended date of retrenchment;
(3) That the employer pays the retrenched employees separation pay equivalent to one (1) month pay or at least one-
half (1/2) month pay for every year of service, whichever is higher;

(4) That the employer exercises its prerogative to retrench employees in good faith for the advancement of its interest
and not to defeat or circumvent the employees' right to security of tenure; and

(5) That the employer used fair and reasonable criteria in ascertaining who would be dismissed and who would be
retained among the employees, such as status, efficiency, seniority, physical fitness, age, and financial hardship for
certain workers.

The burden of proving that the termination of services is for a valid or authorized cause rests upon the employer. In
termination by retrenchment, not every loss incurred or expected to be incurred by an employer can justify
retrenchment. The employer must prove, among others, that the losses are substantial and that the retrenchment is
reasonably necessary to avert such losses.

In this case, while the respondent may have presented its Financial Statements, the respondent, nevertheless, failed
to establish with reasonable certainty that the proportion of its revenues are largely expended for its elementary and
high school personnel salaries, wages and other benefits. The expenses for the elementary and high school
departments were not set out in detail and instead, were lumped together with the college department. Such detail
becomes material in the light of the respondent's claim that the personnel expenses for the elementary and high
school departments were "eating into" the portion of its budget allocated for other purposes. There could be no
practical basis from which the respondent's claim finds support. Aside from this, the respondent failed to present any
proof establishing how the continued operations of the elementary and high school departments has become
impracticable. Finally, on the petitioners' allegation that the closure and their retrenchment amounted to unfair labor
practice, suffice it to say that the petitioners failed to discharge its burden of proving that the retrenchment was
motivated by ill will, bad faith or malice, or that it was aimed at interfering with their right to self-organize.
97. Radio Mindanao Network, Inc. vs. Amurao III, GR No. 167225, October 22, 2014

Facts:

On February 16, 1989, petitioner Radio Mindanao Network, Inc. (RMN) hired respondent Michael Maximo R. Amurao
III (Michael) as a radio broadcaster for its DWKC-FM station and production manager for its metropolitan radio
operations at a monthly salary of P28,400.00.

Years later, RMN decided to reformat and restructure the programming of its DWKC-FM station to meet the demands
of the broadcasting industry. On April 25, 2002, the president of RMN met with Michael and other personnel of the
station to inform them of the management’s decision, advising them that the reformatting and restructuring of the
station’s programs would necessarily affect their employment; but assuring that they would be paid their retirement
pay and other benefits. To formalize the discussions had in their meeting, RMN furnished Michael and other
personnel separate letters dated May 14, 2002. However, Michael and the other personnel refused to sign in receipt
when the letters were served on them. Not long after, however, they accepted the offer of RMN and executed
affidavits relinquishing all their claims against the employer. 5 months after receiving his benefits and his execution of
the quitclaim, Michael filed a complaint against RMN for illegal dismissal with money claims in the National Labor
Relations Commission (NLRC).

Labor Arbiter rendered a decision declaring the dismissal of Michael as illegal, holding the quitclaim Michael signed
as void because it was not voluntarily executed.

RMN appealed to the NLRC, contending that the quitclaim signed in its favor was valid and binding because it
represented a voluntary and reasonable settlement of Michael’s claims; and that Michael was estopped from filing the
illegal dismissal case against it. NLRC held that the quitclaim was null and void for not being voluntarily executed.
RMN then appealed to the CA, and the CA denied due course to the petition and dismissed it for lack of merit.

Issue:

Whether or not the Affidavit of Release/Quitclaim executed by Michael was valid and binding; and

Ruling:

The Court finds and considers the CA’s ruling unfounded.

Not all quitclaims are per se invalid or against public policy. A quitclaim is invalid or contrary to public policy only: (1)
where there is clear proof that the waiver was wrangled from an unsuspecting or gullible person; or (2) where the
terms of settlement are unconscionable on their face. In instances of invalid quitclaims, the law steps in to annul the
questionable waiver. Indeed, there are legitimate waivers that represent the voluntary and reasonable settlements of
laborers’ claims that should be respected by the Court as the law between the parties. Where the party has
voluntarily made the waiver, with a full understanding of its terms as well as its consequences, and the consideration
for the quitclaim is credible and reasonable, the transaction must be recognized as a valid and binding undertaking,
and may not later be disowned simply because of a change of mind. A waiver is essentially contractual.

In our view, the requisites for the validity of Michael’s quitclaim were satisfied. Firstly, Michael acknowledged in his
quitclaim that he had read and thoroughly understood the terms of his quitclaim and signed it of his own volition.
Being a radio broadcaster and production manager, he occupied a highly responsible position in the company. It
would be implausible to hold, therefore, that he could be easily duped into simply signing away his rights. Secondly,
the settlement pay of P311,922.00 was credible and reasonable considering that Michael did not even assail such
amount as unconscionably low, or even state that he was entitled to a higher amount. Thirdly, that he was required to
sign the quitclaim as a condition to the release of the settlement pay did not prove that its execution was coerced.
Having agreed to part with a substantial amount of money, RMN took steps to protect its interest and obtain its
release from all obligations once it paid Michael his settlement pay, which it did in this case.

With the quitclaim having been freely and voluntarily signed, RMN was released and absolved from any liability in
favor of Michael. Suffice it to say that the quitclaim is ineffective in barring recovery of the full measure of an
employee’s rights only when the transaction is shown to be questionable and the consideration is scandalously low
and inequitable. Such is not true here.
98. Imasen Philippine Manufacturing Corp., vs. Alcon et al., GR No. 194884, October 22, 2014

Facts:

Petitioner Imasen Philippine Manufacturing Corporation is a domestic corporation engaged in the manufacture of auto
seat-recliners and slide-adjusters. It hired the respondents as manual welders in 2001. On October 5, 2002, the
respondents reported for work on the second shift - from 8:00 pm to 5:00 am of the following day. At around 12:40
am, Cyrus A. Altiche, Imasen's security guard on duty, went to patrol and inspect the production plant's premises.
When Altiche reached Imasen's Press Area, he heard the sound of a running industrial fan. Intending to turn the fan
off, he followed the sound that led him to the plant's "Tool and Die" section. At the "Tool and Die" section, Altiche saw
the respondents having sexual intercourse on the floor, using a piece of carton as mattress. Altiche immediately went
back to the guard house and relayed what he saw to Danilo S. Ogana, another security guard on duty.

On October 14, 2002, Imasen issued the respondents separate inter¬office memoranda informing them of Altiche's
report on the October 5, 2002 incident and directing them to submit their individual explanation. The respondents
complied with the directive; they claimed that they were merely sleeping in the "Tool and Die" section at the time of
the incident. They also claimed that other employees were near the area, making the commission of the act charged
impossible.

On December 4, 2002, Imasen issued the respondents separate inter¬office memoranda terminating their services. It
found the respondents guilty of the act charged which it considered as "gross misconduct contrary to the existing
policies, rules and regulations of the company."

On December 5, 2002, the respondents filed before the LA the complaint for illegal dismissal. The respondents
maintained their version of the incident. The LA found the respondents' dismissal valid, i.e., for the just cause of gross
misconduct and with due process. The LA additionally pointed out that the respondents did not show any ill motive or
intent on the part; of Altiche and Ogano sufficient to render their accounts of the incident suspicious.

The CA nullified the NLRC's ruling. The CA disagreed with the conclusion that the respondents' sexual intercourse
inside company premises constituted serious misconduct that the Labor Code considers sufficient to justify the
penalty of dismissal. The CA pointed out that the respondents' act, while provoked by "reckless passion in an inviting
environment and time," was not done with wrongful intent or with the grave or aggravated character that the law
requires. To the CA, the penalty of dismissal is not commensurate to the respondents' act, considering especially that
the respondents had not committed any infraction in the past. Accordingly, the CA reduced the respondents' penalty
to a three-month suspension.

Issue:

The sole issue for this Court's resolution is whether the respondents' infraction — engaging in sexual intercourse
inside company premises during work hours — amounts to serious misconduct within the terms of Article 282 (now
Article 296) of the Labor Code justifying their dismissal.

Ruling:

The just causes for dismissing an employee are provided under Article 282 (now Article 296) of the Labor Code.
Under Article 282(a), serious misconduct by the employee justifies the employer in terminating his or her
employment.

Misconduct is defined as an improper or wrong conduct. It is a transgression of some established and definite rule of
action, a forbidden act, a dereliction of duty, willful in character, and implies wrongful intent and not mere error in
judgment. To constitute a valid cause for the dismissal within the text and meaning of Article 282 of the Labor Code,
the employee's misconduct must be serious, i.e., of such grave and aggravated character and not merely trivial or
unimportant.

Additionally, the misconduct must be related to the performance of the employee's duties showing him to be unfit to
continue working for the employer. Further, and equally important and required, the act or conduct must have been
performed with wrongful intent.c

To summarize, for misconduct or improper behavior to be a just cause for dismissal, the following elements must
concur: (a) the misconduct must be serious; (b) it must relate to the performance of the employee's duties showing
that the employee has become unfit to continue working for the employer; and (c) it must have been performed with
wrongful intent.

After due consideration, we find the NLRC legally correct and well within its jurisdiction when it affirmed the validity of
the respondents' dismissal on the ground of serious misconduct. Sexual acts and intimacies between two consenting
adults belong, as a principled ideal, to the realm of purely private relations. Whether aroused by lust or inflamed by
sincere affection, sexual acts should be carried out at such place, time and circumstance that, by the generally
accepted norms of conduct, will not offend public decency nor disturb the generally held or accepted social morals.
Under these parameters, sexual acts between two consenting adults do not have a place in the work environment.

Indisputably, the respondents engaged in sexual intercourse inside company premises and during work hours. These
circumstances, by themselves, are already punishable misconduct. Added to these considerations, however, is the
implication that the respondents did not only disregard company rules but flaunted their disregard in a manner that
could reflect adversely on the status of ethics and morality in the company. Under these factual premises and in the
context of legal parameters we discussed, we cannot help but consider the respondents' misconduct to be of grave
and aggravated character so that the company was justified in imposing the highest penalty available — dismissal.
Their infraction transgressed the bounds of socially and morally accepted human public behavior, and at the same
time showed brazen disregard for the respect that their employer expected of them as employees. By their
misconduct, the respondents, in effect, issued an open invitation for others to commit the same infraction, with like
disregard for their employer's rules, for the respect owed to their employer, and for their co-employees' sensitivities.
Taken together, these considerations reveal a depraved disposition that the Court cannot but consider as a valid
cause for dismissal.
15. SUSPENSION OF BUSINESS OPERATIONS

99. Mindanao Terminal & Brokerage Service Inc et al., vs. Nagkahiusang Mamumuo sa Minterbro-Southern
Phils Federation of Labor, G.R. No. 174300, December 5, 2012

Facts:

Minterbro, the employer-company, is engaged in arrastre and stevedoring services. On April 14, 1997, it temporarily
ceased its operation because its port was in a bad shape then. On August 1, 1997, the employer formally informed
DOLE of the temporary cessation of operation up to December 16, 1997; repair works on the pier were commenced
on the same day.

The employees, thru its union, complained of constructive dismissal and consequently sought for separation benefits.
They argued that Minterbro did not operate for more than six months which constitutes dismissal thereby entitling
them to their separation claims.

In denying complainants their separation benefits, the Executive Labor Arbiter considered the period embraced within
August 1, 1997, when Minterbro formally informed the DOLE of the temporary cessation of operation up to December
16, 1997, when it was issued a certificate declaring the wharf safe and ready for operations and December 22-28,
1997, when the employer-company serviced a vessel MV Uranus which obviously did not exceed six (6) months, thus
denying complainants their monetary benefits. Incidentally, the period reckoned is incorrect.

Issue:

WON the suspension of business operations was valid?

Ruling:

No, because the suspension of business operations herein exceeded six months.

Under Article 286 of the Labor Code, the bona fide suspension of the operation of a business or undertaking for a
period not exceeding six months shall not terminate employment. Consequently, when the bona fide suspension of
the operation of a business or undertaking exceeds six months, then the employment of the employee shall be
deemed terminated. By the same token and applying said rule by analogy, if the employee was forced to remain
without work or assignment for a period exceeding six months, then he is in effect constructively dismissed.

The union members/employees were not given work starting April 14, 1997 and that more than six months have
elapsed after the union members were laid off when the next vessel was serviced at the Minterbro pier on December
22 to 28, 1997.

A lay-off, used interchangeably with "retrenchment," is a recognized prerogative of management. When a lay-off is
temporary, the employment status of the employee is not deemed terminated, but merely suspended. Article 286 of
the Labor Code provides, in part, that the bona fide suspension of the operation of the business or undertaking for a
period not exceeding six months does not terminate employment.
100. Leopard Security & Investigation Agency vs. Quitoy, G.R. No. 186344, Feb. 20, 2013

Facts:

Quitoy, et. al (respondents) were assigned to the different branches of its only client in Cebu City (Union Bank) by
petitioner Leopard Security & Investigation Agency (LSIA). On April 1, 2005, Union Bank decided to terminate the
services of LSIA, effective at the end of the same month. LSIA only informed the guards a day before the effectivity of
said termination made by Union Bank. On May 10, 2005 LSIA ordered the guards to report to its main office in
Mandaluyong City.

The guards did not heed to the said order. Few days prior however, the guards had already filed a complaint for
illegal dismissal (May 3, 2005).

Issue:

WON the floating status of the guards per se amounted to illegal dismissal.

Ruling:

No.

Applying Article 286 of the Labor Code of the Philippines by analogy, this Court has repeatedly recognized that
security guards may be temporarily sidelined by their security agency as their assignments primarily depend on the
contracts entered into by the latter with third parties. Temporary "off-detail" or "floating status" is the period of time
when security guards are in between assignments or when they are made to wait after being relieved from a previous
post until they are transferred to a new one. It takes place when, as here, the security agency’s clients decide not to
renew their contracts with the agency, resulting in a situation where the available posts under its existing contracts
are less than the number of guards in its roster. For as long as such temporary inactivity does not continue for a
period exceeding six months, it has been ruled that placing an employee on temporary "off-detail" or "floating status"
is not equivalent to dismissal.

In the case at bench, respondents were informed on 29 April 2005 that they were going to be relieved from duty as a
consequence of the 30 April 2005 expiration of the security service contract between Union Bank and LSIA. While
respondents lost no time in immediately filing their complaint, the record equally shows that they were directed by
LSIA to report for work at its Mandaluyong City office on 10 May 2005 or a mere ten days from the time the former
were effectively sidelined. Considering that a security guard is only considered illegally dismissed from service when
he is sidelined from duty for a period exceeding six months we find that the CA correctly upheld the NLRC’s ruling
that respondents were not illegally dismissed by LSIA. Parenthetically, said ruling is binding on respondents who did
not appeal either the decision rendered by the NLRC or the CA in line with the entrenched procedural rule in this
jurisdiction that a party who did not appeal cannot assign such errors as are designed to have the judgment modified.
101. SKM Art Craft Corp., vs. Bauca, et al., GR No. 171282, November 27, 2013

Facts:

The 23 respondents in G.R. No. 171282 were employed by petitioner SKM Art Craft Corporation which is engaged in
the handicraft business. On April 18, 2000, around 1:12 a.m., a fire occurred at the inspection and
receiving/repair/packing area of petitioner's premises in Intramuros, Manila. The fire investigation report stated that
the structure and the beach rubber building were totally damaged. Also burned were four container vans and a trailer
truck. The estimated damage was P22 million. On May 8, 2000, petitioner informed respondents that it will suspend
its operations for six months, effective May 9, 2000.

On May 16, 2000, only eight days after receiving notice of the suspension of petitioner's operations, the 23
respondents (and other co-workers) filed a complaint for illegal dismissal. They alleged that there was discrimination
in choosing the workers to be laid off and that petitioner had discovered that most of them were members of a newly-
organized union.

Petitioner denied the claim of illegal dismissal and said that Article 286 of the Labor Code allows the bona fide
suspension of a business or undertaking for a period not exceeding six months. Petitioner claimed that the fire cost it
millions in losses and that it is impossible to resume its normal operations for a significant period of time.

The Labor Arbiter ruled that the employees were illegally dismissed, on the ground that the petitioner failed to admit
them back to work after the expiration of the 6-month period, and ordered petitioner to pay them backwages
amounting to P59,918 each. The NLRC reversed the decision of the LA on the ground that it was premature to file a
case for illegal dismissal only 8 days into the suspension of operations considering the 6-month period had not yet
lapsed. It ordered reinstatement but without payment of backwages. On appeal to the CA, the CA ruled that the
petitioner failed to prove that the suspension of its operations was bona fide.

Issues:

(1) WON the employees were illegally dismissed considering the following:

a.) The complaint for illegal dismissal was filed only 8 days after notice of suspension of operations

b.) The petitioner failed to admit the respondents back to work after the lapse of the 6-month period

(2) WON the suspension of business operations of SKM Art Craft was bona fide

Ruling:

On the issue of illegal dismissal, while we agree with the NLRC that the suspension of petitioner's operation is valid,
the Labor Arbiter and the CA are correct that respondents were illegally dismissed since they were not recalled after
six months, after the bona fide suspension of petitioner's operations.

It is admitted that petitioner's premises was burned on April 18, 2000. Petitioner also submitted pictures of its
premises after the fire, the certification by the Barangay Chairman that petitioner's factory was burned, and the fire
investigation report of the Bureau of Fire Protection. To prove the damages, petitioner submitted a list of burned
machines, its inventory for April 2000 and the fire investigation report which stated that the estimated damage is P22
million.

We therefore agree with the NLRC that petitioner's suspension of operations is valid because the fire caused
substantial losses to petitioner and damaged its factory. On this point, we disagree with the CA that petitioner failed to
prove that its suspension of operations is bona fide. The list of materials burned was not the only evidence submitted
by petitioner. It was corroborated by pictures and the fire investigation report, and they constitute substantial evidence
of petitioner's losses.

Under Article 286 of the Labor Code, the bona fide suspension of the operations of a business or undertaking for a
period not exceeding six months shall not terminate employment. Article 286 provides,

ART. 286.When employment not deemed terminated. — The bona fide suspension of the operations of a business or
undertaking for a period not exceeding six (6) months, or the fulfillment by the employee of a military or civic duty
shall not terminate employment.

In all such cases, the employer shall reinstate the employee to his former position without loss of seniority rights if he
indicates his desire to resume his work not later than one (1) month from the resumption of operations of his
employer or from his relief from the military or civic duty.

The NLRC correctly noted that the complaint for illegal dismissal filed by respondents was premature since it was
filed only eight days after petitioner announced that it will suspend its operations for six months. In Nippon Housing
Phil., Inc. v. Leynes, we said that a complaint for illegal dismissal filed prior to the lapse of said six months is
generally considered as prematurely filed.

In this case, however, we agree with the Labor Arbiter and the CA that respondents were already considered illegally
dismissed since petitioner failed to recall them after six months, when its bona fide suspension of operations lapsed.
We stress that under Article 286 of the Labor Code, the employment will not be deemed terminated if the bona fide
suspension of operations does not exceed six months. But if the suspension of operations exceeds six months, the
employment will be considered terminated.

WHEREFORE, we DENY the petition in G.R. No. 171282 and AFFIRM the Decision dated November 9, 2005 and
Resolution dated January 24, 2006 of the Court of Appeals in CA-G.R. SP No. 76670, subject to the settlement
agreements and quitclaims signed by almost all of the respondents.
102. Navotas Shipyard Corp., vs. Montallana et al., GR No. 190053, March 24, 2014

Facts:

The case arose when respondents Innocencio Montallana, Alfredo Bautista, Teodoro Judloman, Guillermo Bongas,
Rogelio Bongas, Diosdado Busante, Emiliano Badu and Rosendo Subing-Subing filed a complaint for illegal
(constructive) dismissal, with money claims, against the petitioners, Navotas Shipyard Corporation (company) and its
President/General Manager, Jesus Villaflor.

The respondents alleged that on October 20, 2003, the company's employees (about 100) were called to a meeting
where Villaflor told them: "Magsasara na ako ng negosyo, babayaran ko na lang kayo ng separation pay dahil wala
na akong pangsweldo sa inyo. Marami akong mga utang sa krudo, yelo, at iba pa."4 Since then, they were not
allowed to report for work but Villaflor's promise to give them separation pay never materialized despite their
persistent demands and follow-ups. ScaHDT

The petitioners, on the other hand, claimed that due to the "seasonal lack of fish caught and uncollected
receivables[,]" 5 the company suffered financial reverses. It was thus constrained to temporarily cease operations.
They projected that the company could resume operations before the end of six months or on April 22, 2004. It
reported the temporary shutdown to the Department of Labor and Employment, National Capital Region (DOLE-NCR)
and filed an Establishment Termination Report.

The Labor Arbiter initially denied the complaint but awarded the complainants 13th month pay and SIL for the year
2003. The LA argued that the respondents could not have been illegally dismissed as the temporary shutdown of
operations merely suspends the employment relationship. The NLRC affirmed the LA decision in toto. On appeal to
the CA, the appellate court ruled that the closure was intended to be permanent and that in the absence of proper
notice given by the petitioners, the respondents should be considered illegally dismissed.

Petitioners then appealed the case pursuant to Rule 45 questioning the propriety of the award for separation pay and
backwages notwithstanding the closure of the company’s business operations.

Issues:

(1) What is the applicable law to the case, Art. 283 or Art. 286?

(2) WON the respondents were illegally dismissed

(3) WON the respondents are entitled to backwages/nominal damages

(4) WON the respondents are entitled to separation pay, service incentive leave and 13th month pay

Ruling:

To place the case in perspective, we first examine the applicable law in view of the disagreement between the
petitioners and the respondents in that respect. According to the CA, the "[p]etitioners anchor their arguments mainly
on Article 283 of the Labor Code, stating that private respondents resorted to retrenchment and permanent closure of
business, while private respondents maintain that what is applicable is Article 286 . . . as the closure of business was
merely temporary." Articles 283 and 286 of the Labor Code provide:

ART. 283. Closure of establishment and reduction of personnel. — The employer may also terminate the employment
of any employee due to the installation of labor-saving devices, redundancy, retrenchment to prevent losses or the
closing or cessation of operation of the establishment or undertaking unless the closing is for the purpose of
circumventing the provisions of this Title, by serving a written notice on the workers and the [Department of Labor]
and Employment at least one (1) month before the intended date thereof. In case of termination due to the installation
of labor-saving devices or redundancy, the worker affected thereby shall be entitled to a separation pay equivalent to
at least his one (1) month pay or to at least one (1) month pay for every year of service, whichever is higher. In case
of retrenchment to prevent losses and in cases of closures and cessation of operations of establishment or
undertaking not due to serious business losses or financial reverses, the separation pay shall be equivalent to one (1)
month pay or to at least one-half (1/2) month pay for every year of service, whichever is higher. A fraction of at least
six (6) months shall be considered one (1) whole year.

ART. 286. When employment not deemed terminated. — The bona-fide suspension of the operation of a business or
undertaking for a period not exceeding six (6) months, or the fulfillment by the employee of a military or civic duty
shall not terminate employment. In all such cases, the employer shall reinstate the employee to his former position
without loss of seniority rights if he indicates his desire to resume his work not later than one (1) month from the
resumption of operations of his employer or from his relief from the military or civic duty.

As we earlier stated, the petitioners undertook a temporary shutdown. In fact, the company notified the DOLE of the
shutdown and filed an Establishment Termination Report containing the names of the affected employees. The
petitioners expected the company to recover before the end of the six-month shutdown period, but unfortunately, no
recovery took place. Thus, the shutdown became permanent. According to the petitioners, they gave the company's
employees their separation pay.

We disagree with the company's position that it resorted to a retrenchment under Article 283 of the Labor Code; it
was a temporary shutdown under Article 286 where the employees are considered on floating status or whose
employment is temporarily suspended.
Under the circumstances, we cannot say that the company's employees were illegally dismissed; rather, they lost
their employment because the company ceased operations after failing to recover from their financial reverses. The
CA itself recognized what happened to the company when it observed: "The temporary shutdown has ripened into a
closure or cessation of operations. In this situation[,] private respondents are definitely entitled to the corresponding
benefits of separation." Even the respondents had an inkling of the company's fate when they claimed before the LA
that on October 20, 2003, they were called, together with all the other employees of the company, by Villaflor; the
latter allegedly told them that he would be closing the company, but would give them their separation pay. He also
disclosed to them the reason — he could no longer pay their salaries due to the company's unsettled financial
obligations on fuel and ice and other indebtedness.

Since there was no illegal dismissal, the respondents are not entitled to backwages. The term "backwages"
presupposes illegal termination of employment. It is restitution of earnings unduly withheld from the employee
because of illegal termination. Hence, where there is no illegal termination, there is no basis for claim or award of
backwages.

The lack of basis for backwages notwithstanding, we note that the respondents claimed that they were not given
individual written notices of the company's temporary shutdown or of its closure. The records support the
respondents' position. Other than the Establishment Termination Report submitted by the company to the DOLE-
NCR when it temporarily shut down its operations and which included the respondents' names, there is no evidence
(other than the petitioner's informal talk with its employees, which did not strictly comply with the legal requirement)
that they were served individual written notices at least thirty (30) days before the effectivity of the termination, as
required under Section 1 (iii), Rule I, Book VI of the Omnibus Rules Implementing the Labor Code. Pursuant to
existing jurisprudence, if the dismissal is by virtue of a just or authorized cause, but without due process, the
dismissed workers are entitled to an indemnity in the form of nominal damages.

Under Article 283 of the Labor Code quoted earlier, the employer may terminate the employment of any employee
due to, among other causes, the closure or cessation of operations of the establishment or undertaking. In such an
eventuality, the employee may or may not be entitled to separation pay. On this point, Article 283 provides: in cases
of closures or cessation of operations of establishment or undertaking not due to serious business losses or financial
reverses, the separation pay shall be equivalent to one (1) month pay or to at least one-half (1/2) month pay for every
year of service, whichever is higher. A fraction of least six months shall be considered one (1) whole year.

Considering that the company's closure was due to serious financial reverses, it is not legally bound to give the
separated employees separation pay. In Reahs Corporation v. NLRC,the Court explained that "[t]he grant of
separation pay, as an incidence of termination of employment under Article 283, is a statutory obligation on the part
of the employer and a demandable right on the part of the employee, except only where the closure or cessation of
operations was due to serious business losses or financial reverses and there is sufficient proof of this fact or
condition."

We note, however, that in his meeting with the employees, including the respondents, on October 20, 2003, Villaflor
told them that he would be giving them separation pay as a consequence of the company's closure. He should now
honor his undertaking to the respondents and grant them separation pay. Except for the petitioners' claim that "they
gave the separation pays of their employees," they failed to present proof of actual payment. In this light, Villaflor's
grant of separation pay to the respondents has still to be fulfilled.

Finally, the petitioners did not appeal the LA's award of service incentive leave pay and 13th month pay for the year
2003 to the respondents. Accordingly, the award stands.
103. Emeritus Security & Maintenance Systems Inc., vs. Dailig, GR No. 204761, April 2, 2014

Facts:

Respondent Dailig is one of the security guards of petitioner agency, who was assigned to one of its clients
(Panasonic) in Calamba, Laguna. On December 10, 2005 respondent was relieved from his post. On June 16, 2006
he filed a complaint for illegal dismissal claiming that on various dates in December 2005 to January to May 2006, he
went to petitioner’s office to follow-up his next assignment, but was still not given one. He argued that if an employee
is on floating status for more than 6 months, such an employee is deemed illegally dismissed.

Petitioner countered that he never dismissed respondent, and even sent notices to him to report to work on the day
he received the order of relief and once on January 27, 2006. Despite these notices, respondent failed to report for
work, which meant according to petitioner that he was no longer interested to continue his employment.

The Labor Arbiter, NLRC, and CA ruled for respondent saying that he was illegally dismissed, and gave him
separation pay instead of reinstatement, because of the strained relations doctrine. Petitioner questions the finding of
illegal dismissal, and separation pay – because it had already reinstated respondent with its sister company, Emme.

Issues:

A. Does a “floating status” of a security guard lasting for more than 6 months constitute constructive dismissal?

B. Is separation pay still proper when an employee is reinstated with a sister company?

Ruling:

A. Yes, in this case for 2 reasons:

Petitioner's allegation of sending respondent a notice sometime in January 2006, requiring him to report for work, is
unsubstantiated, and thus, self-serving. The Court agrees with the ruling of the Labor Arbiter, NLRC and Court of
Appeals that a floating status of a security guard, such as respondent, for more than six months constitutes
constructive dismissal. In Nationwide Security and Allied Services, Inc. v. Valderama, 8 the Court held: . . . the
temporary inactivity or "floating status" of security guards should continue only for six months. Otherwise, the security
agency concerned could be liable for constructive dismissal. The failure of petitioner to give respondent a work
assignment beyond the reasonable six-month period makes it liable for constructive dismissal. . . . .

Further, the Court notes that the Labor Arbiter, NLRC, and Court of Appeals unanimously found that respondent was
illegally dismissed by petitioner. Factual findings of quasi-judicial bodies like the NLRC, if supported by substantial
evidence, are accorded respect and even finality by this Court, more so when they coincide with those of the Labor
Arbiter. 10 Such factual findings are given more weight when the same are affirmed by the Court of Appeals

B. Yes

Article 279 of the Labor Code of the Philippines mandates the reinstatement of an illegally dismissed employee, to
wit:

Security of Tenure. — . . . An employee who is unjustly dismissed from work shall be entitled to reinstatement without
loss of seniority rights and other privileges and to his full back wages, inclusive of allowances, and to his other
benefits or their monetary equivalent computed from the time his compensation was withheld from him up to the time
of his actual reinstatement.

Thus, reinstatement is the general rule, while the award of separation pay is the exception. The circumstances
warranting the grant of separation pay, in lieu of reinstatement, are laid down by the Court in Globe-Mackay Cable
and Radio Corporation v. National Labor Relations Commission, thus:

Over time, the following reasons have been advanced by the Court for denying reinstatement under the facts of the
case and the law applicable thereto; that reinstatement can no longer be effected in view of the long passage of time
(22 years of litigation) or because of the realities of the situation; or that it would be 'inimical to the employer's
interest;' or that reinstatement may no longer be feasible; or, that it will not serve the best interests of the parties
involved; or that the company would be prejudiced by the workers' continued employment; or that it will not serve any
prudent purpose as when supervening facts have transpired which make execution on that score unjust or inequitable
or, to an increasing extent, due to the resultant atmosphere of 'antipathy and antagonism' or 'strained relations' or
'irretrievable estrangement' between the employer and the employee.

Petitioner counters that Emeritus and Emme are sister companies with the same Board of Directors and officers,
arguing that Emeritus and Emme are in effect one and the same corporation.

Considering petitioner's undisputed claim that Emeritus and Emme are one and the same, there is no basis in
respondent's allegation that he was not reinstated to his previous employment. Besides, respondent assails the
corporate personalities of Emeritus and Emme only in his Comment filed before this Court. Further, respondent did
not appeal the Labor Arbiter's reinstatement order.

Contrary to the Court of Appeals' ruling, there is nothing in the records showing any strained relations between the
parties to warrant the award of separation pay. There is neither allegation nor proof that such animosity existed
between petitioner and respondent. In fact, petitioner complied with the Labor Arbiter's reinstatement order.
Considering that (1) petitioner reinstated respondent in compliance with the Labor Arbiter's decision, and (2) there is
no ground, particularly strained relations between the parties, to justify the grant of separation pay, the Court of
Appeals erred in ordering the payment thereof, in lieu of reinstatement.
104. Lopez vs. Irvine Construction Corp. GR No. 207253, August 20, 2014

Facts:

Petitioner is a construction worker of respondent construction firm who sometimes acts as a guard at one of
respondent’s warehouses in Cavite. On December 18 2005 he was purportedly laid off work, where he was told “ikaw
ay lay-off muna.” Thus he filed a case for illegal dismissal. Respondent denied this by saying that he was not illegally
dismissed but was temporarily laid off when the project in Cavite was finished, and that he was sent a return to work
order within the 6 month period for bona fide suspension of the operation of a business or undertaking in the Labor
Code.

The LA and NLRC found petitioner to have been illegally dismissed, but the CA reversed the ruling dismissing the
case for illegal dismissal.

Issue:

Granting that petitioner is a regular employee, was there a proper temporary lay-off due to bona-fide suspension of
business operations?

Ruling:

Note: At the outset, the Court found petitioner to be a regular employee and not a project employee, entitled to
security of tenure. The discussion is limited to suspension of business operations.

NO, THERE WAS NO PROPER LAY-OFF DUE TO BONA-FIDE SUSPENSION OF BUSINESS

Among the authorized causes for termination under Article 283 of the Labor Code is retrenchment, or what is
sometimes referred to as a "lay-off":

Art. 283. Closure of Establishment and Reduction of Personnel. — The employer may also terminate the employment
of any employee due to the installation of labor-saving devices, redundancy, retrenchment to prevent losses or the
closing or cessation of operation of the establishment or undertaking unless the closing is for the purpose of
circumventing the provisions of this Title, by serving a written notice on the workers and the Ministry of Labor and
Employment at least one (1) month before the intended date thereof….

It is defined as the severance of employment, through no fault of and without prejudice to the employee, resorted to
by management during the periods of business recession, industrial depression, or seasonal fluctuations, or during
lulls caused by lack of orders, shortage of materials, conversion of the plant to a new production program or the
introduction of new methods or more efficient machinery, or of automation. Elsewise stated, lay-off is an act of the
employer of dismissing employees because of losses in the operation, lack of work, and considerable reduction on
the volume of its business, a right recognized and affirmed by the Court. However, a lay-off would be tantamount to a
dismissal only if it is permanent. When a lay-off is only temporary, the employment status of the employee is not
deemed terminated, but merely suspended.

Pursuant to Article 286 of the Labor Code, the suspension of the operation of business or undertaking in a temporary
lay-off situation must not exceed six (6) months:

ART. 286. When Employment not Deemed Terminated. — The bona-fide suspension of the operation of a business
or undertaking for a period not exceeding six (6) months….

Within this six-month period, the employee should either be recalled or permanently retrenched. Otherwise, the
employee would be deemed to have been dismissed, and the employee held liable therefor.

Notably, in both a permanent and temporary lay-off, jurisprudence dictates that the one-month notice rule to both the
DOLE and the employee under Article 283 of the Labor Code, as above cited, is mandatory. Also, in both cases, the
lay-off, being an exercise of the employer's management prerogative, must be exercised in good faith — that is, one
which is intended for the advancement of employers' interest and not for the purpose of defeating or circumventing
the rights of the employees under special laws or under valid agreements. Instructive on the nature of a lay-off as a
management prerogative is the following excerpt from the case of Industrial Timber Corporation v. NLRC:

Closure or [suspension] of operations for economic reasons is, therefore, recognized as a valid exercise of
management prerogative. The determination to cease [or suspend] operations is a prerogative of management, which
the State does not usually interfere with, as no business or undertaking [is] required to continue operating at a loss
simply because it has to maintain its workers in employment. Such an act would be tantamount to a taking of property
without due process of law.

As the NLRC correctly ruled in this case, Lopez, who, as earlier discussed was a regular employee of Irvine, was not
merely temporarily laid off from work but was terminated from his employment without any valid cause therefor; thus,
the proper disposition is to affirm the LA's ruling that Lopez had been illegally dismissed.

Although the NLRC did not expound on the matter, it is readily apparent that the supposed lay-off of Lopez was
hardly justified considering the absence of any causal relation between the cessation of Irvine's project in Cavite with
the suspension of Lopez's work. To repeat, Lopez is a regular and not a project employee. Hence, the continuation of
his engagement with Irvine, either in Cavite, or possibly, in any of its business locations, should not have been
affected by the culmination of the Cavite project alone. In light of the well-entrenched rule that the burden to prove the
validity and legality of the termination of employment falls on the employer, 47 Irvine should have established the
bona fide suspension of its business operations or undertaking that would have resulted in the temporary lay-off of its
employees for a period not exceeding six (6) months in accordance with Article 286 of the Labor Code.

In this case, Irvine failed to prove compliance with the parameters of Article 286 of the Labor Code. As the records
would show, it merely completed one of its numerous construction projects which does not, by and of itself, amount to
a bona fide suspension of business operations or undertaking. In invoking Article 286 of the Labor Code, the
paramount consideration should be the dire exigency of the business of the employer that compels it to put some of
its employees temporarily out of work. 51 This means that the employer should be able to prove that it is faced with a
clear and compelling economic reason which reasonably forces it to temporarily shut down its business operations or
a particular undertaking, incidentally resulting to the temporary lay-off of its employees.

Due to the grim economic consequences to the employee, case law states that the employer should also bear the
burden of proving that there are no posts available to which the employee temporarily out of work can be assigned.

Verily, Irvine cannot conveniently suspend the work of any of its employees in the guise of a temporary lay-off when it
has not shown compliance with the legal parameters under Article 286 of the Labor Code. With Irvine failing to prove
such compliance, the resulting legal conclusion is that Lopez had been constructively dismissed; and since the same
was effected without any valid cause and due process, the NLRC properly affirmed the LA's ruling that Lopez's
dismissal was illegal.
105. Exocet Security & Allied Services Corp., vs. Serrano, GR No. 198538, Sept. 29, 2014

Facts:

Petitioner Exocet Security and Allied Services Corporation (Exocet) is engaged in the provision of security personnel
to its various clients or principals. By virtue of its contract with JG Summit Holdings Inc. (JG Summit), Exocet
assigned respondent Armando D. Serrano (Serrano) on September 24, 1994 as “close-in”security personnel for one
of JG Summit’s corporate officers, Johnson Robert L. Go. After eight years, Serrano was re-assigned as close-in
security for Lance Gokongwei, and then to his wife, Mary Joyce Gokongwei. As close-in security, records show that
Serrano was receiving a monthly salary of P11,274.30.

On August 15, 2006, Serrano was relieved by JG Summit from his duties. For more than six months after he reported
back to Exocet, Serrano was without any reassignment. On March 15, 2007, Serrano filed a complaint for illegal
dismissal against Exocet with the National Labor Relations Commission (NLRC).

For its defense, Exocet denied dismissing Serrano alleging that, after August 15, 2006, Serrano no longer reported
for duty assignment as VIP security for JG Summit, and that on September 2006, he was demanding for VIP Security
detail to another client. However, since, at that time, Exocet did not have clients in need of VIP security assignment,
Serrano was temporarily assigned to general security service.6 Exocet maintained that it was Serrano who declined
the assignment on the ground that he is not used to being a regular security guard. Serrano, Exocet added, even
refused to report for immediate duty, as he was not given a VIP security assignment.

Considering the parties’ respective allegations, the Labor Arbiter ruled that Serrano was illegally dismissed. In its
June 30, 2008 Decision, the Labor Arbiter found that Serrano, while not actually dismissed, was placed on a floating
status for more than six months and so, was deemed constructively dismissed. Thus, the Labor Arbiter ordered
Exocet to pay Serrano separation pay.

Not satisfied with the award, Serrano appealed the Labor Arbiter’s Decision to the NLRC. In its March 5, 2009
Resolution, the NLRC initially affirmed the ruling of the Labor Arbiter, but modified the monetary award to include the
payment of backwages for six months that Serrano was not given a security assignment.

Acting on Exocet’s motion for reconsideration, however, the NLRC, in its September 2, 2009 Resolution, further
modified its earlier decision by removing the award for backwages.11 The NLRC deviated from its earlier findings and
ruled that Serrano was not constructively dismissed, as his termination was due to his own fault, stubborn refusal,
and deliberate failure to accept a re-assignment.12 Nevertheless, the NLRC proceeded to affirm in toto the decision
of the Labor Arbiter on the ground that Exocet did not interpose the appeal.

On January 22, 2010, the NLRC issued another Resolution denying Serrano’s motion for reconsideration.14 Hence,
not satisfied with the NLRC’s ruling, Serrano filed a petition for certiorari with the CA assailing the September 2, 2009
Resolution of the NLRC. Serrano insisted that he was constructively dismissed and, thus, is entitled to reinstatement
without loss of seniority rights and to full backwages from the time of the alleged dismissal up to the time of the finality
of the Decision.

On March 31, 2011, the appellate court rendered a Decision in Serrano’s favor, reversing and setting aside the
NLRC’s September 2, 2009 Resolution and ordering Exocet to pay Serrano separation pay and backwages.15 In so
ruling, the CA found that Serrano was constructively dismissed, as Exocet failed to re-assign him within six months
after placing him on “floating status.”Petitioner Exocet’s Motion for Reconsideration was denied by the appellate court
in its September 7, 2011 Resolution.17

Issue:

The sole issue for resolution is whether or not Serrano was constructively dismissed.

Ruling:

The petition has merit.

The crux of the controversy lies on the consequence of the lapse of the six-month period, during which respondent
Serrano was placed on a “floating status” and petitioner Exocet could not assign him to a position he wants. The
appellate court was of the view that Serrano was constructively dismissed. The Court maintains otherwise.

As the circumstance is generally outside the control of the security agency or the employer, the Court has ruled that
when a security guard is placed on a “floating status,” he or she does not receive any salary or financial benefit
provided by law.

Verily, a floating status requires the dire exigency of the employer’s bona fidesuspension of operation of a business
or undertaking. In security services, this happens when the security agency’s clients which do not renew their
contracts are more than those that do and the new ones that the agency gets. Also, in instances when contracts for
security services stipulate that the client may request the agency for the replacement of the guards assigned to it
even for want of cause, the replaced security guard may be placed on temporary “off-detail” if there are no available
posts under respondent’s existing contracts.

When a security guard is placed on a “floating status,” he does not receive any salary or financial benefit provided by
law. Due to the grim economic consequences to the employee, the employer should bear the burden of proving that
there are no posts available to which the employee temporarily out of work can be assigned.”
It must be emphasized, however, that although placing a security guard on “floating status” or a temporary “off-detail”
is considered a temporary retrenchment measure, there is similarly no provision in the Labor Code which treats of a
temporary retrenchment or lay-off. Neither is there any provision which provides for its requisites or its duration.22
Nevertheless, since an employee cannot be laid-off indefinitely, the Court has applied Article 292 (previously Article
286) of the Labor Code by analogy to set the specific period of temporary lay-off to a maximum of six (6) months.

In the controversy now before the Court, there is no question that the security guard, Serrano, was placed on floating
status after his relief from his post as a VIP security by his security agency’s client. Yet, there is no showing that his
security agency, petitioner Exocet, acted in bad faith when it placed Serrano on such floating status. What is more,
the present case is not a situation where Exocet did not recall Serrano to work within the six-month period as required
by law and jurisprudence. Exocet did, in fact, make an offer to Serrano to go back to work. It is just that the
assignment—although it does not involve a demotion in rank or diminution in salary, pay, benefits or privileges—was
not the security detail desired by Serrano.

Clearly,Serrano’s lack of assignment for more than six months cannot be attributed to petitioner Exocet. On the
contrary, records show that, as early as September 2006, or one month after Serrano was relieved as a VIP security,
Exocet had already offered Serrano a position in the general security service because there were no available clients
requiring positions for VIP security. Notably, even though the new assignment does not involve a demotion in rank or
diminution in salary, pay, or benefits, Serrano declined the position because it was not the post that suited his
preference, as he insisted on being a VIP Security.

Anent the client’s action, respondent agency had no recourse but to assign complainant to a new posting. However,
complainant, having had a taste of VIP detail and perhaps the perks that come with such kind of assignment,
vaingloriously assumed that he can only be assigned to VIP close-in posting and that he would accept nothing less.
In fact, after his relief and tardy appearance at respondent’s office, he was offered re-assignment albeit to general
security services which he refused. Respondents clearly made known to him that as of the moment no VIP detail was
vacant or sought by other clients but complainant was adamant in his refusal. Complainant even had the nerve to
assert that he just be informed if there is already a VIP detail available for him and that he will just report for re-
assignment by then. It is also well to note that to these allegations, complainant made no denial.29

Thus, it is manifestly unfair and unacceptable to immediately declare the mere lapse of the six-month period of
floating status as a case of constructive dismissal, without looking into the peculiar circumstances that resulted in the
security guard’s failure to assume another post. This is especially true in the present case where the security guard’s
own refusal to accept a non-VIP detail was the reason that he was not given an assignment within the six-month
period. The security agency, Exocet, should not then be held liable.

Indeed, from the facts presented, Serrano was guilty of wilful disobedience to a lawful order of his employer in
connection with his work, which is a just cause for his termination under Art.288 (previously Art. 282)of the Labor
Code.31 Nonetheless, Exocet did not take Serrano’s wilful disobedience against him. Hence, Exocet is considered to
have waived its right to terminate Serrano on such ground.

Petitioner Exocet Security and Allied Services Corporation is neither guilty of illegal dismissal nor constructive
dismissal. Petitioner is hereby ORDERED to look for a security assignment for respondent within a period of thirty
(30) days from finality of judgment. If one is available, petitioner is ordered to notify respondent Armando D. Serrano
to report to such available guard position within ten (10) days from notice. If respondent fails to report for work within
said time period, he shall be deemed to have abandoned his employment with petitioner.In such case, respondent
Serrano is notentitled to any backwages, separation pay, or similar benefits.

If no security assignment is available for respondent within a period of thirty (30) days from finality of judgment,
petitioner Exocet should comply with the requirements of DOLE Department Order No. 14, Series of 2001, in relation
to Art. 289 of the Labor Code, and serve a written notice on respondent Serrano and the DOLE one (1) month before
the intended date of termination; and pay Serrano separation pay equivalent to half month pay for every year of his
service.
16.DISEASE AS GROUND FOR TERMINATION

106. Deoferio vs. Intel Technology Phils., GR No. 202996, June 18, 2014

Facts:

On February 1, 1996, respondent Intel Technology Philippines, Inc. (Intel) employed Deoferio as a product quality
and reliability engineer with a monthly salary of P9,000.00. In July2001, Intel assigned him to the United States as a
validation engineer for an agreed period of two years and with a monthly salary of US$3,000.00. On January 27,
2002, Deoferio was repatriated to the Philippines after being confined at Providence St. Vincent Medical Center for
major depression with psychosis.4 In the Philippines, he worked as a product engineer with a monthly salary of
P23,000.00.

After several consultations, Dr. Lee issued a psychiatric report dated January 17, 2006 concluding and stating that
Deoferio’s psychotic symptoms are not curable within a period of six months and "will negatively affect his work and
social relation with his co-worker[s]." Pursuant to these findings, Intel issued Deoferio a notice of termination on
March 10, 2006.

Deoferio responded to his termination of employment by filing a complaint for illegal dismissal with prayer for money
claims against respondents Intel and Mike Wentling (respondents). He denied that he never had mental illness and
insisted that he satisfactorily performed his duties as a product engineer. He argued that Intel violated his statutory
right to procedural due process when it summarily issued a notice of termination. He further claimed that he was
entitled to a salary differential equivalent to the pre-terminated period of his assignment in the United States minus
the base pay that he had already received. Deoferio also prayed for backwages, separation pay, moral and
exemplary damages, as well as attorney’s fees.

In defense, the respondents argued that Deoferio’s dismissal was based on Dr. Lee’s certification that: (1) his
schizophrenia was not curable within a period of six months even with proper medical treatment; and (2) his
continued employment would be prejudicial to his and to the other employees’ health. The respondents also insisted
that Deoferio’s presence at Intel’s premises would pose an actual harm to his co-employees as shown by his
previous acts. On May 8, 2003, Deoferio emailed an Intel employee with this message: "All soul’s day back to work
Monday WW45." On January 18, 2005, he cut the mouse cables, stepped on the keyboards, and disarranged the
desks of his co-employees. The respondents also highlighted that Deoferio incurred numerous absences from work
due to his mental condition, specifically, from January 31, 2002 until February 28, 2002, from August 2002 until
September 2002, and from May 2003 until July 2003. Deoferio also took an administrative leave with pay from
January 2005 until December 2005.

The respondents further asserted that the twin-notice requirement in dismissals does not apply to terminations under
Article 284 of the Labor Code. They emphasized that the Labor Code’s implementing rules (IRR) only requires a
competent public health authority’s certification to effectively terminate the services of an employee.They insisted that
Deoferio’s separation and retirement payments for P247,517.35 were offset by his company car loan which amounted
to P448,132.43. He was likewise not entitled to moral and exemplary damages, as well as attorney’s fees, because
the respondents faithfully relied on Dr. Lee’s certification that he was not fit to work as a product engineer.

The Labor Arbitration Ruling

In a decision dated March 6, 2008,the Labor Arbiter (LA) ruled that Deoferio had been validly dismissed. The LA gave
weight to Dr. Lee’s certification that Deoferio had been suffering from schizophrenia and was not fit for employment.
The evidence on record shows that Deoferio’s continued employment at Intel would pose a threat to the health of his
co-employees. The LA further held that the Labor Code and its IRR do not require the employer to comply with the
twin-notice requirement in dismissals due to disease. The LA also found unmeritorious Deoferio’s money claims
against Intel.

On appeal by Deoferio, the National Labor Relations Commission (NLRC) wholly affirmed the LA’s ruling.The NLRC
also denied Deoferio’s motion for reconsideration, prompting him to seek relief from the CA through a petition for
certiorari under Rule 65 of the Rules of Court.

The CA’s Ruling

On February 24, 2012, the CA affirmed the NLRC decision. It agreed with the lower tribunals’ findings that Deoferio
was suffering from schizophrenia and that his continued employment at Intel would be prejudicial to his health and to
those of his co-employees. It ruled that the only procedural requirement under the IRR is the certification by a
competent public health authority on the non-curability of the disease within a period of six months even with proper
medical treatment. It also concurred with the lower tribunals that Intel was justified in not paying Deoferio separation
pay as required by Article 284 of the Labor Code because this obligation had already been offset by the matured car
loan that Deoferio owed Intel.

Deoferio filed the present petition after the CA denied his motion for reconsideration.

The Petition

In the present petition before the Court, Deoferio argues that the uniform finding that he was suffering from
schizophrenia is belied by his subsequent employment at Maxim Philippines Operating Corp. and Philips
Semiconductors Corp., which both offered him higher compensations. He also asserts that the Labor Code does not
exempt the employer from complying with the twin-notice requirement in terminations due to disease.

The Respondents’ Position

In their Comment, the respondents posit that the petition raises purely questions of fact which a petition for review on
certiorari does not allow. They submit that Deoferio’s arguments have been fully passed upon and found
unmeritorious by the lower tribunals and by the CA. They additionally argue that Deoferio’s subsequent employment
in other corporations is irrelevant in determining the validity of his dismissal; the law merely requires the non-curability
of the disease within a period of six months even with proper medical treatment.

The respondents also maintain that Deoferio’s claim for salary differential is already barred by prescription under
Article 291 of the Labor Code. Even assuming that the claim for salary differential has been timely filed, the
respondents assert that the parties expressly agreed in the International Assignment Relocation Agreement that "the
assignment length is only an estimate and not a guarantee of employment for any particular length of time."
Moreover, his assignment in the United States was merely temporary and did not change his salary base, an amount
which he already received.

Issues:

This case presents to us the following issues:

(1) Whether Deoferio was suffering from schizophrenia and whether his continued employment was prejudicial to his
health, as well as to the health of his co-employees;

(2) Whether the twin-notice requirement in dismissals applies to terminations due to disease; and

As part of the second issue, the following issues are raised:

(a) Whether Deoferio is entitled to nominal damages for violation of his right to statutory procedural due process; and

(b) Whether the respondents are solidarily liable to Deoferio for nominal damages.

(3) Whether Deoferio is entitled to salary differential, backwages, separation pay, moral and exemplary damages, as
well as attorney’s fees.

Ruling:

We find the petition partly meritorious. Intel had an authorized cause to dismiss Deoferio from employment
Concomitant to the employer’s right to freely select and engage an employee is the employer’s right to discharge the
employee for just and/or authorized causes.

The present case involves termination due to disease – an authorized cause for dismissal under Article 284 of the
Labor Code. As substantive requirements, the Labor Code and its IRR require the presence of the following
elements:

(1) An employee has been found to be suffering from any disease.

(2) His continued employment is prohibited by law or prejudicial to his health, as well as to the health of his co-
employees.

(3) A competent public health authority certifies that the disease is of such nature or at such a stage that it cannot be
cured within a period of six months even with proper medical treatment. With respect to the first and second
elements, the Court liberally construed the phrase "prejudicial to his health as well as to the health of his co-
employees" to mean "prejudicial to his health or to the health of his co-employees." We did not limit the scope of this
phrase to contagious diseases for the reason that this phrase is preceded by the phrase "any disease" under Article
284 of the Labor Code, to wit:

The third element substantiates the contention that the employee has indeed been suffering from a disease that: (1)
is prejudicial to his health as well as to the health of his co-employees; and (2) cannot be cured within a period of six
months even with proper medical treatment. Without the medical certificate, there can be no authorized cause for the
employee’s dismissal. The absence of this element thus renders the dismissal void and illegal.

Simply stated, this requirement is not merely a procedural requirement, but a substantive one. The certification from
a competent public health authority is precisely the substantial evidence required by law to prove the existence of the
disease itself, its non-curability within a period of six months even with proper medical treatment, and the prejudice
that it would cause to the health of the sick employee and to those of his co-employees.

In the current case, we agree with the CA that Dr. Lee’s psychiatric report substantially proves that Deoferio was
suffering from schizophrenia, that his disease was not curable within a period of six months even with proper medical
treatment, and that his continued employment would be prejudicial to his mental health. This conclusion is further
substantiated by the unusual and bizarre acts that Deoferio committed while at Intel’s employ.

The twin-notice requirement applies to terminations under Article 284 of the Labor Code. The Labor Code and its IRR
are silent on the procedural due process required in terminations due to disease. Despite the seeming gap in the law,
Section 2, Rule 1, Book VI of the IRR expressly states that the employee should be afforded procedural due process
in all cases of dismissals.

In Sy v. Court of Appeals and Manly Express, Inc. v. Payong, Jr., promulgated in 2003 and 2005, respectively, the
Court finally pronounced the rule that the employer must furnish the employee two written notices in terminations due
to disease, namely: (1) the notice to apprise the employee of the ground for which his dismissal is sought; and (2) the
notice informing the employee of his dismissal, to be issued after the employee has been given reasonable
opportunity to answer and to be heard on his defense. These rulings reinforce the State policy of protecting the
workers from being terminated without cause and without affording them the opportunity to explain their side of the
controversy.

From these perspectives, the CA erred in not finding that the NLRC gravely abused its discretion when it ruled that
the twin-notice requirement does not apply to Article 284 of the Labor Code. This conclusion is totally devoid of any
legal basis; its ruling is wholly unsupported by law and jurisprudence. In other words, the NLRC’s unprecedented,
whimsical and arbitrary ruling, which the CA erroneously affirmed, amounted to a jurisdictional error.

Deoferio is entitled to nominal damages for violation of his right to statutory procedural due process.

We award Deoferio the sum of P30,000.00 as nominal damages for violation of his statutory right to procedural due
process. In so ruling, we take into account Intel’s faithful compliance with Article 284 of the Labor Code and Section
8, Rule 1, Book 6 of the IRR. We also note that Deoferio’s separation pay equivalent to one-half month salary for
every year of service45 was validly offset by his matured car loan. Under Article 1278 of the Civil Code, in relation to
Article 1706 of the Civil Code46 and Article 113(c) of the Labor Code, compensation shall take place when two
persons are creditors and debtors of each other in their own right. We likewise consider the fact that Intel exhibited
real concern to Deoferio when it financed his medical expenses for more than four years. Furthermore, prior to his
termination, Intel liberally allowed Deoferio to take lengthy leave of absences to allow him to attend to his medical
needs.

Intel shall be solely liable to Deoferio for the satisfaction of nominal damages. Wentling, as a corporate officer, cannot
be held liable for acts done in his official capacity because a corporation, by legal fiction, has a personality separate
and distinct from its officers, stockholders, and members. There is also no ground for piercing the veil of corporate
fiction because Wentling acted in good faith and merely relied on Dr. Lee’s psychiatric report in carrying out the
dismissal.48

Deoferio is not entitled to salary differential, backwages, separation pay, moral and exemplary damages, as well as
attorney's fees.

Deoferio's claim for salary differential is already barred by prescription. Under Article 291 of the Labor Code, all
money claims arising from employer-employee relations shall be filed within three years from the time the cause of
action accrued. In the current case, more than four years have elapsed from the pre-termination of his assignment to
the United States until the filing of his complaint against the respondents. We thus see no point in further discussing
this matter.

SO ORDERED.
17.OTHER CAUSES OF SEVERANCE OF EMPLOYMENT RELATION

107. Gan vs. Galderma Philippines, Inc. G.R. No. 177167, January 17, 2013

Facts:

Respondent Galderma Philippines, Inc. (Galderma), a wholly-owned subsidiary of Galderma Pharma S.A., is
engaged in the business of selling, marketing, and distribution of Cetaphil Brand Product Lines (CBPL) that include
Cetaphil liquid and bar cleansers, and pharmaceutical products, such as Locetar, Benzac and other prescription
drugs. CBPL, which are over-the-counter products sold and/or distributed through supermarkets and health and
beauty outlets, are handled by Galderma's Consumer Products Division, while pharmaceutical products, which are
mostly prescription drugs sold and/or distributed through drug stores, are handled by its Ethical Products Division.

On February 9, 2001, petitioner Nelson B. Gan (Gan) was hired by Galderma as Product Manager for its Consumer
Products Division to handle the marketing of CBPL effective March 1, 2001.

Gan was initially under the immediate supervision of Sales and Marketing Manager, Stephen C. Peregrino
(Peregrino). Starting September 1, 2001, however, in view of Peregrino’s resignation, he directly reported to
Galderma's President and General Manager, respondent Rosendo C. Veneracion (Veneracion).

With his satisfactory performance during the first year, Gan was acknowledged and rewarded by Galderma through
positive performance appraisal, salary and benefits increases, and informal notations on his marketing reports.
Pursuant to its intention to give him additional product management responsibilities, Galderma provided Gan with
product knowledge training on Benzac and Locetar brands in December 2001. Thereafter, Gan’s incentive program
was revised and took effect in April 2002.

On April 11, 2002, Gan severed his employment ties with Galderma, and the company responded positively, making
his severance effective on July 15, 2002, which is three months allowance for the company to seek replacement.

Three months passed, on July 25, 2002, Gan filed a Complaint for illegal constructive dismissal, full backwages,
separation pay, damages, attorney’s fees, and cost of suit against respondents Galderma and Veneracion, alleging
that the resignation was not voluntary due to series of incidents where he said that Veneracion made several acts
that made him feel humiliated and embarrassed, alleging multiple acts of harassment by Veneracion due to the new
responsibilities given to him.

The respondents, on the other hand, denied sunch allegations saying that Galderma's senior managers noticed that
Gan had a change of attitude from the time the management decided to include the Benzac and Locetar brands
under his responsibility. Despite the fact that the company provided Gan with product knowledge training on the said
brands, he initially refused to accept the additional assignment. The company had to remind Gan that the assignment
was part of his Job Description, which allowed the company to assign him to undertake additional tasks as may be
deemed necessary by operations. Thereafter transpired series of activities necessary for the job and a number of
meetings happened. Later on, Gan asked the company that he wanted to resign and even asked for his arrangement
of favorable terms.

Issue:

whether or not there was constructive dismissal of Gan.

Ruling:

To begin with, constructive dismissal is defined as quitting or cessation of work because continued employment is
rendered impossible, unreasonable or unlikely; when there is a demotion in rank or a diminution of pay and other
benefits. It exists if an act of clear discrimination, insensibility, or disdain by an employer becomes so unbearable on
the part of the employee that it could foreclose any choice by him except to forego his continued employment. There
is involuntary resignation due to the harsh, hostile, and unfavorable conditions set by the employer. The test of
constructive dismissal is whether a reasonable person in the employee's position would have felt compelled to give
up his employment/position under the circumstances.

On the other hand, "resignation is the voluntary act of an employee who is in a situation where one believes that
personal reasons cannot be sacrificed in favor of the exigency of the service, and one has no other choice but to
dissociate oneself from employment. It is a formal pronouncement or relinquishment of an office, with the intention of
relinquishing the office accompanied by the act of relinquishment. As the intent to relinquish must concur with the
overt act of relinquishment, the acts of the employee before and after the alleged resignation must be considered in
determining whether he or she, in fact, intended to sever his or her employment."

Since Gan submitted a resignation letter, it is incumbent upon him to prove with clear, positive, and convincing
evidence that his resignation was not voluntary but was actually a case of constructive dismissal; that it is a product
of coercion or intimidation. He has to prove his allegations with particularity.

Gan could not have been coerced. Coercion exists when there is a reasonable or well-grounded fear of an imminent
evil upon a person or his property or upon the person or property of his spouse, descendants or ascendants. Neither
do the facts of this case disclose that Gan was intimidated
The instances of "harassment" alleged by Gan are more apparent than real. Aside from the need to treat his
accusations with caution for being self-serving due to lack of substantial documentary or testimonial evidence to
corroborate the same, the acts of "harassment," if true, do not suffice to be considered as "peculiar circumstances"
material to the execution of the subject resignation letter.

What the records of this case reveal is that Gan deliberately wrote and filed a resignation letter that is couched in a
clear, concise, and categorical language. Its content confirmed his unmistakable intent to resign. The resignation
letter indicates that he was resigning "to pursue the establishment of his own business or explore opportunities with
other companies." The reasons stated for relinquishing his position are but logical options for a person of his
experience and standing.

He was a managerial employee holding a responsible position and receiving more than the mandated minimum
wage. He also appears to have a good professional track record that highlights his marketability. At the time he
resigned, he had more than a decade of experience in sales and marketing with expertise in product management.
Indeed, it would be absurd to assume that he did not understand the full import of the words he used in his
resignation letter and the consequences of executing the same.

What is evident, therefore, is that Gan's resignation is NOT "a case of adherence, not of choice," but was a product of
a mutually beneficial arrangement. We agree with respondents that the result of the negotiation leading to Gan's
resignation is a "win-win" solution for both parties. On one hand, Gan was able to obtain a favorable severance pay
while getting flexible working hours to implement his post-resignation career options. On the other hand, Galderma
was able to cut its relation with an employee perceived to be unwilling to perform additional product responsibilities
while being given ample time to look for an alternative to hire and train. Indeed, Gan voluntarily resigned from
Galderma for a valuable consideration. He negotiated for an improvement of the resignation package offered and he
managed to obtain an acceptable one. As opposed to the case of San Miguel Corporation v. NLRC, Gan was not
tricked or was "morally and psychologically hoodwinked" to draft, sign, and tender his resignation letter. It was not
made without proper discernment and time to reflect; nor was it a knee-jerk reaction that left him with no alternative
but to accede.
108. Padillo vs. Rural bank of Nabunturan Inc. G.R. No. 199338, January 21, 2013.

Facts:

On October 1, 1977, petitioner, the late Eleazar Padillo (Padillo), was employed by respondent Rural Bank of
Nabunturan, Inc. (Bank) as its SA Bookkeeper. Due to liquidity problems which arose sometime in 2003, the Bank
took out retirement/insurance plans with Philippine American Life and General Insurance Company (Philam Life) for
all its employees in anticipation of its possible closure and the concomitant severance of its personnel. In this regard,
the Bank procured Philam Plan Certificate of Full Payment No. 88204, Plan Type 02FP10SC, Agreement No.
PP98013771 (Philam Life Plan) in favor of Padillo for a benefit amount of P100,000.00 and which was set to mature
on July 11, 2009.

On October 14, 2004, respondent Mark S. Oropeza (Oropeza), the President of the Bank, bought majority shares of
stock in the Bank and took over its management which brought about its gradual rehabilitation. The Bank's finances
improved and eventually, its liquidity was regained.

During the latter part of 2007, Padillo suffered a mild stroke due to hypertension which consequently impaired his
ability to effectively pursue his work. In particular, he was diagnosed with Hypertension S/P CVA (Cerebrovascular
Accident) with short term memory loss, the nature of which had been classified as a total disability. On September
10, 2007, he wrote a letter addressed to respondent Oropeza expressing his intention to avail of an early retirement
package. Despite several follow-ups, his request remained unheeded.

On October 3, 2007, Padillo was separated from employment due to his poor and failing health as reflected in a
Certification dated December 4, 2007 issued by the Bank. Not having received his claimed retirement benefits,
Padillo filed on September 23, 2008 with the NLRC Regional Arbitration Branch No. XI of Davao City a complaint for
the recovery of unpaid retirement benefits. He asserted, among others, that the Bank had adopted a policy of
granting its aging employees early retirement packages, pointing out that one of his co-employees, Nenita Lusan
(Lusan), was accorded retirement benefits in the amount of P348,672.72 when she retired at the age of only fifty-
three (53). The Bank and Oropeza (respondents) countered that the claim of Padillo for retirement benefits was not
favorably acted upon for lack of any basis to grant the same.

Issue:

Was it legal for the employer Bank to withhold Padillo’s retirement benefits considering that it was he who separated
himself from employment due to failing health and that he was not terminated by the Bank?

Is the grant to Luson of an early retirement package considered a company practice such as would entitle Padillo to
the same benefit?

Did the Bank act in bad faith and hence, liable for damages in discriminating against Padillo?

Ruling:

At the outset, it must be maintained that the Labor Code provision on termination on the ground of disease under
Article 297 does not apply in this case, considering that it was the petitioner and not the Bank who severed the
employment relations. As borne from the records, the clear import of Padillo's September 10, 2007 letter and the fact
that he stopped working before the foregoing date and never reported for work even thereafter show that it was
Padillo who voluntarily retired and that he was not terminated by the Bank.

Article 297 of the Labor Code contemplates a situation where the employer, and not the employee, initiates the
termination of employment on the ground of the latter's disease or sickness, viz.:

A plain reading of the [Article 297 of the Labor Code] clearly presupposes that it is the employer who terminates the
services of the employee found to be suffering from any disease and whose continued employment is prohibited by
law or is prejudicial to his health as well as to the health of his co-employees. It does not contemplate a situation
where it is the employee who severs his or her employment ties. This is precisely the reason why Section 8, Rule 1,
Book VI of the Omnibus Rules Implementing the Labor Code, directs that an employer shall not terminate the
services of the employee unless there is a certification by a competent public health authority that the disease is of
such nature or at such a stage that it cannot be cured within a period of six (6) months even with proper medical
treatment. (Emphasis, underscoring and words in brackets supplied)

Thus, given the inapplicability of Article 297 of the Labor Code to the case at bar, it necessarily follows that
petitioners' claim for separation pay anchored on such provision must be denied.

What is applicable, however, is the Labor Code provision on retirement. In particular, Article 300 of the Labor Code
as amended by Republic Act Nos. 7641 32 and 8558 33 partly provides:

Art. 300. Retirement. — Any employee may be retired upon reaching the retirement age established in the collective
bargaining agreement or other applicable employment contract.

In case of retirement, the employee shall be entitled to receive such retirement benefits as he may have earned
under existing laws and any collective bargaining agreement and other agreements: Provided, however, that an
employee's retirement benefits under any collective bargaining and other agreements shall not be less than those
provided herein.
In the absence of a retirement plan or agreement providing for retirement benefits of employees in the establishment,
an employee upon reaching the age of sixty (60) years or more, but not beyond sixty-five (65) years which is hereby
declared the compulsory retirement age, who has served at least five (5) years in the said establishment, may retire
and shall be entitled to retirement pay equivalent to at least one-half (1/2) month salary for every year of service, a
fraction of at least six (6) months being considered as one whole year.

Unless the parties provide for broader inclusions, the term one half (1/2) month salary shall mean fifteen (15) days
plus one-twelfth (1/12) of the 13th month pay and the cash equivalent of not more than five (5) days of service
incentive leaves.

Simply stated, in the absence of any applicable agreement, an employee must (1) retire when he is at least sixty (60)
years of age and (2) serve at least (5) years in the company to entitle him/her to a retirement benefit of at least one-
half (1/2) month salary for every year of service, with a fraction of at least six (6) months being considered as one
whole year. Notably, these age and tenure requirements are cumulative and non-compliance with one negates the
employee's entitlement to the retirement benefits under Article 300 of the Labor Code altogether.

In this case, it is undisputed that there exists no retirement plan, collective bargaining agreement or any other
equivalent contract between the parties which set out the terms and condition for the retirement of employees, with
the sole exception of the Philam Life Plan which premiums had already been paid by the Bank.

Neither was it proven that there exists an established company policy of giving early retirement packages to the
Bank's aging employees. In the case of Metropolitan Bank and Trust Company v. National Labor Relations
Commission, it has been pronounced that to be considered a company practice, the giving of the benefits should
have been done over a long period of time, and must be shown to have been consistent and deliberate. In this
relation, petitioners' bare allegation of the solitary case of Lusan cannot — assuming such fact to be true —
sufficiently establish that the Bank's grant of an early retirement package to her (Lusan) evolved into an established
company practice precisely because of the palpable lack of the element of consistency. As such, petitioners' reliance
on the Lusan incident cannot bolster their claim.

All told, in the absence of any applicable contract or any evolved company policy, Padillo should have met the age
and tenure requirements set forth under Article 300 of the Labor Code to be entitled to the retirement benefits
provided therein. Unfortunately, while Padillo was able to comply with the five (5) year tenure requirement — as he
served for twenty-nine (29) years — he, however, fell short with respect to the sixty (60) year age requirement given
that he was only fifty-five (55) years old when he retired. Therefore, without prejudice to the proceeds due under the
Philam Life Plan, petitioners' claim for retirement benefits must be denied.

Nevertheless, the Court concurs with the CA that financial assistance should be awarded but at an increased amount.
With a veritable understanding that the award of financial assistance is usually the final refuge of the laborer,
considering as well the supervening length of time which had sadly overtaken the point of Padillo's death — an
employee who had devoted twenty-nine (29) years of dedicated service to the Bank — the Court, in light of the
dictates of social justice, holds that the CA's financial assistance award should be increased from P50,000.00 to
P75,000.00, still exclusive of the P100,000.00 benefit receivable by the petitioners under the Philam Life Plan which
remains undisputed.

Finally, the Court finds no bad faith in any of respondents' actuations as they were within their right, absent any proof
of its abuse, to ignore Padillo's misplaced claim for retirement benefits. Respondents' obstinate refusal to accede to
Padillo's request is precisely justified by the fact that there lies no basis under any applicable agreement or law which
accords the latter the right to demand any retirement benefits from the Bank. While the Court mindfully notes that
damages may be recoverable due to an abuse of right under Article 21 in conjunction with Article 19 of the Civil
Code of the Philippines, 36 the following elements must, however, obtain: (1) there is a legal right or duty; (2)
exercised in bad faith; and (3) for the sole intent of prejudicing or injuring another. Records reveal that none of these
elements exists in the case at bar and thus, no damages on account of abuse of right may be recovered.

Neither can the grant of an early retirement package to Lusan show that Padillo was unfairly discriminated upon.
Records show that the same was merely an isolated incident and petitioners have failed to show that any bad faith or
motive attended such disparate treatment between Lusan and Padillo. Irrefragably also, there is no showing that
other Bank employees were accorded the same benefits as that of Lusan which thereby dilutes the soundness of
petitioners' imputation of discrimination and bad faith. Verily, it is axiomatic that bad faith can never be presumed — it
must be proved by clear and convincing evidence. This petitioners were unable to prove in the case at bar.
109 .Grace Christian High School vs. Lavandera, GR No. 177845, August 20, 2014

Facts:

Filipinas was employed by petitioner Grace Christian High School (GCHS) as high school teacher since June 1977,
with a monthly salary of P18,662.00 as of May 31, 2001.

On August 30, 2001, Filipinas filed a complaint for illegal (constructive) dismissal, non-payment of service incentive
leave (SIL) pay, separation pay, service allowance, damages, and attorney’s fees against GCHS and/or its principal,
Dr. James Tan. She alleged that on May 11, 2001, she was informed that her services were to be terminated
effective May 31, 2001, pursuant to GCHS’ retirement plan which gives the school the option to retire a teacher who
has rendered at least 20 years of service, regardless of age. The retirement pay would be one-half (½) month for
every year of service. At that time, Filipinas was only 58 years old and still physically fit to work. She pleaded with
GCHS to allow her to continue teaching but her services were terminated, contrary to the provisions of RA 7641
(“Retirement Pay Law.”)

GCHS denied that they illegally dismissed Filipinas. They asserted that the latter was considered retired on May 31,
1997 after having rendered 20 years of service pursuant to GCHS’ retirement plan and that she was duly advised that
her retirement benefits in the amount of P136,210.00 based on her salary at the time of retirement, i.e., P13,621.00,
had been deposited to the trustee-bank in her name. Nonetheless, her services were retained on a yearly basis until
May 11, 2001 when she was informed that her year-to-year contract would no longer be renewed.

LA Ruling:

LA ruled that Filipinas was not terminated but was considered as retired and thus, eligible to receive retirement pay.
But denied Filipinas’ claims for service allowance, salary increase, and damages for lack of sufficient bases.

NLRC Ruling:

NLRC set aside the LA’s award, and held that under Article 287 of the Labor Code, as amended by RA 7641, the
retirement package consists of 15 days salary, plus13th month pay and SIL pay pro-rated to their one-twelfth (1/12)
equivalent.

CA Ruling:

CA affirmed with modification the NLRC’s Decision. It held the computation of “one-half month salary” by equating it
to “22.5 days” which is “arrived at after adding 15 days plus 2.5 days representing one-twelfth of the 13th month pay,
plus 5 days of service incentive leave

Issue:

Whether or not the CA committed reversible error in using the multiplier “22.5 days” in computing the retirement pay
differentials of Filipinas.

Ruling:

No.

RA 7641 provides for the rules on retirement pay to qualified private sector employees in the absence of any
retirement plan in the establishment. The said law states that “an employee’s retirement benefits under any CBA and
other agreements shall not be less than those provided” under the same – that is, at least one-half (½) month salary
for every year of service, a fraction of at least six (6) months being considered as one whole year – and that “[u]nless
the parties provide for broader inclusions, the term one-half (½) month salary shall mean fifteen (15) days plus one-
twelfth (1/12) of the 13th month pay and the cash equivalent of not more than five (5) days of service incentive
leaves.”

In this case, GCHS has a retirement plan for its faculty and non-faculty members, which gives it the option to retire a
teacher who has rendered at least 20 years of service, regardless of age, with a retirement pay of one-half (½) month
for every year of service. However, that GCHS computed Filipinas’ retirement pay without including one-twelfth (1/12)
of her 13th month pay and the cash equivalent of her five (5) days SIL, both the NLRC and the CA correctly ruled that
Filipinas’ retirement benefits should be computed in accordance with Article 287 of the Labor Code, as amended by
RA 7641, being the more beneficent retirement scheme.

Thus, the term “one-half month salary” shall include all the following:

(a) Fifteen (15) days salary of the employee based on his latest salary rate

(b) The cash equivalent of not more than five (5) days of service incentive leave;

(c) One-twelfth of the 13th month pay due the employee.

(d) All other benefits that the employer and employee may agree upon that should be included in the computation of
the employee’s retirement pay.
110. Intel Technology Phils Inc. vs. NLRC et al., GR No. 200575, February 5, 2014

Facts:

Cabiles was initially hired by Intel Phil. on April 16, 1997 as an Inventory Analyst. He was subsequently promoted
several times over the years and was also assigned at Intel Arizona and Intel Chengdu. He later applied for a position
at Intel Semiconductor Limited Hong Kong (Intel HK).

Cabiles was offered the position of Finance Manager by Intel HK. On January 31, 2007, Cabiles signed the job offer
and started working on February 1, 2007. On September 8, 2007, after seven (7) months of employment, Cabiles
resigned from Intel HK.

About two years thereafter, Cabiles filed a complaint for non-payment of retirement benefits and for moral and
exemplary damages with the NLRC Regional Arbitration Branch-IV. He insisted that he was employed by Intel for 10
years and 5 months from April 1997 to September 2007 – a period which included his seven (7) month stint with Intel
HK. Thus, he believed he was qualified to avail of the benefits under the company’s retirement policy allowing an
employee who served for 10 years or more to receive retirement benefits.

Petitioner’ Position:

It contends that he is disqualified to receive the benefits for his failure to complete the required minimum ten (10)
years of service as he resigned to assume new responsibilities with Intel HK effective February 1, 2007.

Respondent’s Position:

Cabiles submits for his entitlement to retirement pay as he was under the employ of Intel Phil. for more than ten (10)
years in accordance with the prevailing retirement policy. Cabiles views his employment with Intel HK as a
continuation of his service with Intel Phil. alleging that it was but an assignment by his principal employer, similar to
his assignments to Intel Arizona and Intel Chengdu. Having rendered 9.5 years of service with Intel Phil. and an
additional seven months with Intel HK, he claims that he had completed the required 10 year continuous service with
Intel Phil., thus, qualifying him for retirement benefits.

Labor Arbiter’s Ruling

On March 18, 2010, the LA found that Cabiles did not sever his employment with Intel Phil. when he moved to Intel
HK, similar to the instances when he was assigned at Intel Arizona and Intel Chengdu, thus entitled to Retirement
pay.

NLRC Ruling

NLRC determined that his decision to move to Intel HK was not definitive proof of permanent severance of his ties
with Intel Phil. It treated his transfer to Hong Kong as akin to his overseas assignments in Arizona and Chengdu.

Issue:

WON there was severance of relation between Cabiles and Intel Philippines when he assumed new responsibilities
with Intel HK, thus disqualified to receive retirement benefits for failure to complete the required 10 years of service

Ruling:

Cabiles Resigned from Intel Philippines

Resignation is the formal relinquishment of an office, the overt act of which is coupled with an intent to renounce. This
intent could be inferred from the acts of the employee before and after the alleged resignation.

In this case, Cabiles, while still on a temporary assignment in Intel Chengdu, was offered by Intel HK the job of a
Finance Manager. His acceptance of the offer meant letting go of the retirement benefits he now claims as he was
informed through email correspondence that his 9.5 years of service with Intel Phil. would not be rounded off in his
favor. All these are indicative of the clearest intent of Cabiles to sever ties with Intel Phil. He chose to forego his
tenure with Intel Phil., with all its associated benefits, in favor of a more lucrative job for him and his family with Intel
HK.

Undoubtedly, Cabiles’ decision to move to Hong Kong required the abandonment of his permanent position with Intel
Phil. in order for him to assume a position in an entirely different company. Clearly, the "transfer" was more than just
an assignment. It constituted a severance of Cabiles’ relationship with Intel Phil., for the assumption of a position with
a different employer, rank, compensation and benefits.
111. Sutherland & Global Services Phils Inc., vs. Labrador, GR No. 193107, March 24, 2014

Facts:

Petitioner Sutherland Global Services (Philippines), Inc. (Sutherland)is engaged in the business of process
outsourcing and technology consultingservices for international clients.In August 2006, Sutherland hired Labrador as
one of its call center agents with the main responsibility ofanswering various queries and complaints through phoned-
in calls.

In his two years of working at Sutherland, Labrador committed several infractions.But it was only on June 17, 2008
that Labrador was finally charged with violation for transgressing the “Non-Compliance Sale Attribute” policy clause
stated in the Employee Handbook. Allegedly, on May 13, 2008, one of Sutherland’s customers complained that
Labrador

Initially asked for her credit card account, but only for purposes of verification. As it turned out, a second account was
created and a new order was placed under the same customer’s name. Thus, two sets of packages were shipped to
the customer who had to pay twice for the same product.

Under Sutherland’s Employee Handbook, Labrador’s action is classified as an act of dishonesty or fraud.6 On May
24, 2008, Sutherland sent Labrador a Notice to Explain7 in writing why he should not be held administratively liable.

After investigation, a recommendation was issued finding Labrador guilty of violating the Employee Handbook due to
gross or habitual neglect of duty.The recommendation further stated:

With (sic) the request of Mr. Larry Labrador (Customer ServiceRepresentative – UOLIB Sales) for resignation instead
of termination, due to humanitarian purposes and his stay and contribution to the account, SGS Management allows
his request of resigning from the company, ergo: he shall resign from the company effective immediately.

On June 17, 2008, Labrador submitted his resignation letter.

Issue:

Whether the CA erred in ruling that Labrador was illegally terminated and did not voluntarily resign

Rulings:

We have consistently ruled that the power to dismiss an employee is a recognized prerogative inherent in the
employer's right to freely manage and regulate his business. The law, however, in protecting the rights of the
laborers, authorizes neither oppression nor self-destruction of the employer. The worker's right to security of tenure is
not an absolute right, for the law provides that he may be dismissed for cause. Furthermore, Article 282 of the Labor
Code provides that an employee may be terminated from the service on either of the following just causes:

Art. 282. Termination by employer. - An employer may terminate an employment for any of the following causes:

xxxxxxxxxx

2. Gross and habitual neglect by the employee of his duties;

xxxxxxxxx

5. Other causes analogous to the foregoing.

The failure to faithfully comply with the company rules and regulations is considered to be a just cause in terminating
one’s employment, depending on the nature, severity and circumstances of non-compliance. “An employer ‘has the
right to regulate, according to its discretion and best judgment, all aspects of employment, including work assignment,
working methods, processes to be followed, working regulations, transfer of employees, work supervision, lay-off of
workers and the discipline, dismissal and recall of workers.’”

Thus, it was within Sutherland’s prerogative to terminate Labrador’s employment when he committed a serious
infraction and, despite a previous warning, repeated it. To reiterate, he opened another client account without the
latter’s consent, with far-reaching and costly effects on the company.

For one, the repeated past infractions would have resulted in negative feedbacks on Sutherland’s performance and
reputation. It would likewise entail additional administrative expense since Sutherland would have to address the
complaints – an effort that would entail investigation costs and the return of the doubly-delivered merchandise. As a
rule, “an employer cannot be compelled to continue with the employment of workers when continued employment will
prove inimical to the employer's interests.”

To Sutherland’s credit, it duly complied with the procedural requirement in dismissing an employee; it clearly
observed both substantive and procedural due process. Its action was based on a just and authorized cause, and the
dismissal was effected after due notice and hearing.29 After Labrador’s subsequent infraction, Sutherland sent him a
Notice to Explain and an administrative hearing was thereafter conducted. During the hearing,Labrador himself
admitted his faults. These incidents were properlyrecorded and were properly discussed in Sutherland's
recommendation Butbefore Sutherland could finally pronounce its verdict, Labrador submittedhis resignation letter,
impelled no doubt, as Sutherland alleged, by the needto protect his reputation and his future employment chances.
To be sure,Sutherland's explanation was not remote, far-fetched or unbelievable eventhe undisputable evidence on
record of infractions.
112. Chiang Kai Shek College et al., vs. Torres, GR No. 189456, April 2, 2014

Facts:

Petitioner Chiang Kai Shek College is a private educational institution that offers elementary to college education to
the public. Individual petitioner Carmelita Espino is the Vice-President of the school. Respondent had been employed
as a grade school teacher of the school from July 1970 until 31 May 2003. The manner of her severance from
employment is the matter at hand. Respondent was accused of leaking a copy of a special quiz given to Grade 5
students of HEKASI (HEKASI 5). HEKASI stands for Heograpiya, Kasaysayan at Sibika(Geography, History and
Civics). Petitioners came to know about the leakage from one of the teachers of HEKASI 5, Aileen Benabese (Ms.
Benabese). Ms. Benabese narrated that after giving a special quiz, she borrowed the book of one of her students,
Aileen Regine M. Anduyan (Aileen), for the purpose of making an answer key. When she opened Aileen’s book, a
piece of paper fell. Said paper turned out to be a copy of the same quiz she had just given and the same already
contained answers.

Ms. Benabese informed the school’s Assistant Supervisor Mrs. Gloria Caneda (Mrs. Caneda) about the incident. Mrs.
Caneda conferred with Assistant Supervisor Encarnacion Koo (Mrs. Koo), who was in charge of the HEKASI area,
and Supervisor LuningningTibi (Ms. Tibi). Mrs. Koo confronted respondent, who had initially denied leaking the test
paper but later on admitted that she gave the test paper to Mrs. TeresitaAnduyan (Mrs. Anduyan), her co-teacher and
the mother of Aileen. Respondent and Mrs. Anduyan were both directed to submit their written statement on the
incident.

According to petitioners, their Investigating Committee had actually decided to terminate respondent and had in fact
prepared a memorandum of termination,10 but respondent allegedly pleaded for a change of punishment in a short
letter dated 5 September 2002, to wit:

Request for change of punishment from termination to suspensionand I am resigning at the end of the school year.

Mrs. Rosalinda M. Torres

On 10 June 2003, respondent filed a complaint for constructive dismissal and illegal suspension with the Labor
Arbiter.

Issue:

Whether or not in this case the school’s act of imposing the penalty of suspension instead of immediate dismissal
from service at the behest of the erring employee, in exchange for the employee’s resignation at the end of the school
year, constitutes constructive dismissal.

Rulings:

Resignation is the voluntary act of an employee who is in a situation where one believes that personal reasons
cannot be sacrificed for the favor of employment, and opts to leave rather than stay employed. It is a formal
pronouncement or relinquishment of an office, with the intention of relinquishing the office accompanied by the act of
relinquishment. As the intent to relinquish must concur with the overt act of relinquishment, the acts of the employee
before and after the alleged resignation must be considered in determining whether, he or she, in fact, intended to
sever his or her employment.19

Respondent had admitted to leaking a copy of the HEKASI 5 special quiz. She reluctantly made the admission and
apologized to Mrs. Koo when the latter confronted her. She admitted during the 28 August 2002 hearing of executing
two (2) contradictory statements. On 30 August 2002, the Investigating Committee found respondent guilty of leaking
a copy of the special quiz. Based on this infraction alone, Chiang Kai Shek College would have been justified to
validly terminate respondent from service. As Associate Justice Antonio T. Carpio emphasized, academic dishonesty
is the worst offense a teacher can make because teachers caught committing academic dishonesty lose their
credibility as educators and cease to be role models for their students. More so that under Chiang Kai Shek College
Faculty Manual, leaking and selling of test questions is classified as a grave offense punishable by
dismissal/termination.

On 5 September 2002, respondent was verbally informed by Mrs. Caneda, Mrs. Carmelita Espino and Ms. Tibi that
she was being dismissed from service. Before the Investigating Committee could formalize respondent’s dismissal,
respondent handwrote a letter requesting that the penalty be lowered from dismissal to suspension in exchange for
respondent’s resignation at the end of the school year.

Given the indications of voluntary resignation, we rule that there is no constructive dismissal in this case. There is
constructive dismissal when there is cessation of work, because continued employment is rendered impossible,
unreasonable or unlikely, as an offer involving a demotion in rank or a diminution in pay and other benefits. Aptly
called a dismissal in disguise or an act amounting to dismissal but made to appear as if it were not, constructive
dismissal may, likewise, exist if an act of clear discrimination, insensibility, or disdain by an employer becomes so
unbearable on the part of the employee that it could foreclose any choice by him except to forego his continued
employment. There was here no discrimination committed by petitioners. While respondent did not tender her
resignation wholeheartedly, circumstances of her own making did not give her any other option. With due process,
she was found to have committed the grave offense of leaking test questions. Dismissal from employment was the
justified equivalent penalty. Having realized that, she asked for, and was granted, not just a deferred imposition of,
but also an acceptable cover for the penalty.
113. Goodyear Philippines Inc. vs. Angus, GR No. 185449, November 12, 2014

Principle: In the absence of an express or implied prohibition against it, collection of both retirement benefits and
separation pay upon severance from employment is allowed. This is grounded on the social justice policy that doubts
should always be resolved in favor of labor rights.

FACTS:

Angus was employed by Goodyear on November 16, 1966 and occupied the position of Secretary to the Manager of
Quality and Technology. In order to maintain the viability of its operations in the midst of economic reversals,
Goodyear implemented a cost saving measure. Consequently, Angus received from the Human Resources Director
of Goodyear a letter terminating her services since her position is redundant and no longer necessary for its effective
operation.

Angus was given by Goodyear her early retirement benefit having served the company for almost 35 years and have
reached the required minimum age of 55 to qualify for early retirement to which Angus replied accepting her
termination however demanding that she be given her separation pay as mandated by law, separate from her early
retirement benefit, arguing that nothing in the company's Retirement Plan under the CBA, the CBA itself or the
Employment Contract prohibits the grant of more than one land of separation pay.

Both the Labor Arbiter and the NLRC ruled in favor of Goodyear, that the grant of both separation pay and retirement
benefit is not allowed under the retirement Plan/CBA. Court of Appeals reversed hence Goodyear appealed to the
Supreme Court.

ISSUE:

Whether or not Angus is entitled to both retirement benefits and separation pay

RULING:

Yes.

In Aquino v. National Labor Relations Commission,33 citing Batangas Laguna Tayabas Bus Company v. Court of
Appeals and University of the East v. Hon. Minister of Labor 35 the Court held that an employee is entitled to recover
both separation pay and retirement benefits in the absence of a specific prohibition in the Retirement Plan or CBA.
Concomitantly, the Court ruled that an employee's right to receive separation pay in addition to retirement benefits
depends upon the provisions of the company's Retirement Plan and/or CBA.
Angus presented the parties' 2001-2004 CBA and upon examination of the same, the Court agrees with her that it
does not contain any restriction on the availment of benefits under the company's Retirement Plan and of separation
pay.

Moreover, the Court agrees with the CA that the amount Angus received from petitioners represented only her
retirement pay and not separation pay.  A cursory reading of petitioners' September 18, 2001 letter notifying Angus of
her termination from employment shows that they granted her early retirement benefits pegged at 47 days' pay per
year of service. In fact, petitioners were even explicit in stating in the said letter that the amount she was to receive
would come from the company's Pension Fund, which, as correctly asserted by Angus, was created to cover
retirement benefit payment of employees. In addition, the document showing a detailed account of Angus' termination
benefits speaks for itself as the same is entitled "Summary of Retirement Pay and other Company Benefits."
It is worthy to mention at this point that retirement benefits and separation pay are not mutually exclusive. Retirement
benefits are a form of reward for an employee's loyalty and service to an employer and are earned under existing
laws, CBAs, employment contracts and company policies.On the other hand, separation pay is that amount which an
employee receives at the time of his severance from employment, designed to provide the employee with the
wherewithal during the period that he is looking for another employment and is recoverable only in instances
enumerated under Articles 283 and 284 of the Labor Code or in illegal dismissal cases when reinstatement is not
feasible.  In the case at bar, Article 283 clearly entitles Angus to separation pay apart from the retirement benefits she
received from petitioners.
18.PRESCRIPTION OF CLAIMS
19.JURISDICTION OF THE LABOR ARBITER

114. Portillo vs. Rudolf Lietz, Inc. et al., G.R. No. 196539, October 10, 2012

FACTS:

Portillo was employed by Lietz Inc. Intheir letter agreement, one of the terms and conditions is upon termination of
Portillo’s employment, she will not engaged, for a period of 3 years, directly or indirectly as employee, manager,
proprietor, or solicitor for yourself or others in a similar or competitive business or the same character of work which
she was employed by Lietz Inc. The agreement is contained in a Goodwill Clause.

The employment relation of the two parties ended however Lietz Inc. learned that Portillo had been hired by Ed Keller
Philippines to head its Pharma Raw Material Department. Ed Keller is a direct competitor of Lietz. As a consequence
of which, Lietz Inc. did not release Portillo’s salaries and commission.

Portillo then filed a complaint with the NLRC. Lietz Inc. admitted liability for Portillo’s money claim however raised the
defense of legal compensation for Portillo’s alleged breached of the Goodwill Clause.

Labor Arbiter ruled in favor of Portillo. CA however ruled that while Portillo is entitled to her claim, legal compensation
is proper relying on Paragraph 4 of Article 217 of the Labor Code that there is "causal connection between [Portillo’s]
monetary claims against [respondents] and the latter’s claim from liquidated damages against the former."

ISSUE:

Whether or not the legal compensation is proper on the premise set by CA that the claim of Portillo and the claim of
Lietz arises from employer-employee relation

RULING:

The Labor Arbiter has no jurisdiction.

Art. 217. Jurisdiction of Labor Arbiters and the Commission. – (a) Except as otherwise provided under this code,
the Arbiters shall have original and exclusive jurisdiction to hear and decide, within thirty (30) calendar days after the
submission of the case by the parties for decision without extension, even in the absence of stenographic notes, the
following case involving all workers, whether agricultural or nonagricultural:

x xxx

4. Claims for actual, moral, exemplary and other forms of damages arising from the employer-employee relations;
(Underscoring supplied)

Evidently, the Court of Appeals is convinced that the claim for liquidated damages emanates from the "Goodwill
Clause of the employment contract and, therefore, is a claim for damages arising from the employer-employee
relations."

We thereafter ruled that the "reasonable causal connection with the employer-employee relationship" is a
requirement not only in employees’ money claims against the employer but is, likewise, a condition when the claimant
is the employer.

There is no causal connection between the petitioner employees’ claim for unpaid wages and the respondent
employers’ claim for damages for the alleged "Goodwill Clause" violation. Portillo’s claim for unpaid salaries did not
have anything to do with her alleged violation of the employment contract as, in fact, her separation from employment
is not "rooted" in the alleged contractual violation. She resigned from her employment. She was not dismissed.
Portillo’s entitlement to the unpaid salaries is not even contested. Indeed, Lietz Inc.’s argument about legal
compensation necessarily admits that it owes the money claimed by Portillo.

The alleged contractual violation did not arise during the existence of the employer-employee relationship. It was a
post-employment matter, a post-employment violation.

In Dai-Chi Electronics Manufacturing Corporation v. Villarama, Jr., which reiterated the San Miguel ruling and allied
jurisprudence, we pronounced that a non-compete clause, as in the "Goodwill Clause" referred to in the present case,
with a stipulation that a violation thereof makes the employee liable to his former employer for liquidated damages,
refers to post-employment relations of the parties, hence falls under the jurisdiction of the civil courts.

We reiterated that Article 217, paragraph 4 does not automatically cover all disputes between an employer and its
employee(s). We noted that the cause of action was within the realm of Civil Law, thus, jurisdiction over the
controversy belongs to the regular courts. At bottom, we considered that the stipulation referred to post-employment
relations of the parties.

It is clear, therefore, that while Portillo’s claim for unpaid salaries is a money claim that arises out of or in connection
with an employer-employee relationship, Lietz Inc.’s claim against Portillo for violation of the goodwill clause is a
money claim based on an act done after the cessation of the employment relationship. And, while the jurisdiction
over Portillo’s claim is vested in the labor arbiter, the jurisdiction over Lietz Inc.’s claim rests on the regular
courts.

As it is, petitioner does not ask for any relief under the Labor Code. It merely seeks to recover damages based on the
parties' contract of employment as redress for respondent's breach thereof. Such cause of action is within the realm
of Civil Law, and jurisdiction over the controversy belongs to the regular courts. More so must this be in the present
case, what with the reality that the stipulation refers to the postemployment relations of the parties.

In the case at bar, the difference in the nature of the credits that one has against the other, conversely, the
nature of the debt one owes another, which difference in turn results in the difference of the forum where the different
credits can be enforced, prevents the application of compensation. Simply, the labor tribunal in an employee’s
claim for unpaid wages is without authority to allow the compensation of such claims against the post-
employment claim of the former employer for breach of a post-employment condition. The labor tribunal
does not have jurisdiction over the civil case of breach of contract.
115. Ace Navigation Co. Inc. et al., vs. Fernandez, G.R. No. 197309, October 10, 2012

Facts:

Seaman Teodorico Fernandez, filed with the NLRC a complaint for disability benefits, with prayer for moral and
exemplary damages, plus attorney’s fees, against Ace Navigation Co., Inc., Vela International Marine Ltd., and/or
Rodolfo Pamintuan.

The petitioners moved to dismiss the complaint, contending that the labor arbiter had no jurisdiction over the dispute.
They argued that exclusive original jurisdiction is with the voluntary arbitrator or panel of voluntary arbitrators,
pursuant to Section 29 of the POEA Standard Employment Contract (POEA-SEC), since the parties are covered by
the AMOSUP-TCC or AMOSUP-VELA (as later cited by the petitioners) collective bargaining agreement (CBA).
Under Section 14 of the CBA, a dispute between a seafarer and the company shall be settled through the grievance
machinery and mandatory voluntary arbitration.

Fernandez opposed the motion. He argued that inasmuch as his complaint involves a money claim, original and
exclusive jurisdiction over the case is vested with the labor arbiter.

Issue:

WON the labor arbiter has jurisdiction over the dispute.

Ruling:

Section 3, Article XIII (on Social Justice and Human Rights) of the Constitution declares:

xxxx

The State shall promote the principle of shared responsibility between workers and employers and the preferential
use of voluntary modes in settling disputes, including conciliation, and shall enforce their mutual compliance therewith
to foster industrial peace.

Article 260 of the Labor Code (Grievance machinery and voluntary arbitration) states:

The parties to a Collective Bargaining Agreement shall include therein provisions that will ensure the mutual
observance of its terms and conditions. They shall establish a machinery for the adjustment and resolution of
grievances arising from the interpretation or implementation of their Collective Bargaining Agreement and those
arising from the interpretation or enforcement of company personnel policies.

Article 261 of the Labor Code (Jurisdiction of Voluntary Arbitrators or panel of Voluntary Arbitrators), on the other
hand, reads in part:

The Voluntary Arbitrator or panel of Voluntary Arbitrators shall have original and exclusive jurisdiction to hear and
decide all unresolved grievances arising from the interpretation or implementation of the Collective Bargaining
Agreement and those arising from the interpretation or enforcement of company personnel policies[.]

Article 262 of the Labor Code (Jurisdiction over other labor disputes) declares:

The Voluntary Arbitrator or panel of Voluntary Arbitrators, upon agreement of the parties, shall also hear and decide
all other labor disputes including unfair labor practices and bargaining deadlocks.

Further, the POEA-SEC, which governs the employment of Filipino seafarers, provides in its Section 29 on Dispute
Settlement Procedures:

In cases of claims and disputes arising from this employment, the parties covered by a collective bargaining
agreement shall submit the claim or dispute to the original and exclusive jurisdiction of the voluntary arbitrator or
panel of voluntary arbitrators.

xxxx

Under the above-quoted constitutional and legal provisions, the voluntary arbitrator or panel of voluntary arbitrators
has original and exclusive jurisdiction over Fernandez’s disability claim. There is no dispute that the claim arose out
of Fernandez’s employment with the petitioners and that their relationship is covered by a CBA. The CBA provides for
a grievance procedure for the resolution of grievances or disputes which occur during the employment relationship
and, like the grievance machinery created under Article 261 of the Labor Code, it is a two-tiered mechanism, with
voluntary arbitration as the last step.

Since the parties used unequivocal language in their CBA for the submission of their disputes to voluntary arbitration
(a condition laid down in Vivero for the recognition of the submission to voluntary arbitration of matters within the
original and exclusive jurisdiction of labor arbiters), we find that the CA committed a reversible error in its ruling; it
disregarded the clear mandate of the CBA between the parties and the POEA-SEC for submission of the present
dispute to voluntary arbitration.

It bears stressing at this point that we are upholding the jurisdiction of the voluntary arbitrator or panel of voluntary
arbitrators over the present dispute, not only because of the clear language of the parties’ CBA on the matter; more
importantly, we so uphold the voluntary arbitrator’s jurisdiction, in recognition of the State’s express preference for
voluntary modes of dispute settlement, such as conciliation and voluntary arbitration as expressed in the Constitution,
the law and the rules.

In closing, we quote with approval a most recent Court pronouncement on the same issue, thus –

It is settled that when the parties have validly agreed on a procedure for resolving grievances and to submit a dispute
to voluntary arbitration then that procedure should be strictly observed.
116. Cosare vs. Broadcom Asia, Inc. GR No. 201298, February 5, 2014, citing 2010 Matling Industrial and
Commercial Corp et al.,

Facts:

The case stems from a complaint for constructive dismissal, illegal suspension and monetary claims filed with the
National Capital Region Arbitration Branch of the National Labor Relations Commission (NLRC) by Cosare against
the respondents. Cosare was named an incorporator of Broadcom and was also its Assistant Vice President for Sales
(AVP for Sales) and Head of the Technical Coordination.

Issue:

WON the controversy falls under the jurisdiction of the LA or the regular courts.

Ruling:

It is the LA, and not the regular courts, which has the original jurisdiction over the subject controversy. An intra-
corporate controversy, which falls within the jurisdiction of regular courts, has been regarded in its broad sense to
pertain to disputes that involve any of the following relationships: (1) between the corporation, partnership or
association and the public; (2) between the corporation, partnership or association and the state in so far as its
franchise, permit or license to operate is concerned; (3) between the corporation, partnership or association and its
stockholders, partners, members or officers; and (4) among the stockholders, partners or associates, themselves.
Settled jurisprudence, however, qualifies that when the dispute involves a charge of illegal dismissal, the action may
fall under the jurisdiction of the LAs upon whose jurisdiction, as a rule, falls termination disputes and claims for
damages arising from employer-employee relations as provided in Article 217 of the Labor Code. Consistent with this
jurisprudence, the mere fact that Cosare was a stockholder and an officer of Broadcom at the time the subject
controversy developed failed to necessarily make the case an intra-corporate dispute.

In Matling Industrial and Commercial Corporation v. Coros, the Court distinguished between a "regular employee"
and a "corporate officer" for purposes of establishing the true nature of a dispute or complaint for illegal dismissal and
determining which body has jurisdiction over it. Succinctly, it was explained that "[t]he determination of whether the
dismissed officer was a regular employee or corporate officer unravels the conundrum" of whether a complaint for
illegal dismissal is cognizable by the LA or by the RTC. "In case of the regular employee, the LA has jurisdiction;
otherwise, the RTC exercises the legal authority to adjudicate.

Applying the foregoing to the present case, the LA had the original jurisdiction over the complaint for illegal dismissal
because Cosare, although an officer of Broadcom for being its AVP for Sales, was not a "corporate officer" as the
term is defined by law. We emphasized in Real v. Sangu Philippines, Inc. the definition of corporate officers for the
purpose of identifying an intra-corporate controversy. Citing Garcia v. Eastern Telecommunications Philippines, Inc.,
we held:

" ‘Corporate officers’ in the context of Presidential Decree No. 902-A are those officers of the corporation who are
given that character by the Corporation Code or by the corporation’s by-laws. There are three specific officers whom
a corporation must have under Section 25 of the Corporation Code. These are the president, secretary and the
treasurer. The number of officers is not limited to these three. A corporation may have such other officers as may be
provided for by its by-laws like, but not limited to, the vice-president, cashier, auditor or general manager. The
number of corporate officers is thus limited by law and by the corporation’s by-laws."

As may be deduced from the foregoing, there are two circumstances which must concur in order for an individual to
be considered a corporate officer, as against an ordinary employee or officer, namely: (1) the creation of the position
is under the corporation’s charter or by-laws; and (2) the election of the officer is by the directors or stockholders. It is
only when the officer claiming to have been illegally dismissed is classified as such corporate officer that the issue is
deemed an intra-corporate dispute which falls within the jurisdiction of the trial courts.

Finally, the mere fact that Cosare was a stockholder of Broadcom at the time of the case’s filing did not necessarily
make the action an intra- corporate controversy. "Not all conflicts between the stockholders and the corporation are
classified as intra-corporate. There are other facts to consider in determining whether the dispute involves corporate
matters as to consider them as intra-corporate controversies." Time and again, the Court has ruled that in
determining the existence of an intra-corporate dispute, the status or relationship of the parties and the nature of the
question that is the subject of the controversy must be taken into account. Considering that the pending dispute
particularly relates to Cosare’s rights and obligations as a regular officer of Broadcom, instead of as a stockholder of
the corporation, the controversy cannot be deemed intra-corporate. This is consistent with the "controversy test"
explained by the Court in Reyes v. Hon. RTC, Br. 142, to wit:

Under the nature of the controversy test, the incidents of that relationship must also be considered for the purpose of
ascertaining whether the controversy itself is intra-corporate. The controversy must not only be rooted in the
existence of an intra-corporate relationship, but must as well pertain to the enforcement of the parties’ correlative
rights and obligations under the Corporation Code and the internal and intra-corporate regulatory rules of the
corporation. If the relationship and its incidents are merely incidental to the controversy or if there will still be conflict
even if the relationship does not exist, then no intra-corporate controversy exists.
117. 2011 Real Amecos Innovations Inc. et al., vs. Lopez, GR No. 178055, July 2, 2014

Facts:

Amecos received a Subpoena from the Office of the City Prosecutor of Quezon City in connection with a complaint
filed by the Social Security System (SSS) for alleged delinquency in the remittance of SSS contributions. Amecos
attributed its failure to remit the SSS contributions to herein respondent Eliza R. Lopez (respondent). Amecos claimed
respondent refused to provide Amecos with her SSS Number and to be deducted her contributions; that on the basis
of the foregoing, Amecos no longer enrolled respondent with the SSS and did not deduct her corresponding
contributions.

Amecos eventually settled its obligations with the SSS. Thereafter, petitioners sent a demand letter to respondent for
P27,791.65 representing her share in the SSS contributions but to no avail. Thus, petitioners filed the instant
Complaint for sum of money and damages against respondent before the Metropolitan Trial Court (MeTC) of
Caloocan City.

Respondent filed her Answer with Motion to Dismiss and averred that the regular courts do not have Jurisdiction over
the instant case as it arose out of their employer-employee relationship.

Issue:

WHETHER THE REGULAR CIVIL COURT AND NOT THE LABOR ARBITER OR THE NATIONAL LABOR
RELATIONS COMMISSION HAS JURISDICTION OVER CLAIM[S] FOR REIMBURSEMENT ARISING FROM
EMPLOYER-EMPLOYEE RELATIONS.

Ruling:

This Court holds that as between the parties, Article 217(a)(4) of the Labor Code is applicable. Said provision
bestows upon the Labor Arbiter original and exclusive jurisdiction over claims for damages arising from employer-
employee relations. The observation that the matter of SSS contributions necessarily flowed from the employer-
employee relationship between the parties is correct; thus, petitioners’ claims should have been referred to the labor
tribunals. In this connection, it is noteworthy to state that “the Labor Arbiter has jurisdiction to award not only the
reliefs provided by labor laws, but also damages governed by the Civil Code.”

At the same time, it cannot be assumed that since the dispute concerns the payment of SSS premiums, petitioners’
claim should be referred to the Social Security Commission (SSC). As far as SSS is concerned, there is no longer a
dispute, with respect to petitioners’ accountability to the System; petitioners already settled their pecuniary obligations
to it.
118. Indophil Textile Mills Inc. vs. Engr. Adviento, GR No. 171212, August 4, 2014

Facts:

Petitioner hired respondent Engr. Salvador Adviento as Civil Engineer to maintain its facilities. Respondent consulted
a physician due to recurring weakness and dizziness. He was diagnosed with Chronic Poly Sinusitis, and thereafter,
with moderate, severe and persistent Allergic Rhinitis. Accordingly, respondent was advised by his doctor to totally
avoid house dust mite and textile dust as it will transmute into health problems.

Distressed, respondent filed a complaint against petitioner with the National Labor Relations Commission for alleged
illegal dismissal and for the payment of backwages, separation pay, actual damages and attorney’s fees.

Subsequently, respondent filed another Complaint with the Regional Trial Court (RTC) alleging that he contracted
such occupational disease by reason of the gross negligence of petitioner to provide him with a safe, healthy and
workable environment.

In reply, petitioner filed a Motion to Dismiss on the ground that: (1) the RTC has no jurisdiction over the subject matter
of the complaint because the same falls under the original and exclusive jurisdiction of the Labor Arbiter (LA) under
Article 217(a)(4) of the Labor Code; and (2) there is another action pending with the Regional Arbitration Branch III of
the NLRC involving the same parties for the same cause.

Issue:

Whether or not the RTC has jurisdiction over the subject matter of respondent’s complaint praying for moral
damages, exemplary damages, compensatory damages, anchored on petitioner’s alleged gross negligence in failing
to provide a safe and healthy working environment for respondent

Ruling:

The RTC has jurisdiction over respondent’s complaint.

We have recognized that not all claims involving employees can be resolved solely by our labor courts, specifically
when the law provides otherwise. For this reason, we have formulated the “reasonable causal connection rule,”
wherein if there is a reasonable causal connection between the claim asserted and the employer-employee relations,
then the case is within the jurisdiction of the labor courts; and in the absence thereof, it is the regular courts that have
jurisdiction.

We ruled in the recent case of Portillo v. Rudolf Lietz, Inc. that not all disputes between an employer and his
employees fall within the jurisdiction of the labor tribunals such that when the claim for damages is grounded on the
"wanton failure and refusal" without just cause of an employee to report for duty despite repeated notices served
upon him of the disapproval of his application for leave of absence, the same falls within the purview of Civil Law. As
early as Singapore Airlines Limited v. Paño, we differentiated between abandonment per se and the manner and
consequent effects of such abandonment and ruled that the first, is a labor case, while the second, is a civil law case:

...petitioner's claim for damages is grounded on the "wanton failure and refusal" without just cause of private
respondent Cruz to report for duty despite repeated notices served upon him of the disapproval of his application for
leave of absence without pay. This, coupled with the further averment that Cruz "maliciously and with bad faith"
violated the terms and conditions of the conversion training course agreement to the damage of petitioner, removes
the present controversy from the coverage of the Labor Code and brings it within the purview of Civil Law.

In the case at bench, we find that such reasonable causal connection is nil. True, the maintenance of a safe and
healthy workplace is ordinarily a subject of labor cases. More, the acts complained of appear to constitute matters
involving employee-employer relations since respondent used to be the Civil Engineer of petitioner. However, it
should be stressed that respondent’s claim for damages is specifically grounded on petitioner’s gross negligence to
provide a safe, healthy and workable environment for its employees - a case of quasi-delict.

In the case at bar, respondent alleges that due to the continued and prolonged exposure to textile dust seriously
inimical to his health, he suffered work-contracted disease which is now irreversible and incurable, and deprived him
of job opportunities. Clearly, injury and damages were allegedly suffered by respondent, an element of quasi-delict.
Secondly, the previous contract of employment between petitioner and respondent cannot be used to counter the
element of “no pre-existing contractual relation” since petitioner’s alleged gross negligence in maintaining a
hazardous work environment cannot be considered a mere breach of such contract of employment since the
negligence is direct, substantive and independent.

It also bears stressing that respondent is not praying for any relief under the Labor Code of the Philippines. He
neither claims for reinstatement nor backwages or separation pay resulting from an illegal termination. The cause of
action herein pertains to the consequence of petitioner’s omission which led to a work-related disease suffered by
respondent, causing harm or damage to his person. Such cause of action is within the realm of Civil Law, and
jurisdiction over the controversy belongs to the regular courts.
Further, it cannot be gainsaid that the claim for damages occurred after the employer-employee relationship of
petitioner and respondent has ceased. Given that respondent no longer demands for any relief under the Labor
Code, Article 217(a)(4) of the Labor Code is inapplicable to the instant case.
20.2011 NLRC RULES OF PROCEDURE OF THE NLRC

119. Aujero vs. Phil Communications Satellite Corp. G.R. No. 193484, January 18, 2012

Facts:

It was in 1967 that the petitioner started working for respondent Philippine Communications Satellite Corporation
(Philcomsat) as an accountant in the latter’s Finance Department. On August 15, 2001 or after 34 years of service,
the petitioner applied for early retirement. His application for retirement was approved, effective September 15, 2001,
entitling him to receive retirement benefits at a rate equivalent to one and a half of his monthly salary for every year of
service. At that time, the petitioner was Philcomsat’s Senior Vice-President with a monthly salary of P274,805.00.

On September 12, 2001, the petitioner executed a Deed of Release and Quitclaim in Philcomsat’s favor, following his
receipt from the latter of a check in the amount of P9,439,327.91.

Almost three (3) years thereafter, the petitioner filed a complaint for unpaid retirement benefits, claiming that the
actual amount of his retirement pay is P14,015,055.00 and the P9,439,327.91 he received from Philcomsat as
supposed settlement for all his claims is unconscionable, which is more than enough reason to declare his quitclaim
as null and void. According to the petitioner, he had no choice but to accept a lesser amount as he was in dire need
thereof and was all set to return to his hometown and he signed the quitclaim despite the considerable deficiency as
no single centavo would be released to him if he did not execute a release and waiver in Philcomsat’s favor.

The petitioner claims that his right to receive the full amount of his retirement benefits, which is equivalent to one and
a half of his monthly salary for every year of service, is provided under the Retirement Plan that Philcomsat created
on January 1, 1977 for the benefit of its employees.

Issue:

Whether the delay in the filing of Philcomsat’s appeal and posting of surety bond is inexcusable.

Whether the quitclaim executed by the petitioner in Philcomsat’s favor is valid, thereby foreclosing his right to institute
any claim against Philcomsat.

Ruling:

Procedural rules may be relaxed to give way to the full determination of a case on its merits. The emerging trend in
our jurisprudence is to afford every party-litigant the amplest opportunity for the proper and just determination of his
cause free from the constraints of technicalities.24 Far from having gravely abused its discretion, the NLRC correctly
prioritized substantial justice over the rigid and stringent application of procedural rules. This, by all means, is not a
case of grave abuse of discretion calling for the issuance of a writ of certiorari.

While the law looks with disfavor upon releases and quitclaims by employees who are inveigled or pressured into
signing them by unscrupulous employers seeking to evade their legal responsibilities, a legitimate waiver
representing a voluntary settlement of a laborer’s claims should be respected by the courts as the law between the
parties.Considering the petitioner’s claim of fraud and bad faith against Philcomsat to be unsubstantiated, this Court
finds the quitclaim in dispute to be legitimate waiver.

While the petitioner bewailed as having been coerced or pressured into signing the release and waiver, his failure to
present evidence renders his allegation self-serving and inutile to invalidate the same. That no portion of his
retirement pay will be released to him or his urgent need for funds does not constitute the pressure or coercion
contemplated by law.
120. Sarona vs. NLRC G.R. No. 185280, January 18, 2012

Facts:

Petitioner, a security guard in Sceptre since April 1976, was asked by Sceptre’s operations manager on June 2003,
to submit a resignation letter as a requirement for an application in Royale and to fill up an employment application
form for the said company. He was then assigned at Highlight Metal Craft Inc. from July 29 to August 8, 2003 and
was later transferred to Wide Wide World Express Inc. On September 2003, he was informed that his assignment at
WWWE Inc. was withdrawn because Royale has been allegedly replaced by another security agency which he later
discovered to be untrue. Nevertheless, he was once again assigned at Highlight Metal sometime in September 2003
and when he reported at Royale’s office on October 1, 2003, he was informed that he would no longer be given any
assignment as instructed by Sceptre’s general manager.

He thus filed a complaint for illegal dismissal. The LA ruled in petitioner’s favor as he found him illegally dismissed
and was not convinced by the respondent’s claim on petitioner’s abandonment. Respondents were ordered to pay
back wages computed from the day he was dismissed up to the promulgation of his decision on May 11, 2005.The
LA also ordered for the payment of separation pay but refused to pierce Royale’s corporate veil.

Respondents appealed to the NLRC claiming that the LA acted with grave abuse of discretion upon ruling on the
illegal dismissal of petitioner. NLRC partially affirmed the

LA’s decision with regard to petitioner’s illegal dismissal and separation pay but modified the amount of backwages
and limited it to only 3 months of his last month salary reducing P95, 600 to P15, 600 since he worked for Royale for
only 1 month and 3 days.

Petitioner did not appeal but raised the validity of LA’s findings on piercing Royale’s corporate personality and
computation of his separation pay and such petition was dismissed by the NLRC. Petitioner elevated NLRC’s
decision to the CA on a petition for certiorari, and the CA disagreed with the NLRC’s decision of not proceeding to
review the evidence for determining if Royale is Sceptre’s alter ego that would warrant the piercing of its corporate
veil.

Issue:

Whether Petitions under Rule 45 should only be confined to questions of law.

Whether NLRC committed grave abuse of discretion when it limited itself to reviewing and deciding only the issues
that were elevated on appeal.

Whether Royale’s corporate fiction should be pierced for the purpose of compelling it to recognize the petitioner’s
length of service with Sceptre and for holding it liable for the benefits that have accrued to him arising from his
employment with Sceptre.

Ruling:

By way of exception to the general rule, this Court will scrutinize the facts if only to rectify the prejudice and injustice
resulting from an incorrect assessment of the evidence presented.

A Rule 45 Petition should be confined to questions of law. Nevertheless, this Court has the power to resolve a
question of fact, such as whether a corporation is a mere alter ego of another entity or whether the corporate fiction
was invoked for fraudulent or malevolent ends, if the findings in assailed decision is not supported by the evidence on
record or based on a misapprehension of facts. The question of whether one corporation is merely an alter ego of
another is purely one of fact. So is the question of whether a corporation is a paper company, a sham or subterfuge
or whether the petitioner adduced the requisite quantum of evidence warranting the piercing of the veil of the
respondent’s corporate personality.

Nevertheless, this Court will not hesitate to deviate from what are clearly procedural guidelines and disturb and strike
down the findings of the CA and those of the labor tribunals if there is a showing that they are unsupported by the
evidence on record or there was a patent misappreciation of facts. Indeed, that the impugned decision of the CA is
consistent with the findings of the labor tribunals does not per se conclusively demonstrate the correctness thereof.

NLRC cannot be accused of grave abuse of discretion because under the NLRC Rules of Procedure, the NLRC shall
limit itself to reviewing and deciding only the issues that were elevated on appeal. The NLRC, while not totally bound
by technical rules of procedure, is not licensed to disregard and violate the implementing rules it implemented. In all
instances, however, technicalities were not allowed to stand in the way of equitably and completely resolving the
labor disputes. Technical rules are not binding in labor cases and are not to be applied strictly if the result would be
detrimental to the working man.

Consistent with the mandates of the Labor Code and prevailing jurisprudence notwithstanding, the NLRC in
adjudicating labor cases choose not to encumber itself with technicalities and limitations consequent to procedural
rules if such will only serve as a hindrance to its duty to decide cases judiciously and in a manner that would put an
end with finality to all existing labor conflicts between the employer and the employee.

The respondents’ scheme reeks of bad faith and fraud and compassionate justice dictates that Royale and Sceptre
be merged as a single entity, compelling Royale to credit and recognize the petitioner’s length of service with
Sceptre. The respondents cannot use the legal fiction of a separate corporate personality for ends subversive of the
policy and purpose behind its creation or which could not have been intended by law to which it owed its being.
121. (a) Salenga et al., vs. Court of Appeals, G.R. No. 174941, February 1, 2012

FACTS:

Petitioner Salenga filed a Complaint for illegal dismissal with a claim for reinstatement and payment of back wages,
benefits, and moral and exemplary damages against respondent Clark Develoment Corporation (CDC) and Colayco.
Earlier, petitioner was informed that the position of head executive assistant, which he held, was declared redundant
by the CDC board of directors. Thus his employment was to be terminated.Respondents CDC were represented by
the Office of the Government Corporate Counsel (OGCC).

Labor ArbiterDarlucio issued a Decision in favor of petitioner Salenga. At the time the above Decision was
rendered, respondent CDC was already under the leadership of Sergio T. Naguiat. He subsequently instructed the
manager of the Corporate and Legal Services Department and concurrent corporate board secretary, not to appeal
the Decision and to so inform the OGCC.

Despite these instructions, two separate appeals were filed before LA Darlucio. One appeal was filed by former CDC
President/CEO RufoColayco. Another appeal was from the OGCC on behalf of respondent CDC and RufoColayco.
The Memorandum of Appeal was verified and certified by HilanaTimbol-Roman, the executive vice president of
respondent CDC. Timbol-Roman and OGCC lawyer Roy Christian Mallari also executed a Joint Affidavit of
Declaration wherein they swore that they were the "respective authorized representative and counsel" of
respondentcorporation. However, the Memorandum of Appeal and the Joint Affidavit of Declaration were not
accompanied by a board resolution from respondent's board of directors authorizing either Timbol-Roman or
Atty. Mallari, or both, to pursue the case or to file the appeal on behalf of respondent.

Petitioner Salana thereafter opposed the two appeals on the grounds that both appellants, respondent CDC — as
allegedly represented by Timbol-Roman and Atty. Mallari — and Rufo Colayco had failed to observe Rule VI,
Sections 4 to 6 of the NLRC Rules of Procedure; and that appellants had not been authorized by respondent's board
of directors to represent the corporation and, thus, they were not the "employer" whom the Rules referred to.

The First Division of the NLRC upheld LA Darlucio's ruling. However, the NLRC held that the award of exemplary and
moral damages were unsubstantiated. Moreover, it also dropped Colayco as a respondent to the case, since LA
Darlucio had failed to provide any ground on which to anchor the former's solidary liability.

Petitioner Salenga thereafter moved for a partial reconsideration of the above-mentioned Decision. He sought the
reinstatement of the award of exemplary and moral damages. He likewise insisted that the NLRC should not have
entertained the appeal. Petitioner's theory revolved on the fact that neither Timbol-Roman nor Colayco was
authorized to represent the corporation, so the corporation itself did not appeal LA Darlucio's Decision. As a
result, that Decision should be considered as final and executory.His second Motion for Reconsideration
reiterated his claim that the NLRC should not have entertained the imperfect appeal, absent a proper verification and
certification against forum-shopping from the duly authorized representative of respondent CDC. Without that
authority, neither could the OGCC act on behalf of the corporation.

The parties underwent several hearings before the NLRC First Division. During these times, petitioner Salenga
demanded from the OGCC to present a board resolution authorizing it or any other person to represent the
corporation in the proceedings. This, the OGCC failed to do.

The First Division of the NLRC held that, absent a board resolution authorizing Timbol-Roman to file the
appeal on behalf of respondent CDC, the appeal was not perfected and was thus a mere scrap of paper. In
other words, the NLRC had no jurisdiction over the appeal filed before it.

Respondent CDC filed with the CA Petition for Certiorari. However, the Petition still lacked a board resolution from the
board of directors of respondent corporation authorizing its then President Angeles to verify and certify the Petition on
behalf of the board.

Petitioner Salenga maintains that the CA had no jurisdiction to entertain CDC’s petition as a remedy for an appeal
that had actually not been filed, absent a board resolution allowing the appeal. He claims the CA acted with grave
abuse of discretion when it failed to dismiss the original and supplemental Petitions despite the lack of a board
resolution authorizing the filing thereof and the absence of a proper verification and certification against non-forum
shopping.

ISSUE:

Was it proper for the NLRC and CA to entertain respondent CDC’s appeal of the decision rendered by LA Darlucio?

RULING:

The NLRC had no jurisdiction to entertain the appeal filed by Timbol-Roman and former CDC CEO Colayco.

LA Darlucio rendered a Decision in favor of petitioner. The OGCC, representing respondent CDC and former CEO
Colayco separately appealed from said Decision. Both alleged that they had filed the proper bond to cover the award
granted by LA Darlucio. It is clear from the NLRC Rules of Procedure that appeals must be verified and certified
against forum-shopping by the parties-in-interest themselves. In the case at bar, the parties-in-interest are petitioner
Salenga, as the employee, and respondent Clark Development Corporation as the employer.

A corporation can only exercise its powers and transact its business through its board of directors and through its
officers and agents when authorized by a board resolution or its bylaws. The power of a corporation to sue and be
sued is exercised by the board of directors. The physical acts of the corporation, like the signing of documents, can
be performed only by natural persons duly authorized for the purpose by corporate bylaws or by a specific act of the
board. The purpose of verification is to secure an assurance that the allegations in the pleading are true and correct
and have been filed in good faith.
Thus, we agree with petitioner that, absent the requisite board resolution, neither Timbol-Roman nor Atty.
Mallari, who signed the Memorandum of Appeal and Joint Affidavit of Declaration allegedly on behalf of
respondent corporation, may be considered as the "appellant" and "employer" referred to by Rule VI, Sections
4 to 6 of the NLRC Rules of Procedure, which state:

SECTION 4. REQUISITES FOR PERFECTION OF APPEAL. — (a) The Appeal shall be filed
within the reglementary period as provided in Section 1 of this Rule; shall be verified by
appellant himself in accordance with Section 4, Rule 7 of the Rules of Court, with proof of
payment of the required appeal fee and the posting of a cash or surety bond as provided in
Section 6 of this Rule; shall be accompanied by memorandum of appeal in three (3) legibly
typewritten copies which shall state the grounds relied upon and the arguments in support
thereof; the relief prayed for; and a statement of the date when the appellant received the
appealed decision, resolution or order and a certificate of non-forum shopping with proof of
service on the other party of such appeal. A mere notice of appeal without complying with the
other requisites aforestated shall not stop the running of the period for perfecting an appeal.

Xxx

The OGCC failed to produce any valid authorization from the board of directors despite petitioner Salenga's repeated
demands. In fact, many of its pleadings, representations, and submissions lacked board authorization.

We cannot agree with the OGCC's attempt to downplay this procedural flaw by claiming that, as the statutorily
assigned counsel for GOCCs, it does not need such authorization. In Constantino-David v. Pangandaman-Gania, we
exhaustively explained why it was necessary for government agencies or instrumentalities to execute the verification
and the certification against forum-shopping through their duly authorized representatives. We ruled thereon as
follows:

But the rule is different where the OSG is acting as counsel of record for a government agency.
For in such a case it becomes necessary to determine whether the petitioning government
body has authorized the filing of the petition and is espousing the same stand
propounded by the OSG. Verily, it is not improbable for government agencies to adopt a
stand different from the position of the OSG since they weigh not just legal
considerations but policy repercussions as well. They have their respective mandates for
which they are to be held accountable, and the prerogative to determine whether further
resort to a higher court is desirable and indispensable under the circumstances.

The verification of a pleading, if signed by the proper officials of the client agency itself,
would fittingly serve the purpose of attesting that the allegations in the pleading are true
and correct and not the product of the imagination or a matter of speculation, and that the
pleading is filed in good faith. Of course, the OSG may opt to file its own petition as a
"People's Tribune" but the representation would not be for a client office but for its own perceived
best interest of the State.

The case of Commissioner of Internal Revenue v. S.C. Johnson and Son, Inc., is not also a
precedent that may be invoked at all times to allow the OSG to sign the certificate of non-forum
shopping in place of the real party-in-interest. The ruling therein mentions merely that the
certification of non-forum shopping executed by the OSG constitutes substantial compliance with
the rule since "the OSG is the only lawyer for the petitioner, which is a government agency
mandated under Section 35, Chapter 12, Title III, Book IV, of the 1987 Administrative Code
(Reiterated under Memorandum Circular No. 152 dated May 17, 1992) to be represented only by
the Solicitor General."

By its very nature, "substantial compliance" is actually inadequate observance of the


requirements of a rule or regulation which are waived under equitable circumstances to facilitate
the administration of justice there being no damage or injury caused by such flawed compliance.
This concept is expressed in the statement "the rigidity of a previous doctrine was thus subjected
to an inroad under the concept of substantial compliance." In every inquiry on whether to accept
"substantial compliance," the focus is always on the presence of equitable conditions to
administer justice effectively and efficiently without damage or injury to the spirit of the legal
obligation.

Xxxxxxxxx

The fact that the OSG under the 1987 Administrative Code is the only lawyer for a
government agency wanting to file a petition, or complaint for that matter, does not
operate per se to vest the OSG with the authority to execute in its name the certificate of
non-forum shopping for a client office. For, in many instances, client agencies of the OSG
have legal departments which at times inadvertently take legal matters requiring court
representation into their own hands without the intervention of the OSG. Consequently,
the OSG would have no personal knowledge of the history of a particular case so as to
adequately execute the certificate of non-forum shopping; and even if the OSG does have
the relevant information, the courts on the other hand would have no way of ascertaining
the accuracy of the OSG's assertion without precise references in the record of the case.
Thus, unless equitable circumstances which are manifest from the record of a case
prevail, it becomes necessary for the concerned government agency or its authorized
representatives to certify for non-forum shopping if only to be sure that no other similar
case or incident is pending before any other court. ASTIED
We recognize the occasions when the OSG has difficulty in securing the attention and signatures
of officials in charge of government offices for the verification and certificate of non-forum
shopping of an initiatory pleading. This predicament is especially true where the period for filing
such pleading is non-extendible or can no longer be further extended for reasons of public
interest such as in applications for the writ of habeas corpus, in election cases or where sensitive
issues are involved. This quandary is more pronounced where public officials have stations
outside Metro Manila.

But this difficult fact of life within the OSG, equitable as it may seem, does not excuse it from
wantonly executing by itself the verification and certificate of non-forum shopping. If the OSG is
compelled by circumstances to verify and certify the pleading in behalf of a client agency, the
OSG should at least endeavor to inform the courts of its reasons for doing so, beyond
instinctively citing City Warden of the Manila City Jail v. Estrella and Commissioner of Internal
Revenue v. S.C. Johnson and Son, Inc.

Henceforth, to be able to verify and certify an initiatory pleading for non-forum shopping
when acting as counsel of record for a client agency, the OSG must (a) allege under oath
the circumstances that make signatures of the concerned officials impossible to obtain
within the period for filing the initiatory pleading; (b) append to the petition or complaint
such authentic document to prove that the party-petitioner or complainant authorized the
filing of the petition or complaint and understood and adopted the allegations set forth
therein, and an affirmation that no action or claim involving the same issues has been
filed or commenced in any court, tribunal or quasi-judicial agency; and, (c) undertake to
inform the court promptly and reasonably of any change in the stance of the client
agency.

Anent the document that may be annexed to a petition or complaint under letter (b)
hereof, the letter-endorsement of the client agency to the OSG, or other correspondence
to prove that the subject-matter of the initiatory pleading had been previously discussed
between the OSG and its client, is satisfactory evidence of the facts under letter (b) above.
In this exceptional situation where the OSG signs the verification and certificate of non-
forum shopping, the court reserves the authority to determine the sufficiency of the
OSG's action as measured by the equitable considerations discussed herein. (Emphasis
ours, italics provided)

The ruling cited above may have pertained only to the Office of the Solicitor General's representation of government
agencies and instrumentalities, but we see no reason why this doctrine cannot be applied to the case at bar insofar
as the OGCC is concerned.

While in previous decisions we have excused transgressions of these rules, it has always been in the context of
upholding justice and fairness under exceptional circumstances. In this case, though, respondent failed to provide
any iota of rhyme or reason to compel us to relax these requirements. Instead, what is clear to us is that the so-called
appeal was done against the instructions of then President/CEO Naguiat not to file an appeal. Timbol-Roman, who
signed the Verification and the Certification against forum-shopping, was not even an authorized representative of the
corporation. The OGCC was equally remiss in its duty. It ought to have advised respondent corporationthe proper
procedure for pursuing an appeal. Instead, it maintained the appeal and failed to present any valid authorization from
respondent corporation even after petitioner had questioned OGCC's authority all throughout the proceedings. Thus,
it is evident that the appeal was made in bad faith.

Time and again, we have said that the perfection of an appeal within the period prescribed by law is jurisdictional, and
the lapse of the appeal period deprives the courts of jurisdiction to alter the final judgment. Thus, there is no other
recourse but to respect the findings and ruling of the labor arbiter. Clearly, therefore, the CA committed grave abuse
of discretion in entertaining the Petition filed before it after the NLRC had dismissed the case based on lack of
jurisdiction. The assailed CA Decision did not even resolve petitioner Salenga's consistent and persistent claim that
the NLRC should not have taken cognizance of the appeal in the first place, absent a board resolution. Thus, LA
Darlucio's Decision with respect to the liability of the corporation still stands.
121. (b) Lockheed Detective & Watchman Agency, G.R. No. 185918, April 18, 2012

FACTS:

Lockheed Detective and Watchman Agency, Inc. entered into a contract for security services with University of the
Philippines. Separate claims were filed against them by security guards for payment of underpaid wages, among
others.

Lockheed and UP as job contractor and principal, respectively, were held solidarily liable for all the claims. Further,
UP, as third-party respondent, was held liable to Lockheed, as third-party complainant and cross-claimant, for the
unpaid legislated salary increases of the security guards.

A writ of execution was later issued, with NLRC directing that the satisfaction of the judgment award in favor of
Lockheed will be only against the funds of UP which are not identified as public funds. Consequently, Lockheed filed
and was granted its motion for the issuance of an alias writ of execution. A Notice of Garnishment was issued to
Philippine National Bank UP Diliman Branch for the satisfaction of the award.

UP filed an Urgent Motion to Quash Garnishment. UP contended that the funds being subjected to garnishment at
PNB are government/public funds. The funds are under the name of UP System Trust Receipts, earmarked for
Student Guaranty Deposit, Scholarship Fund, Student Fund, etc. UP argued that as public funds, the subject PNB
account cannot be disbursed except pursuant to an appropriation required by law. The Labor Arbiter, however,
dismissed the urgent motion for lack of merit. The amount was withdrawn by the sheriff from UP's PNB account.

Upon UP’s Petition for Certiorari, the CA did not depart from its findings that the funds covered in the savings account
sought to be garnished do not fall within the classification of public funds. However, it reconsidered UP’s Motion to
Quash Garnishment, in light of the ruling in the case of National Electrification Administration v. Moraleswhich
mandates that all money claims against the government must first be filed with the Commission on Audit (COA).

Lockheed contends that UP has its own separate and distinct juridical entity from the national government and has its
own charter. Thus, it can be sued and be held liable. Lastly, Lockheed contends that UP cannot anymore seek the
quashal of the writ of execution and notice of garnishment as they are already fait accompli.

ISSUE:

Was the garnishment proper?

RULING:

We agree with UP that there was no point for Lockheed in discussing the doctrine of state immunity from suit as this
was never an issue in this case. Clearly, UP consented to be sued when it participated in the proceedings below.
What UP questions is the hasty garnishment of its funds in its PNB account.

This Court finds that the CA correctly applied the NEA case. Like NEA, UP is a juridical personality separate and
distinct from the government and has the capacity to sue and be sued. Thus, also like NEA, it cannot evade
execution, and its funds may be subject to garnishment or levy. However, before execution may be had, a
claim for payment of the judgment award must first be filed with the COA. Under Commonwealth Act No. 327,
as amended by Section 26 of P.D. No. 1445, it is the COA which has primary jurisdiction to examine, audit and settle
"all debts and claims of any sort" due from or owing the Government or any of its subdivisions, agencies and
instrumentalities, including government-owned or controlled corporations and their subsidiaries. With respect to
money claims arising from the implementation of Republic Act No. 6758, their allowance or disallowance is for COA
to decide, subject only to the remedy of appeal by petition for certiorari to this Court.

We cannot subscribe to Lockheed's argument that NEA is not similarly situated with UP because the COA's
jurisdiction over the latter is only on post-audit basis. A reading of the pertinent Commonwealth Act provision clearly
shows that it does not make any distinction as to which of the government subdivisions, agencies and
instrumentalities, including government-owned or controlled corporations and their subsidiaries whose debts should
be filed before the COA.

As to the fait accompli argument of Lockheed, contrary to its claim that there is nothing that can be done since the
funds of UP had already been garnished, since the garnishment was erroneously carried out and did not go through
the proper procedure (the filing of a claim with the COA), UP is entitled to reimbursement of the garnished funds plus
interest of 6% per annum, to be computed from the time of judicial demand to be reckoned from the time UP filed a
petition for certiorari before the CA which occurred right after the withdrawal of the garnished funds from PNB.
122. 3rd Alert Security & Detective Services Inc. vs. Navia, G.r. No. 200653, June 13, 2012

FACTS:

The labor arbiter, NLRC and CA were unanimous in deciding that Navia was illegally dismissed by 3 rd Alert Security
and Detective Services. 3rd Alert’s appeals and motions for reconsideration with NLRC and CA were all denied.

In the meantime, the NLRC issued an Entry of Judgment certifying that its resolution declaring Navia’s dismissal
illegal has become final and executory. Thus, Navia filed with the labor arbiter an ex-parte motion for recomputation
of back wages and an ex-parte motion for execution based on the recomputed back wages. The labor arbiter issued
a writ of execution to enforce the recomputed monetary awards.

3rd Alert appealed the recomputed amount stated in the writ of execution to the NLRC. 3rd Alert also alleged that the
writ was issued with grave abuse of discretion since there was already a notice of reinstatement sent to Navia.

The NLRC dismissed the appeal, ruling that 3rd Alert is guilty of bad faith since there was no earnest effort to
reinstate Navia. The NLRC also ruled that there was no notice or reinstatement sent to Navia's counsel. The CA also
found the petition without merit because Navia had not been reinstated either physically or in the payroll.

ISSUE:

Was the writ of execution issued with grave abuse of discretion?

RULING:

Article 223 of the Labor Code provides that in case there is an order of reinstatement, the employer must admit the
dismissed employee under the same terms and conditions, or merely reinstate the employee in the payroll. The order
shall be immediately executory. Thus, 3rd Alert cannot escape liability by simply invoking that Navia did not report
for work. The law states that the employer must still reinstate the employee in the payroll. Where reinstatement is no
longer viable as an option, separation pay equivalent to one (1) month salary for every year of service could be
awarded as an alternative.

Since the proceedings below indicate that 3rd Alert failed to adduce additional evidence to show that it tried to
reinstate Navia, either physically or in the payroll, we adopt as correct the finding that there was no earnest effort to
reinstate Navia. The CA was correct in affirming the judgment of the NLRC in this regard.To reiterate, no indication
exists showing that 3rd Alert exerted any efforts to reinstate Navia; worse, 3rd Alert's lame excuse of having sent a
notice of reinstatement to a certain "Biznar" only compounded the intent to mislead the courts.

Also, the main issue of this case, finding Navia to have been illegally dismissed, has already attained finality.
Litigation must end and terminate sometime and somewhere, and it is essential for an effective and efficient
administration of justice that, once a judgment has become final, the winning party be not deprived of the fruits of the
verdict. The order is to reinstate Navia; sadly, the mere execution of this judgment has to even reach the highest
court of the land, thereby frustrating the entire judicial process.

"It is settled that in actions for recovery of wages or where an employee was forced to litigate and incur expenses to
protect his right and interest, he is entitled to an award of attorney's fees." Navia, having been compelled to litigate
due to 3rd Alert's failure to satisfy his valid claim, is also entitled to attorney's fees of ten percent (10%) of the total
award at the time of actual payment, following prevailing jurisprudence.
123. Estate of Dulay vs. Aboitiz Jebsen Maritime, Inc. G.R. No. 172642, June 13, 2012

Facts:

Nelson R. Dulay (Nelson, for brevity) was employed by [herein respondent] General Charterers Inc. (GCI), a
subsidiary of co-petitioner [herein co-respondent] Aboitiz Jebsen Maritime Inc. since 1986. He initially worked as an
ordinary seaman and later as bosun on a contractual basis. From September 3, 1999 up to July 19, 2000, Nelson
was detailed in petitioners’ vessel, the MV Kickapoo Belle.

On August 13, 2000, or 25 days after the completion of his employment contract, Nelson died due to acute renal
failure secondary to septicemia. At the time of his death, Nelson was a bona fide member of the Associated Marine
Officers and Seaman’s Union of the Philippines (AMOSUP), GCI’s collective bargaining agent. Nelson’s widow,
Merridy Jane, thereafter claimed for death benefits through the grievance procedure of the Collective Bargaining
Agreement (CBA) between AMOSUP and GCI. However, on January 29, 2001, the grievance procedure was
"declared deadlocked" as petitioners refused to grant the benefits sought by the widow.

On March 5, 2001, Merridy Jane filed a complaint with the NLRC Sub-Regional Arbitration Board in General Santos
City against GCI for death and medical benefits and damages.

Herein respondents, on the other hand, asserted that the NLRC had no jurisdiction over the action on account of the
absence of employer-employee relationship between GCI and Nelson at the time of the latter’s death.

Issue:

Whether or not the CA committed error in ruling that the Labor Arbiter has no jurisdiction over the case.

Ruling:

It is true that R.A. 8042 is a special law governing overseas Filipino workers. However, a careful reading of this
special law would readily show that there is no specific provision thereunder which provides for jurisdiction over
disputes or unresolved grievances regarding the interpretation or implementation of a CBA. Section 10 of R.A. 8042,
which is cited by petitioner, simply speaks, in general, of "claims arising out of an employer-employee relationship or
by virtue of any law or contract involving Filipino workers for overseas deployment including claims for actual, moral,
exemplary and other forms of damages." On the other hand, Articles 217(c) and 261 of the Labor Code are very
specific in stating that voluntary arbitrators have jurisdiction over cases arising from the interpretation or
implementation of collective bargaining agreements. Stated differently, the instant case involves a situation where the
special statute (R.A. 8042) refers to a subject in general, which the general statute (Labor Code) treats in particular.5
In the present case, the basic issue raised by Merridy Jane in her complaint filed with the NLRC is: which provision of
the subject CBA applies insofar as death benefits due to the heirs of Nelson are concerned. The Court agrees with
the CA in holding that this issue clearly involves the interpretation or implementation of the said CBA. Thus, the
specific or special provisions of the Labor Code govern.
124. Prudential Guarantee and Assurance Employee labor Union vs. NLRC, et al., G.R. No. 185335,

Facts:

June 13, 2012Vallota commenced his employment with respondent Prudential Guarantee and Assurance, Inc. (PGAI)
on May 16, 1995 as a Junior Programmer assigned to the Electronic Data Processing (EDP) Department. He
reported directly to Gerald Dy Victory, then head of the EDP, until his replacement by respondent Jocelyn Retizos
(Retizos)sometime in 1997.

In August of 2005, Vallota was elected to the Board of Directors of the Union.

On November 11, 2005, PGAI’s Human Resource Manager, Atty. Joaquin R. Rillo (Atty. Rillo), invited Union
President, Mike Apostol (Apostol) to his office. Atty. Rillo informed Apostol that PGAI was going to conduct an on-the-
spot security check in the Information and Technology (IT) Department. Atty. Rillo also requested that Union
representatives witness the inspection to which Apostol agreed.

The inspection team proceeded to the IT Department, and the EDP head, through PGAI network administrator
Angelo Gutierrez (Gutierrez), initiated the spot check of IT Department computers, beginning with the one assigned to
Vallota. After exploring the contents of all the folders and subfolders in the "My Documents" folder, Gutierrez
apparently did not find anything unusual with Vallota’s computer and said "Wala naman, saan dito?" Retizos insisted,
"Nandyan yan," and took over the inspection until she found a folder named "MAA." She then exclaimed, "Heto oh!
Ano to? Bakit may MAA dito?" Retizos asked Vallota, "Are you working for MAA?" Vallota replied, "Hindi po, MAA
mutual life po yan na makikita po sa internet." Gutierrez saved a copy of the contents of the MAA folder in a floppy
disk.3

Sensing that Vallota was being singled out, Apostol insisted that all the computers in the IT Department, including
that of Retizos, be also subjected to a spot security check. Later, at Retizos’ office, and in the presence of Atty. Rillo,
Vallota was informed that Retizos and Atty. Rillo would print the files found in his computer under the folder "MAA."
Vallota did not object. After the files were printed, Vallota and the Union Secretary were asked to sign each page of
the printout. Vallota, however, was not given a copy of the printed file.

On November 14, 2005, Vallota received a memorandum4 directing him to explain within 72 hours why highly
confidential files were stored in his computer. Vallota responded in writing on November 21, 2005.6

On December 21, 2005, Vallota was given a notice of termination of his employment effective January 10, 2006 on
the ground of loss of trust and confidence. The decision (AC-05-02) was embodied in a memorandum12 dated
December 21, 2005.

Thus, the petitioners filed a complaint for illegal dismissal with claims for full backwages, moral and exemplary
damages, and attorney’s fees.

Issue:

WHETHER OR NOT THE HONORABLE COURT OF APPEALS COMMITTED GRAVE ABUSE OF DISCRETION IN
GIVING LIBERALITY TO PRIVATE RESPONDENTS['] FOUR BLATANT VIOLATIONS OF THE NLRC RULES OF
PROCEDURE

Ruling:

The allegation of grave abuse of discretion is misplaced, as this is an issue appropriate for a petition for certiorari
under Rule 65, not a petition for review on certiorari under Rule 45. There is no question that grave abuse of
discretion or errors of jurisdiction may be corrected only by the special civil action of certiorari. Such special remedy
does not avail in instances of error of judgment which can be corrected by appeal or by a petition for review. Because
the petitioners availed of the remedy under Rule 45, recourse to Rule 65 cannot be allowed either as an add-on or as
a substitute for appeal.
125. Radio Philippines Network Inc et al., vs. Yap et al., G.R. No. 187713, August 1, 2012

Facts:

Respondents were employees of RPN and former members of the RPN Employees Union (RPNEU) whom the
RPNEU's Grievance and Investigation Committee recommended for expulsion from the union. Thereafter, RPN
notified the respondents that their employment would be terminated, whereupon the respondents filed with the LA a
complaint for illegal dismissal and non-payment of benefits.

LA rendered a decision ordering the reinstatement of the respondents with payment of backwages and full benefits
and without loss of seniority rights.

Petitioner submitted a Manifestation that it has complied with the reinstatement of the complainants. RPN General
Manager Linao informed respondents that they had been reinstated, but only in the payroll. Four (4) days later, the
respondents returned to RPN to collect their salaries, it being a payday; but they were barred entry and the guards
manhandled them, pulled them by the hair and arms and pushed them back to the street.

LA then cited the petitioners for indirect contempt for "committing disobedience to lawful order." When brought before
the NLRC, the body dismissed the same and also denied the petitioners' motion for reconsideration. It was appealed
to the CA which also dismissed the petition as well as the motion for reconsideration, for failure to submit the
documents it enumerated in its Resolution.

Issue:

Whether or not thepayroll reinstatement of the employees availed by RPN is proper.

Ruling:

Yes. The manner of reinstating a dismissed employee in the payroll generally involves an exercise of management
prerogative.

In the case of Pioneer Texturizing Corp. v. NLRC, it was held that an order reinstating a dismissed employee is
immediately self-executory without need of a writ of execution, in accordance with the third paragraph of Article 223
of the Labor Code. The article states that the employee entitled to reinstatement "shall either be admitted back to
work under the same terms and conditions prevailing prior to his dismissal or separation or, at the option of the
employer, merely reinstated in the payroll." Thus, even if the employee is able and raring to return to work, the option
of payroll reinstatement belongs to the employer. 

The new NLRC Rules of Procedure, which took effect on January 7, 2006, now requires the employer to submit a
report of compliance within ten (10) calendar days from receipt of the LA's decision, disobedience to which clearly
denotes a refusal to reinstate. The employee need no longer file a motion for issuance of a writ of execution, since
the LA shall thereafter motu proprio issue the writ. With the new rules, there will be no difficulty in determining the
employer's intransigence in immediately complying with the order.

It has been held that in case of strained relations or non-availability of positions, the employer is given the option to
reinstate the employee merely in the payroll, precisely in order to avoid the intolerable presence in the workplace of
the unwanted employee. The Court explained in Maranaw Hotel Resort Corporation v. NLRC, thus:

This option [to reinstate a dismissed employee in the payroll] is based on practical considerations. The employer may
insist that the dismissal of the employee was for a just and valid cause and the latter's presence within its premises is
intolerable by any standard; or such presence would be inimical to its interest or would demoralize the co-employees.
Thus, while payroll reinstatement would in fact be unacceptable because it sanctions the payment of salaries to one
not rendering service, it may still be the lesser evil compared to the intolerable presence in the workplace of an
unwanted employee.

The circumstances of the present case have more than amply shown that the physical restoration of the respondents
to their former positions would be impractical and would hardly promote the best interest of both parties.
Respondents have accused the petitioners of being directly complicit in the plot to expel them from the union and to
terminate their employment, while petitioners have charged the respondents with trying to sabotage the peace of the
workplace in "furthering their dispute with the union." The resentment and enmity between the parties have so
strained their relationship and even provoked antipathy and antagonism, as amply borne out by the physical clashes
that had ensued every time the respondents attempted to enter the RPN compound, that respondents' presence in
the workplace will not only be distracting but even disruptive, to say the least.

The proposal to pay the respondents' salaries through ATM cards, now a wide practice cannot be said to be
prejudicial or oppressive since it would not entail any unusual effort by the respondents to collect their money. As to
the respondents' demand to be paid their salaries on the 15th and 30th of the month along with the other employees,
instead of on the 5th and 20th days, petitioners reason that the salaries must be staggered due to RPN's erratic cash
flows. The law only requires that the fortnightly intervals be observed.
126. Ace Navigation Co., Inc. vs. Fernandez, et al., G.r. No. 197309, October 10, 2012

Facts:

Seaman Teodorico Fernandez, assisted by his wife Glenita, filed with the NLRC a complaint for disability benefits.

The petitioners moved to dismiss the complaint, contending that the labor arbiter had no jurisdiction over the dispute.
They argued that exclusive original jurisdiction is with the voluntary arbitrator or panel of voluntary arbitrators,
pursuant to Section 29 of the POEA Standard Employment Contract, since the parties are covered by the AMOSUP-
TCC or AMOSUP-VELA. Under Section 14 of the CBA, a dispute between a seafarer and the company shall be
settled through the grievance machinery and mandatory voluntary arbitration.

The LA denied the motion to dismiss, holding that under Section 10 of RA 8042, the labor arbiter has original and
exclusive jurisdiction over money claims arising out of an employer-employee relationship or by virtue of any law or
contract, notwithstanding any provision of law to the contrary. NLRC agreed with the LA that the case involves a
money claim and is within the jurisdiction of the labor arbiter.

CA also rejected the petitioners' submission and stressed that the jurisdiction of voluntary arbitrators is limited to the
seafarers' claims which do not fall within the labor arbiter's original and exclusive jurisdiction or even in cases where
the labor arbiter has jurisdiction, the parties have agreed in unmistakable terms (through their CBA) to submit the
case to voluntary arbitration.

Issue:
Who has the original and exclusive jurisdiction over Fernandez's disability claim — the labor arbiter under Section 10
of R.A. No. 8042, as amended, or the voluntary arbitration mechanism as prescribed in the parties' CBA and the
POEA-SEC?

Ruling:
The answer lies in the State's labor relations policy laid down in the Constitution and fleshed out in the enabling
statute, the Labor Code. Section 3, Article XIII (on Social Justice and Human Rights) of the Constitution declares:

The State shall promote the principle of shared responsibility between workers and employers and the preferential
use of voluntary modes in settling disputes, including conciliation, and shall enforce their mutual compliance therewith
to foster industrial peace.

Article 260 of the Labor Code (Grievance machinery and voluntary arbitration) states:
The parties to a Collective Bargaining Agreement shall include therein provisions that will ensure the mutual
observance of its terms and conditions. They shall establish a machinery for the adjustment and resolution of
grievances arising from the interpretation or implementation of their Collective Bargaining Agreement and those
arising from the interpretation or enforcement of company personnel policies.

Article 261 of the Labor Code (Jurisdiction of Voluntary Arbitrators or panel of Voluntary Arbitrators), on the other
hand, reads in part:

The Voluntary Arbitrator or panel of Voluntary Arbitrators shall have original and exclusive jurisdiction to hear and
decide all unresolved grievances arising from the interpretation or implementation of the Collective Bargaining
Agreement and those arising from the interpretation or enforcement of company personnel policies[.]

Article 262 of the Labor Code (Jurisdiction over other labor disputes) declares:

The Voluntary Arbitrator or panel of Voluntary Arbitrators, upon agreement of the parties, shall also hear and decide
all other labor disputes including unfair labor practices and bargaining deadlocks.

Further, the POEA-SEC, which governs the employment of Filipino seafarers, provides in its Section 29 on Dispute
Settlement Procedures:

In cases of claims and disputes arising from this employment, the parties covered by a collective bargaining
agreement shall submit the claim or dispute to the original and exclusive jurisdiction of the voluntary
arbitrator or panel of voluntary arbitrators. If the parties are not covered by a collective bargaining agreement, the
parties may at their option submit the claim or dispute to either the original and exclusive jurisdiction of the National
Labor Relations Commission (NLRC), pursuant to Republic Act (RA) 8042 otherwise known as the Migrant Workers
and Overseas Filipinos Act of 1995 or to the original and exclusive jurisdiction of the voluntary arbitrator or panel of
voluntary arbitrators. If there is no provision as to the voluntary arbitrators to be appointed by the parties, the same
shall be appointed from the accredited voluntary arbitrators of the National Conciliation and Mediation Board of the
Department of Labor and Employment. [emphasis ours] 

Under the above-quoted constitutional and legal provisions, the voluntary arbitrator or panel of voluntary arbitrators
has original and exclusive jurisdiction over Fernandez's disability claim. There is no dispute that the claim arose out
of Fernandez's employment with the petitioners and that their relationship is covered by a CBA — the AMOSUP/TCC
or the AMOSUP-VELA CBA. The CBA provides for a grievance procedure for the resolution of grievances or disputes
which occur during the employment relationship and, like the grievance machinery created under Article 261 of the
Labor Code, it is a two-tiered mechanism, with voluntary arbitration as the last step.

Contrary to the CA's reading of the CBA's Article 14, there is unequivocal or unmistakable language in the agreement
which mandatorily requires the parties to submit to the grievance procedure any dispute or cause of action they may
have against each other. The relevant provisions of the CBA state:
14.6Any Dispute, grievance, or misunderstanding concerning any ruling, practice, wages or working
conditions in the COMPANY or any breach of the Contract of Employment, or any dispute arising from the
meaning or application of the provisions of this Agreement or a claim of violation thereof or any complaint or
cause of action that any such Seaman may have against the COMPANY, as well as complaints which the
COMPANY may have against such Seaman shall be brought to the attention of the GRIEVANCE
RESOLUTION COMMITTEE before either party takes any action, legal or otherwise. Bringing such a dispute
to the Grievance Resolution Committee shall be unwaivable prerequisite or condition precedent for bringing
any action, legal or otherwise, in any forum and the failure to so refer the dispute shall bar any and all legal
or other actions.

14.7a)If by reason of the nature of the Dispute, the parties are unable to amicably settle the dispute, either
party may refer the case to a MANDATORY ARBITRATION COMMITTEE. The MANDATORY ARBITRATION
COMMITTEE shall consist of one representative to be designated by the UNION, and one representative to be
designated by the COMPANY and a third member who shall act as Chairman and shall be nominated by mutual
choice of the parties. . . .

h)Referral of all unresolved disputes from the Grievance Resolution Committee to the Mandatory Arbitration
Committee shall be unwaivable prerequisite or condition precedent for bringing any action, claim, or cause
of action, legal or otherwise, before any court, tribunal, or panel in any jurisdiction. The failure by a party or
seaman to so refer and avail oneself to the dispute resolution mechanism contained in this action shall bar
any legal or other action. All parties expressly agree that the orderly resolution of all claims in the prescribed
manner served the interests of reaching settlements or claims in an orderly and uniform manner, as well as
preserving peaceful and harmonious labor relations between seaman, the Union, and the Company. 

What might have caused the CA to miss the clear intent of the parties in prescribing a grievance procedure in their
CBA is, as the petitioners' have intimated, the use of the auxiliary verb "may" in Article 14.7 (a) of the CBA which, to
reiterate, provides that "[i]f by reason of the nature of the Dispute, the parties are unable to amicably settle
the dispute, either party may refer the case to a MANDATORY ARBITRATION COMMITTEE."

While the CA did not qualify its reading of the subject provision of the CBA, it is reasonable to conclude that it viewed
as optional the referral of a dispute to the mandatory arbitration committee when the parties are unable to amicably
settle the dispute.

We find this a strained interpretation of the CBA provision. The CA read the provision separately, or in isolation of the
other sections of Article 14, especially 14.7 (h), which, in clear, explicit language, states that the "referral of all
unresolved disputes from the Grievance Resolution Committee to the Mandatory Arbitration Committee shall
be unwaivable prerequisite or condition precedent for bringing any action, claim, or cause of action, legal or
otherwise, before any court, tribunal, or panel in any jurisdiction" and that the failure by a party or seaman to
so refer the dispute to the prescribed dispute resolution mechanism shall bar any legal or other action.

Read in its entirety, the CBA's Article 14 (Grievance Procedure) unmistakably reflects the parties' agreement to
submit any unresolved dispute at the grievance resolution stage to mandatory voluntary arbitration under Article 14.7
(h) of the CBA. And, it should be added that, in compliance with Section 29 of the POEA-SEC which requires that in
cases of claims and disputes arising from a seafarer's employment, the parties covered by a CBA shall submit the
claim or dispute to the original and exclusive jurisdiction of the voluntary arbitrator or panel of voluntary arbitrators.

Consistent with this finding, Fernandez's contention — that his complaint for disability benefits is a money claim that
falls within the original and exclusive jurisdiction of the labor arbiter under Section 10 of R.A. No. 8042 — is
untenable. We likewise reject his argument that he never referred his claim to the grievance machinery (so that no
unresolved grievance exists as required under Article 261 of the Labor Code), and that the parties to the case are not
the union and the employer. Needless to state, no such distinction exists in the parties' CBA and the POEA-SEC.

It bears stressing at this point that we are upholding the jurisdiction of the voluntary arbitrator or panel of voluntary
arbitrators over the present dispute, not only because of the clear language of the parties' CBA on the matter; more
importantly, we so uphold the voluntary arbitrator's jurisdiction, in recognition of the State's express preference for
voluntary modes of dispute settlement, such as conciliation and voluntary arbitration as expressed in the Constitution,
the law and the rules.

In closing, we quote with approval a most recent Court pronouncement on the same issue, thus — It is settled that
when the parties have validly agreed on a procedure for resolving grievances and to submit a dispute to
voluntary arbitration then that procedure should be strictly observed.
127. Portillo vs. Lietz Inc. et al., G.R. No. 196539, October 10, 2012

Facts:

In a letter agreement dated 3 May 1991, signed by individual respondent Rudolf Lietz (Rudolf) and conformed to by
Portillo, the latter was hired by the former under the following terms and conditions:

A copy of [Lietz, Inc.'s] work rules and policies on personnel is enclosed and an inherent part of
the terms and conditions of employment.

We acknowledge your proposal in your application specifically to the effect that you will not
engage in any other gainful employment by yourself or with any other company either directly or
indirectly without written consent of [Lietz, Inc.], and we hereby accept and henceforth consider
your proposal an undertaking on your part, a breach of which will render you liable to [Lietz, Inc.]
for liquidated damages.

On her tenth (10th) year with Lietz, Inc., specifically on 1 February 2002, Portillo was promoted to Sales
Representative and received a corresponding increase in basic monthly salary and sales quota. In this regard, Portillo
signed another letter agreement containing a "Goodwill Clause:"

It remains understood and you agreed that, on the termination of your employment by act of
either you or [Lietz, Inc.], and for a period of three (3) years thereafter, you shall not engage
directly or indirectly as employee, manager, proprietor, or solicitor for yourself or others in a
similar or competitive business or the same character of work which you were employed by
[Lietz, Inc.] to do and perform. Should you breach this good will clause of this Contract, you shall
pay [Lietz, Inc.] as liquidated damages the amount of 100% of your gross compensation over the
last 12 months, it being agreed that this sum is reasonable and just. 

Three (3) years thereafter, on 6 June 2005, Portillo resigned from Lietz, Inc. During her exit interview, Portillo
declared that she intended to engage in business — a rice dealership, selling rice in wholesale.

Subsequently, Lietz, Inc. learned that Portillo had been hired by Ed Keller Philippines, Limited to head its Pharma
Raw Material Department. Ed Keller Limited is purportedly a direct competitor of Lietz, Inc.

On 14 September 2005, Portillo filed a complaint with the National Labor Relations Commission (NLRC) for non-
payment of 1 1/2 months' salary, two (2) months' commission, 13th month pay, plus moral, exemplary and actual
damages and attorney's fees.

In its position paper, Lietz, Inc. admitted liability for Portillo's money claims in the total amount of P110,662.16.
However, Lietz, Inc. raised the defense of legal compensation: Portillo's money claims should be offset against her
liability to Lietz, Inc. for liquidated damages in the amount of P869,633.09 7 for Portillo's alleged breach of the
"Goodwill Clause" in the employment contract when she became employed with Ed Keller Philippines, Limited.

Issue:

Simply, the issue is whether Portillo's money claims for unpaid salaries may be offset against respondents' claim for
liquidated damages.

Ruling:

Paragraph 4 of Article 217 of the Labor Code appears to have caused the reliance by the Court of Appeals on the
"causal connection between [Portillo's] monetary claims against [respondents] and the latter's claim from liquidated
damages against the former."

Art. 217. Jurisdiction of Labor Arbiters and the Commission. — (a) Except as otherwise
provided under this code, the Arbiters shall have original and exclusive jurisdiction to hear and
decide, within thirty (30) calendar days after the submission of the case by the parties for
decision without extension, even in the absence of stenographic notes, the following case
involving all workers, whether agricultural or non-agricultural:

xxx xxx xxx

4. Claims for actual, moral, exemplary and other forms of damages arising from the
employer-employee relations;
That the "Goodwill Clause" in this case is likewise a post-employment issue should brook no argument. There is no
dispute as to the cessation of Portillo's employment with Lietz, Inc.  She simply claims her unpaid salaries and
commissions, which Lietz, Inc. does not contest. At that juncture, Portillo was no longer an employee of Lietz,
Inc.  The "Goodwill Clause" or the "Non-Compete Clause" is a contractual undertaking effective after the cessation of
the employment relationship between the parties. In accordance with jurisprudence, breach of the undertaking is a
civil law dispute, not a labor law case.

It is clear, therefore, that while Portillo's claim for unpaid salaries is a money claim that arises out of or in connection
with an employer-employee relationship, Lietz, Inc.'s claim against Portillo for violation of the goodwill clause is a
money claim based on an act done after the cessation of the employment relationship. And, while the jurisdiction over
Portillo's claim is vested in the labor arbiter, the jurisdiction over Lietz, Inc.'s claim rests on the regular courts.

In the case at bar, the difference in the nature of the credits that one has against the other, conversely, the nature of
the debt one owes another, which difference in turn results in the difference of the forum where the different credits
can be enforced, prevents the application of compensation. Simply, the labor tribunal in an employee's claim for
unpaid wages is without authority to allow the compensation of such claims against the post employment claim of the
former employer for breach of a post employment condition. The labor tribunal does not have jurisdiction over the civil
case of breach of contract.

We are aware that in Bañez v. Hon. Valdevilla, we mentioned that: 

Whereas this Court in a number of occasions had applied the jurisdictional provisions of Article
217 to claims for damages filed by employees [citation omitted], we hold that by the designating
clause "arising from the employer-employee relations" Article 217 should apply with equal force
to the claim of an employer for actual damages against its dismissed employee, where the basis
for the claim arises from or is necessarily connected with the fact of termination, and should be
entered as a counterclaim in the illegal dismissal case. 

While on the surface, Bañez supports the decision of the Court of Appeals, the facts beneath premise an opposite
conclusion. There, the salesman-employee obtained from the NLRC a final favorable judgment of illegal dismissal.
Afterwards, the employer filed with the trial court a complaint for damages for alleged nefarious activities causing
damage to the employer. Explaining further why the claims for damages should be entered as a counterclaim in the
illegal dismissal case, we said:

Even under Republic Act No. 875 (the 'Industrial Peace Act,' now completely superseded by the
Labor Code), jurisprudence was settled that where the plaintiff's cause of action for damages
arose out of, or was necessarily intertwined with, an alleged unfair labor practice committed by
the union, the jurisdiction is exclusively with the (now defunct) Court of Industrial Relations, and
the assumption of jurisdiction of regular courts over the same is a nullity. To allow otherwise
would be "to sanction split jurisdiction, which is prejudicial to the orderly administration of justice."
Thus, even after the enactment of the Labor Code, where the damages separately claimed by
the employer were allegedly incurred as a consequence of strike or picketing of the union, such
complaint for damages is deeply rooted from the labor dispute between the parties, and should
be dismissed by ordinary courts for lack of jurisdiction. As held by this Court in National
Federation of Labor vs. Eisma, 127 SCRA 419:

Certainly, the present Labor Code is even more committed to the view that on policy
grounds, and equally so in the interest of greater promptness in the disposition of labor
matters, a court is spared the often onerous task of determining what essentially is a
factual matter, namely, the damages that may be incurred by either labor or
management as a result of disputes or controversies arising from employer-employee
relations. 

There is no causal connection between the petitioner employees' claim for unpaid wages and the respondent
employers' claim for damages for the alleged "Goodwill Clause" violation. Portillo's claim for unpaid salaries did not
have anything to do with her alleged violation of the employment contract as, in fact, her separation from employment
is not "rooted" in the alleged contractual violation. She resigned from her employment. She was not dismissed.
Portillo's entitlement to the unpaid salaries is not even contested. Indeed, Lietz, Inc.'s argument about legal
compensation necessarily admits that it owes the money claimed by Portillo.

The alleged contractual violation did not arise during the existence of the employer-employee relationship. It was a
post-employment matter, a post-employment violation. Reminders are apt. That is provided by the fairly recent case
of Yusen Air and Sea Services Phils., Inc. v. Villamor,  which harked back to the previous rulings on the necessity of
"reasonable causal connection" between the tortious damage and the damage arising from the employer-employee
relationship. Yusen proceeded to pronounce that the absence of the connection results in the absence of jurisdiction
of the labor arbiter. Importantly, such absence of jurisdiction cannot be remedied by raising before the labor tribunal
the tortious damage as a defense.
128. Gonzales vs. Solid Cement Corp., G.R. No. 198423, October 23, 2012

Facts:

The current petition arose from the execution of the final and executory judgment in the parties' illegal dismissal
dispute (referred to as "original case," docketed in this Court as G.R. No. 165330 and entitled Solid Cement
Corporation, et al. v. Leo Gonzales). The Labor Arbiter (LA) resolved the case at his level on December 12, 2000.
Since the LA found that an illegal dismissal took place, the company reinstated  petitioner Gonzales in the
payroll  on January 22, 2001. 

In the meanwhile, the parties continued to pursue the original case on the merits. The case was appealed to the
National Labor Relations Commission (NLRC) and from there to the Court of Appeals (CA) on a petition
for certiorari under Rule 65 of the Rules of Court. The LA's ruling of illegal dismissal was largely left undisturbed in
these subsequent recourses. The original case  eventually came to this Court. In our Resolutions of March 9,
2005  and June 8, 2005,  we denied the petition of respondent Solid Cement Corporation (Solid Cement) for lack of
merit. Our ruling became final and entry of judgment took place on July 12, 2005.

Soon after its finality, the original case was remanded to the LA for execution. The LA decision dated December 12,
2000 declared the respondents guilty of illegal dismissal and ordered the reinstatement of Gonzales to his former
position "with full backwages and without loss of seniority rights and other benefits[.]"  Under this ruling, as modified
by the NLRC ruling on appeal, Gonzales was awarded the following:

(1)Backwages in the amount of P636,633.33;

(2)Food and Transportation Allowance in the amount of P18,080.00;

(3)Moral damages in the amount of P100,000.00;

(4)Exemplary damages in the amount of P50,000.00; and

(5)Ten percent (10%) of all sums owing to the petitioner as attorney's fees.

Actual reinstatement and return to work for Gonzales (who had been on payroll reinstatement since January
22, 2001) came on July 15, 2008. 

When Gonzales moved for the issuance of an alias writ of execution on August 4, 2008, he included several items as
components in computing the amount of his backwages. Acting on the motion, the LA added P57,900.00 as rice
allowance and P14,675.00 as medical reimbursement (with the company's apparent conformity), and excluded the
rest of the items prayed for in the motion, either because these items have been paid or that, based on the records of
the case, Gonzales was not entitled thereto. Under the LA's execution order dated August 18, 2009, Gonzales was
entitled to a total of P965,014.15. 

The NLRC, in its decision dated February 19, 2010 and resolution dated May 18, 2010, modified the LA's execution
order by including the following amounts as part of the judgment award: 

Additional backwages from Dec. 13, 2000 to Jan. 21, 2001 P50,800.00 9

Salary differentials from year 2000 until August 2008 617,517.48

13th month pay differential 51,459.79

13th month pay for years 2000 and 2001 80,000.00

12% interest from July 12, 2005 878,183.42

This ruling increased Gonzales' entitlement to P2,805,698.04.

On a petition for certiorari  under Rule 65 of the Rules of Court, the CA set aside the NLRC's decision and reinstated
the LA's order, prompting Gonzales to come to the Court via  a petition for review on certiorari (docketed as G.R.
No. 198423) under Rule 45 of the Rules of Court. In our Minute Resolutions, we denied Gonzales' Rule 45
petition. At this point came the two motions now under consideration.

LA ruling:

WHEREFORE, premises considered, respondents are hereby declared guilty of ILLEGAL DISMISSAL and
ordered to reinstate complainant to his former position with full backwages and without loss of seniority
rights and other benefits which to date amounts (sic)to Six Hundred Thirty Six Thousand and Six Hundred
Thirty Three Pesos and Thirty Three Centavos (P636,633.33).

Further, respondents are jointly and severally liable to pay the following: 

1.P18,080 as reimbursement for food and transportation allowance;

2.Five Hundred Thousand (P500,000.00) Pesos as moral damages;

3.Two Hundred Fifty Thousand (P250,000.00) Pesos as exemplary damages; and

4.10% of all sums owing to complainant as attorney's fees.  

NLRC Ruling:

WHEREFORE, premises considered, the decision under review is hereby, MODIFIED by REDUCING the
amount of moral and exemplary damages due the complainant to the sum of P100,000.00 and P50,000.00,
respectively.

Further, joint and several liability for the payment of backwages, food and transportation allowance and
attorney's fees as adjudged in the appealed decision is hereby imposed only upon respondents Allen
Querubin and Solid Cement Corporation, the latter having a personality which is distinct and separate from
its officers.

The relief of reinstatement is likewise, AFFIRMED. 

CA Ruling:

IN VIEW OF ALL THE FOREGOING, the instant petition is hereby dismissed for lack of merit. Accordingly,
the decision of the Second Division of the NLRC dated 26 March 2002 in NLRC CA No. 027452-01 is
hereby AFFIRMED. 

Issue:

whether the CA was legally correct in finding that the NLRC acted outside its jurisdiction when it modified
the LA's execution order

Ruling:

Thus, even without proof of nonpayment, the NLRC was right in requiring the payment of the 13th month pay and
the salaries due after the LA's decision until the illegally dismissed petitioner was reinstated in the payroll,  i.e., from
December 13, 2000 to January 21, 2001. It follows that the CA was wrong when it concluded that the NLRC acted
outside its jurisdiction by including these monetary awards as items for execution.

These amounts are not excluded from the concept of backwages as the salaries fell due after Gonzales should have
been reinstated, while the 13th month pay fell due for the same period by legal mandate. These are entitlements that
cannot now be glossed over if the final decision on the merits in this case were to be respected.

The Legal Obstacle: the prohibition


on 2nd motion for reconsideration

The above discussions unavoidably lead to the conclusion that the Court's Minute Resolutions denying Gonzales'
petition were not properly issued and are tainted by the nullity of the CA decision these Resolutions effectively
approved. We do not aim to defend these actions, however, by mechanically and blindly applying the principle of
immutability of judgment, nor by tolerating the CA's inappropriate application of this principle. The immutability
principle, rather than being absolute, is subject to well-settled exceptions, among which is its inapplicability when a
decision claimed to be final is not only erroneous, but null and void. 

We cannot also be oblivious to the legal reality that the matter before us is no longer the validity of Gonzales'
dismissal and the legal consequences that follow — matters long laid to rest and which we do not and cannot now
disturb. Nor is the matter before us the additional monetary benefits that Gonzales claims in his petition, since these
essentially involve factual matters that are beyond a Rule 45 petition to rule upon and correct.
The matter before us — in the Rule 45 petition questioning the CA's Rule 65 determination — is the  scope of the
benefits awarded by the LA, as modified on appeal and ultimately affirmed by this Court, which ruling has
become final and which now must be implemented as a matter of law.

Given these considerations, to reopen this case on second motion for reconsideration would not actually embroil the
Court with changes in the decision on the merits of the case, but would confine itself solely to the issue of the CA's
actions in the course of determining lack or excess of jurisdiction or the presence of grave abuse of discretion in
reviewing the NLRC's ruling on the execution aspect of the case.

Additionally, while continued consideration of a case on second motion for reconsideration very strongly remains an
exception, our action in doing so in this case is not without sound legal justification. An order of execution that varies
the tenor of a final and executory judgment is null and void.  This was what the CA effectively did — it varied the final
and executory judgment of the LA, as modified on appeal and ultimately affirmed by the Court. We would simply be
enforcing our own Decision on the merits of the original case by nullifying what the CA did.

Viewed in these lights, the recognition of, and our corrective action on, the nullity of the CA's ruling on the current
petition is a duty this Court is under obligation to undertake pursuant to Section 1, Article VIII of the Constitution. We
undertake this corrective action by restoring what the CA should have properly recognized to be covered by the
Decision on the merits of the original case.
129. Martos et al., vs. New San Jose Builders, G.R. No. 192650, October 24, 2012

FACTS:

New San Jose Builders, Inc. is a corporation engaged in the construction of road, bridges, etc. San Jose was
constrained to slow down and suspend most of the works on projects due to lack of funds. Thus, the workers were
informed that many of them would be laid off and the rest would be reassigned to other projects. Thus several
complaints were filed with the Labor Arbiter for illegal dismissal and other monetary benefits.

The LA handed down a decision declaring, among others, that petitioner Felix Martoswas illegally dismissed and
entitled to separation pay, backwages and other monetary benefits; and dismissing, without prejudice, the
complaints/claims of the other complainants.

In the NLRC, the petitioner-workers appealed their dismissal without prejudice. Respondent San Jose also appealed
the part where Martos was declared regular and illegally dismissed. The NLRC ruled in favor of some of the workers.

In the certiorari to the CA, it upheld the ruling of the LA. The CA explained that the NLRC committed grave abuse of
discretion in reviving the complaints of petitioners despite their failure to verify the same. Out of the 102
complainants, only Martos verified the position paper and his counsel never offered any explanation for his failure to
secure the verification of the others.

Petitioners insist that the lack of verification of a position paper is only a formal and not a jurisdictional defect.

Respondent says that while it is true that the NLRC Rules must be liberally construed and that the NLRC is not bound
by the technicalities of law and procedure, it should not be the first to arbitrarily disregard specific provisions of the
rules which are precisely intended to assist the parties in obtaining just, expeditious and inexpensive settlement of
labor disputes. It was only Felix Martos who verified their position paper and their memorandum of appeal. It was only
he alone who was vigilant in looking after his interest and enforcing his rights. Petitioners should be considered to
have waived their rights and interests in the case for their consistent neglect and passive attitude.

ISSUES:

1] whether or not the CA was correct in dismissing the complaints filed by those petitioners who failed to verify their
position papers; and

2] whether or not Martos should be reinstated.

RULING:

1.The absence of a proper verification is cause to treat the pleading as unsigned and dismissible. 

The lone signature of Martos would have been sufficient if he was authorized by his co-petitioners to sign for them.
Unfortunately, petitioners failed to adduce proof that he was so authorized. The liberal construction of the rules may
be invoked in situations where there may be some excusable formal deficiency or error in a pleading, provided that
the same does not subvert the essence of the proceeding and it at least connotes a reasonable attempt at
compliance with the rules.

Considering that the dismissal of the other complaints by the LA was without prejudice, the other complainants should
have taken the necessary steps to rectify their procedural mistake after the decision of the LA was rendered. They
should have corrected this procedural flaw by immediately filing another complaint with the correct verification this
time. Surprisingly, they did not even attempt to correct this technical blunder. Worse, they committed the same
procedural error when they filed their appeal  with the NLRC.

2. As to Martos, the Court agrees that the reinstatement being sought by him was no longer practicable because of
strained relation between the parties. Indeed, he can no longer question this fact. Thus, the Court deems it fair to
award separation pay in lieu of reinstatement. In addition to his separation pay, Martos is also entitled to payment of
full backwages, 13th month pay, service incentive leave pay, and attorney's fees
130. Building Care Corp. vs. Macaraeg, G.R. No. 198357, December 10, 2012

FACTS:

Petitioners are in the business of providing security services to their clients. They hired respondent on 1996. However
she was relieved of her post and was reassigned on 2008, but after said period, she was allegedly no longer given
any assignment. Thus the filing of a complaint for illegal dismissal, underpayment of salaries, non-payment of
separation pay and refund of cash bond. Conciliation and mediation proceedings failed, so the parties were ordered
to submit their respective position papers.

On May 13, 2009, the LA decided the case as wanting of merit but ordered the agency to pay P5000 as financial
assistance.

Respondent then filed a Notice of Appeal with the National Labor Relations Commission (NLRC), but in a Decision
dated October 23, 2009, the NLRC dismissed the appeal for having been filed out of time, thereby declaring that
the Labor Arbiter's Decision had become final and executory on June 16, 2009.

In a petition for certiorari elevated to the CA, the CA awarded reinstatement and backwages.

ISSUE:

Whether the CA erred in liberally applying the rules of procedure and ruling that respondent's appeal should be
allowed and resolved on the merits despite having been filed out of time

RULING:

The Court cannot sustain the CA's Decision.

It should be emphasized that the resort to a liberal application, or suspension of the application of procedural rules,
must remain as the exception to the well-settled principle that rules must be complied with for the orderly
administration of justice.

In this case, the justifications given by the CA for its liberalitywas the fault of respondent's former counsel. Note,
however, that neither respondent nor her former counsel gave any explanation or reason citing extraordinary
circumstances for her lawyer's failure to abide by the rules for filing an appeal.

It is, however, an oft-repeated ruling that the negligence and mistakes of counsel bind the client. A departure from
this rule would bring about never-ending suits. The only exception would be, where the lawyer's gross negligence
would result in the grave injustice of depriving his client of the due process of law.

“It should also be borne in mind that the right of the winning party to enjoy the finality of the resolution of the case is
also an essential part of public policy and the orderly administration of justice. Hence, such right is just as weighty or
equally important as the right of the losing party to appeal or seek reconsideration within the prescribed period. “

In sum, the Court cannot countenance relaxation of the rules absent the showing of extraordinary circumstances to
justify the same. In this case, no compelling reasons can be found to convince this Court that the CA acted correctly
by according respondent such liberality. 
131. Kapisanang Pangkaunlaran ng Kababaihang Potrero, Inc. (KPKPI) v. Barreno,Ametin, Nonay, Dionisio
and Casio, G.R. No. 175900, June 10, 2013

Facts:

On November 1997, the Technology and Livelihood Resource Center tapped KPKI to participate in its microlending
program and was granted a loan for microfinance or re-lending for the poor. As such, KPKPI hired respondents for its
KPKPI Mile Program

On September 20, 2001, respondents filed a Complaint before DOLE-NCR for underpayment of wages, non-payment
of labor standard benefits, namely, legal/special holiday pay, 13th month pay and service incentive leave pay, and
non-coverage with the Social Security System and Home Development Mutual Fund against KPKPI and its Program
Manager (DOLE CASE).

During its pendency, however, respondents were informed of their termination. This prompted the filing of another
Complaint against petitioners, this time for illegal dismissal with prayer for reinstatement and payment of their money
claims before the NLRC (NLRC CASE).

LA: NO forum shopping; the subsequent dismissal of the respondents affected the jurisdiction of the DOLE-NCR
since illegal dismissal cases are beyond the latter's jurisdiction. Necessarily therefore, the case for money claims
pending before the DOLE-NCR had to be consolidated with the illegal dismissal case before the NLRC.

NLRC: set aside the LA's ruling; respondents guilty of forum shopping in filing the same complaint against petitioners
in two (2) fora, namely the DOLE and the NLRC.

CA: agreed with the NLRC that respondents committed forum shopping in seeking their money claims before the
DOLE and the NLRC. Nonetheless, it declared that the ends of justice would be better served if respondents would
be given the opportunity to be heard on their complaint for illegal dismissal. Accordingly, CA ordered the remand of
the case to the NLRC for further proceedings on the matter of illegal dismissal, separation pay, damages, and
attorney's fees.

Issue: Whether the CA erred in ordering the reinstatement and remand of the NLRC CASE to the NLRC despite its
finding of forum shopping.

Ruling: Respondents are not guilty of forum shopping. SC concurs in the result arrived at by the CA in
remanding the cases for illegal dismissal to the NLRC for resolution of the appeal.

Forum shopping exists "when one party repetitively avails of several judicial remedies in different courts,
simultaneously or successively, all substantially founded on the same transactions and the same essential facts and
circumstances, and all raising substantially the same issues either pending in, or already resolved adversely, by
some other court." What is truly important to consider in determining whether it exists or not is the vexation caused
the courts and parties-litigants by a party who asks different courts and/or administrative agencies to rule on the
same or related causes and/or grant the same or substantially the same reliefs, in the process creating the possibility
of conflicting decisions being rendered by different  fora upon the same issues.

In the case at bar, there is no identity of causes of action between the cases pending with the DOLE and the NLRC.
The DOLE CASE involved violations of labor standard provisions where an employer-employee relationship exists.
On the other hand, the NLRC CASE questioned the propriety of respondents' dismissal. No less than the Labor Code
provides for these two (2) separate remedies for distinct causes of action. More importantly, at the time the DOLE
CASE was initiated, respondents' only cause of action was petitioners' violation of labor standard laws which falls
within the jurisdiction of the DOLE. It was only after the same was filed that respondents were dismissed from
employment, prompting the filing of the NLRC CASE, which is within the mantle of the NLRC's jurisdiction. Under the
foregoing circumstances, respondents had no choice but to avail of different fora.

In cases where the complaint for violation of labor standard laws preceded the termination of the employee and the
filing of the illegal dismissal case, it would not be in consonance with justice to charge the complainants with
engaging in forum shopping when the remedy available to them at the time their causes of action arose was to file
separate cases before different fora.
132. Philippine Transmarine Carriers, Inc. (PTCI) v. Legaspi, G.R. No. 202791, June 10, 2013

Facts: Legaspi was employed as Utility Pastry on board the vessel "Azamara Journey" under the employment of
petitioner PTCI. His employment was covered by a CBA where it was agreed that the company shall pay a maximum
disability compensation of up to US$60,000.00 only.

While on board, Legaspi suffered "Cardiac Arrest S/P ICD Insertation." He was checked by the ship’s doctor,
prescribed medications and then repatriated for further medical treatment and exam. The company-designated
physician assessed his condition to be Disability Grade 2.

Not satisfied, Legaspi filed a complaint for full and permanent disability compensation against PTCI before the Labor
Arbiter. LA ruled in favor of Legaspi. LA awarded US$80,000.00 based on the ITF Cruise Ship Model Agreement for
Catering Personnel, not on the CBA. Unsatisfied, PTCI appealed before NLRC. NLRC affirmed the decision of the
LA. On September 5, 2010, NLRC issued the Entry of Judgment stating that its resolution affirming the LA decision
had become final and executory.

On October 22, 2010, during the hearing on the motion for execution before the NLRC, PTCI agreed to pay Legaspi
US$81,320.00. The terms and conditions of which were embodied in the Receipt of Judgment Award with
Undertaking, wherein Legaspi acknowledged receipt of the said amount and undertook to return it to PTCI in the
event the latter's petition for certiorari would be granted, without prejudice to Legaspi’s right to appeal. It was also
agreed upon that the remaining balance would be given on the next scheduled conference.

On November 8, 2010, PTCI timely filed a petition for certiorari with the CA. In the meantime, on March 2, 2011, LA
issued a writ of execution which noted PTCI’s payment of the amount of US$81,320.00. In compliance, PTCI
tendered to NLRC Cashier the US$8,132.00 attorney’s fees and P3,042.95 execution fees. LA ordered their release
to Legaspi.

Unaware of the entry of judgment of the NLRC, payment of US$81,320.00 and the writ of execution issued by LA, CA
partially granted the petition for certiorari and modified the assailed resolutions of the NLRC, awarding only
US$60,000.00 pursuant to the CBA.

PTCI filed its Manifestation with Motion to Amend the Dispositive Portion, submitting to the CA the writ of execution
issued by the LA in support of its motion. PTCI contended that since it had already paid the total amount of
US$89,452.00, it was entitled to the return of the excess payment in the amount of US$29,452.00. CA denied it and
ruled that the petition should have been dismissed for being moot and academic because the assailed decision of the
NLRC had become final and executory and that the said judgment had been satisfied even before the filing of the
petition for certiorari.

Issues: Whether the petition for certiorari has become moot and academic.

Ruling: The petition for certiorari is not moot.

Section 14, Rule VII of the 2011 NLRC Rules of Procedure provides that decisions, resolutions or orders of the NLRC
shall become final and executory after ten (10) calendar days from receipt thereof by the parties, and entry of
judgment shall be made upon the expiration of the said period.  In St. Martin Funeral Home v. NLRC,  however, it was
ruled that judicial review of decisions of the NLRC may be sought via a petition for certiorari before the CA under Rule
65 of the Rules of Court; and under Section 4 thereof, petitioners are allowed sixty (60) days from notice of the
assailed order or resolution within which to file the petition. Hence, in cases where a petition for certiorari is filed after
the expiration of the 10-day period under the 2011 NLRC Rules of Procedure but within the 60-day period under Rule
65 of the Rules of Court, the CA can grant the petition and modify, nullify and reverse a decision or resolution of the
NLRC.

Accordingly, in this case, although the petition for certiorari was not filed within the 10-day period, PTCI timely filed it
before the CA within the 60-day reglementary period under Rule 65. It has, thus, been held that the CA's review of
the decisions or resolutions of the NLRC under Rule 65, particularly those which have already been executed, does
not affect their statutory finality, considering that Section 4,  Rule XI of the 2011 NLRC Rules of Procedure, provides
that a petition for certiorari filed with the CA shall not stay the execution of the assailed decision unless a restraining
order is issued.

In the present case, the Receipt of the Judgment Award with Undertaking was fair to both the employer and the
employee. It stipulated that Legaspi should return the amount to PTCI if the petition for  certiorari would be granted
but without prejudice to Legaspi’s right to appeal. The agreement, thus, provided available remedies to both parties. It
is clear that PTCI paid Legaspi subject to the terms and conditions stated in the Receipt of the Judgment Award with
Undertaking. Both parties signed the agreement. Legaspi neither refuted the agreement nor claimed that he was
forced to sign it against his will. Therefore, the petition for certiorari was not rendered moot despite PTCI’s
satisfaction of the judgment award, as Legaspi had obliged himself to return the payment if the petition would be
granted.

Legaspi agreed to the stipulation that he would return the amount paid to him in the event that the petition
for  certiorari  would be granted. Since the petition was indeed granted by the CA, albeit partially, Legaspi must
comply with the condition to return the excess amount. To allow now Legaspi to retain the excess money judgment
would amount to his unjust enrichment to the prejudice of PTCI. Legaspi is ORDERED to return the excess amount of
payment in the sum of US$29,452.00 to PTCI, the amount shall earn interest at the rate of 12% per annum from the
finality of this judgment
133. Pasos vs. Phil National Construction Corp. G.R. No. 192394, July 3, 2013

Facts:

Petitioner contends that PNCC’s appeal from the Labor Arbiter’s decision should not have been allowed since the
appeal bond filed was insufficient. He likewise argues that the appellate court erred in heavily relying in the case of
Cagayan Valley Drug Corporation v. Commissioner of Internal Revenue 23 which enumerated the officials and
employees who can sign the verification and certification without need of a board resolution. He contends that in said
case, there was substantial compliance with the requirement since a board resolution was submitted albeit belatedly
unlike in the instant case where no board resolution was ever submitted even belatedly.

As to the substantive issue, petitioner submits that the CA erroneously concluded that he was a project employee
when there are indicators which point otherwise. He contends that even if he was just hired for the NAIA 2 Project
from April 26, 1996 to July 25, 1996, he was made to work until August 4, 1998. He also avers the DOLE had certified
that he was not among the employees listed in the termination reports submitted by PNCC which belies the
photocopies of termination reports attached by PNCC to its pleadings listing petitioner as one of the affected
employees. Petitioner points out that said termination reports attached to PNCC’s pleadings are mere photocopies
and were not even certified by the DOLE-NCR as true copies of the originals on file with said office. Further, he
argues that in violation of the requirement of Department Order No. 19 that the duration of the project employment is
reasonably determinable, his contracts for the SM projects did not specify the date of completion of the project nor
was the completion determinable at the time that petitioner was hired.

PNCC counters that documentary evidence would show that petitioner was clearly a project employee and remained
as such until his last engagement. It argues that the repeated rehiring of petitioner as accounting clerk in different
projects did not make him a regular employee. It also insists that it complied with the reportorial requirements and
that it filed and reported the termination of petitioner upon every completion of project to which he was employed.

Issues:

(1) Should an appeal be dismissed outright if the appeal bond filed is less than the adjudged amount?
(2) Can the head of the personnel department sign the verification and certification on behalf of the corporation sans
any board resolution or secretary’s certificate authorizing such officer to do the same? and
(3) Is petitioner a regular employee and not a mere project employee and thus can only be dismissed for cause?

Ruling:

1) Not necessarily. While perfection of an appeal within the reglementary period and in the manner prescribed
by law is jurisdictional, the bond requirement on appeals involving monetary awards has been relaxed in
certain cases. This can only be done where there was substantial compliance of the Rules or where the
appellants, at the very least, exhibited willingness to pay by posting a partial bond. In the instant case, the
Labor Arbiter in his decision ordered PNCC to pay petitioner back wages amounting toP422,630.41 and
separation pay of P37,662 or a total of P460,292.41. When PNCC filed an appeal bond amounting
to P422,630.41 or at least 90% of the adjudged amount, there is no question that this is substantial
compliance with the requirement that allows relaxation of the rules.

2) Yes, the head of the personnel department may sign the verification and certification because he was in a
position to assure that the allegations were true and correct and on the basis of liberality in applying the
rules.

We have held that the following officials or employees of the company can sign the verification and
certification without need of a board resolution: (1) the Chairperson of the Board of Directors, (2) the
President of a corporation, (3) the General Manager or Acting General Manager, (4) Personnel Officer, and
(5) an Employment Specialist in a labor case.

While the above cases do not provide a complete listing of authorized signatories to the verification and
certification required by the rules, the determination of the sufficiency of the authority was done on a case to
case basis. The rationale applied in the foregoing cases is to justify the authority of corporate officers or
representatives of the corporation to sign the verification or certificate against forum shopping, being "in a
position to verify the truthfulness and correctness of the allegations in the petition." 

While we agree with petitioner that in Cagayan Valley, the requisite board resolution was submitted though
belatedly unlike in the instant case, this Court still recognizes the authority of Mr. Erece, Jr. to sign the
verification and certification on behalf of PNCC sans a board resolution or secretary’s certificate as we have
allowed in Pfizer, Inc. v. Galan, one of the cases cited in Cagayan Valley. In Pfizer, the Court ruled as valid
the verification signed by an employment specialist as she was in a position to verify the truthfulness and
correctness of the allegations in the petition despite the fact that no board resolution authorizing her was
ever submitted by Pfizer, Inc. even belatedly. We believe that like the employment specialist in Pfizer, Mr.
Erece, Jr. too, as head of the Personnel Services Department of PNCC, was in a position to assure that the
allegations in the pleading have been prepared in good faith and are true and correct.

3) Petitioner was a regular employee and enjoys the right to security of tenure.
While for first three months, petitioner can be considered a project employee of PNCC, his employment
thereafter, when his services were extended without any specification of as to the duration, made him a
regular employee of PNCC. And his status as a regular employee was not affected by the fact that he was
assigned to several other projects and there were intervals in between said projects since he enjoys security
of tenure.
134. Nacar vs. Gallery Frames, G.R. No. 189871, August 13, 2013, En banc

Facts:

Petitioner argues that notwithstanding the fact that there was a computation of backwages in the Labor Arbiter’s
decision, the same is not final until reinstatement is made or until finality of the decision, in case of an award of
separation pay. Petitioner maintains that considering that the October 15, 1998 decision of the Labor Arbiter did not
become final and executory until the April 17, 2002 Resolution of the Supreme Court in G.R. No. 151332 was entered
in the Book of Entries on May 27, 2002, the reckoning point for the computation of the backwages and separation pay
should be on May 27, 2002 and not when the decision of the Labor Arbiter was rendered on October 15, 1998.
Further, petitioner posits that he is also entitled to the payment of interest from the finality of the decision until full
payment by the respondents.

On their part, respondents assert that since only separation pay and limited backwages were awarded to petitioner by
the October 15, 1998 decision of the Labor Arbiter, no more recomputation is required to be made of said awards.
Respondents insist that since the decision clearly stated that the separation pay and backwages are "computed only
up to [the] promulgation of this decision," and considering that petitioner no longer appealed the decision, petitioner is
only entitled to the award as computed by the Labor Arbiter in the total amount ofP158,919.92. Respondents added
that it was only during the execution proceedings that the petitioner questioned the award, long after the decision had
become final and executory. Respondents contend that to allow the further recomputation of the backwages to be
awarded to petitioner at this point of the proceedings would substantially vary the decision of the Labor Arbiter as it
violates the rule on immutability of judgments.

Issue:

Whether or not a re-computation in the course of execution of the labor arbiter's original computation of the awards
made, pegged as of the time the decision was rendered and confirmed with modification by a final CA decision, is
legally proper

Ruling:

Yes it was legally proper. A re-computation is necessary as it essentially considered the labor arbiter's original
decision in accordance with its basic component parts as we discussed above. To reiterate, the first part contains the
finding of illegality and its monetary consequences; the second part is the computation of the awards or monetary
consequences of the illegal dismissal, computed as of the time of the labor arbiter's original decision.

The instant case is similar to the case of Session Delights Ice Cream and Fast Foods v. Court of Appeals (Sixth
Division), wherein the issue submitted to the Court for resolution was the propriety of the computation of the awards
made, and whether this violated the principle of immutability of judgment. Like in the present case, it was a distinct
feature of the judgment of the Labor Arbiter in the above-cited case that the decision already provided for the
computation of the payable separation pay and backwages due and did not further order the computation of the
monetary awards up to the time of the finality of the judgment. Also in Session Delights, the dismissed employee
failed to appeal the decision of the labor arbiter.

A source of misunderstanding in implementing the final decision in this case proceeds from the way the original labor
arbiter framed his decision. The decision consists essentially of two parts.

The first is that part of the decision that cannot now be disputed because it has been confirmed with finality. This is
the finding of the illegality of the dismissal and the awards of separation pay in lieu of reinstatement, backwages,
attorney's fees, and legal interests.

The second part is the computation of the awards made. On its face, the computation the labor arbiter made shows
that it was time-bound as can be seen from the figures used in the computation. This part, being merely a
computation of what the first part of the decision established and declared, can, by its nature, be re-computed. This is
the part, too, that the petitioner now posits should no longer be re-computed because the computation is already in
the labor arbiter's decision that the CA had affirmed. The public and private respondents, on the other hand, posit that
a re-computation is necessary because the relief in an illegal dismissal decision goes all the way up to reinstatement
if reinstatement is to be made, or up to the finality of the decision, if separation pay is to be given in lieu
reinstatement.

That the labor arbiter's decision, at the same time that it found that an illegal dismissal had taken place, also made a
computation of the award, is understandable in light of Section 3, Rule VIII of the then NLRC Rules of Procedure
which requires that a computation be made. This Section in part states:

[T]he Labor Arbiter of origin, in cases involving monetary awards and at all events, as far as practicable, shall
embody in any such decision or order the detailed and full amount awarded.
Clearly implied from this original computation is its currency up to the finality of the labor arbiter's decision. As we
noted above, this implication is apparent from the terms of the computation itself, and no question would have arisen
had the parties terminated the case and implemented the decision at that point.
However, the petitioner disagreed with the labor arbiter's findings on all counts - i.e., on the finding of illegality as well
as on all the consequent awards made. Hence, the petitioner appealed the case to the NLRC which, in turn, affirmed
the labor arbiter's decision. By law, the NLRC decision is final, reviewable only by the CA on jurisdictional grounds.
The petitioner appropriately sought to nullify the NLRC decision on jurisdictional grounds through a timely filed Rule
65 petition for certiorari. The CA decision, finding that NLRC exceeded its authority in affirming the payment of 13th
month pay and indemnity, lapsed to finality and was subsequently returned to the labor arbiter of origin for execution.

It was at this point that the present case arose. Focusing on the core illegal dismissal portion of the original labor
arbiter's decision, the implementing labor arbiter ordered the award re-computed; he apparently read the figures
originally ordered to be paid to be the computation due had the case been terminated and implemented at the labor
arbiter's level. Thus, the labor arbiter re-computed the award to include the separation pay and the backwages due
up to the finality of the CA decision that fully terminated the case on the merits. Unfortunately, the labor arbiter's
approved computation went beyond the finality of the CA decision (July 29, 2003) and included as well the payment
for awards the final CA decision had deleted - specifically, the proportionate 13th month pay and the indemnity
awards. Hence, the CA issued the decision now questioned in the present petition.

Consequently, from the above disquisitions, under the terms of the decision which is sought to be executed by the
petitioner, no essential change is made by a recomputation as this step is a necessary consequence that flows from
the nature of the illegality of dismissal declared by the Labor Arbiter in that decision. A recomputation (or an original
computation, if no previous computation has been made) is a part of the law – specifically, Article 279 of the Labor
Code and the established jurisprudence on this provision – that is read into the decision. By the nature of an illegal
dismissal case, the reliefs continue to add up until full satisfaction, as expressed under Article 279 of the Labor Code.
The recomputation of the consequences of illegal dismissal upon execution of the decision does not constitute an
alteration or amendment of the final decision being implemented. The illegal dismissal ruling stands; only the
computation of monetary consequences of this dismissal is affected, and this is not a violation of the principle of
immutability of final judgments.

That the amount respondents shall now pay has greatly increased is a consequence that it cannot avoid as it is the
risk that it ran when it continued to seek recourses against the Labor Arbiter's decision. Article 279 provides for the
consequences of illegal dismissal in no uncertain terms, qualified only by jurisprudence in its interpretation of when
separation pay in lieu of reinstatement is allowed. When that happens, the finality of the illegal dismissal decision
becomes the reckoning point instead of the reinstatement that the law decrees. In allowing separation pay, the final
decision effectively declares that the employment relationship ended so that separation pay and backwages are to be
computed up to that point.
135. Castells vs. Saudi Arabia Airlines, G.R. No. 188514, Aug 28, 2013

Facts:

Respondent SAUDIA AIRLINES issued a memorandum transferring the petitioners from Manila to Jeddah due to
operational requirements. Centi-Mandanas complied with the transfer order while Castiled did not. Upon arrival of
Cent-Mandanas, she was told that her contract would no longer be renewed and that she was asked to sign a pre-
typed resignation letter. She averred that while she never wished to resign, SAUDIA left her with no other viable
choice as it would terminate her services anyway. Thus, she filled out the resignation form handed to her. Castells
alleged that upon her non-compliance with the transfer order, she prepared a resignation letter stating that she felt
she was being forced to resign. She then alleged that the SAUDIA Manila Office Manager told her to amend the
same to state that she was voluntarily resigning; this she reluctantly followed.

Petitioners filed this complaint for illegal dismissal against the respondent with the LA, with prayer for reinstatement,
full backwages, moral and exemplary damages, and attorney’s fees. They alleged that the Jeddah-based flight
attendants aged 39 to 40 years old, were already processing their respective resignations and that the transfer order
was made so that they would be terminated upon their arrival in Jeddah. Respondent maintained that the
resignations were intelligently and voluntarily made and that they were penned and duly signed by them. It asserted
further that the petitioners voluntarily executed an undertaking acknowledging receipt of various sums of money and
irrevocably and unconditionally releasing the respondent from any claim or demand which they may gave in
connection with their employment.

LA ruled in favor of the petitioners and held the respondent guilty of illegal dismissal and ordered to pay them full;
backwages less the amount they already received. It founded that petitioners did not voluntarily resigned but they
were forced to do so because of old age under pain of actual termination. Further held that the subject undertaking
was akin to a quitclaim, cannot bar the petitioners from filing the case against the respondents. Respondent appealed
to NLRC, and the latter reversed the decision of LA and dismissed the illegal dismissal case against the petitioners
and ruled that there was no coercion on employed by the respondent and that the undertaking executed by
petitioners was valid. Petitioner filed their Motion for Reconsideration which was subsequently denied by the NLRC
in its resolution dated October 26, 2007. .

Aggrieved, the petitioners elevated the matter to the CA. On January 16, 2008, petitioners filed with the CA a Motion
for Extension to File a Petition for Certiorari, praying that they be given a period of 15 days from January 18, 2008, or
until February 2, 2008, within which to file the subject petition. The said motion was granted in a Resolution dated
January 29, 2008.

SAUDIA filed a Motion for Reconsideration, primarily contending that A.M. No. 07-7-12-SC, which took effect on
December 27, 2007, no longer allowed the filing of an extension of time to file a petition for certiorari; thus, the CA
should not have admitted the subject petition. In a Resolution dated August 28, 2008, the CA reconsidered its earlier
resolution and granted SAUDIA’s motion. It deemed the subject petition not admitted due to petitioners’ non-
compliance with the reglementary period prescribed by Section 4, Rule 65 of the Rules of Court (Rules), as amended
by A.M. No. 07-7-12-SC. Hence, it considered the case closed and terminated.

Petitioners filed a Motion for Reconsideration dated September 26, 2008, which was, however, denied in a
Resolution dated June 16, 2009, prompting them to institute the instant petition.

Issue: Whether or not the CA correctly refused admission of the subject petition

Ruling:

It is well-settled that procedural rules should be treated with utmost respect and due regard, since they are designed
to facilitate the adjudication of cases to remedy the worsening problem of delay in the resolution of rival claims and in
the administration of justice. However, Court has recognized exceptions to the strict application of such rules, but only
for the most compelling reasons where stubborn obedience to the Rules would defeat rather than serve the ends of
justice.24 These exceptions, as enumerated in the case of Labao v. Flores, are as follows:

(1) most persuasive and weighty reasons;


(2) to relieve a litigant from an injustice not commensurate with his failure to comply with the prescribed
procedure;
(3) good faith of the defaulting party by immediately paying within a reasonable time from the time of the
default;
(4) the existence of special or compelling circumstances;
(5) the merits of the case;
(6) a cause not entirely attributable to the fault or negligence of the party favored by the suspension of the
rules;
(7) a lack of any showing that the review sought is merely frivolous and dilatory;
(8) the other party will not be unjustly prejudiced thereby;
(9) fraud, accident, mistake, or excusable negligence without appellant’s fault;
(10) peculiar legal and equitable circumstances attendant to each case;
(11) in the name of substantial justice and fair play;
(12) importance of the issues involved; and
(13) Exercise of sound discretion by the judge guided by all the attendant circumstances.
Despite the rigid wording of Section 4, Rule 65 of the Rules, as amended by A.M. No. 07-7-12-SC – which
now disallows an extension of the 60-day reglementary period to file a petition for certiorari – courts may
nevertheless extend the same, subject to its sound discretion.

General rule is that a petition for certiorari must be filed within sixty (60) days from notice of the judgment,
order, or resolution sought to be assailed. Under exceptional circumstances, however, and subject to the sound
discretion of the Court, said period may be extended.

In this case, the CA had already exercised its sound discretion in granting the extension to file the subject
petition thru a Resolution dated January 29, 2008. Consequently, it could not renege on such grant by rendering
another issuance almost seven months later, i.e., Resolution dated August 28, 2008, which resulted in the
refusal to admit the same petition.

136. Dongon vs. Rapid Movers and Forwarders Co Inc., et al., G.R. No. 163431, Aug. 28, 2013

Facts:

Rapid is engaged in the hauling and trucking business while private respondent Nathaniel T. Dongon is a former truck
helper leadman. Private respondent’s area of assignment is the Tanduay Otis Warehouse where he has a job of
facilitating the loading and unloading petitioner’s trucks. On 23 April 2001, private respondent and his driver, Vicente
Villaruz, were in the vicinity of Tanduay as they tried to get some goods to be distributed to their clients. Tanduay’s
security guard called the attention of private respondent as to the fact that Mr. Villaruz was not wearing an
Identification Card (I.D. Card). Private respondent, then, assured the guard that he will secure a special permission
from the management to warrant the orderly release of goods. Instead of complying with his compromise, private
respondent lent his I.D. Card to Villaruz; and by reason of such misrepresentation , private respondent and Mr.
Villaruz got a clearance from Tanduay for the release of the goods. However, the security guard, who saw the
misrepresentation committed by private respondent and Mr. Villaruz, accosted them and reported the matter to the
management of Tanduay. On 23 May 2001, after conducting an administrative investigation, private respondent was
dismissed from the petitioning Company.

On 01 June 2001, private respondent filed a Complaint for Illegal Dismissal.


September I 0, 2001, LA ruled against the private respondent and ruled that Rapid Movers rightly exercised its
prerogative to dismiss the petitioner considering that: he had admitted lending his company ID to the driver; his act
had constituted mental dishonesty and deceit amounting to breach of trust; Rapid Mover’s relationship with Tanduay
had been jeopardized by his act; and that he had been banned from all the warehouses of Tanduay as a result,
leaving Rapid Movers with no available job for him. June 17, 2002, NLRC reversed the LA ruling and held the Rapid
Movers had not discharged its burden to prove the validity of petitioner’s dismissal. It opined that Rapid Movers did
not suffered any pecuniary damage and that the dismissal was a penalty disproportionate to the act of the petitioner.
It awarded separation pay and damages.

The case was elevated to the CA averring grave abuse of discretion on the part of NLRC. October 24, 2003, the Ca
reinstated the decision of the LA and upheld the right of Rapid Movers to discipline its workers. Petitioner file a motion
for reconsideration but the same was denied on March 22, 2004. Hence, the case was appealed with the SC.

Petitioner argued that his dismissal was illegal; that the CA gravely abused its discretion in disregarding his showing
the he did not violate Rapid Movers rules and regulation but simply performed work in line with his duties and it did
not appreciate his good faith and lack of any intention to willfully disobey the company rules; that the penalty of
dismissal is too harsh and disproportionate to his supposed violation. Respondent Rapid movers, prays that the
petition for certiorari be dismissed for being an improper remedy for a lost appeal; it insist that the CA did not commit
grave abuse of discretion.

Issues:

1. Whether or not the petition should be dismissed for being an improper remedy for a lost appeal.
2. Whether or not the petitioner was guilty of willful disobedience which will justify his dismissal.

Ruling:

The petition should not be dismissed. It has been clarified that a party seeking the review of decisions of the NLRC
should file his petition for certiorari in the CA under rule 65 on the ground of grave abuse of discretion amounting to
lack or excess of jurisdiction on the part of NLRC. Thereafter, the remed y of the aggrieved party from the CA
decision is petition for review on certiorari under rule 45.

The petition filed here is self-styled as a petition for review on certiorari, but Rapid Movers points out that the petition
was really one for certiorari under Rule 65 of the Rules of Court due to its basis being the commission by the CA of a
grave abuse of its discretion and because the petition was filed beyond the reglementary period of appeal under Rule
45.

The Court deems it proper to allow due course to the petition as one for certiorari under Rule 65 in the broader
interest of substantial justice, particularly because the NLRC’s appellate adjudication was set aside by the CA, and in
order to put at rest the doubt that the CA, in so doing, exercised its judicial authority oppressively. Whether the
petition was proper or not should be of less importance than whether the CA gravely erred in undoing and setting
aside the determination of the NLRC as a reviewing forum vis-àvis the Labor Arbiter. We note in this regard that the
NLRC had declared the dismissal of petitioner to be harsh and not commensurate to the infraction committed. Given
the spirit and intention underlying our labor laws of resolving a doubtful situation in favor of the working man, we will
have to review the judgment of the CA to ascertain whether the NLRC had really committed grave abuse of its
discretion. This will settle the doubts on the propriety of terminating petitioner, and at the same time ensure that
justice is served to the parties.

For willful disobedience to be a ground, it is required that: (a) the conduct of the employee must be willful or
intentional; and (b) the order the employee violated must have been reasonable, lawful, made known to the
employee, and must pertain to the duties that he had been engaged to discharge. Willfulness must be attended by a
wrongful and perverse mental attitude rendering the employee’s act inconsistent with proper subordination. In any
case, the conduct of the employee that is a valid ground for dismissal under the Labor Code constitutes harmful
behavior against the business interest or person of his employer. It is implied that in every act of willful disobedience,
the erring employee obtains undue advantage detrimental to the business interest of the employer. Under the
foregoing standards, the disobedience attributed to petitioner could not be justly characterized as willful within the
contemplation of Article 296 of the Labor Code. He neither benefitted from it, nor thereby prejudiced the business
interest of Rapid Movers. His explanation that his deed had been intended to benefit Rapid Movers was credible.
There could be no wrong or perversity on his part that warranted the termination of his employment based on willful
disobedience.

Although we recognize the inherent right of the employer to discipline its employees, we should still ensure that the
employer exercises the prerogative to discipline humanely and considerately, and that the sanction imposed is
commensurate to the offense involved and to the degree of the infraction. The discipline exacted by the employer
should further consider the employee’s length of service and the number of infractions during his employment. The
employer should never forget that always at stake in disciplining its employee are not only his position but also his
livelihood, and that he may also have a family entirely dependent on his earnings. Considering that petitioner’s motive
in lending his company ID to Villaruz was to benefit Rapid Movers as their employer by facilitating the loading of
goods at the Tanduay Otis Warehouse for distribution to Rapid Movers’ clients, and considering also that petitioner
had rendered seven long unblemished years of service to Rapid Movers, his dismissal was plainly unwarranted. The
NLRC’s reversal of the decision of the Labor Arbiter by holding that penalty too harsh and disproportionate to the
wrong attributed to him was legally and factually justified, not arbitrary or whimsical.

137. Malayang Manggagawa ng Stayfast Phils Inc vs. NLRC et al., G.r. No. 155306, Aug. 28, 2013

FACTS:

The Labor Arbiter and the National Labor Relations Commission (NLRC) made similar findings of fact. Petitioner
MMSP and NLMS-Olalia) sought to be the exclusive bargaining agent of the employees of respondent company,
Stayfast Philippines, Inc. A certification election was conducted and out of the 223 valid votes cast, petitioner
garnered 109 votes while NLMS-Olalia received 112 votes and 2 votes were for "No Union." NLMS-Olalia was
certified by the Med-Arbiter as the sole and exclusive bargaining agent of all rank and file employees of respondent
company.

Petitioner appealed the Order of the Med-Arbiter to the Secretary of Labor and Employment. The Secretary of Labor
and Employment initially set aside the Order of the Med-Arbiter and called for run-off election between petitioner and
NLMS-Olalia. On motion of NLMS-Olalia, however, the Secretary of Labor and Employment reconsidered his earlier
decision and restored the Med-Arbiter’s Order dated January 9, 1996.

Meanwhile, NLMS-Olalia demanded to collectively bargain with respondent company. The latter rejected petitioner’s
demand, insisting that it would negotiate a collective bargaining agreement only with whichever union is finally
certified as the sole and exclusive bargaining agent of the workers. Nevertheless, NLMS-Olalia went on strike on April
1, 1997 until it was temporarily restrained eight days later.

Subsequently, Petitioner filed its own notice of strike in the National Conciliation and Mediation Board (NCMB).
Respondent company opposed petitioner’s move and filed a motion to dismiss on the ground that petitioner was not
the certified bargaining agent and therefore lacked personality to file a notice of strike. Thereafter, the parties were
able to make concessions during the conciliation-mediation stage in the NCMB which led petitioner to withdraw its
notice of strike.
In this connection, the NCMB issued a Certification dated July 31, 1997 which stated that the notice of strike filed by
MMSP-Independent on June 1997 against Stayfast is dropped. It further certified that there was no new Notice of
Strike filed by the same union.

On July 21, 1997, however, petitioner’s members staged a "sit-down strike" to dramatize their demand for a fair and
equal treatment as respondent company allegedly continued to discriminate against them. Respondent company
issued a memorandum requiring the alleged participants in the "sit-down strike" to explain within 24 hours why they
should not be terminated or suspended from work for infraction of company rules and regulations pertaining to
unauthorized work stoppage, acts inimical to company interest, and disregard of instruction of immediate supervisor
to perform assigned task.

As no one complied with the memorandum within the 24-hour deadline, respondent company promptly terminated
the service of the participants in the "sit-down strike" on July 22, 1997. Consequently, on July 23, 1997, petitioner
staged a strike and filed a complaint for unfair labor practice, union busting and illegal lockout against respondent
company and its General Manager, Maria Almeida, in the NLRC.

Petitioner alleged that respondents had committed acts of discrimination, such as the denial of the use of the canteen
to conduct strike votes, or denial of petitioners’ leave to attend hearings in labor cases, among others.

For its part, respondent company claimed that petitioner lacked legal authority to go on strike since it is a minority
union. Petitioner committed illegal acts during the strike and obstructed the free ingress and egress from respondent
company’s premises.

On April 27, 1999, the Labor Arbiter rendered a Decision which ruled that, while petitioner may file a notice of strike
on behalf of its members, petitioner failed to cite any instance of discrimination or harassment when it filed its notice
of strike on June 5, 1997 and the incidents mentioned as discriminatory occurred after the filing of the said notice.
Moreover, assuming the strike was legal at the beginning, it became illegal when petitioner committed acts prohibited
under Article 264(e) of the Labor Code, such as acts of violence, coercion and intimidation and obstruction of free
ingress and egress.

Petitioner appealed but, NLRC upheld the Labor Arbiter’s Decision. According to the NLRC, the actuations of
petitioner were patently illegal because the sit-down strike staged on July 21, 1997 was made barely a week after
petitioner withdrew its notice of strike, with prejudice, on account of the concessions agreed upon by the parties.
Petitioner filed no new notice of strike that could have supported its charges of discriminatory acts and unfair labor
practice. Moreover, no evidence was presented to establish such charges. Also, petitioner’s members were given the
opportunity to explain their violation of respondent company’s rules on unauthorized work stoppage, acts inimical to
company interest and disregard of instruction of immediate supervisor to perform assigned task.

Hence, this petition for certiorari under Rule 65 of the Rules of Court.

ISSUES:

1. That the CA committed grave abuse of discretion amounting to lack or excess of jurisdiction when they upheld the
rulings of the NLRC and disregarded the constitutional protection of labor as well as Article 248 (e) and Article 263 of
the Labor Code.

2. That the CA committed grave abuse of discretion amounting to lack or excess of jurisdiction when they upheld the
decision of the NLRC that the termination of complainants/appellants were valid and corollary thereto no
reinstatement[,] backwages, damages and attorney’s fees were awarded.

Petitioner claims that the discriminatory acts of respondent company and its General Manager against petitioner’s
members constituted unfair labor practice under Article 248(e) of the Labor Code, as amended. The termination of
employment of petitioner’s 127 officers and members constituted union-busting and unlawful lockout. As the said
officers and members were unlawfully dismissed from employment, they are entitled to reinstatement with full
backwages. The arbitrary action of respondent company and its General Manager wantonly disregarded the legal
rights of petitioner’s officers and members thereby entitling said officers and members to damages and attorney’s
fees.

RULING:

The petition fails for many reasons.

1. Certiorari is the wrong remedy.

For purposes of appeal, the Decision dated July 1, 2002 of the Court of Appeals was a final judgment as it denied
due course to, and dismissed, the petition. Thus, the Decision disposed of the petition of petitioner in a manner that
left nothing more to be done by the Court of Appeals in respect to the said case. Thus, petitioner should have filed an
appeal by petition for review on certiorari under Rule 45, not a petition for certiorari under Rule 65, in this Court.

Moreover, certiorari is not and cannot be made a substitute for an appeal where the latter remedy is available but was
lost through fault or negligence. In this case, petitioner received the Decision dated July 1, 2002 on August 2, 2002
and, under the rules, had until August 19, 2002 to file an appeal by way of a petition for review in this Court. Petitioner
let this period lapse without filing an appeal and, instead, filed this petition for certiorari on October 1, 2002.

2. Petitioner unable to establish allegation of grave abuse of discretion on the part of CA.

Where a petition for certiorari under Rule 65 of the Rules of Court alleges grave abuse of discretion, the petitioner
should establish that the respondent court or tribunal acted in a capricious, whimsical, arbitrary or despotic manner in
the exercise of its jurisdiction as to be equivalent to lack of jurisdiction

The term "grave abuse of discretion" has a specific meaning. An act of a court or tribunal can only be considered as
with grave abuse of discretion when such act is done in a "capricious or whimsical exercise of judgment as is
equivalent to lack of jurisdiction." The abuse of discretion must be so patent and gross as to amount to an "evasion of
a positive duty or to a virtual refusal to perform a duty enjoined by law, or to act at all in contemplation of law, as
where the power is exercised in an arbitrary and despotic manner by reason of passion and hostility."

In this case, nowhere in the petition did petitioner show that the issuance of the Decision dated July 1, 2002 of the
Court of Appeals was patent and gross that would warrant striking it down through a petition for certiorari. Aside from
a general statement in the Jurisdictional Facts portion of the petition and the sweeping allegation of grave abuse of
discretion in the general enumeration of the grounds of the petition, petitioner failed to substantiate its imputation of
grave abuse of discretion on the part of the Court of Appeals. No argument was advanced to show that the Court of
Appeals exercised its judgment capriciously, whimsically, arbitrarily or despotically by reason of passion and hostility.
Petitioner did not even discuss how or why the conclusions of the Court of Appeals were made with grave abuse of
discretion. Instead, petitioner limited its discussion on its version of the case, which had been already rejected both
by the Labor Arbiter and the NLRC. Thus, petitioner failed in its duty to demonstrate with definiteness the grave
abuse of discretion that would justify the proper availment of a petition for certiorari under Rule 65 of the Rules of
Court.

3. Even on the merits, petition must fail.

Petitioner’s case rests on the alleged discriminatory acts of respondent company against petitioner’s officers and
members. However, both the Labor Arbiter and the NLRC held that there was no sufficient proof of respondent
company’s alleged discriminatory acts. Thus, petitioner’s unfair labor practice, union-busting and unlawful lockout
claims do not hold water. Moreover, the established facts as found by the NLRC are as follows: the "sit-down strike"
made by petitioner’s officers and members on July 21, 1997 was in violation of respondent company’s rules, and
petitioner’s officers and members ignored the opportunity given by respondent company for them to explain their
misconduct, which resulted in the termination of their employment.

138. Borra et al., vs. Court of Appeals, et al., G.R. No. 167484, Sept. 9, 2013

FACTS:

On September 12, 1997, herein petitioners filed with the National Labor Relations Commission (NLRC) Regional
Arbitration Branch No. VIin Bacolod City two separate complaints against herein private respondent alone, while RAB
Case No. 06-09-10699-97 impleaded herein private respondent and a certain Fela Contractor as respondents. In
RAB Case No. 06-09-10698-97, petitioners asked that they be recognized and confirmed as regular employees of
herein private respondent and further prayed that they be awarded various benefits received by regular employees
for three (3) years prior to the filing of the complaint, while in RAB Case No. 06-09-10699-97,herein petitioners sought
for payment of unpaid wages, holiday pay, allowances, 13th month pay, service incentive leave pay, moral and
exemplary damages also during the three (3) years preceding the filing of the complaint.

On October 16, 1997, private respondent filed a Motion to Consolidate the above mentioned cases, but the Labor
Arbiter in charge of the case denied the said Motion.

On January 9, 1998, private respondent filed a Motion to Dismiss RAB Case No. 06-09-0698-97 on the ground of res
judicata. Private respondent cited an earlier decided case entitled " Humphrey Perez, et al. v. Hawaiian Philippine Co.
et al. "(Perez case) and docketed as RAB Case No.06-04-10169-95, which was an action for recovery of 13th month
pay and service incentive leave pay, and it includes herein petitioners among the complainants and herein private
respondent and one Jose Castillon (Castillon) as respondents. Private respondent contended that the Perez case,
which has already become final and executory, as no appeal was taken there from, serves as a bar to the litigation of
RAB Case No. 06-09-10698-97, because it was ruled therein that petitioners are not employees of private respondent
but of Castillon.

In an Order dated July 9, 1998, the Labor Arbiter granted private respondent's Motion to Dismiss.

Petitioners appealed to the NLRC which set aside the Order of the Labor Arbiter, reinstated the complaint in RAB
Case No. 06-09-10698-97and remanded the same for further proceedings.

Private respondent appealed to the CA. On January 12, 2001, the CA rendered judgment, affirming the Decision of
the NLRC and denied the subsequent motion for reconsideration.

Aggrieved, private respondent filed a petition for review on certiorari before this Court. The case was entitled as
"Hawaiian Philippine Company v. Borra" and docketed as G.R. No. 151801. On November 12, 2002, this Court
rendered its Decision denying the petition and affirming the Decision of the CA.
On June 22, 2004, the CA rendered its questioned Decision, the dispositive portion of which reads, thus:

WHEREFORE, foregoing premises considered, the petition is GRANTED. Accordingly, the Order dated August 12,
2003 of public respondent is hereby ANNULLED and SET ASIDE. RAB Case No. 09-10698-97 is ordered
DISMISSED.

SO ORDERED.

Petitioners filed a Motion for Reconsideration, but the CA denied it in its Resolution dated January 14, 2005.

Hence, the present petition for certiorari based on the following grounds:

ISSUES:

1. Does the CA have jurisdiction to entertain the petition for certiorari? Yes.

2. Is there an employer-employee relationship between petitioners and private respondent? No.

RULING:

1. This Court is not persuaded by petitioners' argument that the CA has no jurisdiction over private respondent's
petition for certiorari because this Court, in G.R. No. 151801, lodged jurisdiction in the Labor Arbiter by directing the
remand of RAB Case No. 06-09-10698-97 thereto for further proceedings.

It is settled that jurisdiction over the subject matter is conferred by law and it is not within the courts, let alone the
parties, to themselves determine or conveniently set aside.

In this regard, Rule 41 of the Rules of Court, which is applied in a suppletory character to cases covered by the NLRC
Rules, provides that in all the instances enumerated under the said Rule, where the judgment or final order is not
appealable, the aggrieved party may file an appropriate special civil action under Rule 65.

On the basis of the foregoing, it is clear that the CA has jurisdiction over the special civil action for certiorari filed by
private respondent as the latter was able to allege and establish that the denial of its motion to dismiss was tainted
with grave abuse of discretion. Petitioners are wrong to argue that this Court's directive in G.R. No. 151801 to
remand RAB Case No. 06-09-10698-97 to the Labor Arbiter for further proceedings deprives the CA of its jurisdiction
over private respondent's petition for certiorari.

2. As earlier mentioned, this issue has already been settled. In the already final and executory decision of the Labor
Arbiter in RAB Case No.06-09-10699-97, it was ruled therein that no employer-employee relationship exists between
private respondent and petitioners because the latter's real employer is Fela Contractor. Thus, insofar as the question
of employer and employee relations between private respondent and petitioners is concerned, the final judgment in
RAB Case No. 06-09-10699-97 has the effect and authority of res judicata by conclusiveness of judgment.

Hence, there is no point in determining the main issue raised in RAB Case No. 06-09-10698-97, whether petitioners
may be considered regular employees of private respondent, because, in the first place, they are not even employees
of the latter. As such, the CA correctly held that the Labor Arbiter committed grave abuse of discretion in denying
private respondent's motion to dismiss RAB Case No. 06-09-10698-97.

139. McBurnie vs. Ganzon et al., GR Nol. 178034 & 178117 & 186984-85, Oct. 17, 2013, (En banc)

Facts:

On October 4, 2002, McBurnie, an Australian national, instituted a complaint for illegal dismissal and other monetary
claims against the respondents. McBurnie claimed that on May 11, 1999, he signed a five-year employment
agreement with the company EGI as an Executive Vice-President who shall oversee the management of the
company's hotels and resorts within the Philippines. He performed work for the company until sometime in November
1999, when he figured in an accident that compelled him to go back to Australia while recuperating from his injuries.
While in Australia, he was informed by respondent Ganzon that his services were no longer needed because their
intended project would no longer push through.

The respondents opposed the complaint, contending that their agreement with McBurnie was to jointly invest in and
establish a company for the management of hotels. They did not intend to create an employer-employee relationship,
and the execution of the employment contract that was being invoked by McBurnie was solely for the purpose of
allowing McBurnie to obtain an alien work permit in the Philippines. At the time McBurnie left for Australia for his
medical treatment, he had not yet obtained a work permit. 

The LA declared McBurnie as having been illegally dismissed from employment, and thus entitled to receive from the
respondents the following amounts: (a) US$985,162.00 as salary and benefits for the unexpired term of their
employment contract, (b) P2,000,000.00 as moral and exemplary damages, and (c) attorney's fees equivalent to 10%
of the total monetary award.
Feeling aggrieved, the respondents appealed the LA's Decision to the NLRC. On November 5, 2004, they filed their
Memorandum of Appeal and Motion to Reduce Bond,  and posted an appeal bond in the amount of P100,000.00. The
respondents contended in their Motion to Reduce Bond, inter alia,  that the monetary awards of the LA were null and
excessive, allegedly with the intention of rendering them incapable of posting the necessary appeal bond.

Issue:

The crucial issue in this case concerns the sufficiency of the appeal bond that was posted by the respondents.

Ruling:

To clarify, the prevailing jurisprudence on the matter provides that the filing of a motion to reduce bond, coupled with
compliance with the two conditions emphasized in Garcia v. KJ Commercial  for the grant of such motion, namely,
(1) a meritorious ground, and (2) posting of a bond in a reasonable amount, shall suffice to suspend the running of
the period to perfect an appeal from the labor arbiter's decision to the NLRC. To require the full amount of the
bond within the 10-day reglementary period would only render nugatory the legal provisions which allow an appellant
to seek a reduction of the bond.

A serious error of the NLRC was its outright denial of the motion to reduce the bond, without even considering the
respondents' arguments and totally unmindful of the rules and jurisprudence that allow the bond's reduction. Instead
of resolving the motion to reduce the bond on its merits, the NLRC insisted on an amount that was equivalent to the
monetary award.

Although the general rule provides that an appeal in labor cases from a decision involving a monetary award may be
perfected only upon the posting of a cash or surety bond, the Court has relaxed this requirement under certain
exceptional circumstances in order to resolve controversies on their merits. These circumstances include: (1) the
fundamental consideration of substantial justice; (2) the prevention of miscarriage of justice or of unjust enrichment;
and (3) special circumstances of the case combined with its legal merits, and the amount and the issue involved. 

The bond requirement in appeals involving monetary awards has been and may be relaxed in meritorious cases,
including instances in which (1) there was substantial compliance with the Rules, (2) surrounding facts and
circumstances constitute meritorious grounds to reduce the bond, (3) a liberal interpretation of the requirement of an
appeal bond would serve the desired objective of resolving controversies on the merits, or (4) the appellants, at the
very least, exhibited their willingness and/or good faith by posting a partial bond during the reglementary period. 

As the Court, nonetheless, remains firm on the importance of appeal bonds in appeals from monetary awards of LAs,
we stress that the NLRC, pursuant to Section 6, Rule VI of the NLRC Rules of Procedure, shall only accept motions
to reduce bond that are coupled with the posting of a bond in a reasonable amount.

At the time of a motion to reduce appeal bond's filing, the question of what constitutes "a reasonable amount of bond"
that must accompany the motion may be subject to differing interpretations of litigants. The judgment of the NLRC
which has the discretion under the law to determine such amount cannot as yet be invoked by litigants until after their
motions to reduce appeal bond are accepted.

It is in this light that the Court finds it necessary to set a parameter for the litigants' and the NLRC's guidance on the
amount of bond that shall hereafter be filed with a motion for a bond's reduction. To ensure that the provisions of
Section 6, Rule VI of the NLRC Rules of Procedure that give parties the chance to seek a reduction of the appeal
bond are effectively carried out, without however defeating the benefits of the bond requirement in favor of a winning
litigant, all motions to reduce bond that are to be filed with the NLRC shall be accompanied by the posting of a cash
or surety bond equivalent to 10% of the monetary award that is subject of the appeal, which shall provisionally be
deemed the reasonable amount of the bond in the meantime that an appellant's motion is pending resolution by the
Commission. In conformity with the NLRC Rules, the monetary award, for the purpose of computing the necessary
appeal bond, shall exclude damages and attorney's fees.  Only after the posting of a bond in the required percentage
shall an appellant's period to perfect an appeal under the NLRC Rules be deemed suspended.

Notably, in the present case, following the CA's rendition of its Decision which allowed a reduced appeal bond, the
respondents have posted a bond in the amount of P10,000,000.00.
Given the circumstances in this case and the merits of the respondents' arguments before the NLRC, the Court holds
that the respondents had posted a bond in a "reasonable amount", and had thus complied with the requirements for
the perfection of an appeal from the LA's decision.

Moreover, in all cases, the reduction of the appeal bond shall be justified by meritorious grounds and accompanied by
the posting of the required appeal bond in a reasonable amount.

The requirement on the existence of a "meritorious ground" delves on the worth of the parties' arguments, taking into
account their respective rights and the circumstances that attend the case.

140. Lepanto Consolidated Mining Corp., vs. Icao, GR No. 196047, January 15, 2014

Facts:

The instant petition stemmed from a complaint for illegal dismissal and damages filed by private respondent Belio C.
Icao [Icao] against petitioners Lepanto Consolidated Mining Company (LCMC) and its Chief Executive Officer [CEO]
Felipe U. Yap [Yap] before the Arbitration Branch of the NLRC.

The Labor Arbiter ruled that petitioner and its CEO liable for illegal dismissal and ordering them to pay respondent
Icao P345,879.45, representing his full backwages and separation pay. 

The petitioner and its CEO filed an Appearance with Memorandum of Appeal  before the NLRC. Instead of posting
the required appeal bond in the form of a cash bond or a surety bond in an amount equivalent to the monetary award
of P345,879.45 adjudged in favor of Icao, they filed a Consolidated Motion for Release of Cash Bond and to Apply
Bond Subject for Release As Payment for Appeal Bond (Consolidated Motion).  They requested therein that the
NLRC release the cash bond of P401,610.84, which they had posted in the separate case Dangiw Siggaao v.
LCMC,  and apply that same cash bond to their present appeal bond liability. They reasoned that since this Court had
already decided Dangiw Siggaao in their favor, and that the ruling therein had become final and executory, the cash
bond posted therein could now be released.  They also cited financial difficulty as a reason for resorting to this course
of action and prayed that, in the interest of justice, the motion be granted.

The NLRC First Division dismissed the appeal of petitioner and the latter's CEO for non-perfection.  It found that they
had failed to post the required appeal bond equivalent to the monetary award of P345,879.45. It explained that their
Consolidated Motion for the release of the cash bond in another case (Dangiw Siggaao), for the purpose of applying
the same bond to the appealed case before it, could not be considered as compliance with the requirement to post
the required appeal bond.

Issue:

The sole issue before the Court is whether or not petitioner complied with the appeal bond requirement under the
Labor Code and the NLRC Rules by filing a Consolidated Motion to release the cash bond it posted in another case,
which had been decided with finality in its favor, with a view to applying the same cash bond to the present case.

Ruling:

While it is true that the procedure undertaken by petitioner is not provided under the Labor Code or in the NLRC
Rules, we consider such procedure compliant with the appeal bond requirement under the Labor Code.

When the law does not clearly provide a rule or norm for the tribunal to follow in deciding a question submitted, but
leaves to the tribunal the discretion to determine the case in one way or another, the judge must decide the question
in conformity with justice, reason and equity, in view of the circumstances of the case. Applying this doctrine, we rule
that petitioner substantially complied with the mandatory requirement of posting an appeal bond for the reasons
explained below. 

First, there is no question that the appeal was filed within the 10-day reglementary period.  Except for the alleged
failure to post an appeal bond, the appeal to the NLRC was therefore in order.
Second, it is also undisputed that petitioner has an unencumbered amount of money in the form of cash in the
custody of the NLRC. To reiterate, petitioner had posted a cash  bond of P401,610.84 in the separate case Dangiw
Siggaao, which was earlier decided in its favor. As claimed by petitioner and confirmed by the Judgment Division of
the Judicial Records Office of this Court, the Decision of the Court in  Dangiw Siggaao had become final and
executory as of 28 April 2008, or more than seven months before petitioner had to file its appeal in the present case.
This fact is shown by the Entry of Judgment on file with the aforementioned office. Hence, the cash bond in that case
ought to have been released to petitioner then.

Under the Rule VI, Section 6 of the 2005 NLRC Rules, "[a] cash or surety bond shall be valid and effective from the
date of deposit or posting, until the case is finally decided, resolved or terminated, or the award satisfied." Hence, it is
clear that a bond is encumbered and bound to a case only for as long as 1) the case has not been finally decided,
resolved or terminated; or 2) the award has not been satisfied. Therefore, once the appeal is finally decided and no
award needs to be satisfied, the bond is automatically released. Since the money is now unencumbered, the
employer who posted it should now have unrestricted access to the cash which he may now use as he pleases — as
appeal bond in another case, for instance. This is what petitioner simply did.

Third, the cash bond in the amount of P401,610.84 posted in  Dangiw Siggaao is more than enough to cover the
appeal bond in the amount of P345,879.45 required in the present case.

Fourth, this ruling remains faithful to the spirit behind the appeal bond requirement which is to ensure that workers will
receive the money awarded in their favor when the employer's appeal eventually fails.  There was no showing at all of
any attempt on the part of petitioner to evade the posting of the appeal bond. On the contrary, petitioner's move
showed a willingness to comply with the requirement. Hence, the welfare of Icao is adequately protected.

Having complied with the appeal bond requirement, petitioner's appeal before the NLRC must therefore be
reinstated.

141. Building Care Corp. vs. Macaraeg, G.R. No. 198357, December 10, 2012

Facts:

Petitioner is engaged in the business of providing security services. From 8/25/1986 she was assigned at the Genato
Building in Caloocan City up until March 9, 2008, where she was relieved of her duty. Following her relief from duty,
she was re-assigned to bayview park hotel from March 9-13, 2008. However, after said date, she was no longer given
an assignment.

Because of this, respondent was prompted to file a case for illegal dismissal against petitioner as well as
underpayment of salaries, non-payment of separation pay and refund of cash bond.

Petitioner alleged that respondent Macaraeg was relieved from her duty at the Genato building for sleeping on the
job, as well as continuously borrowing money from the employees and clients of the Genato Building. Moreover,
Bayview Hotel also found that the services of respondent did not meet their standards, which was the reason why her
assignment at bayview was cut short.

At the Labor Arbiter, the complaint for illegal dismissal was dismissed. However, herein respondent elevated the case
to the NLRC by filing a notice of appeal. The appeal, however, was dismissed for having been filed out of time.

As a result, the case was elevated by the respondent to the CA via a petition for certiorari, which was granted in favor
of respondent. Hence, this case.

Issue:

1. WON respondent entitled to an appeal, or if the appeal filed before NLRC was filed out of time.

Ruling: PETITION GRANTED

The Supreme Court held that respondent failed to file a timely appeal before the NLRC, and this precluded her from
availing of other remedies. In other words, the decision of the LA had become final and executory for her failure to file
an appeal within the prescribed period. The supreme court decided the case in this manner, to wit:

Discussion:

It should be emphasized that the resort to a liberal application, or suspension of the application of
procedural rules, must remain as the exception to the well-settled principle that rules must be
complied with for the orderly administration of justice. In Marohomsalic v. Cole, the Court stated:

While procedural rules may be relaxed in the interest of justice, it is well- settled
that these are tools designed to facilitate the adjudication of cases. The
relaxation of procedural rules in the interest of justice was never intended to
be a license for erring litigants to violate the rules with impunity. Liberality in
the interpretation and application of the rules can be invoked only in proper cases
and under justifiable causes and circumstances. While litigation is not a game of
technicalities, every case must be prosecuted in accordance with the
prescribed procedure to ensure an orderly and speedy administration of
justice.

The later case of Daikoku Electronics Phils., Inc. v. Raza, further explained that:

To be sure, the relaxation of procedural rules cannot be made without any


valid reasons proffered for or underpinning it. To merit liberality, petitioner
must show reasonable cause justifying its non-compliance with the rules
and must convince the Court that the outright dismissal of the petition
would defeat the administration of substantial justice. . . . The desired
leniency cannot be accorded absent valid and compelling reasons for such a
procedural lapse. . ..

We must stress that the bare invocation of "the interest of substantial justice" line
is not some magic want that will automatically compel this Court to suspend
procedural rules. Procedural rules are not to be belittled, let alone dismissed
simply because their non-observance may have resulted in prejudice to a
party's substantial rights. Utter disregard of the rules cannot be justly
rationalized by harping on the policy of liberal construction.

xxxxxxxxx

…considering that the belated filing of respondent's appeal before the NLRC was the fault of
respondent's former counsel. Note, however, that neither respondent nor her former counsel gave
any explanation or reason citing extraordinary circumstances for her lawyer's failure to abide by the
rules for filing an appeal. Respondent merely insisted that she had not been remiss in following up
her case with said lawyer.

It is, however, an oft-repeated ruling that the negligence and mistakes of counsel bind the client. A
departure from this rule would bring about never-ending suits, so long as lawyers could allege their
own fault or negligence to support the client's case and obtain remedies and reliefs already lost by
the operation of law. The only exception would be, where the lawyer's gross negligence would result
in the grave injustice of depriving his client of the due process of law. In this case, there was no
such deprivation of due process. Respondent was able to fully present andargue her case before
the Labor Arbiter. She was accorded the opportunity to be heard. Her failure to appeal the Labor
Arbiter's Decision cannot, therefore, be deemed as a deprivation of her right to due process. In
Heirs of TeofiloGaudiano v. Benemerito, the Court ruled, thus:

"The perfection of an appeal within the period and in the manner prescribed by
law is jurisdictional and non-compliance with such legal requirements is fatal and
has the effect of rendering the judgment final and executory. The limitation on the
period of appeal is not without reason. They must be strictly followed as they are
considered indispensable to forestall or avoid unreasonable delays in the
administration of justice, to ensure an orderly discharge of judicial business, and
to put an end to controversies. . . .

xxxxxxxx

The right to appeal is not a natural right or part of due process; it is merely a
statutory privilege and may be exercised only in the manner and in
accordance with the provisions of law. Thus, one who seeks to avail of the
right to appeal must strictly comply with the requirements of the rules, and
failure to do so leads to the loss of the right to appeal."

xxxxxxxxx

Clearly, allowing an appeal, even if belatedly filed, should never be taken lightly. The judgment
attains finality by the lapse of the period for taking an appeal without such appeal or motion for
reconsideration being filed. In Ocampo v. Court of Appeals (Former Second Division), the Court
reiterated the basic rule that "when a party to an original action fails to question an adverse
judgment or decision by not filing the proper remedy within the period prescribed by law, he loses
the right to do so, and the judgment or decision, as to him, becomes final and binding." The
Decision of the Labor Arbiter, therefore, became final and executory as to respondent when she
failed to file a timely appeal therefrom. The importance of the concept of finality of judgment cannot
be gainsaid. As elucidated in Pasiona, Jr. v. Court of Appeals, to wit:

xxxxxxxxx
It should also be borne in mind that the right of the winning party to enjoy
the finality of the resolution of the case is also an essential part of public
policy and the orderly administration of justice. Hence, such right is just as
weighty or equally important as the right of the losing party to appeal or
seek reconsideration within the prescribed period.

When the Labor Arbiter's Decision became final, petitioners attained a vested right to said judgment.
They had the right to fully rely on the immutability of said Decision. In Sofio v. Valenzuela, it was
amply stressed that:

The Court will not override the finality and immutability of a judgment based
only on the negligence of a party's counsel in timely taking all the proper
recourses from the judgment. To justify an override, the counsel's negligence
must not only be gross but must also be shown to have deprived the party the
right to due process.
In sum, the Court cannot countenance relaxation of the rules absent the showing of
extraordinary circumstances to justify the same. In this case, no compelling reasons can be
found to convince this Court that the CA acted correctly by according respondent such liberality.
CIA
HDT
142. Co Say Coco Products Phils Inc., vs. Baltasar, et al., GR No. 188828, March 5, 2014

Facts:

Petitioner Co Say Coco entered into a contract for cargo handling services with co-petitioner Tanawan Port to
operate and manage the arrastre and stevedoring services of the port.

To commence operations, respondents et al were employed to various positions such as computer operator, crane
operator, crane helper, and fork lift operator. However, the business of tanawan port failed to take off due to a lack of
clientele. As a result, petitioner tanawan port decided to close its business and cease operations.

Upon approval of its retirement from business by the city treasurer, petitioner tanawan port called its employees to
inform them of their intention to close despite attempts to try to revive its business.

Because of this, respondents were terminated from employment but were given separation pay and 13 th month pay
totaling P86,416.68.

Respondents, however, alleged that petitioners were only feigning business losses to ease out EEs pointing out the
absence of evidence of petitioners to prove business reverses. They had also raised the failure of petitioners to send
notices to the EEs concerned and DOLE one month before the intended date of closure as required by law.

Petitioner tanawan port, on the other hand, raised the defense that severance from employment was brought about
by cessation of business, an authorized cause of termination for employment under the labor code. Tanawan port
reiterated that cessation of business was caused due to a lack of clientele, resulting to serious financial drain to the
company, leaving the management with no choice but to shut down operations.

The LA rendered decision in favor of respondents for failure to comply with procedural and substantial requirements.
The LA decision was reversed in the NLRC, stating that dismissal was not illegal. Respondents moved for the
reconsideration of the NLRC decision but was met with failure.

In the CA, the NLRC decision was reversed due to failure of petitioners to perfect their appeal. The CA held that
petitioners posted the appeal bond after the period to perfect the appeal had expired, resulting in non-perfection of
the appeal.

Instrumental to the issue is the presence of two certificates indicating the posting of a surety bond in the amount of
the monetary award due to the respondents. Petitioner uses this allegation in imputing error on the ruling of the CA
that the appeal was not perfected on time.
The first certificate indicated the receipt of the surety bond on sept. 24, 2003 while the second certificate indicated
the receipt of the surety bond on oct. 28, 2003.

Issues:

1. WON appeal bond was timely posted by the petitioner.

Ruling: Petition DENIED

In this case, the SC ruled that the appeal of the petitioners was not timely filed for failure to post the appeal bond
within the prescribed 10-day period to file an appeal under the NLRC Rules. As an effect, the decision of the labor
arbiter became final and executory between the parties.

Therefore, the decision became binding and the petitioners could no longer contest the decision of the labor arbiter.
The SC further stressed that the appeal period should be strictly applied since an appeal is not a matter of right.
Rather, it is a statutory privilege.

Discussion:

The crucial issue in the resolution of the instant petition concerns the timely posting of the appeal bond. The
pertinent rule on the matter is Article 223 of the Labor Code, as amended, which sets forth the rules on appeal from
the Labor Arbiter's monetary award:

ART. 223. Appeal. — Decisions, awards, or orders of the Labor Arbiter are final and executory unless appealed to
the Commission by any or both

parties within ten (10) calendar days from receipt of such decisions, awards, or orders. . . . .

xxxxxxxxx

In case of a judgment involving a monetary award, an appeal by the employer may be perfected only upon the
posting of a cash or surety bond issued by a reputable bonding company duly accredited by the Commission in the
amount equivalent to the monetary award in the judgment appealed from. (Emphasis ours).

Implementing the aforestated provisions of the Labor Code are the provisions of Rule VI of the 2011 NLRC Rules of
Procedure on perfection of appeals which read:

SECTION 1. PERIODS OF APPEAL. — Decisions, awards, or orders of the Labor Arbiter shall be final and executory
unless appealed to the Commission by any or both parties within ten (10) calendar days from receipt thereof; and in
case of decisions or resolutions of the Regional Director of the Department of Labor and Employment pursuant to
Article 129 of the Labor Code, within five (5) calendar days from receipt thereof. If the 10th or 5th day, as the case
may be, falls on a Saturday, Sunday or holiday, the last day to perfect the appeal shall be the first working day
following such Saturday, Sunday or holiday.

xxxxxxxx

These statutory and regulatory provisions explicitly provide that an appeal from theLabor Arbiter to the NLRC must be
perfected within ten calendar days from receipt of such decisions, awards or orders of the Labor Arbiter. In a
judgment involving a monetary award, the appeal shall be perfected only upon; (1) proof of payment of the required
appeal fee; (2) posting of a cash or surety bond issued by a reputable bonding company; and (3) filing of a
memorandum of appeal.

No appeal was perfected by the petitioners within the 10-day period under Article 223 of the Labor Code.

xxxxxxxxx

The Court of Appeals therefore, correctly ruled that petitioners failed to perfect their appeal on time. In holding so, the
appellate court only applied the appeal bond requirement as already well explained in our previous pronouncements
that there is legislative and administrative intent to strictly apply the appeal bond requirement, and the Court should
give utmost regard to this intention. The clear intent of both statutory and procedural law is to require the employer to
post a cash or surety bond securing the full amount of the monetary award within the ten 10-day reglementary period.
Rules on perfection of an appeal, particularly in labor cases,must be strictly construed because to extend the period
of the appeal is to delay the case, a circumstance which would give the employer a chance to wear out the efforts
and meager resources of the worker to the point that the latter is constrained to give up for less than what is due him.
This is to assure the workers that if they finally prevail in the case the monetary award will be given to them both upon
dismissal of the employer's appeal. It is further meant to discourage employers from using the appeal to delay or
evade payment of their obligations to the employees. The appeal bond requirement precisely aims to prevent empty
or inconsequential victories secured by laborers in consonance with the protection of labor clause ensconced and
zealously guarded by our Constitution.

It is entrenched in our jurisprudence that perfection of an appeal in a manner and within the period prescribed by law
is not only mandatory but jurisdictional, and failure to perfect an appeal has the effect of making judgment final
and executory. While dismissal of an appeal on technical grounds is frowned upon, Article 223 of the Labor Code
which prescribes the appeal bond requirement, however, is a rule of jurisdiction and not of procedure. Hence, there is
a little leeway for condoning a liberal interpretation thereof, and certainly none premised on the ground that its
requirements are mere technicalities. It is axiomatic that an appeal is only a statutory privilege and it may only be
exercised in the manner provided by law. The timely perfection of an appeal is a mandatory requirement, which
cannot be trifled with a "mere technicality" to suit the interest of party. We cannot condone the practice of parties who,
either by their own or their counsel's inadvertence, have allowed the judgment to become final and executory and,
after the same had reached finality, seeks the shield of substantial justice to assail it.

All considered then, the finding of the Labor Arbiter holding the petitioners liable for illegal dismissal is binding on
them. Not having been timely appealed, this issue is already beyond our jurisdiction to resolve, and the finding of the
Labor Arbiter can no longer be disturbed without violating the fundamental principle that final judgment is immutable
and unalterable and may no longer be modified in any respect, even if the modification is meant to correct erroneous
conclusion of fact and law.

143. Olores vs. Manila Doctors College et al., GR No. 201663, March 31, 2014

Facts:

Before us is a Petition for Review on Certiorari under Rule 45 of the Rules of Court seekingt he reversal of the Janary
9, 2012 and April 27, 2012 Resolutions of the Court of Appeals (CA) in CA- GR. SP No. 122596.

The facts, as found by the NLRC are as follows:

Respondent is a private higher educational institution dedicated to providing academic degrees and
certificate courses related to Allied Medical Services and Liberal Arts and Sciences.

(Petitioner) was hired as a part-time faculty of respondent on 07 November 2005. He was assigned at the
Humanities Department of the College of Arts and Sciences. Thereafer, he signed fixed term employment contracts
as par-time instructor from 03 November 2008, (peititioner) signed fixed term employemet, this time as a full time
instructor.

(Petitioner) submitted the final grades of his students to Mr. Jacinto Bernanrdo, Jr (Bernardo), the char of the
Humanities Area. On 13 April 2010, Bernando charged (petitioner) with gross misconduct and gross inefficiency in
the performance of duty. (Petitioner) was accused of employment a grading system not in accordance with the
system

On 14 April 2010, (petitioner) submited his answer refuting the allegations of the chair of Humanites. Also, on this
date, (petitioner) wrote a letter to respondent’s Human Resource Manager asking that he should now be granted a
permanent status.

Meanwhile, summer classes started on 15 April 2010 without (petitioner) having signed an employment
contract.
Acting on the report of Bernardo, respondent created the Manila Doctors tribunal (MDT) which was tasked to
ascertain the truth. The MDT sent notices of hearing to (petitioner)

On 31 May 2010, The MDT submitted its recoomendation to the president of respondnt. The culpability of
(petitioner) was established, hence dismissal was recoomended. On 07 June 2010, respondent terminated the
services of petitioner for grave misconduct and gross inefficieny and incompetence.

LA: found merit in petitioner’s charge for illegal dismissal. However it dismissed petitioner’s charge for illegal
dismissal.

NLRC: denied resolution. NLRC reasoned that respondent’s appeal was not accompanied by netiher a cash nor
surety bond, thus no appeal was perfected from the deicsion fo the LA.

Issues:

(1) Whether respondent’s appeal with the NLRC was perfected despite its failure to post a bond;
(2) Whether the CA erred in dismissing petitioner’s Rule 65 petition.

Ruling:

There is merit in the petition.

At the outset, it must be emphized that Artcile 223 of the Labor Code states that an appeal by the employer
to the NLRC from a judgment of Labor Arbiter, which involves a monetary award, may be perfected only upon the
posting of a cash or surety bond issued by a reputablebonding company duly accredited by the NLRC, in an amount
equivalent ot he moentary award in the judgment appealed from.

Section 4 (a) and 6 of Rule VI of the New rules of Procedure of the NLRC, as amended, reaffirm the explicit
jursidictional principle in Article 223.

The posting of a bond is indispensable to the perfection of an appeal in cases involving monetary awards
from the decisions of the Labor Arbiter. The lawmakers clearly intended to make the bond a mandatory requisite for
the perfection of the appeal by the employer as inferred from the provision that an appealby the employer may be
perfected “only upon the posting of a cash or surety bond”. The word “only” makes it clear that the posting of a cash
or surety bond by the employer is the essential and exclusive means by which an employer’s appeal may be
perfected. Moreover, the filing of the bond is not only mandatory, but a jurisdictional requirement as well that must be
complied with in order to confer jurisdiction upon the NLRC. Non compliance therewith renders the deicsion of the
Labor Arbiter final and executory. This requirement is intended to assure the workders that if they prevail in the case,
they will receive the money judgment in their upon the dismissal of the employer’s appeal. It is intended to discourage
employers from using an appeal to delay or evade their obligation to satisfy their employees’ just and lawful claims.

Here, it is undisputed that respondent’s applea was not accompanied by any appealbond despite the clear
monetary obligation to pay petitioner his separation pay in the amount of Php 100,000.00. Since the posting of a bond
for the perfection of an appeal is both mandatory and jurisdictional, the decision of the Labor Arbiter sought to be
appealed before the NLRC had already become final and executory. Therefore, the NLRC had no authroity to
entertain the appeal, much less to reverse the decision of the Labor Arbiter.

Nevertheless, assuming that the NLRC has jurisdction to take cognizance of the instant case, this Court is
inclined to favor petitioner because the instant case falls under one of the recognized exception to the rule that a
motion for reconsideration is necessary prior to the filing of a certiorari petition.

The general rule is that a motion for reconsideration is indispensable before resort to the special civil action
for certiorari to afford the court or tribunal the opportunity to correct its error, if any. The rule is well settled that the
filing of a motion for reconsideration is an indispens able condition to the filing of a special civil action for certiorari.

However, said rule is subject to several recognized exceptions:


(b) Where the questions raised in the certiorari proceedings have been duly raised and passed upon by the
lower court, or are the same as those raised and passed upon in the lower court

Here. The NLRC had all the opprotunity to review its ruling and correct itself.

All told, the petition is meritorious. However, since this Court is not a trier of facts, we cannot rule on the
substantive issue of the case, i.e., whether petitioner has attained regular status, inasmuch as the CA has not yet
passed upon The factual issues raised by the parties

144. Bergonio et al., vs. South east Asian Airlines, GR No. 195227,.April 21, 2014

Facts:

On April 30, 2004, the petitioners filed before the LA a complaint for illegal dismissal and illegal suspension
with prayer for reinstatement against respondents South East Asian Airlines (SEAIR) and Irene Dornier as SEAIR’s
President (collectively, the respondents).

In a decision dated May 31, 2005, the LA found the petitioners illegally dismissed and ordered the
respondents, among others, to immediately reinstate the petitioners with full backwages. The respondents received
their copy of this decision on July 8, 2005.6

On August 20, 2005, the petitioners filed before the LA a Motion for issuance of Writ of Execution for their
immediate reinstatement.

During the scheduled pre-execution conference held on September 14, 2005, the respondents manifested
their option to reinstate the petitioners in the payroll. The payroll reinstatement, however, did not materialize. Thus, on
September 22, 2005, the petitioners filed before the LA a manifestation for their immediate reinstatement

On October 3, 2005, the respondents filed an opposition to the petitioners’ motion for execution. 7 They
claimed that the relationship between them and the petitioners had already been strained because of the petitioners’
threatening text messages, thus precluding the latter’s reinstatement.

On October 7, 2005, the LA granted the petitioners’ motion and issued a writ of execution.8

The respondents moved to quash the writ of execution with a prayer to hold in abeyance the implementation of the
reinstatement order.9 They maintained that the relationship between them and the petitioners had been so strained
that reinstatement was no longer possible.

In an order dated August 15, 2006, 13 the NLRC dismissed the respondents’ appeal for non-perfection. The
NLRC likewise denied the respondents’ motion for reconsideration in its November 29, 2006 resolution, prompting the
respondents to file before the CA a petition for certiorari.

The records show that the petitioners appealed the December 18, 2007 CA decision with this Court. In a
resolution dated August 4, 2008, the Court denied the petition. The Court likewise denied the petitioners’ subsequent
motion for reconsideration, and thereafter issued an Entry of Judgment certifying that its August 4, 2008 resolution
had become final and executory on March 9, 2009.

On January 31, 2008, the petitioners filed with the LA an Urgent Ex-Parte Motion for the Immediate Release
of the Garnished Amount.

In its March 13, 2008 order, the LA granted the petitioners’ motion; it directed Metrobank-San Lorenzo to
release the P1,900,000.00 garnished amount. The LA found valid and meritorious the respondents’ claim for accrued
wages in view of the respondents’ refusal to reinstate the petitioners despite the final and executory nature of the
reinstatement aspect of its (LA’s) May 31, 2005 decision. The LA noted that as of the December 18, 2007 CA
decision (that reversed the illegal dismissal findings of the LA), the petitioners’ accrued wages amounted to
P3,078,366.33.

In its July 16, 2008 resolution, the NLRC affirmed in toto the LA’s March 13, 2008 order. The NLRC
afterwards denied the respondents’ motion for reconsideration for lack of merit.

Issue: Whether or not the order of reinstatement is immediately executory

Ruling:

Nature of the reinstatement aspect of the LA’s decision on a finding of illegal dismissal

Article 223 (now Article 229) of the Labor Code governs appeals from, and the execution of, the LA’s decision.
Pertinently, paragraph 3, Article 223 of the Labor Code provides:

Article 223. APPEAL

xxxx

In any event, the decision of the Labor Arbiter reinstating a dismissed or separated employee, insofar as the
reinstatement aspect is concerned, shall immediately be executory, pending appeal. The employee shall either be
admitted back to work under the same terms and conditions prevailing prior to his dismissal or separation or, at the
option of the employer, merely reinstated in the payroll. The posting of a bond by the employer shall not stay the
execution for reinstatement provided herein. [Emphasis and underscoring supplied]

Under paragraph 3, Article 223 of the Labor Code, the LA’s order for the reinstatement of an employee
found illegally dismissed is immediately executory even during pendency of the employer’s appeal from the decision.
Under this provision, the employer must reinstate the employee – either by physically admitting him under the
conditions prevailing prior to his dismissal, and paying his wages; or, at the employer’s option, merely reinstating the
employee in the payroll until the decision is reversed by the higher court. 22 Failure of the employer to comply with the
reinstatement order, by exercising the options in the alternative, renders him liable to pay the employee’s salaries.

Otherwise stated, a dismissed employee whose case was favorably decided by the LA is entitled to receive
wages pending appeal upon reinstatement, which reinstatement is immediately executor. Unless the appellate
tribunal issues a restraining order, the LA is duty bound to implement the order of reinstatement and the employer
has no option but to comply with it.

Moreover, and equally worth emphasizing, is that an order of reinstatement issued by the LA is self-
executory, i.e., the dismissed employee need not even apply for and the LA need not even issue a writ of execution
to trigger the employer’s duty to reinstate the dismissed employee.

In short, therefore, with respect to decisions reinstating employees, the law itself has determined a
sufficiently overwhelming reason for its immediate and automatic execution even pending appeal. The employer is
duty-bound to reinstate the employee, failing which, the employer is liable instead to pay the dismissed employee’s
salary. The Court’s consistent and prevailing treatment and interpretation of the reinstatement order as immediately
enforceable, in fact, merely underscores the right to security of tenure of employees that the Constitution protects.

145. Arabit et al., vs. Jardine Pacific Finance Inc. GR No. 181719, April 21, 2014

Facts:

Petitioners were former regular employees of respondent Jardine Pacific Finance, Inc. (formerly MB Finance)
(Jardine). The petitioners were also officers and members of MB Finance Employees Association-FFW Chapter (the
Union), a legitimate labor union and the sole exclusive bargaining agent of the employees of Jardine.

On the claim of financial losses, Jardine decided to reorganize and implement a redundancy program among its
employees. The petitioners were among those affected by the redundancy program. Jardine thereafter hired
contractual employees to undertake the functions these employees used to perform.

On June 1, 1999, the petitioners and the Union filed a complaint against Jardine with the NLRC for illegal dismissal
and unfair labor practice.
The LA ruled in the petitioners’ favor. The LA held that the hiring of contractual employees to replace the petitioners
directly contradicts the concept of redundancy which involves the trimming down of the workforce because a task is
being carried out by too many people. The LA explained that the company’s action was a circumvention of the right of
the petitioners to security of tenure.

The LA further held that it was not enough for Jardine to simply focus on its losses. According to the LA, it was error
for Jardine to simply lump together the seven petitioners as employees whose positions have become redundant
without explaining why their respective positions became superfluous in relation to the other positions and employees
of the company.23 On the petitioners’ allegation of unfair labor practice, the LA held that not enough evidence was
presented to prove the claim against Jardine.

Both parties appealed the LA’s decision to the NLRC. But the NLRC dismissed the appeals and affirmed the LA’s
decision in its entirety. Jardine moved for the reconsideration of the NLRC’s decision, which motion the NLRC also
denied. Jardine thereafter sought recourse with the CA via a petition for certiorari under Rule 65.

The CA reversed the LA’s and the NLRC’s rulings, and granted Jardine’s petition for certiorari.

The CA found that Jardine’s act of hiring contractual employees in replacement of the petitioners does not run
counter to the argument that their positions are already superfluous. According to the CA, the hiring of contractual
employees is a management prerogative that Jardine has the right to exercise. In the absence of any showing of
malice or arbitrariness on the part of Jardine in implementing its redundancy program, the courts must not interfere
with the company’s exercise of a bona fide management decision.

The CA denied the petitioners’ subsequent motion for reconsideration.

Issue:

whether or not the petitioners were illegally dismissed.

Ruling:

Redundancy in contrast with retrenchment

Jardine, in its petition for certiorari with the CA, posited that the distinction between redundancy and retrenchment is
not material.48 It contended that employers resort to these causes of dismissal for purely economic
considerations.49 Jardine further argued that the immateriality of the distinction between these two just causes for
dismissal is shown by the fact that redundancy and retrenchment are found and lumped together in just one single
provision of the Labor Code (Article 283 thereof).

We cannot accept Jardine’s shallow understanding of the concepts of redundancy and retrenchment in determining
the validity of the severance of an employer-employee relationship. The fact that they are found together in just one
provision does not necessarily give rise to the conclusion that the difference between them is immaterial. This Court
has already ruled before that retrenchment and redundancy are two different concepts; they are not synonymous;
thus, they should not be used interchangeably.50 The clear distinction between these two concepts was discussed in
Andrada, et al., v. NLRC, citing the case of Sebuguero v. NLRC, where this Court clarified:

Redundancy exists where the services of an employee are in excess of what is reasonably demanded by the actual
requirements of the enterprise. A position is redundant where it is superfluous, and superfluity of a position or
positions may be the outcome of a number of factors, such as over hiring of workers, decreased volume of business,
or dropping of a particular product line or service activity previously manufactured or undertaken by the enterprise.

Retrenchment, on the other hand, is used interchangeably with the term "lay-off." It is the termination of employment
initiated by the employer through no fault of the employee’s and without prejudice to the latter, resorted to by
management during periods of business recession, industrial depression, or seasonal fluctuations, or during lulls
occasioned by lack of orders, shortage of materials, conversion of the plant for a new production program or the
introduction of new methods or more efficient machinery, or of automation. Simply put, it is an act of the employer of
dismissing employees because of losses in the operation of a business, lack of work, and considerable reduction on
the volume of his business, a right consistently recognized and affirmed by this Court.

These rulings appropriately clarify that redundancy does not need to be always triggered by a decline in the business.
Primarily, employers resort to redundancy when the functions of an employee have already become superfluous or in
excess of what the business requires. Thus, even if a business is doing well, an employer can still validly dismiss an
employee from the service due to redundancy if that employee’s position has already become in excess of what the
employer’s enterprise requires.

From this perspective, it is illogical for Jardine to terminate the petitioners’ employment and replace them with
contractual employees. The replacement effectively belies Jardine’s claim that the petitioners’ positions were
abolished due to superfluity. Redundancy could have been justified if the functions of the petitioners were transferred
to other existing employees of the company.

To dismiss the petitioners and hire new contractual employees as replacements necessarily give rise to the sound
conclusion that the petitioners’ services have not really become in excess of what Jardine’s business requires. To
replace the petitioners who were all regular employees with contractual ones would amount to a violation of their right
to security of tenure. For this, we affirm the NLRC’s ruling, citing the LA’s decision, when it ruled:
In the case at bench, respondents did not dispute that after laying-off complainants herein, they engaged the services
of an agency to perform the tasks used to be done by complainants. This is [in direct] contradiction to the concept of
redundancy which precisely requires the trimming down of the [workforce] because a task is being carried out by just
too many people. The subsequent contracting out to an agency the functions or duties that used to be the domain of
individual complainants herein is a circumvention of their constitutional rights to security of tenure, and therefore
illegal.

146. Mirant (Phils) Corp., vs. Caro, GR No. 181490, April 23, 2014

Facts:

Respondent was hired by Mirant Pagbilao on January 3, 1994 as its Logistics Officer. In 2002, when Southern
Company was sold to Mirant, respondent was already a Supervisor of the Logistics and Purchasing Department of
petitioner. At the time of the severance of his employment, respondent was the Procurement Supervisor of Mirant
Pagbilao assigned at petitioner corporation's corporate office. As Procurement Supervisor, his main task was to serve
as the link between the Materials Management Department of petitioner corporation and its staff, and the suppliers
and service contractors in order to ensure that procurement is carried out in conformity with set policies, procedures
and practices. In addition, respondent was put incharge of ensuring the timely, economical, safe and expeditious
delivery of materials at the right quality and quantity to petitioner corporation's plant. Respondent was also
responsible for guiding and overseeing the welfare and training needs of the staff of the Materials Management
Department. Due to the nature of respondent's functions, petitioner corporation considers his position as confidential.

Respondent filed a complaint for illegal dismissal and money claims for 13th and 14th month pay, bonuses and other
benefits, as well as the payment of moral and exemplary damages and attorney's fees.

On November 3, 2004, petitioner corporation conducted a random drug test where respondent was randomly chosen
among its employees who would be tested for illegal drug use. Through an Intracompany Correspondence, these
employees were informed that they were selected for random drug testing to be conducted on the same day that they
received the correspondence. Respondent was duly notified that he was scheduled to be tested after lunch on that
day. His receipt of the notice was evidenced by his signature on the correspondence.

Respondent avers that at around 11:30 a.m. of the same day, he received a phone call from his wife's colleague who
informed him that a bombing incident occurred near his wife's work station in Tel Aviv, Israel where his wife was then
working as a caregiver. Respondent attached to his Position Paper a Press Release of the Department of Foreign
Affairs (DFA) in Manila to prove the occurrence of the bombing incident and a letter from the colleague of his wife
who allegedly gave him a phone call from Tel Aviv.

On that same day, at around 6:15 p.m., respondent returned to petitioner corporation's office. When he was finally
able to charge his cellphone at the office, he received a text message from Tina Cecilia (Cecilia), a member of the
Drug Watch Committee that conducted the drug test, informing him to participate in the said drug test. He
immediately called up Cecilia to explain the reasons for his failure to submit himself to the random drug test that day.
He also proposed that he would submit to a drug test the following day at his own expense. Respondent never heard
from Cecilia again. The Investigating Panel found respondent's explanations as to his whereabouts on that day to be
inconsistent, and recommended that he be suspended for four weeks without pay.

Issue:

Whether or not there was illegal dismissal

Ruling:

There was illegal dismissal in the case at bar.

While the adoption and enforcement by petitioner corporation of its Anti-Drugs Policy is recognized as a valid
exercise of its management prerogative as an employer, such exercise is not absolute and unbridled. Managerial
prerogatives are subject to limitations provided by law, collective bargaining agreements, and the general principles
of fair play and justice. In the exercise of its management prerogative, an employer must therefore ensure that the
policies, rules and regulations on work-related activities of the employees must always be fair and reasonable and the
corresponding penalties, when prescribed, commensurate to the offense involved and to the degree of the infraction.
47 The Anti-Drugs Policy of Mirant fell short of these requirements.

Petitioner corporation's subject Anti-Drugs Policy fell short of being fair and reasonable.

First. The policy was not clear on what constitutes "unjustified refusal" when the subject drug policy prescribed that
an employee's "unjustified refusal" to submit to a random drug test shall be punishable by the penalty of termination
for the first offense. To be sure, the term "unjustified refusal" could not possibly cover all forms of "refusal" as the
employee's resistance, to be punishable by termination, must be "unjustified." To the mind of the Court, it is on this
area where petitioner corporation had fallen short of making it clear to its employees — as well as to management —
as to what types of acts would fall under the purview of "unjustified refusal."

It is not a mere jurisprudential principle, but an enshrined provision of law, that all doubts shall be resolved in favor of
labor. Thus, in Article 4 of the Labor Code, as amended, "[a]ll doubts in the implementation and interpretation of the
provisions of [the Labor] Code, including its implementing rules and regulations, shall be resolved in favor of labor." In
Article 1702 of the New Civil Code, a similar provision states that "[i]n case of doubt, all labor legislation and all labor
contracts shall be construed in favor of the safety and decent living for the laborer." Applying these provisions of law
to the circumstances in the case at bar, it is not fair for this Court to allow an ambiguous policy to prejudice the rights
of an employee against illegal dismissal. To hold otherwise and sustain the stance of petitioner corporation would be
to adopt an interpretation that goes against the very grain of labor protection in this jurisdiction. As correctly stated by
the Labor Arbiter, "when a conflicting interest of labor and capital are weighed on the scales of social justice, the
heavier influence of the latter must be counter-balanced by the sympathy and compassion the law must accord the
underprivileged worker."

Second. The penalty of termination imposed by petitioner corporation upon respondent fell short of being reasonable.
Company policies and regulations are generally valid and binding between the employer and the employee unless
shown to be grossly oppressive or contrary to law — as in the case at bar.

To be sure, the unreasonableness of the penalty of termination as imposed in this case is further highlighted by a fact
admitted by petitioner corporation itself: that for the ten-year period that respondent had been employed by petitioner
corporation, he did not have any record of a violation of its company policies.

147. Castro Jr. vs. Ateneo de Naga University et al., GR No. 175293, July 23, 2014

FACTS:

The petitioner started his employment with respondent Ateneo de Naga University (University) in the first semester of
school year 1960-1961. At the time of his dismissal, he was a regular and full-time faculty member of the University's
Accountancy Department in the College of Commerce with a monthly salary of P29,846.20. Allegedly, he received on
February 22, 2000 a letter from respondent Fr. Joel Tabora, SJ., the University President, informing him that his
contract (which was set to expire on May 31, 2000) would no longer be renewed. After several attempts to discuss the
matter with Fr. Tabora in person, and not having been given any teaching load or other assignments effective June
2000, he brought his complaint for illegal dismissal.

The University denied the allegation of illegal dismissal, and maintained that the petitioner was a participant and
regular contributor to the Ateneo de Naga Employees Retirement Plan (Plan); that upon reaching the age of 60 years
on June 26, 1999, he was deemed automatically retired under the Plan; and that he had been allowed to teach after
his retirement only on contractual basis.

On September 3, 2001, Labor Arbiter (LA) Jesus Orlando M. Quinones ruled in favor of the petitioner.

Aggrieved, the respondents appealed to the NLRC.Simultaneously, they submitted a manifestation stating that
neither actual nor payroll reinstatement of the petitioner could be effected because he had meanwhile been employed
as a Presidential Assistant for Southern Luzon Affairs with the position of Undersecretary; and that his reinstatement
would result in dual employment and double compensation which were prohibited by existing civil service rules and
regulations.

On July 12, 2002, the petitioner, citing the executory nature of the order for his reinstatement, filed his motion
to order the respondents to pay his salaries and benefits accruing in the period from September 3, 2001 until
July 3, 2002.

In his order dated October 10, 2002, LA Quinones, explaining that Article 223 of the Labor Code granted to the
employer the option to implement either a physical or a payroll reinstatement, and that, therefore, the
respondents must first exercise the option regardless of the petitioner's employment with the Government,
denied the petitioner's motion, but ordered the respondents to exercise the option of either actual or payroll
reinstatement of the petitioner.

Dissatisfied, the petitioner filed a notice of partial appeal, but the notice was denied due course on June 30, 2003.

Upon the denial of his motion for reconsideration, the petitioner elevated the matter to the CA by petition for
certiorari.

In the interim, on June 26, 2004, the petitioner executed a receipt and quitclaim in favor of the University
respecting his claim for the benefits under the Plan, to wit:

RECEIPT and QUITCLAIM

Date: June 26, 2004

This is to acknowledge receipt from ATENEO DE NAGA UNIVERSITY the total sum of SIX HUNDRED FORTY SIX
THOUSAND EIGHT HUNDRED TWENTY EIGHT PESOS & 42/100 (P646,828.42) represen: ing full payment of
benefits due me pursuant to the Employees retirement plan. In view of this payment, I hereby waive all my rights,
title, interest in and over my retirement benefits under said plan which is presently under trusteeship of Bank of
the Philippine Islands. BPI is hereby instructed to reimburse the company for the amount paid by it to me out of
whatever amount due me under the said retirement plan.

(sgd.)
CRISANTO F. CASTRO, JR.
Employee

A few days later, the petitioner sent the following letter to Fr. Tabora, viz:
Dear Fr. fabora,

This is tJ inform you that I am getting my retirement pay as you have approved, together with the "RECEIPT
AND QUIT CLAIM" which your Treasurer forced me to sign upon your order and/or your lawyer. I will receive
pay UNDER PROTEST, and under the following conditions:

1. That I am getting this retirement pay without prejudice to the case that I have filled [sic] against Ateneo,
Fr. Joel Tabora and Edwin Bernal.

2. That I do not agree nor confirm with your computation as to the number of years of service I have
rendered.

3. That the total amount is still subject to verification.

For your information.

(sgd.)
CRISANTO F. CASTRO, JR.

Meanwhile, the NLRC rendered its decision affirming with modification the ruling of the LA on the petitioner's
illegal dismissal case.

However, upon motion for reconsideration, the NLRC REVERSED its ruling. In justifying its reversal of its
decision, the NLRC held that his execution of the receipt and quitclaim respecting his benefits under the Plan
estopped the petitioner from pursuing other claims arising from his employer-employee relationship with the
University, opining that:

Actually, the execution and signing of the Receipt and Quitclaim by complainantappellee, in this case, only
indicates that he voluntarily waived his rights to his money awards, as stated in the Labor Arbiter's Decision, as
affirmed with modification by the Commission (Second Division). A person is precluded from maintaining a position
inconsistent with one, in which he has acquiesced x x x. Also, in his signing the said Receipt and Quitclaim, the
necessary implication is that the said document would cover any and all claims arising out of the employment
relationship.

On May 31, 2006, the CA dismissed the petitioner's petition for certiorari on the ground of its having been rendered
moot and academic by the aforecited August 31, 2005 decision of the NLRC.

ISSUE:

1. WON the petitioner was stopped from pursuing his complaint for illegal dismissal upon execution of the
receipt and quitclaim

2. WON Petitioner has a right to accrued benefits.

RULING:

1. NO. Petitioner was stopped from pursuing his complaint for illegal dismissal upon execution of the receipt and
quitclaim. Execution of the receipt and quitclaim was not a settlement of the petitioner's claim for accrued
salaries.

The NLRC held that the petitioner was estopped from pursuing his complaint for illegal dismissal upon his receipt of
the benefits and his execution of the receipt and quitclaim. He insists, however, that the payment he had received in
protest pertained only to his retirement benefits.

We agree with the petitioner.

The text of the receipt and quitclaim was clear and straightforward, and it was to the effect that the sum received by
the petitioner represented ''full payment of benefits ... pursuant to the Employee's retirement plan." As such, both the
NLRC and the CA should have easily seen that the quitclaim related only to the settlement of the retirement benefits,
which benefits could not be confused with the reliefs related to the complaint for illegal dismissal.

Worthy to stress is that retirement is of a different species from the reliefs awarded to an illegally dismissed
employee. Retirement is a form of reward for an employee's loyalty and service to the employer, and is intended to
help the employee enjoy the remaining years of his life, and to lessen the burden of worrying about his financial
support or upkeep. In contrast, the reliefs awarded to an illegally dismissed employee are in recognition of the
continuing employer-employee relationship that has been severed by the employer without just or authorized cause,
or without compliance with due process.

2. YES. He is entitled to the benefits. Claim for accrued benefits should be sustained despite dismissal of the
petitioner's complaint

The petitioner argues that according to Roquero v. Philippine Airlines, Inc., the employer is obliged to reinstate and to
pay the wages of the dismissed employee during the period of appeal until its reversal by the higher Court; and that
because he was not reinstated either actually or by payroll, he should be held entitled to the accrued salaries.

The argument of the petitioner is correct.

Article 279 of the Labor Code, as amended, entitles an illegally dismissed employee to reinstatement. Article 223 of
the Labor Code requires the reinstatement to be immediately executory even pending appeal. With its intent being
ostensibly to promote the benefit of the employee, Article 223 cannot be the source of any right of the employer to
remove the employee should he fail to immediately comply with the order of reinstatement. In Roquero, the Comi
ruled that the unjustified refusal of the employer to reinstate the dismissed employee would entitle the latter to the
payment of his salaries effective from the time when the employer failed to reinstate him; thus, it became the
ministerial duty of the LA to implement the order of reinstatement. According to Triad Security & Allied Services v.
Ortega, Jr., the law mandates the prompt reinstatement of the dismissed or separated employee, without need of any
writ of execution.

The provision of Article 223 is clear that an award for reinstatement shall be immediately executory even
pending appeal and the posting of a bond by the employer shall not stay the execution for reinstatement. The
legislative intent is quite obvious, i.e., to make an award of reinstatement immediately enforceable, even penoing
appeal. To require the application for and issuance of a wit of execution as prerequisites for the execution of a
reinstatement award v10uld certainly betray and run counter to the very object and intent of Article 223, i.e., the
immediate execution of a reinstatement order. The reason is simple. An application for a writ of execution and its
issuance could be delayed for numerous reasons. A mere continuance of postponement of a scheduled hearing, for
instance, or an inaction on the part of the Labor Arbiter or the NLRC could easily delay the issuance of the writ
thereby setting at naught the strict mandate and noble purpose envisioned by Article 223. In other words, if the
requirements of Article 224 were to govern, as we so declared in Maranaw, then the executory nature of a
reinstatement order or award contemplated by Article 223 will be unduly circumscribed and rendered ineffectual. In
enacting the law, the legislature is presumed to have ordained a valid and sensible law, one which operates no
further than may be necessary to achieve its specific purpose. Statutes, as a rule, are to be construed in the lights of
the purpose to be achieved and the evil sought to be remedied. And where the statute is fairly susceptible of two or
more constructions, that construction should be adopted which will most tend to give effect to the manifest intent of
the lawmaker and promote the object for which the statute was enacted, and a construction should be rejected which
would tend to render abortive other provisions of the statute and to defeat the object which the legislator sought to
attain by its enactment. In introducing a new rule on the reinstatement aspect of a labor decision under R.A. No.
6715, Congress should not be considered to be indulging in mere semantic exercise. On appeal, however, the
apellate tribunal concerned may enjoin or suspend the reinstatement order in the exercise of its sound discretion.

Furthermore, the rule is that all doubts in the interpretation and implementation of labor laws should be resolved in
favor of labor. In ruling that an order or award for reinstatement does not require a writ of execution, the Court is
simply adhering and giving meaning to this rule. Henceforth, we rule that an award or order for reinstatement is
selfexecutory. After receipt of the decision or resolution ordering the employee's reinstatement, the
employer has the right to choose whether to re-admit the employee to work under the same terms and
conditions prevailing prior to his dismissal or to reinstate the employee in the payroll. In either instance, the
employer has to inform the employee of his choice. The notification is based on practical considerations for
without notice, the employee has no way of knowing if he has to report for work or not.

Hence, for as long as the employer continuously fails to actually implement the reinstatement aspect of the
decision of the LA, the employer's obligation to the employee for his accrued backwages and other benefits
continues to accumulate.

The next issue concerns whether or not the petitioner's claim for accrued salaries from the time of the issuance of the
order of reinstatement by LA Quinones until his actual reinstatement in November 2002 was rendered moot and
academic by the reversal of the decision of the LA.

The Court holds that the order of reinstatement of the petitioner was not rendered moot and academic. He
remained entitled to accrued salaries from notice of the LA's order of reinstatement until reversal thereof .
The employee could be barred from claiming accrued salaries only when the failure to reinstate him was
without the fault of the employer.

Considering that the respondents reinstated the petitioner only in November 2002, and that their inability to reinstate
him was without valid ground, they were liable to pay his salaries accruing from the time of the decision of the LA
(i.e., September 3, 2001) until his reinstatement in November 2002. It did not matter that the respondents had yet to
exercise their option to choose between actual or payroll reinstatement at that point because the order of
reinstatement was immediately executory.

148. Phil. Touristers Inc et al., vs. Mas Transit Workers Union-ANGLO-KMU GR No. 201237, Sept. 3, 2014

Facts:

On June 14, 2000, (the union) filed for certification election before (DOLE). DOLE granted the union’s petition,
prompting the client (MTI) to file a motion for reconsideration dated Feb. 7, 2001. Subsequently (MTI) prompted to
sell its buses its franchise to another bus operator (PTI). In view of the sale, MTI informed all its employees of the
said sale and transfer its operations to PTI. Also MTI advised the employees to apply to PTI.

The union filed a complaint for illegal dismissal on the grounds that this scheme was to frustrate their right to self-
organization and there was no actual transfer of ownership of the passenger buses as the stockholders of MTI and
PTI are one and the same. In their defense, MTI claimed that it was already suffering from serious financial reverses.

LA ruling – found MTI guilty of unfair labor practice, i.e illegal lock out. The LA held that MTI’s closure of business and
cessation of operations, allegedly due to serious financial reverses, were actually made to subvert the right of its
employees to self-organization. Accordingly, MTI and petitioners were adjudged jointly and severally liable for the
individual respondents’ backwages, separation pay, and attorney’s fees.

NLRC proceedings – Finding merit in petitioners’ motion for reconsideration, the NLRC, in an Order45 dated
September 30, 2004, reinstated their appeal. It held that there was substantial compliance with the rules considering
the subsequent posting of an additional bond to complete the full judgment award, adding too that petitioners’ initial
motion to reduce bond was based on a meritorious ground – that is, the inability of PTI to post the full amount due to
its liquidity problems as evidenced by its submitted AFS.46 However, considering that PTI’s bonding company,
SSSICI, was not authorized to transact business in all courts all over the country per the Court’s Certification dated
August 6, 2004, petitioners were directed to replace the bond,47 which they timely complied with through the posting
of Supersedeas Bond No. SS-B-10150,48 in the amount of _12,833,000.00, issued on November 8, 2004 by the Far
Eastern Surety & Insurance Company, Inc.

On January 20, 2006, the NLRC rendered a Decision, modifying its April 19, 2004 Decision by dismissing the
complaint against petitioners. The modification was brought about by the NLRC’s finding that there were no factual
and legal bases to hold petitioners jointly and severally liable with MTI as the two corporations are separate and
distinct juridical entities with different stockholders and owners. To this end, it ruled that the individual respondents
were employees of MTI and not PTI, and that the sale of the passenger buses to PTI was not simulated or fictitious
since the deed evidencing said sale was duly notarized and approved by the LTFRB in a Decision dated December
28, 2000.

CA Ruling - CA annulled and set aside the modified ruling of the NLRC finding the latter to have acted with grave
abuse of discretion in applying a liberal interpretation of the rules on perfection of appeal. It held that PTI’s alleged
liquidity problems cannot be considered as a meritorious ground to reduce the bond as there was no showing that
they were incapable of posting at least a surety bond equivalent to the full judgment award

ISSUE:

The central issue for the Court’s resolution is whether or not the CA erred in ascribing grave abuse of discretion on
the part of the NLRC when the latter gave due course to petitioners’ appeal and consequently issued a modified
Decision absolving petitioners from liability.

Ruling:

For an appeal from the LA’s ruling to the NLRC to be perfected, Article 223 (now Article 229)61 of the Labor Code
requires the posting of a cash or surety bond in an amount equivalent to the monetary award in the judgment
appealed from.

In case of a judgment involving a monetary award, an appeal by the employer may be perfected only upon the
posting of a cash or surety bond issued by a reputable bonding company duly accredited by the Commission in the
amount equivalent to the monetary award in the judgment appealed from.

While it has been settled that the posting of a cash or surety bond is indispensable to the perfection of an appeal in
cases involving monetary awards from the decision of the LA,62 the Rules of Procedure of the NLRC63 (the Rules),
particularly Section 6, Rule VI thereof, nonetheless allows the reduction of the bond upon a showing of (a) the
existence of a meritorious ground for reduction, and (b) the posting of a bond in a reasonable amount in relation to
the monetary award be reversed and set aside. However, considering that there were other issues raised in the said
petition relating to the substantial merits of the case which were left undecided, a remand of the case for the CA’s
resolution of these substantive issues remains in order, in line with the doctrine of hierarchy of courts as espoused in
the St. Martin Funeral Home v. NLRC ruling.

149. Azuelo vs. Zameco II Electric Cooperative, Inc. GR No. 192573, October 22, 2014

Facts: Petitioner Ricardo N. Azuelo (Azuelo) was employed by the respondent ZAMECO II Electric Cooperative, Inc.
(ZAMECO) as a maintenance worker. a Complaint for illegal dismissal and non-payment of benefits against
ZAMECO. After several mediations, LA Bactin ordered the parties to submit their respective position papers.

Azuelo, instead of submitting his position paper, moved that the submission of his position paper be extended. This
was granted by the LA. The deadline came, and the LA directed Azuelo to file his position paper. Azuelo, instead,
moved for the issuance of an order to direct ZAMECO to furnish him with a copy of the investigation report with
regard to his dismissal. ZAMECO opposed the motion on ground that they had already furnished Azuelo with a copy.
So the LA dismissed the complaint on ground of "lack of interest".

Azuelo filed another complaint with the same allegations as his first complaint with the RAB of the NLRC. ZAMECO
filed a motion to dismiss on ground of res judicata. Azuelo countered that the dismissal was without prejudice. The LA
dismissed the 2nd complaint. The NLRC denied reconsideration, and the CA affirmed.

Issue:

Whether the dismissal of his first complaint for illegal dismissal, on the ground of lack of interest on his part to
prosecute the same, bars the filing of another complaint for illegal dismissal against ZAMECO based on the same
allegations.

Ruling:

The 2005 Revised Rules of Procedure of the NLRC (2005 Revised Rules), the rules applicable at the time of the
controversy, is silent as to the nature of the dismissal of a complaint on the ground of unreasonable failure to submit
a position paper by the complainant. Nevertheless, the 2005 Revised Rules, particularly Section 3, Rule I thereof,
provides for the suppletory application of the Rules of Court to arbitration proceedings before the LAs and the NLRC
in the absence of any applicable provisions therein, viz:

Section 3. Suppletory Application of the Rules of Court. - In the absence of any applicable provisions in these Rules,
and in order to effectuate the objectives of the Labor Code, the pertinent provisions of the Rules of Court of the
Philippines may, in the interest of expeditious dispensation of labor justice and whenever practicable and convenient,
be applied by analogy or in a suppletory character and effect.

The unjustified failure of a complainant in arbitration proceedings before the LA to submit his position paper is akin to
the case of a complainant's failure to prosecute his action for an unreasonable length of time in ordinary civil
proceedings. In both cases, the complainants are remiss, sans reasonable cause, to prove the material allegations in
their respective complaints. Accordingly, the Court sees no reason not to apply the rules relative to unreasonable
failure to prosecute an action in ordinary civil proceedings to the unjustified failure of a complainant to submit his
position paper in arbitration proceedings before the LA.

In this regard, Section 3, Rule 17 of the Rules of Court provides that:

Section 3. Dismissal due to fault of plaintiff. — If, for no justifiable cause, the plaintiff fails to appear on the date of the
presentation of his evidence in chief on the complaint, or to prosecute his action for an unreasonable length of time,
or to comply with these Rules or any order of the court, the complaint may be dismissed upon motion of the
defendant or upon the court's own motion, without prejudice to the right of the defendant to prosecute his
counterclaim in the same or in a separate action. This dismissal shall have the effect of an adjudication upon the
merits, unless otherwise declared by the court.

"The dismissal of a case for failure to prosecute has the effect of adjudication on the merits, and is necessarily
understood to be with prejudice to the filing of another action, unless otherwise provided in the order of dismissal.
Stated differently, the general rule is that dismissal of a case for failure to prosecute is to be regarded as adjudication
on the merits and with prejudice to the filing of another action, and the only exception is when the order of dismissal
expressly contains a qualification that the dismissal is without prejudice."

Thus, in arbitration proceedings before the LA, the dismissal of a complaint on account of the unreasonable failure of
the complainant to submit his position paper is likewise regarded as an adjudication on the merits and with prejudice
to the filing of another complaint, except when the LA's order of dismissal expressly states otherwise.

The Order which dismissed Azuelo's first complaint due to his unreasonable failure to submit his position paper is
unqualified. It is thus considered as adjudication on the merits and with prejudice to filing of another complaint.
Accordingly, the NLRC did not abuse its discretion when it affirmed LA Abdon's dismissal of the second complaint for
illegal dismissal. Azuelo's filing of a second complaint for illegal dismissal against ZAMECO based on the same
allegations cannot be permitted lest the rule on res judicata be transgressed.
21. RIGHT TO SELF-ORGANIZATION

150. National Union of bank Employees vs. Philnabank Employees Association G.R. No. 174287, Aug. 12,
2013

FACTS:

PNB’s rank-and-file employeeswere represented for collective negotiation by the Philnabank Employees Association
(PEMA). PEMA affiliated with petitioner National Union of Bank Employees (NUBE), which is a labor federation
composed of unions in the banking industry, adopting the name NUBE-PNB Employees Chapter (NUBE-
PEC).NUBE-PEC was certified as the sole and exclusive bargaining agent of the PNB rank-and-file employees.

Pursuant to Article V on Check-off and Agency Fees of the CBA, PNB shall deduct the monthly membership fee and
other assessments imposed by the union from the salary of each union member, and agency fee (equivalent to the
monthly membership dues) from the salary of the rank-and-file employees within the bargaining unit who are not
union members. Moreover, during the effectivity of the CBA, NUBE, being the Federation union, agreed that PNB
shall remit P15.00 of the P65.00 union dues per month collected by PNB from every employee, and that PNB shall
directly credit the amount to NUBE's current account with PNB.

Following the expiration of the CBA, a petition for certification election was filed. While the petition for certification
election was still pending, two significant events transpired — the independent union registration of NUBE-PEC and
its disaffiliation with NUBE.

With a legal personality derived only from a charter issued by NUBE, NUBE-PEC applied for a separate registration
with the DOLE. It was later registered as an independent labor organization.

PEMA then sent a letter to the PNB management informing its disaffiliation from NUBE and requesting to stop,
effective immediately, the check-off of the P15.00 due for NUBE.

Acting thereon, PNB informed NUBE of PEMA's letter and its decision to continue the deduction of the P15.00 fees,
but stop its remittance to NUBE effective July 2003. PNB also notified NUBE that the amounts collected would be
held in a trust account pending the resolution of the issue on PEMA's disaffiliation.
NUBE replied that: it remains as the exclusive bargaining representative of the PNB rank-and-file employees; by
signing the Resolution (on disaffiliation), the chapter officers have abandoned NUBE-PEC and joined another union;
in abandoning NUBE-PEC, the chapter officers have abdicated their respective positions and resigned as such; in
joining another union, the chapter officers committed an act of disloyalty to NUBE-PEC and the general membership.
With regard to the issue of non-remittance of the union dues, NUBE enjoined PNB to comply with the union check-off
provision of the CBA.

ISSUES:

Did PEMA validly disaffiliate itself from NUBE? Consequently, did NUBE still have the right to check-off, or the right to
collect the union dues held in trust by PNB?

RULING:

The right of the local union to exercise the right to disaffiliate from its mother union is well settled in this jurisdiction. In
MSMG-UWP v. Hon. Ramos, We held:

A local union has the right to disaffiliate from its mother union or declare its autonomy. A local
union, being a separate and voluntary association, is free to serve the interests of all its members
including the freedom to disaffiliate or declare its autonomy from the federation which it belongs
when circumstances warrant, in accordance with the constitutional guarantee of freedom of
association.

The purpose of affiliation by a local union with a mother union [or] a federation

". . . is to increase by collective action the bargaining power in respect of the terms and
conditions of labor. Yet the locals remained the basic units of association, free to serve
their own and the common interest of all, subject to the restraints imposed by the
Constitution and By-Laws of the Association, and free also to renounce the affiliation
for mutual welfare upon the terms laid down in the agreement which brought it into
existence."

Thus, a local union which has affiliated itself with a federation is free to sever such affiliation
anytime and such disaffiliation cannot be considered disloyalty. In the absence of specific
provisions in the federation's constitution prohibiting disaffiliation or the declaration of autonomy
of a local union, a local may dissociate with its parent union.

Likewise, Philippine Skylanders, Inc. v. National Labor Relations Commission restated:

The right of a local union to disaffiliate from its mother federation is not a novel thesis unillumined
by case law. In the landmark case of Liberty Cotton Mills Workers Union vs. Liberty Cotton Mills,
Inc., we upheld the right of local unions to separate from their mother federation on the ground
that as separate and voluntary associations, local unions do not owe their creation and
existence to the national federation to which they are affiliated but, instead, to the will of
their members. The sole essence of affiliation is to increase, by collective action, the
common bargaining power of local unions for the effective enhancement and protection
of their interests. Admittedly, there are times when without succor and support local unions may
find it hard, unaided by other support groups, to secure justice for themselves.

Yet the local unions remain the basic units of association, free to serve their own interests
subject to the restraints imposed by the constitution and by-laws of the national federation, and
free also to renounce the affiliation upon the terms laid down in the agreement which brought
such affiliation into existence.

And again, in Coastal Subic Bay Terminal, Inc. v. Department of Labor and Employment — Office of the Secretary,
this Court opined:

Under the rules implementing the Labor Code, a chartered local union acquires legal personality
through the charter certificate issued by a duly registered federation or national union, and
reported to the Regional Office in accordance with the rules implementing the Labor Code. A
local union does not owe its existence to the federation with which it is affiliated. It is a separate
and distinct voluntary association owing its creation to the will of its members. Mere affiliation
does not divest the local union of its own personality, neither does it give the mother
federation the license to act independently of the local union. It only gives rise to a
contract of agency, where the former acts in representation of the latter. Hence, local
unions are considered principals while the federation is deemed to be merely their agent.
As such principals, the unions are entitled to exercise the rights and privileges of a legitimate
labor organization, including the right to seek certification as the sole and exclusive bargaining
agent in the appropriate employer unit.

Finally, the recent case of Cirtek Employees Labor Union-Federation of Free Workers v. Cirtek Electronics, Inc. ruled:
. . . [A] local union may disaffiliate at any time from its mother federation, absent any
showing that the same is prohibited under its constitution or rule. Such, however, does
not result in it losing its legal personality altogether. Verily, Anglo-KMU v. Samahan ng
mgaManggagawangNagkakaisasa Manila Bar Spinning Mills at J.P. Coats enlightens:
ADCSEa

A local labor union is a separate and distinct unit primarily designed to


secure and maintain an equality of bargaining power between the employer
and their employee-members. A local union does not owe its existence
to the federation with which it is affiliated. It is a separate and distinct
voluntary association owing its creation to the will of its members. The mere
act of affiliation does not divest the local union of its own personality,
neither does it give the mother federation the license to act
independently of the local union. It only gives rise to a contract of
agency where the former acts in representation of the latter.

In the case at bar, there is nothing shown in the records nor is it claimed by NUBE that PEMA was expressly
forbidden to disaffiliate from the federation nor were there any conditions imposed for a valid breakaway. This being
so, PEMA is not precluded to disaffiliate from NUBE after acquiring the status of an independent labor organization
duly registered before the DOLE.

Also, there is no merit on NUBE's contention that PEMA's disaffiliation is invalid for non-observance of the procedure
that union members should make such determination through secret ballot and after due deliberation, conformably
with Article 241 (d) of the Labor Code, as amended. Conspicuously, other than citing the opinion of a "recognized
labor law authority," NUBE failed to quote a specific provision of the law or rule mandating that a local union's
disaffiliation from a federation must comply with Article 241 (d) in order to be valid and effective.

Granting, for argument's sake, that Article 241 (d) is applicable, still, We uphold PEMA's disaffiliation from NUBE.
First, non-compliance with the procedure on disaffiliation, being premised on purely technical grounds cannot rise
above the employees' fundamental right to self-organization and to form and join labor organizations of their own
choosing for the purpose of collective bargaining. Second, the Article nonetheless provides that when the nature of
the organization renders such secret ballot impractical, the union officers may make the decision in behalf of the
general membership. In this case, NUBE did not even dare to contest PEMA's representation that "PNB employees,
from where [PEMA] [derives] its membership, are scattered from Aparri to Jolo, manning more than 300 branches in
various towns and cities of the country," hence, "[to] gather the general membership of the union in a general
membership to vote through secret balloting is virtually impossible." It is understandable, therefore, why PEMA's
board of directors merely opted to submit for ratification of the majority their resolution to disaffiliate from NUBE.
Third, and most importantly, NUBE did not dispute the existence of the persons or their due execution of the
document showing their unequivocal support for the disaffiliation of PEMA from NUBE.

Consequently, by PEMA's valid disaffiliation from NUBE, the vinculum that previously bound the two entities was
completely severed. As NUBE was divested of any and all power to act in representation of PEMA, any act performed
by the former that affects the interests and affairs of the latter, including the supposed expulsion of Serranaet al., is
rendered without force and effect.

Also, in effect, NUBE loses it right to collect all union dues held in its trust by PNB. The moment that PEMA
separated from and left NUBE and exists as an independent labor organization with a certificate of registration, the
former is no longer obliged to pay dues and assessments to the latter; naturally, there would be no longer any reason
or occasion for PNB to continue making deductions. As we said inVolkschel Labor Union v. Bureau of Labor
Relations:

. . . In other words, ALUMETAL [NUBE in this case] is entitled to receive the dues from
respondent companies as long as petitioner union is affiliated with it and respondent
companies are authorized by their employees (members of petitioner union) to deduct union
dues. Without said affiliation, the employer has no link to the mother union. The obligation of
an employee to pay union dues is coterminous with his affiliation or membership. "The
employees' check-off authorization, even if declared irrevocable, is good only as long
as they remain members of the union concerned." A contract between an employer and
the parent organization as bargaining agent for the employees is terminated by the
disaffiliation of the local of which the employees are members. . . .

151. Octavio vs. Phil Long Distance Telephone Company, G.R. No. 175492, Feb. 27, 2013

Facts:

PLDT and Gabay ng Unyon sa Telekominaksyon ng mga Superbisor (GUTS) entered into a CBA. Article VI, Section I
thereof provides: Section 1. The COMPANY agrees to grant the following across-the-board salary increase during
the three years covered by this Agreement to all employees covered by the bargaining unit as of the given dates:

Effective January 1, 1999 – 10% of basic wage or P2,000.00 whichever is higher;

Effective January 1, 2000 – 11% of basic wage or P2,250.00 whichever is higher;

Effective January 1, 2001 – 12% of basic wage or P2,500.00 whichever is higher.


When Octavio, a Sales System Analyst and member of GUTS was regularized on January 1, 2001, he was receiving
a monthly basic salary of P10,000.00. On February 1, 2002, he was promoted to the position of Sales System Analyst
2 and his salary was increased to P13,730.00.

On May 31, 2002, PLDT and GUTS entered into another CBA covering the period January 1, 2002 to December 31,
2004 (CBA of 2002-2004) which provided for the following salary increases: 8% of basic wage or P2,000.00
whichever is higher for the first year (2002); 10% of basic wage or P2,700.00 whichever is higher for the second year
(2003); and, 10% of basic wage or P2,400.00 whichever is higher for the third year (2004).

Claiming that he was not given the salary increases of P2,500.00 effective January 1, 2001 and P2,000.00 effective
January 1, 2002, Octavio wrote the President of GUTS, who in turn wrote the PLDT Human Resource Head to inform
management of the GUTS members’ claim for entitlement to the increases. Accordingly, the Union-Management
Grievance Committee convened on October 7, 2002 consisting of representatives from PLDT and GUTS. The
Grievance Committee, however, failed to reach an agreement. In effect, it denied Octavio’s demand for salary
increases. Aggrieved, Octavio filed before the Arbitration Branch of the NLRC a Complaint for payment of said salary
increases.

Issues:

(1) WON Octavio resorted to the proper procedure by filing a Complaint with the LA;
(2) WON the Greivance Committee Resolution was valid;
(3) WON denial of his claim for salary increases constituted a violation of Article 100;

Ruling:

Issue 1: Under Article 260 of the Labor Code, grievances arising from the interpretation or implementation of the
parties’ CBA should be resolved in accordance with the grievance procedure embodied therein. It also provides that
all unsettled grievances shall be automatically referred for voluntary arbitration as prescribed in the CBA.

In its Memorandum, PLDT set forth the grievance machinery and procedure provided under its CBA of 2002-2004
which in part reads: “If the grievance is not settled either because of deadlock or the failure of the committee
(Grievance Committee) to decide the matter, the grievance shall be transferred to a Board of Arbitrators for
the final decision. The Board shall be composed of three (3) arbitrators, one to be nominated by the Union, another
to be nominated by the Management, and the third to be selected by the management and union nominees. The
decision of the board shall be final and binding both the company and the Union in accordance with law. Expenses of
arbitration shall be divided equally between the Company and the Union.”

Indisputably, the present controversy involves the determination of an employee’s salary increases as provided in the
CBAs. When Octavio’s claim for salary increases was referred to the Union-Management Grievance Committee, the
clear intention of the parties was to resolve their differences on the proper interpretation and implementation of the
pertinent provisions of the CBAs. And in accordance with the procedure prescribed therein, the said committee
convened. Unfortunately, it failed to reach an agreement. Octavio’s recourse pursuant to the CBA was to elevate his
grievance to the Board of Arbitrators for final decision. Instead, Octavio filed a Complaint before the NLRC.

It is settled that "when parties have validly agreed on a procedure for resolving grievances and to submit a dispute to
voluntary arbitration then that procedure should be strictly observed." Before a party is allowed to seek the
intervention of the court, it is a precondition that he should have availed of all the means of administrative processes
afforded him (exhaustion of administrative remedies). Hence, if a remedy within the administrative machinery can still
be resorted to by giving the administrative officer concerned every opportunity to decide on a matter that comes
within his jurisdiction, then such remedy should be exhausted first before the court’s judicial power can be sought.
The premature invocation of the court’s judicial intervention is fatal to one’s cause of action. By failing to question the
Committee Resolution through the proper procedure prescribed in the CBA, that is, by raising the same before a
Board of Arbitrators, Octavio is deemed to have waived his right to question the same. Clearly, he departed from the
grievance procedure mandated in the CBA and denied the Board of Arbitrators the opportunity to pass upon a matter
over which it has jurisdiction. Hence, Octavio’s failure to assail the validity and enforceability of the Committee
Resolution makes the same binding upon him.

Issue 2: Octavio cannot claim that the Committee Resolution is not valid, binding and conclusive as to him for being
a modification of the CBA in violation of Article 253 of the Labor Code. The said resolution is a product of the
grievance procedure outlined in the CBA itself arrived at after the management and the union through their respective
representatives conducted negotiations in accordance with the CBA. The union representatives are deemed to have
properly bargained on his behalf since "unions are the agent of its members for the purpose of securing just
and fair wages and good working conditions." In fine, it cannot be gainsaid that the Committee Resolution is a
modification of the CBA. Rather, it only provides for the proper implementation of the CBA provision respecting salary
increases.

Issue 3: Octavio’s argument that the denial of his claim for salary increases constitutes a violation of Article 100 of
the Labor Code is devoid of merit. Even assuming that there has been a diminution of benefits on his part, Article 100
does not prohibit a union from offering and agreeing to reduce wages and benefits of the employees as the right to
free collective bargaining includes the right to suspend it. PLDT averred that one of the reasons why Octavio’s
salary was recomputed as to include in his salary of P13,730.00 the P2,000.00 increase for 2002 is to avoid salary
distortion. Bargaining should not be equated to an "adversarial litigation where rights and obligations are delineated
and remedies applied." Instead, it covers a process of finding a reasonable and acceptable solution to stabilize labor-
management relations to promote stable industrial peace. Clearly, the Committee Resolution was arrived at after
considering the intention of both PLDT and GUTS to foster industrial peace.

152. Takata Phils Corp. vs. Bureau of Labor Relations, et al., GR No. 196276, June 4, 2014

FACTS:

Petioner filed with the DOLE Regional Office a Petitionfor Cancellation of the Certificate of Union Registration of
Respondent SamahangLakasManggagawangTakata (SALAMAT) on the ground that the latter is guilty of
misrepresentation, false statement and fraud with respect to the number of those who participated in the
organizational meeting, the adoption and ratification of its Constitution and By-Laws, and in the election of its
officers. It contended that in the May 1, 2009 organizational meeting of respondent, only 68 attendees signed the
attendance sheet, and which number comprised only 17% of the total number of the 396 regular rank- and-file
employees which respondent sought to represent, and hence, respondent failed to comply with the 20% minimum
membership requirement. Petitioner insisted that the document “PangalanngmgaKasapingUnyon” bore no
signatures of the alleged 119 union members; and that employees were not given sufficient information on the
documents they signed; that the 119 union members were actually only 117; and, that the total number of
petitioner's employees as of May 1, 2009 was 470, and not 396 as respondent claimed.

Respondent denied the charge and claimed that the 119 union members were more than the 20% requirement for
union registration. The document “Sama-SamangPahayagngPagsapisaUnyon” which it presented in its petition for
certification electionsupported their claim of 119 members. Respondent argued that the union members were
informed of the contents of the documents they signed and that the 68 attendees to the organizational meeting
constituted more than 50% of the total union membership, hence, a quorum existed for the conduct of the said
meeting.lawred

DOLE Regional Director issued a Decision granting the petition for cancellation of respondent's certificate of
registration.

Respondent filed an Appeal Memorandum with Formal Entry of Appearance to the Office of the DOLE Secretary,
which the latter eventually referred to the BLR.

The BLR rendered its Decision reversing the Order of the Regional Director.

Petitioner went to the CA via a petition for certiorari under Rule 65 but denied the petition and affirmed the decision
of the BLR.

ISSUE:

Whether or not respondent’s certificate of registration should be cancelled on the grounds of fraud and
misrepresentation.

RULING: There is no merit to cancel respondent’s certificate of registration.

Art. 234 of the Labor Code provides:C

ART. 234. Requirements of Registration. - A federation, national union or industry or trade union center or an
independent union shall acquire legal personality and shall be entitled to the rights and privileges granted by law to
legitimate labor organizations upon issuance of the certificate of registration based on the following requirements:

(a) Fifty pesos (P50.00) registration fee;


(b) The names of its officers, their addresses, the principal address of the labor organization, the minutes of the
organizational meetings and the list of the workers who participated in such meetings;

(c) In case the applicant is an independent union, the names of all its members comprising at least twenty percent
(20%) of all the employees in the bargaining unit where it seeks to operate;

(d) If the applicant union has been in existence for one or more years, copies of its annual financial reports; and

(e) Four copies of the constitution and by-laws of the applicant union, minutes of its adoption or ratification, and the
list of the members who participated in it.

And after the issuance of the certificate of registration, the labor organization's registration could be assailed
directly through cancellation of registration proceedings in accordance with Articles 238 and 239 of the Labor Code.
And the cancellation of union certificate of registration and the grounds thereof are as follows:Ch

ART. 238. Cancellation of Registration. - The certificate of registration of any legitimate labor organization,
whether national or local, may be cancelled by the Bureau, after due hearing, only on the grounds specified in
Article 239 hereof.

ART. 239. Grounds for Cancellation of Union Registration. - The following may constitute grounds for
cancellation of union registration:

(a) Misrepresentation, false statement or fraud in connection with the adoption or ratification of the constitution and
by-laws or amendments thereto, the minutes of ratification, and the list of members who took part in the ratification;

(b) Misrepresentation, false statements or fraud in connection with the election of officers, minutes of the election of
officers, and the list of voters;

(c) Voluntary dissolution by the members.

Petitioner's charge that respondent committed misrepresentation and fraud in securing its certificate of registration
is a serious charge and must be carefully evaluated. Allegations thereof should be compounded with supporting
circumstances and evidence. We find no evidence on record to support petitioner's accusation.

Petitioner's allegation of misrepresentation and fraud is based on its claim that during the organizational meeting
on May 1, 2009, only 68 employees attended, while respondent claimed that it has 119 members as shown in the
document denominated as “PangalanngmgaKasapingUnyon;” hence, respondent misrepresented on the 20%
requirement of the law as to its membership.

We do not agree.

It does not appear in Article 234 (b) of the Labor Code that the attendees in the organizational meeting must
comprise 20% of the employees in the bargaining unit. In fact, even the Implementing Rules and Regulations of the
Labor Code does not so provide. It is only under Article 234 (c) that requires the names of all its members
comprising at least twenty percent (20%) of all the employees in the bargaining unit where it seeks to operate.
Clearly, the 20% minimum requirement pertains to the employees’ membership in the union and not to the
list of workers who participated in the organizational meeting. Indeed, Article 234 (b) and (c) provide for
separate requirements, which must be submitted for the union's registration, and which respondent did submit.
Here, the total number of employees in the bargaining unit was 396, and 20% of which was about 79. Respondent
submitted a document entitled “PangalanngMgaKasapingUnyon” showing the names of 119 employees as union
members, thus respondent sufficiently complied even beyond the 20% minimum membership requirement.
Respondent also submitted the attendance sheet of the organizational meeting which contained the names and
signatures of the 68 union members who attended the meeting. Considering that there are 119 union members
which are more than 20% of all the employees of the bargaining unit, and since the law does not provide
for the required number of members to attend the organizational meeting, the 68 attendees which
comprised at least the majority of the 119 union members would already constitute a quorum for the
meeting to proceed and to validly ratify the Constitution and By-laws of the union. There is, therefore, no basis for
petitioner to contend that grounds exist for the cancellation of respondent's union registration. For fraud and
misrepresentation to be grounds for cancellation of union registration under Article 239 of the Labor Code, the
nature of the fraud and misrepresentation must be grave and compelling enough to vitiate the consent of a majority
of union members.

Petitioner's claim that the alleged union members signed documents without adequate information is not
persuasive. The one who alleges a fact has the burden of proving it and a mere allegation is not evidence.

Petitioner claims that in the list of members, there was an employee whose name appeared twice and another
employee who was merely a project employee. Such could not be considered a misrepresentation in the absence
of showing that respondent deliberately did so for the purpose of increasing their union membership. In fact, even if
those two names were not included in the list of union members, there would still be 117 members which was still
more than 20% of the 396 rank-and-file employees.

As to petitioner's argument that the total number of its employees as of May 1, 2009 was 470, and not 396 as
respondent claimed, still the 117 union members comprised more than the 20% membership requirement for
respondent's registration.
The bare fact that two signatures appeared twice on the list of those who participated in the organizational meeting
would not, to our mind, provide a valid reason to cancel respondent’s certificate of registration. The cancellation of
a union’s registration doubtless has an impairing dimension on the right of labor to self-organization . For
fraud and misrepresentation to be grounds for cancellation of union registration under the Labor Code, the nature
of the fraud and misrepresentation must be grave and compelling enough to vitiate the consent of a
majority of union members.

The alleged failure of respondent to indicate with mathematical precision the total number of employees in the
bargaining unit is of no moment, especially as it was able to comply with the 20% minimum membership
requirement.

22. RIGHTS OF LEGITIMATE LABOR ORGANIZATION

153. Digital Tel. Phils Inc. vs. Digitel Employees Union et al., G.R. No. 184903-04, October 10, 2012

FACTS:

Digitel Employees Union (Union) became the exclusive bargaining agent of all rank and file employees of Digitel in
1994. The Union and Digitel then commenced collective bargainingnegotiations which resulted in a bargaining
deadlock. The Union threatened to go on strike, but then Acting Labor Secretary assumed jurisdiction over the
dispute and eventually directed the parties to execute a CBA.2

However, no CBA was forged between Digitel and the Union. Some Union members abandoned their employment
with Digitel. The Union later became dormant.

Ten (10) years thereafter Digitel received from Esplana, who identified himself as President of the Union, a letter
containing the list of officers, CBA proposals and ground rules.

Digitel was reluctant to negotiate with the Union and demanded that the latter show compliance with the provisions of
the Union’s Constitution and By-laws on union membership and election of officers.

Esplana and his group filed a case based on Digitel’s violation of the duty to bargain. Digitel filed a petition with the
Bureau of Labor Relations (BLR) seeking cancellation of the Union’s registration.

In an Order, the Secretary of Labor directed Digitel to commence the CBA negotiation with the Union.

Thus:

WHEREFORE, all the foregoing premises considered, this Office hereby orders:

1. DIGITEL to commence collective bargaining negotiation with DEU without further delay.

Digitel moved for reconsideration on the contention that the pendency of the petition for cancellation of the
Union’s certificate of registration is a prejudicial question that should first be settled before the DOLE could order the
parties to bargain collectively.

ISSUE:

Whether the Secretary of Labor erred in issuing the assumption order despite the pendency of the petition for
cancellation of union registration.

HELD:

It is well-settled that the pendency of a petition for cancellation of union registration does not preclude collective
bargaining.

The 2005 case of Capitol Medical Center, Inc. v. Hon. Trajano13 is apropos. The respondent union therein sent a
letter to petitioner requesting a negotiation of their CBA. Petitioner refused to bargain and instead filed a petition for
cancellation of the union’s certificate of registration. Petitioner’s refusal to bargain forced the union to file a notice of
strike. They eventually staged a strike. The Secretary of Labor assumed jurisdiction over the labor dispute and
ordered all striking workers to return to work. Petitioner challenged said order by contending that its petition for
cancellation of union’s certificate of registration involves a prejudicial question that should first be settled before the
Secretary of Labor could order the parties to bargain collectively. When the case eventually reached this Court, we
agreed with the Secretary of Labor that the pendency of a petition for cancellation of union registration does not
preclude collective bargaining, thus:

That there is a pending cancellation proceeding against the respondent Union is not a bar to set in motion the
mechanics of collective bargaining. If a certification election may still be ordered despite the pendency of a petition to
cancel the union’s registration certificate, more so should the collective bargaining process continue despite its
pendency. We must emphasize that the majority status of the respondent Union is not affected by the pendency of
the Petition for Cancellation pending against it. Unless its certificate of registration and its status as the certified
bargaining agent are revoked, the Hospital is, by express provision of the law, duty bound to collectively bargain with
the Union.14

Citing the cases of Association of Court of Appeals Employees v. Ferrer- Calleja 16 and Samahan ng Manggagawa sa
Pacific Plastic v. Hon. Laguesma, 17 it was pointed out at the time of the filing of the petition for certification election –
or a CBA process as in the instant case – the union still had the personality to file a petition for certification − or to ask
for a CBA negotiation – as in the present case.

154. Automotive Engine Rebuilders vs. Progresibong Unyon ng mga Manggagawa sa AER G.R. No. 160138,
January 16, 2013 Resolution on the Main decision of July 13, 2011

Facts:
For resolution is the Motion for Partial Reconsideration filed by Progresibong Unyon Ng Mga Manggagawa Sa AER
(Unyon) which questioned the Court’s July 13, 2011 Decision insofar as it failed to award backwages to fourteen (14)
of its members. The decretal portion of the decision reads:

“WHEREFORE, the petitions are DENIED. Accordingly, the complaining employees should be
reinstated without backwages. If reinstatement is no longer feasible, the concerned employees
should be given separation pay up to the date set for their return in lieu of reinstatement.”

The precedents of this case as discussed in the 2011 case are as follows:

Both parties filed charges against each other, blaming the other party for violating labor laws. AER filed a
complaint against Unyon and its 18 members for illegal concerted activities. It likewise suspended 7 union members
who tested positive for illegal drugs. On the other hand, Unyon filed a countercharge accusing AER of unfair labor
practice, illegal suspension and illegal dismissal. In other words, AER claims that Unyon was guilty of staging an
illegal strike while Unyon claims that AER committed an illegal lockout.
 
AER’s fault is obvious from the fact that a day after the union filed a petition for certification election before
the DOLE, it hit back by requiring all its employees to undergo a compulsory drug test. Although AER argues that the
drug test was applied to all its employees, it was silent as to whether the drug test was a regular company policy and
practice in their 35 years in the automotive engine repair and rebuilding business. As the Court sees it, it
was AER’s first ever drug test of its employees immediately implemented after the workers manifested their desire to
organize themselves into a union. Indeed, the timing of the drug test was suspicious.

In awarding reinstatement but denying the award of backwages, the court rejected the conclusion of waiver by the
employee of his right to bring an action against illegal strike and upheld the in pari delicto doctrine or restoration of
conditions prior to the illegal strike and illegal lockout. Adopting the following rationalization in a relevant
jurisprudence, it reasoned out in the 2011 case:

Truly, it is more logical and reasonable for condonation to apply only to


strikers who signified their intention to return and did return to work. The reason
is obvious. These strikers took the initiative in normalizing relations with their
employer and thus helped promote industrial peace. However, as regards the
strikers who decided to pursue with the case, as in the case of the 114 strikers
herein, the employer could not be deemed to have condoned their strike,
because they had not shown any willingness to normalize relations with it. So, if
petitioner really had any intention to pardon the 114 strikers, it would have
included them in its motion to withdraw on November 17, 1980. The fact that it
did not, but instead continued to pursue the case to the end, simply means that it
did not pardon the 114 strikers.

The findings show that both petitioner and the 114 strikers are in pari
delicto, a situation which warrants the maintenance of the status
quo.This means that the contending parties must be brought back to their
respective positions before the controversy; that is, before the strike.
Therefore, the order reinstating the 114 employees is proper.
 
With such restoration of the status quo ante it necessarily follows, as likewise submitted
by the Solicitor General, that the petition must be granted insofar as it seeks the setting aside of the
award of three months' backwages to the 114 employees ordered reinstated on the basis of the
general rule that strikers are not entitled to backwages (with some exceptions not herein applicable,
such as where the employer is guilty of oppression and union-busting activities and strikers ordered
reinstated are denied such reinstatement and therefore are declared entitled to backwages from
the date of such denial). More so, is the principle of "no work, no pay" applicable to the case at
bar, in view of the undisputed finding of illegality of the strike.

The union filed a motion for partial consideration from the decision. It argued that backwages should have been
awarded to the 14 employees who were excluded from the complaint filed by AER and that the latter should have
reinstated them immediately because they did not have any case at all.

Issue:

WON the employees are entitled to backwages, aside from reinstatement when both the employee and employer are
at fault or in pari delicto
Ruling:

YES, they are entitled to REINSTATEMENT AND BACKWAGES despite the facts that these employees also erred
their employer. These excluded nine (9) workers, who signed their names in their petition before the CA, deserve to
be reinstated immediately and granted backwages. It is basic in jurisprudence that illegally dismissed workers are
entitled to reinstatement with back wages pi us interest at the legal rate. Only 9 out of the 14 excluded employees can
be granted the relief they prayed for because the others failed to affix their signatures in the Membership Resolution
of the Union authorizing the filing of the instant Motion for Partial Consideration.

The reinstatement shall be "without prejudice to the right of private respondent AER to subject them for further
medical check-up to determine if subject petitioners are drug dependents.

155. Holy Child Catholic School vs. Hon. Sto Tomas et al., G.R. No. 179146, July 23, 2013

FACTS:

On May 31, 2002, a petition for certification election was filed by private respondent Pinag-IsangTinig at Lakas ng
Anakpawis – Holy Child Catholic School Teachers and Employees Labor Union (HCCS-TELU-PIGLAS), alleging that:
PIGLAS is a legitimate labor organization duly registered with the Department of Labor and Employment (DOLE)
representing HCCS-TELU-PIGLAS; HCCS is a private educational institution duly registered and operating under
Philippine laws; there are approximately one hundred twenty (120) teachers and employees comprising the proposed
appropriate bargaining unit; and HCCS is unorganized, there is no collective bargaining agreement or a duly certified
bargaining agent or a labor organization certified as the sole and exclusive bargaining agent of the proposed
bargaining unit within one year prior to the filing of the petition.6 Among the documents attached to the petition were
the certificate of affiliation with Pinag-IsangTinig at Lakas ng AnakpawisKristiyanongAlyansa ng MakabayangObrero
(PIGLAS-KAMAO) issued by the Bureau of Labor Relations (BLR), charter certificate issued by PIGLAS-KAMAO, and
certificate of registration of HCCS-TELU as a legitimate labor organization issued by the DOLE.7

In its Comment8 and Position Paper,9 petitioner HCCS consistently noted that it is a parochial school with a total of
156 employees as of June 28, 2002, broken down as follows: ninety-eight (98) teaching personnel, twenty-five (25)
non-teaching academic employees, and thirty-three (33) non-teaching non-academic workers. It averred that of the
employees who signed to support the petition, fourteen (14) already resigned and six (6) signed twice. Petitioner
raised that members of private respondent do not belong to the same class; it is not only a mixture of managerial,
supervisory, and rank-and-file employees – as three (3) are vice-principals, one (1) is a department head/supervisor,
and eleven (11) are coordinators – but also a combination of teaching and non-teaching personnel – as twenty-seven
(27) are non-teaching personnel. It insisted that, for not being in accord with Article 24510 of the Labor Code, private
respondent is an illegitimate labor organization lacking in personality to file a petition for certification election, as held
in Toyota Motor Philippines Corporation v. Toyota Motor Philippines Corporation Labor Union; and an inappropriate
bargaining unit for want of community or mutuality of interest, as ruled in Dunlop Slazenger (Phils.), Inc. v. Secretary
of Labor and Employment and De La Salle University Medical Center and College of Medicine v. Laguesma.

On August 10, 2002, Med-Arbiter Agatha Ann L. Daquigan denied the petition for certification election on the ground
that the unit which private respondent sought to represent is inappropriate.

Private respondent appealed before the SOLE, who, on December 27, 2002, ruled against the dismissal of the
petition and directed the conduct of two separate certification elections for the teaching and the non-teaching
personnel.

ISSUE:

Whether or not a petition for certification election is dismissible on the ground that the labor organization’s
membership allegedly consists of supervisory and rank-and-file employees.

RULING:

It was in R.A. No. 875, under Section 3, that such questioned mingling was first prohibited, to wit:

Sec. 3. Employees' right to self-organization. - Employees shall have the right to self-organization and to form, join or
assist labor organizations of their own choosing for the purpose of collective bargaining through representatives of
their own choosing and to engage in concerted activities for the purpose of collective bargaining and other mutual aid
or protection. Individuals employed as supervisors shall not be eligible for membership in a labor organization of
employees under their supervision but may form separate organizations of their own. (Emphasis supplied)

Nothing in R.A. No. 875, however, tells of how the questioned mingling can affect the legitimacy of the labor
organization. Under Section 15, the only instance when a labor organization loses its legitimacy is when it violates its
duty to bargain collectively; but there is no word on whether such mingling would also result in loss of legitimacy.

Then the Labor Code was enacted in 1974 without reproducing Sec. 3 of R.A. No. 875. The provision in the Labor
Code closest to Sec. 3 is Article 290, which is deafeningly silent on the prohibition against supervisory employees
mingling with rank-and-file employees in one labor organization.

Effective 1989, R.A. No. 6715 restored the prohibition against the questioned mingling in one labor organization, viz.:
Sec. 18. Article 245 of the same Code, as amended, is hereby further amended to read as follows:

Art. 245. Ineligibility of managerial employees to join any labor organization; right of supervisory employees.
Managerial employees are not eligible to join, assist or form any labor organization. Supervisory employees shall not
be eligible for membership in a labor organization of the rank-and-file employees but may join, assist or form separate
labor organizations of their own (Emphasis supplied)

Unfortunately, just like R.A. No. 875, R.A. No. 6715 omitted specifying the exact effect any violation of the prohibition
would bring about on the legitimacy of a labor organization.

But then, on June 21, 1997, the 1989 Amended Omnibus Rules was further amended by Department Order No. 9,
series of 1997 (1997 Amended Omnibus Rules). Specifically, the requirement under Sec. 2(c) of the 1989 Amended
Omnibus Rules - that the petition for certification election indicate that the bargaining unit of rank-and-file employees
has not been mingled with supervisory employees - was removed. Instead, what the 1997 Amended Omnibus Rules
requires is a plain description of the bargaining unit.

When a similar issue confronted this Court close to three years later, the above ruling was substantially quoted in
SamahangManggagawasa Charter Chemical Solidarity of Unions in the Philippines for Empowerment and Reforms
(SMCC-Super) v. Charter Chemical and Coating Corporation.47 In unequivocal terms, We reiterated that the alleged
inclusion of supervisory employees in a labor organization seeking to represent the bargaining unit of rank-and-file
employees does not divest it of its status as a legitimate labor organization.

Following the doctrine laid down in Kawashima and SMCC-Super, it must be stressed that petitioner cannot
collaterally attack the legitimacy of private respondent by praying for the dismissal of the petition for certification
election:

Except when it is requested to bargain collectively, an employer is a mere bystander to any petition for certification
election; such proceeding is non-adversarial and merely investigative, for the purpose thereof is to determine which
organization will represent the employees in their collective bargaining with the employer. The choice of their
representative is the exclusive concern of the employees; the employer cannot have any partisan interest therein; it
cannot interfere with, much less oppose, the process by filing a motion to dismiss or an appeal from it; not even a
mere allegation that some employees participating in a petition for certification election are actually managerial
employees will lend an employer legal personality to block the certification election. The employer's only right in the
proceeding is to be notified or informed thereof.

Further, the determination of whether union membership comprises managerial and/or supervisory employees is a
factual issue that is best left for resolution in the inclusion-exclusion proceedings, which has not yet happened in this
case so still premature to pass upon. We could only emphasize the rule that factual findings of labor officials, who are
deemed to have acquired expertise in matters within their jurisdiction, are generally accorded not only with respect
but even finality by the courts when supported by substantial evidence.50 Also, the jurisdiction of this Court in cases
brought before it from the CA via Rule 45 is generally limited to reviewing errors of law or jurisdiction. The findings of
fact of the CA are conclusive and binding. Except in certain recognized instances,51 We do not entertain factual
issues as it is not Our function to analyze or weigh evidence all over again; the evaluation of facts is best left to the
lower courts and administrative agencies/quasi-judicial bodies which are better equipped for the task.

In the same manner, the teaching and non-teaching personnel of [petitioner] school must form separate bargaining
units. Thus, the order for the conduct of two separate certification elections, one involving teaching personnel and the
other involving non-teaching personnel. It should be stressed that in the subject petition, [private respondent] union
sought the conduct of a certification election among all the rank-and-file personnel of [petitioner] school. Since the
decision of the Supreme Court in the U.P. case prohibits us from commingling teaching and non-teaching personnel
in one bargaining unit, they have to be separated into two separate bargaining units with two separate certification
elections to determine whether the employees in the respective bargaining units desired to be represented by [private
respondent].

Indeed, the purpose of a certification election is precisely to ascertain the majority of the employees’ choice of an
appropriate bargaining unit – to be or not to be represented by a labor organization and, if in the affirmative case, by
which one.

156. Visayas Community Medical Center vs. Yballe, et al., GR No. 196156, January 15, 2014

Facts:

Respondents were hired as staff nurses and midwives by (VCMC), formerly (MCCHI). (NFL) is the exclusive
bargaining representative of the rank-and-file employees of MCCHI. Under the 1987 and 1991 (CBAs), the
signatories were. Pongasi,.for MCCHI, and Atty.. Alforque (NFL Legal Counsel) and Lumapguid as President of NFL-
MCCH Chapter. In the CBA effective from January 1994 until December 31, 1995, the signatories were Buot as
Board of Trustees Chairman, Iyoy as MCCH Administrator and Atty. Yu as Legal Counsel of NFL, while Nava,
President of NagkahiusangMamumuosa MCCH (NAMA-MCCH-NFL) signed the proof of Posting.

Nava wrote Rev. Iyoy expressing the union’s desire to renew the CBA, signed/endorsed by 153 union
members. Nava subsequently requested that the following employees be allowed to avail of one-day union leave with
pay However, MCCHI returned the CBA proposal for Nava to secure first the endorsement of the legal counsel of
NFL as the official bargaining representative of MCCHI employees.
Meanwhile, Atty. Alforque informed MCCHI that the proposed CBA submitted by Nava was never referred to
NFL and that NFL has not authorized any other legal counsel or any person for collective bargaining negotiations.

In his letter addressed to Nava, Atty. Alforque suspended their union membership for serious violation of the
Constitution and By-Laws.

The next day, several union members led by Nava and her group launched a series of mass actions such as
wearing black and red armbands/headbands, marching around the hospital premises and putting up placards,
posters and streamers. Atty. Alforque immediately disowned the concerted activities being carried out by union
members which are not sanctioned by NFL.

DOLEissued certifications stating that there is nothing in their records which shows that NAMA-MCCH- NFL
is a registered labor organization, and that said union submitted only a copy of its Charter Certificate on January 31,
1995.

Unfazed, the striking union members held more mass actions. The means of ingress to and egress from the
hospital were blocked so that vehicles carrying patients and employees were barred from entering the premises.

With the volatile situation adversely affecting hospital operations and the condition of confined patients,
MCCHI filed a petition for injunction in the NLRC. Thereafter, several complaints for illegal dismissal and unfair labor
practice were filed by the terminated employees against MCCHI,.Iyoy, UCCP and members of the Board of Trustees
of MCCHI

Issue:

Whether or not the union employees were illegally dismissed.

Ruling:

Yes. Paragraph 3, Article 264(a) of the Labor Code provides that ". . .any union officer who knowingly
participates in an illegal strike and any worker or union officer who knowingly participates in the commission of illegal
acts during a strike may be declared to have lost his employment status .

The dismissal of MCCH employees who participated in the illegal strike conducted by NAMA-MCCH-NFL
which is not a legitimate labor organization, was invalid. Since there was no showing that the complainants committed
any illegal act during the strike, they may not be deemed to have lost their employment status by their mere
participation in the illegal strike. On the other hand, the union leaders (Nava group) who conducted the illegal strike
despite knowledge that NAMA-MCCH-NFL is not a duly registered labor union were declared to have been validly
terminated by petitioner.

We stress that the law makes a distinction between union members and union officers. A worker merely
participating in an illegal strike may not be terminated from employment. It is only when he commits illegal acts during
a strike that he may be declared to have lost employment status. In contrast, a union officer may be terminated from
employment for knowingly participating in an illegal strike or participates in the commission of illegal acts during a
strike. The law grants the employer the option of declaring a union officer who participated in an illegal strike as
having lost his employment. It possesses the right and prerogative to terminate the union officers from service.

While there was indeed no evidence of any illegal act committed by respondents during the strike, the Labor
Arbiter and NLRC were one in finding that respondents actively supported the concerted protest activities, signed the
collective reply of union members manifesting that they launched the mass actions to protest management’s refusal
to negotiate a new CBA, refused to appear in the investigations scheduled by petitioner because it was the union’s
stand that they would only attend these investigations as a group, and failed to heed petitioner’s final directive for
them to desist from further taking part in the illegal strike.

Union members who were illegally dismissed for mere participation in an illegal strike are entitled to
separation pay (in lieu of reinstatement) but not to backwages.

The principle of "fair day’s wage for a fair day’s labor" remains as the basic factor in determining the award
thereof. If there is no work performed by the employee there can be no wage or pay unless, of course, the laborer
was able, willing and ready to work but was illegally locked out, suspended or dismissed or otherwise illegally
prevented from working. For this exception to apply, it is required that the strike be legal.
157. Philtranco Service Enterprises Inc. vs. Philtranco Workers Union-Association of Genuine Labor
Organizations, GR No. 180962, February 26, 2014

Facts:

On the ground that it was suffering business losses, petitioner Philtranco Service Enterprises, Inc., a local land
transportation company engaged in the business of carrying passengers and freight, retrenched 21 of its employees.
Consequently, the company union, herein private respondent Philtranco Workers Union-Association of Genuine
Labor Organizations, filed a Notice of Strike with the Department of Labor and Employment, claiming that petitioner
engaged in unfair labor practices.

Unable to settle their differences at the National Conciliation and Mediation Board, the case was thereafter referred to
the Office of the Secretary of the DOLE.

Acting DOLE Secretary issued a Decision6 dated June 13, 2007,

The PARTIES are enjoined to strictly and fully comply with the provisions of the existing CBA and the other
dispositions of this Decision. Petitioner received a copy of the above Decision on June 14, 2007. It filed a Motion for
Reconsideration on June 25, 2007. Private respondent, on the other hand, submitted a "Partial Appeal."

In an August 15, 2007 Order8 which petitioner received on August 17, 2007, the Secretary of Labor declined to rule
on petitioner’s Motion for Reconsideration and private respondent’s "Partial Appeal", citing a DOLE Regulation9which
provided that voluntary arbitrators’ decisions, orders, resolutions or awards shall not be the subject of motions for
reconsideration. The Secretary of Labor held:

WHEREFORE, the complainant’s and the respondent’s respective pleadings are hereby NOTED as pleadings that
need not be acted upon for lack of legal basis.

On August 29, 2007, petitioner filed before the CA an original Petition for Certiorari and Prohibition, and sought
injunctive relief. On September 20, 2007, the CA issued the assailed Resolution which decreed as follows:

WHEREFORE, premises considered, the instant Petition for Certiorari and Prohibition with Prayer for Temporary
Restraining Order and Preliminary Injunction is hereby DISMISSED.

Issues:

THE HONORABLE COURT OF APPEALS ERRED IN RULING THAT THE PETITIONER AVAILED OF THE
ERRONEOUS REMEDY IN FILING A PETITION FOR CERTIORARI UNDER RULE 65 INSTEAD OF UNDER RULE
43 OF THE RULES OF COURT.

THE HONORABLE COURT OF APPEALS ERRED WHEN IT HELD THAT THE PETITION FOR CERTIORARI WAS
FILED OUT OF TIME.

Ruling:

The Court grants the Petition.

It cannot be said that in taking cognizance of NCMB-NCR CASE No. NS-02-028-07, the Secretary of Labor did so in
a limited capacity, i.e., as a voluntary arbitrator. The fact is undeniable that by referring the case to the Secretary of
Labor, Conciliator-Mediator Aglibut conceded that the case fell within the coverage of Article 263 of the Labor Code;
the impending strike in Philtranco, a public transportation company whose business is imbued with public interest,
required that the Secretary of Labor assume jurisdiction over the case, which he in fact did. By assuming jurisdiction
over the case, the provisions of Article 263 became applicable, any representation to the contrary or that he is
deciding the case in his capacity as a voluntary arbitrator notwithstanding.

It has long been settled that the remedy of an aggrieved party in a decision or resolution of the Secretary of Labor is
to timely file a motion for reconsideration as a precondition for any further or subsequent remedy, and then
seasonably file a special civil action for certiorari under Rule 65 of the 1997 Rules on Civil Procedure. There is no
distinction: when the Secretary of Labor assumes jurisdiction over a labor case in an industry indispensable to
national interest, "he exercises great breadth of discretion" in finding a solution to the parties’ dispute. "The authority
of the Secretary of Labor to assume jurisdiction over a labor dispute causing or likely to cause a strike or lockout in an
industry indispensable to national interest includes and extends to all questions and controversies arising therefrom.
The power is plenary and discretionary in nature to enable him to effectively and efficiently dispose of the primary
dispute." This wide latitude of discretion given to the Secretary of Labor may not be the subject of appeal.

On the question of whether the Petition for Certiorari was timely filed, the Court agrees with petitioner’s submission.
Rule 65 states that where a motion for reconsideration or new trial is timely filed, whether such motion is required or
not, the petition shall be filed not later than 60 days counted from the notice of the denial of the motion. This can only
mean that even though a motion for reconsideration is not required or even prohibited by the concerned government
office, and the petitioner files the motion just the same, the 60-day period shall nonetheless be counted from notice of
the denial of the motion. The very nature of certiorari – which is an extraordinary remedy resorted to only in the
absence of plain, available, speedy and adequate remedies in the course of law – requires that the office issuing the
decision or order be given the opportunity to correct itself. Quite evidently, this opportunity for rectification does not
arise if no motion for reconsideration has been filed.
So also, considering that a decision of the Secretary of Labor is subject to judicial review only through a special civil
action of certiorari and, as a rule, cannot be resorted to without the aggrieved party having exhausted administrative
remedies through a motion for reconsideration, the aggrieved party, must be allowed to move for a reconsideration of
the same so that he can bring a special civil action for certiorari before the Supreme Court.

158. Wesleyan University-Phils., vs. Wesleyan University-Phils., Faculty & Staff Asso., GR No.
181806, March 12, 2014

Principle: A memorandum that imposes a limitation/condition on the employees’ entitlement to vacation and
sick leave credits, when no such limitation/condition is found in the Collective Bargaining Agreement, is
invalid. Like any other contract, a CBA has the force of law between the parties and must be complied with
in good faith. Unilateral changes in the provisions of the CBA will therefore be struck down if the consent of
both parties is not obtained.

Facts:

Petitioner Wesleyan University-Philippines is a non-stock, non-profit educational institution duly organized and
existing under the laws of the Philippines. Respondent Wesleyan University-Philippines Faculty and Staff Association,
on the other hand, is a duly registered labor organization7 acting as the sole and exclusive bargaining agent of all
rank-and-file faculty and staff employees of petitioner.

In December 2003, the parties signed a 5-year CBA 9 effective June 1, 2003 until May 31, 2008.On August 16, 2005,
petitioner issued a Memorandum providing guidelines on the implementation of vacation and sick leave credits as
well as vacation leave commutation. In the said memorandum, the vacation and sick leave credits are not automatic
and should be earned on a month-to-month basis. Respondent questioned the memorandum, saying that it was not
amenable to the unilateral changes and contending that said changes violated the existing CBA between them. The
existing CBA does not impose any limitation on the availability of the vacation and sick leaves.

Issue:

WON the memorandum issued by petitioner is valid

Ruling:

No. It is contrary to the existing CBA between petitioner and respondent.

A Collective Bargaining Agreement (CBA) is a contract entered into by an employer and a legitimate labor
organization concerning the terms and conditions of employment. 1 Like any other contract, it has the force of law
between the parties and, thus, should be complied with in good faith.2 Unilateral changes or suspensions in the
implementation of the provisions of the CBA, therefore, cannot be allowed without the consent of both parties.

Sections 1 and 2 of Article XII of the CBA provide that all covered employees are entitled to 15 days sick leave and
15 days’ vacation leave with pay every year and that after the second year of service, all unused vacation leave shall
be converted to cash and paid to the employee at the end of each school year, not later than August 30 of each year.

The Memorandum dated August 16, 2005, however, states that vacation and sick leave credits are not automatic as
leave credits would be earned on a month-to-month basis. This, in effect, limits the available leave credits of an
employee at the start of the school year. For example, for the first four months of the school year or from June to
September, an employee is only entitled to five days’ vacation leave and five days sick leave. Considering that the
Memorandum dated August 16, 2005 imposes a limitation not agreed upon by the parties nor stated in the CBA, we
agree with the CA that it must be struck down.

In closing, it may not be amiss to mention that when the provision of the CBA is clear, leaving no doubt on the
intention of the parties, the literal meaning of the stipulation shall govem.

However, if there is doubt in its interpretation, it should be resolved in favor of labor, as this is mandated by no less
than the Constitution.

159. Tabangao Shell Refinery Employees Association vs. Pilipinas Shell Petroleum Corp., GR No. 170007,
April 7, 2014

Facts:

In anticipation of the expiration on April 30, 2004 of the 2001-2004 Collective Bargaining Agreement (CBA) between
the petitioner and the respondent Pilipinas Shell Petroleum Corporation, the parties started negotiations for a new
CBA.

After several meetings on the ground rules that would govern the negotiations and on political items, the parties
started their discussion on the economic items on July 27, 2004, their 31st meeting. The union proposed a 20 percent
annual across-the-board basic salary increase for the next three years that would be covered by the new CBA.

In lieu of the annual salary increases, the company made a counter-proposal to grant all covered employees a lump
sum amount of P80,000.00 yearly for the three-year period of the new CBA.
The union requested the company to present its counter-proposal in full detail, similar to the presentation by the union
of its economic proposal. The company explained that the lump sum amount was based on its affordability for the
corporation, the then current salary levels of the members of the union relative to the industry, and the then current
total pay and benefits package of the employees. Not satisfied with the company’s explanation, the union asked for
further justification of the lump sum amount offered by the company. When the company refused to acknowledge any
obligation to give further justification, the union rejected the company’s counter-proposal and maintained its proposal
for a 20% annual increase in basic pay for the next three years.

On the 39th meeting of the parties on August 24, 2004, the union lowered its proposal to 12% annual across-the-
board increase for the next three years. For its part, the company increased its counter-proposal to a yearly lump sum
payment of P88,000.00 for the next three years. The union requested financial data for the manufacturing class of
business in the Philippines. It also requested justification for the company’s counter-offer. In response, the company
stated that financial measures for Tabangao were available in the refinery scorecard regularly cascaded by the
management to the employees. The company reiterated that its counter-offer is based on its affordability for the
company, comparison with the then existing wage levels of allied industry, and the then existing total pay and benefits
package of the employees. The company subsequently provided the union with a copy of the company’s audited
financial statements.

However, the union remained unconvinced and asked for additional documents to justify the company’s counter-offer.
The company invited the attention of the union to the fact that additional data, such as the refinery performance
scorecard, were available from the refinery’s website and shared network drives. The company also declared that the
bases of its counter-offer were already presented to the union and contained in the minutes of previous meetings.
The union thereafter requested for a copy of the comparison of the salaries of its members and those from allied
industries. The company denied the request on the ground that the requested information was entrusted to the
company under a confidential agreement. Alleging failure on the part of the company to justify its offer, the union
manifested that the company was bargaining in bad faith. The company, in turn, expressed its disagreement with the
union’s manifestation.

On the parties’ 41st meeting held on September 2, 2004, the company proposed the declaration of a deadlock and
recommended that the help of a third party be sought. The union replied that they would formally answer the proposal
of the company a day after the signing of the official minutes of the meeting. On that same day, however, the union
filed a Notice of Strike in the National Conciliation and Mediation Board (NCMB), alleging bad faith bargaining on the
part of the company. The NCMB immediately summoned the parties for the mandatory conciliation-mediation
proceedings but the parties failed to reach an amicable settlement.

On September 16, 2004, during the cooling off period, the union conducted the necessary strike vote. The members
of the union, who participated in the voting, unanimously voted for the holding of a strike. Upon being aware of this
development, the company filed a Petition for Assumption of Jurisdiction with the Secretary of Labor and
Employment. The petition was filed pursuant to the first paragraph of Article 263(g) of the Labor Code.

In an Order dated September 20, 2004, the then Secretary of Labor and Employment, Patricia Sto. Tomas, granted
the petition of the company. The Secretary of Labor and Employment took notice of the Notice of Strike filed by the
union in the NCMB which charged the company with unfair labor practice consisting of bad faith in bargaining
negotiations. The Secretary of Labor and Employment also found that the intended strike would likely affect the
company’s capacity to provide petroleum products to the company’s various clientele, including the transportation
sector, the energy sector, and the manufacturing and industrial sectors. The Secretary of Labor and Employment
further observed that a strike by the union would certainly have a negative impact on the price of commodities.
Convinced that such a strike would have adverse consequences on the national economy, the Secretary of Labor
and Employment ruled that the labor dispute between the parties would cause or likely to cause a strike in an industry
indispensable to the national interest. Thus, the Secretary of Labor and Employment assumed jurisdiction over the
dispute of the parties.

The union now comes to this Court to press its contentions. It insists that the corporation is guilty of unfair labor
practice through bad faith bargaining. According to the union, bad faith bargaining and a CBA deadlock cannot legally
co-exist because an impasse in negotiations can only exist on the premise that both parties are bargaining in good
faith. Besides, there could have been no deadlock between the parties as the union had not given its consent to it,
pursuant to item 8 of the ground rules governing the parties’ negotiations which required mutual consent for a
declaration of deadlock. The union also posits that its filing of a CBA deadlock case against the company was a
separate and distinct case and not an offshoot of the company’s unfair labor practice through bargaining in bad faith.
According to the union, as there was no deadlock yet when the union filed the unfair labor practice of bargaining in
bad faith, the subsequent deadlock case could neither be an offshoot of, nor an incidental issue in, the unfair labor
practice case. Because there was no deadlock yet at the time of the filing of the unfair labor practice case, the union
claims that deadlock was not an incidental issue but a non-issue.

Issue: WON  the Court of Appeals misapplied St. Scholastica’s College and the Secretary of Labor and Employment
committed grave abuse of discretion when it presumed deadlock in its Order dated September 20, 2004 assuming
jurisdiction over the labor dispute between the union and the company

Ruling:

No, the union’s petition must fail. The CA and the SOLE are correct. The assumption of jurisdiction was in
order.
As discussed above, there was already an actual existing deadlock between the parties. What was lacking was the
formal recognition of the existence of such a deadlock because the union refused a declaration of deadlock. Thus,
the union’s view that, at the time the Secretary of Labor and Employment exercised her power of assumption of
jurisdiction, the issue of deadlock was neither an incidental issue to the matter of unfair labor practice nor an existing
issue is incorrect.

More importantly, however, the union’s mistaken theory that the deadlock issue was neither incidental nor existing is
based on its premise that the case is all about the company’s alleged unfair labor practice of bargaining in bad faith,
which is the ground stated in its first Notice of Strike

While the first Notice of Strike is indeed significant in the determination of the existing labor dispute between the
parties, it is not the sole criterion. As this Court explained in Union of Filipro Employees-Drug, Food and Allied
Industries Unions-Kilusang Mayo Uno v. Nestle Philippines, Inc.:

The Secretary of the DOLE has been explicitly granted by Article 263(g) of the Labor Code the
authority to assume jurisdiction over a labor dispute causing or likely to cause a strike or lockout
in an industry indispensable to the national interest, and decide the same accordingly. And, as
a matter of necessity, it includes questions incidental to the labor dispute; that is, issues
that are necessarily involved in the dispute itself, and not just to that ascribed in the
Notice of Strike or otherwise submitted to him for resolution. x x x (Emphasis supplied.)

The totality of the company’s Petition for Assumption of Jurisdiction, including every allegation therein, also guided
the Secretary of Labor and Employment in the proper determination of the labor dispute over which he or she was
being asked to assume jurisdiction.

The labor dispute between the union and the company concerned the unresolved matters between the parties in
relation to their negotiations for a new CBA. The power of the Secretary of Labor and Employment to assume
jurisdiction over this dispute includes and extends to all questions and controversies arising from the said dispute,
such as, but not limited to the union’s allegation of bad faith bargaining. It also includes and extends to the various
unresolved provisions of the new CBA such as compensation, particularly the matter of annual wage increase or
yearly lump sum payment in lieu of such wage increase, whether or not there was deadlock in the negotiations.
Indeed, nowhere does the Order dated September 20, 2004 of the Secretary of Labor and Employment mention a
CBA deadlock. What the union viewed as constituting the inclusion of a CBA deadlock in the assumption of
jurisdiction was the inclusion of the economic issues, particularly the company’s stance of yearly lump sum payment
in lieu of annual wage increase, in the directive for the parties to submit their respective position papers. The union’s
Motion for Reconsideration (With Urgent Prayer to Compel the Company to Justify Offer of Wage [Increase]
Moratorium) and Second Motion for Reconsideration questioning the Order dated September 20, 2004 of the
Secretary of Labor and Employment actually confirm that the labor dispute between the parties essentially and
necessarily includes the conflicting positions of the union, which advocates annual wage increase, and of the
company, which offers yearly lump sum payment in lieu of wage increase. In fact, that is the reason behind the
union’s prayer that the company be ordered to justify its offer of wage increase moratorium. As there is already an
existing controversy on the matter of wage increase, the Secretary of Labor and Employment need not wait for a
deadlock in the negotiations to take cognizance of the matter. That is the significance of the power of the Secretary of
Labor and Employment under Article 263(g) of the Labor Code to assume jurisdiction over a labor dispute causing or
likely to cause a strike or lockout in an industry indispensable to the national interest. As this Court elucidated in
Bagong Pagkakaisa ng Manggagawa ng Triumph International v. Secretary of the Department of Labor and
Employment:

Article 263(g) is both an extraordinary and a preemptive power to address an extraordinary


situation - a strike or lockout in an industry indispensable to the national interest. This grant is not
limited to the grounds cited in the notice of strike or lockout that may have preceded the strike or
lockout; nor is it limited to the incidents of the strike or lockout that in the meanwhile may have
taken place. As the term "assume jurisdiction" connotes, the intent of the law is to give the Labor
Secretary full authority to resolve all matters within the dispute that gave rise to or which arose
out of the strike or lockout; it includes and extends to all questions and controversies arising from
or related to the dispute, including cases over which the labor arbiter has exclusive jurisdiction.
23. REVISED GUIDELINES OF THE NCMB FOR THE CONDUCT OF VOLUNTARY ARBITRATION
PROCEEDINGS

160. (162.) 7K Corp. vs. Albarico, G.R. No. 182295, June 26, 2013

Facts:

Respondent Eddie Albarico (Albarico) was a regular employee of petitioner 7K Corporation, a company selling water
purifiers. He started working for the company in 1990 as a salesman. 4 Because of his good performance, his
employment was regularized. He was also promoted several times: from salesman, he was promoted to senior sales
representative and then to acting team field supervisor.

In 1992, he was awarded the President’s Trophy for being one of the company’s top water purifier specialist
distributors. However, in April of 1993, the chief operating officer of petitioner 7K Corporation terminated Albarico’s
employment allegedly for his poor sales performance.5 Respondent had to stop reporting for work, and he
subsequently submitted his money claims against petitioner for arbitration before the National Conciliation and
Mediation Board (NCMB). The issue for voluntary arbitration before the NCMB, according to the parties’ Submission
Agreement dated 19 April 1993, was whether respondent Albarico was entitled to the payment of separation pay and
the sales commission reserved for him by the corporation.6

Issue:

WON the voluntary arbitrator properly assumed jurisdiction to decide the issue of the legality of the dismissal of
respondent as well as the latter’s entitlement to backwages, even if neither the legality nor the entitlement was
expressly claimed in the Submission Agreement of the parties.

Ruling:

NO. Petitioner overlooks the proviso in the said article, thus:

Art. 217. Jurisdiction of the Labor Arbiters and the Commission.

a. Except as otherwise provided under this Code, the Labor Arbiters shall have original and exclusive jurisdiction
to hear and decide, within thirty (30) calendar days after the submission of the case by the parties for decision without
extension, even in the absence of stenographic notes, the following cases involving all workers, whether agricultural
or non-agricultural:cralavvonlinelawlibrary

xxxx
2. Termination disputes;

xxxx

6. Except claims for Employees Compensation, Social Security, Medicare and maternity benefits, all other claims
arising from employer-employee relations, including those of persons in domestic or household service, involving
an amount exceeding five thousand pesos (P5,000.00) regardless of whether accompanied with a claim for
reinstatement. (Emphases supplied)

Thus, although the general rule under the Labor Code gives the labor arbiter exclusive and original jurisdiction over
termination disputes, it also recognizes exceptions. One of the exceptions is provided in Article 262 of the Labor
Code. In San Jose v. NLRC, we said:

The phrase “Except as otherwise provided under this Code” refers to the following exceptions:

A. Art. 217. Jurisdiction of Labor Arbiters . . .

xxxx

B. Art. 262. Jurisdiction over other labor disputes. The Voluntary Arbitrator or panel of Voluntary Arbitrators,
upon agreement of the parties, shall also hear and decide all other labor disputes including unfair labor
practices and bargaining deadlocks.

From the above discussion, it is clear that voluntary arbitrators may, by agreement of the parties, assume jurisdiction
over a termination dispute such as the present case, contrary to the assertion of petitioner that they may not.

Petitioner argues that, assuming that the voluntary arbitrator has jurisdiction over the present termination dispute, the
latter should have limited his decision to the issue contained in the Submission Agreement of the parties – the
issue of whether respondent Albarico was entitled to separation pay and to the sales commission the latter earned
before being terminated.Thus, it contends that the voluntary arbitrator exceeded his jurisdiction.

The Court notes that even the NLRC was of the understanding that the NCMB arbitration case sought to resolve the
issue of the legality of the dismissal of the respondent. In fact, the identity of the issue of the legality of his dismissal,
which was previously submitted to the NCMB, and later submitted to the NLRC, was the basis of the latter’s finding of
forum shopping and the consequent dismissal of the case before it. In fact, petitioner also implicitly acknowledged
this when it filed before the NLRC its Motion to Dismiss respondent’s Complaint on the ground of forum shopping.
Thus, it is now estopped from claiming that the issue before the NCMB does not include the issue of the
legality of the dismissal of respondent. Besides, there has to be a reason for deciding the issue of respondent’s
entitlement to separation pay. To think otherwise would lead to absurdity, because the voluntary arbitrator would then
be deciding that issue in a vacuum. The arbitrator would have no basis whatsoever for saying that Albarico was
entitled to separation pay or not if the issue of the legality of respondent’s dismissal was not resolve first.

Hence, the voluntary arbitrator correctly assumed that the core issue behind the issue of separation pay is
the legality of the dismissal of respondent. Moreover, we have ruled in Sime Darby Pilipinas, Inc. v. Deputy
Administrator Magsalin that a voluntary arbitrator has plenary jurisdiction and authority to interpret an agreement to
arbitrate and to determine the scope of his own authority when the said agreement is vague — subject only, in a
proper case, to the certiorari jurisdiction of this Court.

Having established that the issue of the legality of dismissal of Albarico was in fact necessarily – albeit not
explicitly – included in the Submission Agreement signed by the parties, this Court rules that the voluntary
arbitrator rightly assumed jurisdiction to decide the said issue.

Consequently, we also rule that the voluntary arbitrator may award backwages upon a finding of illegal dismissal,
even though the issue of entitlement thereto is not explicitly claimed in the Submission Agreement. Backwages, in
general, are awarded on the ground of equity as a form of relief that restores the income lost by the terminated
employee by reason of his illegal dismissal.34
In Sime Darby we ruled that although the specific issue presented by the parties to the voluntary arbitrator was only
“the issue of performance bonus,” the latter had the authority to determine not only the issue of whether or not a
performance bonus was to be granted, but also the related question of the amount of the bonus, were it to be
granted. We explained that there was no indication at all that the parties to the arbitration agreement had regarded
“the issue of performance bonus” as a two-tiered issue, of which only one aspect was being submitted to arbitration.
Thus, we held that the failure of the parties to limit the issues specifically to that which was stated allowed the
arbitrator to assume jurisdiction over the related issue.

Similarly, in the present case, there is no indication that the issue of illegal dismissal should be treated as a
two-tiered issue whereupon entitlement to backwages must be determined separately.  Besides, “since
arbitration is a final resort for the adjudication of disputes,” the voluntary arbitrator in the present case can
assume that he has the necessary power to make a final settlement. Thus, we rule that the voluntary
arbitrator correctly assumed jurisdiction over the issue of entitlement of respondent Albarico to backwages
on the basis of the former’s finding of illegal dismissal.

161. Lepanto Consolidated Mining Company vs. The Lepanto Capataz Union, G.R. No. 157086, Feb. 18, 2013

FACTS:

Lepanto is a domestic corporation authorized to engage in large-scale mining. Respondent Lepanto Capataz Union
(Union), a labor organization duly registered with DOLE, filed a petition for consent election, thereby proposing to
represent 139 capatazes  of Lepanto.

In due course, Lepanto opposed the petition, contending that the Union was in reality seeking a certification election,
not a consent election, and would be thereby competing with the Lepanto Employees Union (LEU), the current
collective bargaining agent. Lepanto pointed out that the capatazes were already members of LEU, the exclusive
representative of all rank-and-file employees of its Mine Division.

Med-Arbiter Michaela A. Lontoc of DOLE-CAR issued a ruling to the effect that the capatazescould form a separate
bargaining unit due to their not being rank-and-file employees.Lepanto appealed to the DOLE Secretary. The DOLE
Undersecretary Rosalinda Dimapilis-Baldoz, acting by authority of the DOLE Secretary, affirmed the ruling of Med-
Arbiter Lontoc.

In the ensuing certification election, the Union garnered 109 of the 111 total valid votes cast.On the day of the
certification election, however, Lepanto presented an opposition/protest. Hence, a hearing was held on Lepanto’s
opposition/protest. Although the parties were required in that hearing to submit their respective position papers,
Lepanto later opted not to submit its position paper, and contended that the issues identified during the hearing did
not pose any legal issue to be addressed in a position paper.

Med-Arbiter Florence Marie A. Gacad-Ulep of DOLE-CAR rendered a decision certifying the Union as the sole and
exclusive bargaining agent of all capatazes  of Lepanto.Lepanto appealed the decision of Med-Arbiter Gacad-Ulep to
the DOLE Secretary.By her Resolution, DOLE Secretary Patricia A. Sto. Tomas affirmed the decision.

Appellant accused Med-Arbiter Ulep of grave abuse of discretion amounting to lack of jurisdiction based on her failure
to resolve appellant’s motion to modify order to submit position papers and on rendering judgment on the basis only
of appellee’s position paper.

ISSUES: Whether or not Med-Arbiter Ulep is guilty of grave abuse of discretion amounting to lack of jurisdiction;
and, whether the capatazes could form their own union independently of the rank-and-file employees.

RULING:

I. Med-Arbiter Ulep is NOT guilty of grave abuse of discretion amounting to lack of jurisdiction.

Section 5, Rule XXV of Department Order No. 9, otherwise known as the New Rules Implementing Book V of the
Labor Code, states that "in all proceedings at all levels, incidental motions shall not be given due course, but shall
remain as part of the records for whatever they may be worth when the case is decided on the merits".

Section 9, Rule XI of Department Order No. 9, which is applied with equal force in the disposition of protests on the
conduct of election, states that "the Med-Arbiter shall in the same hearing direct all concerned parties, including the
employer, to simultaneously submit their respective position papers within a non-extendible period of ten days". The
parties, including appellant company were required to submit their respective positions on whether there was proper
challenge of the voters, whether LEU failed to participate in the proceedings, if so, whether it should be allowed to
participate at this belated stage and whether the arguments raised during the pre-election conferences and in the
protests are valid. The parties, including appellant company were apprised of these issues and they agreed thereto.
The minutes of the hearing even contained the statement that "no order will issue" and that "the parties are informed
accordingly". If there is any matter that had to be clarified, appellant should have clarified the same during the said
hearing and refused to file its position paper simultaneously with LCU and LEU. It appears that appellant did not do
so and acquiesced to the filing of its position paper within fifteen days from the date of said hearing.
Neither is there merit in appellant’s contention that the Med- Arbiter resolved the protest based solely on appellee
LCU’s position paper. Not only did the Med-Arbiter discuss the demerits of appellant’s motion to modify order to
submit position papers but likewise the demerits of its protest. We do not, however, agree with the Med-Arbiter that
the protest should be dismissed due to appellant’s failure to challenge the individual voters during the election.

II.Capatazes are not rank-and-file employees; hence, they could form their own union.

Thecapatazes were performing functions totally different from those performed by the rank-and-file employees, and
that the capatazes  were "supervising and instructing the miners, mackers and other rank-and-file workers under
them, assess[ing] and evaluat[ing] their performance, mak[ing] regular reports and recommend[ing] new systems and
procedure of work, as well as guidelines for the discipline of employees." Hence, Med-Arbiter Lontoc concluded,
the capatazes "differ[ed] from the rank-and-file and [could] by themselves constitute a separate bargaining unit."

The bargaining unit sought to be represented by the appellee are the capataz employees of the appellant. There is
no other labor organization of capatazes within the employer unit except herein appellant. Thus, appellant is an
unorganized establishment in so far as the bargaining unit of capatazes is concerned. In accordance with the last
paragraph of Section 11, Rule XI, Department Order No. 9 which provides that "in a petition filed by a legitimate labor
organization involving an unorganized establishment, the Med-Arbiter shall, pursuant to Article 257 of the Code,
automatically order the conduct of certification election after determining that the petition has complied with all
requirements under Section 1, 2 and 4 of the same rules and that none of the grounds for dismissal thereof exists",
the order for the conduct of a certification election is proper.

We cannot undo the affirmance by the DOLE Secretary of the correct findings of her subordinates in the DOLE, an
office that was undeniably possessed of the requisite expertise on the matter in issue. In any event, we affirm
that capatazes or foremen are not rank-andfile employees because they are an extension of the management, and
as such they may influence the rank-and-file workers under them to engage in slowdowns or similar activities
detrimental to the policies, interests or business objectives of the employers.

24. UNFAIR LABOR PRACTICE

163. Goya Inc. vs. Goya Inc. Employees Union-FFW, G.R. No. 170054, January 21, 2013

Facts:
Sometime in January 2004, petitioner Goya, Inc. (Company), a domestic corporation engaged in the manufacture,
importation, and wholesale of top quality food products, hired contractual employees from PESO Resources
Development Corporation (PESO) to perform temporary and occasional services in its factory in Parang, Marikina
City. This prompted respondent Goya, Inc. Employees Union-FFW (Union) to request for a grievance conference on
the ground that the contractual workers do not belong to the categories of employees stipulated in the existing
Collective Bargaining Agreement (CBA). When the matter remained unresolved, the grievance was referred to the
National Conciliation and Mediation Board (NCMB) for voluntary arbitration. 

The Union asserted that the hiring of contractual employees from PESO is not a management prerogative and in
gross violation of the CBA tantamount to unfair labor practice (ULP). The CBA merely provides for three (3)
categories of employees in the Company, namely: probationary employees, regular employees and casual
employees.

In countering the Union's allegations, the Company argued that: (a) the law expressly allows contracting and
subcontracting arrangements through Department of Labor and Employment (DOLE) Order No. 18-02; (b) the
engagement of contractual employees did not, in any way, prejudice the Union, since not a single employee was
terminated and neither did it result in a reduction of working hours nor a reduction or splitting of the bargaining unit;
and (c) Section 4, Article I of the CBA merely provides for the definition of the categories of employees and does not
put a limitation on the Company's right to engage the services of job contractors or its management prerogative to
address temporary/occasional needs in its operation.

Issue:

Whether or not the Company is guilty of unfair labor practice in engaging the services of PESO, a third party service
provider, under existing CBA, laws, and jurisprudence.

Ruling:

A careful reading of the above-enumerated categories of employees reveals that the PESO contractual employees do
not fall within the enumerated categories of employees stated in the CBA of the parties. Following the said
categories, [the Company] should have observed and complied with the provision of their CBA. Since the Company
had admitted that it engaged the services of PESO to perform temporary or occasional services which is akin to those
performed by casual employees, the Company should have tapped the services of casual employees instead of
engaging PESO.

In justifying its act, the Company posits that its engagement of PESO was a management prerogative. It bears
stressing that a management prerogative refers to the right of the employer to regulate all aspects of employment,
such as the freedom to prescribe work assignments, working methods, processes to be followed, regulation regarding
transfer of employees, supervision of their work, lay-off and discipline, and dismissal and recall of work, presupposing
the existence of employer-employee relationship. On the basis of the foregoing definition, the Company's
engagement of PESO was indeed a management prerogative. This is in consonance with the pronouncement of the
Supreme Court in the case of Manila Electric Company vs. Quisumbing where it ruled that contracting out of services
is an exercise of business judgment or management prerogative.

This management prerogative of contracting out services, however, is not without limitation. In contracting out
services, the management must be motivated by good faith and the contracting out should not be resorted to
circumvent the law or must not have been the result of malicious arbitrary actions. In the case at bench, the CBA of
the parties has already provided for the categories of the employees in the Company's establishment. These
categories of employees particularly with respect to casual employees serve as limitation to [the Company's]
prerogative to outsource parts of its operations especially when hiring contractual employees. As stated earlier, the
work to be performed by PESO was similar to that of the casual employees. With the provision on casual employees,
the hiring of PESO contractual employees, therefore, is not in keeping with the spirit and intent of their CBA.

164. Park Hotel et al., vs. Soriano et al., G.R. No. 171118, September 10, 2012

Principle: The Labor Code considers it an unfair labor practice when an employer interferes, restrains or coerces
employees in the exercise of their right to self-organization or the right to form an association.

Facts:

Harbutt (Harbutt) and Bill Percy (Percy) are the General Manager and owner, respectively, of Park Hotel.
Percy, Harbutt and Atty. Roberto Enriquez are also the officers and stockholders of Burgos Corporation (Burgos), a
sister company of Park Hotel.

Respondent Manolo Soriano (Soriano) was hired by Park Hotel in July 1990 as Maintenance Electrician, and
then transferred to Burgos in 1992. Respondent Lester Gonzales (Gonzales) was employed by Burgos as Doorman,
and later promoted as Supervisor. Respondent Yolanda Badilla (Badilla) was a bartender of J's Playhouse operated
by Burgos.
In October of 1997, Soriano, Gonzales and Badilla were dismissed from work for allegedly stealing company
properties. As a result, respondents filed complaints for illegal dismissal, unfair labor practice, and payment of moral
and exemplary damages and attorney's fees, before the Labor Arbiter (LA). In their complaints, respondents alleged
that the real reason for their dismissal was that they were organizing a union for the company's employees.

In his Affidavit, respondents claimed that on October 4, 1997, he was barred from entering the company
premises and that the following day, Harbutt shouted at him for having participated in the formation of a union.

Issue: Whether or not petitioners committed unfair labor practice.

Ruling: Yes

Anent the unfair labor practice, Article 248 (a) of the Labor Code27 considers it an unfair labor practice when
an employer interferes, restrains or coerces employees in the exercise of their right to self-organization or the right to
form an association.28 In order to show that the employer committed unfair labor practice under the Labor Code,
substantial evidence is required to support the claim. In the case at bar, respondents were indeed unceremoniously
dismissed from work by reason of their intent to form and organize a union. As found by the LA:

The immediate impulse of petitioners, as in the case at bar, was to terminate the organizers. Petitioners
have to cripple the union at sight, to frustrate attempts of employees from joining or supporting it, preventing them, at
all cost and to frustrate the employees’ bid to exercise their right to self-organization.

Thus, respondents were illegally dismissed.

[***Although relevant, the rulings below are no longer part of the issue emphasized by JMM.]

Requisites for a valid dismissal

The requisites for a valid dismissal are: (a) the employee must be afforded due process, i.e., he must be
given an opportunity to be heard and defend himself; and (b) the dismissal must be for a valid cause as provided in
Article 282 of the Labor Code, or for any of the authorized causes under Articles 283 and 284 of the same Code.25 In
the case before us, both elements are completely lacking. Respondents were dismissed without any just or
authorized cause and without being given the opportunity to be heard and defend themselves. The law mandates that
the burden of proving the validity of the termination of employment rests with the employer. Failure to discharge this
evidentiary burden would necessarily mean that the dismissal was not justified and, therefore, illegal

The doctrine of piercing the veil of corporate fiction in a Labor case

As to whether Park Hotel may be held solidarily liable with Burgos, the Court rules that before a corporation
can be held accountable for the corporate liabilities of another, the veil of corporate fiction must first be pierced. Thus,
before Park Hotel can be held answerable for the obligations of Burgos to its employees, it must be sufficiently
established that the two companies are actually a single corporate entity, such that the liability of one is the liability of
the other.

In the case at bar, respondents utterly failed to prove by competent evidence that Park Hotel was a mere
instrumentality, agency, conduit or adjunct of Burgos, or that its separate corporate veil had been used to cover any
fraud or illegality committed by Burgos against the respondents.

Verily, a corporation, being a juridical entity, may act only through its directors, officers and employees.
Obligations incurred by them, while acting as corporate agents, are not their personal liability but the direct
accountability of the corporation they represent.38 However, corporate officers may be deemed solidarily liable with
the corporation for the termination of employees if they acted with malice or bad faith. In the present case, the lower
tribunals unanimously found that Percy and Harbutt, in their capacity as corporate officers of Burgos, acted
maliciously in terminating the services of respondents without any valid ground and in order to suppress their right to
self-organization.
165. Baptista et al., vs. Villanueva et al., G,.R. No. 194709, July 31, 2013

FACTS:

Petitioners were former union members of Radio Philippines Network Employees Union (RPNEU), a legitimate labor
organization and the sole and exclusive bargaining agent of the rank and file employees of Radio Philippines
Network, while the respondents were the union's elected officers and members.

On suspicion of union mismanagement, petitioners filed a complaint for impeachment against all the union officers
and members of the union before the DOLE. Thereafter, written complaints were filed against petitioners for the
alleged violation of the union's Constitution and By-Laws, specifically for urging that a member start an action in any
court of justice or external investigative body against the Union or its officer without first exhausting all internal
remedies open to him or available in accordance with the CBL. Petitioners denied the charges imputed against them.

After the investigation, petitioners were infromed of their expulsion from the union and their termination from
employment in compliance with their CBA's union security clause. Aggrieved, petitioners filed complaints for ULP
against the respondents pursuant to Article 249 (a) and (b) of the Labor Code. The LA adjudged the respondents
guilty of ULP. The NLRC set aside the LA decision and dismissed the complaint for ULP for lack of merit. The CA
sustained the NLRC decision.

ISSUE:

Whether or not the respondents committed ULP under Article 249 (a) and (b) of the Labor Code.

RULING: NO. There is no ULP.

The primary concept of ULP is embodied in Article 247 of the Labor Code. In essence, ULP relates to the commission
of acts that transgress the workers' right to organize. As specified in Articles 248 and 249 of the Labor Code, the
prohibited acts must necessarily relate to the workers' right to self-organization and to the observance of a CBA.
Absent the said vital elements, the acts complained, although seemingly unjust, would not constitute ULP.

In this case, petitioners claim that the respondents, as union officers, are guilty of ULP for violating paragraphs (a)
and (b) of Article 249 of the Labor Code, to wit:

ART. 249.UNFAIR LABOR PRACTICES OF LABOR ORGANIZATIONS. — It shall be unfair labor practice for a labor
organization, its officers, agents or representatives:

(a)To restrain or coerce employees in the exercise of their rights to self-organization. However, a labor organization
shall have the right to prescribe its own rules with respect to the acquisition or retention of membership;

(b)To cause or attempt to cause an employer to discriminate against an employee, including discrimination against an
employee with respect to whom membership in such organization has been denied or to terminate an employee on
any ground other than the usual terms and conditions under which membership or continuation of membership is
made available to other members.

It is well-settled that workers' and employers' organizations shall have the right to draw up their constitutions and
rules. In this case, RPNEU's Constitution and By-Laws expressly mandate that before a party is allowed to seek the
intervention of the court, it is a pre-condition that he should have availed of all the internal remedies within the
organization. Petitioners were found to have violated the provisions of the union's Constitution and By-Laws when
they filed petitions for impeachment against their union officers and for audit before the DOLE without first exhausting
all internal remedies available within their organization. This act is a ground for expulsion from union membership.
Thus, petitioners' expulsion from the union was not a deliberate attempt to curtail or restrict their right to organize, but
was triggered by the commission of an act, expressly sanctioned by the union's Constitution and By-Laws.

For a charge of ULP against a labor organization to prosper, the onus probandi rests upon the party alleging it. In
labor cases, substantial evidence is required. Moreover, all the prohibited acts constituting unfair labor practice
should materially relate to the workers' right to self-organization.

Unfortunately, petitioners failed to discharge the burden required to prove the charge of ULP against the respondents.
Aside from their self-serving allegations, petitioners were not able to establish how they were restrained or coerced
by their union in a way that curtailed their right to self-organization. The records likewise failed to sufficiently show
that the respondents unduly persuaded management into discriminating against petitioners other than to bring to its
attention their expulsion from the union, which in turn, resulted in the implementation of their CBA's union security
clause.

166. TH Shopfitters Corp., et al., vs. T&H Shopfitters Corp., Union, GR No. 191714, Feb 26, 2014

FACTS:
On September 7, 2004, Respondents filed their Complaint for Unfair Labor Practice (ULP) by way of union busting,
and Illegal Lockout, with moral and exemplary damages and attorney's fees, against Petitioners, T&H Shopfitters
Corporation (T&H Shopfitters) and Gin Queen Corporation (Gin Queen).

Respondents treated T&H Shopfitters and Gin Queen as a single entity and their sole employer. In their desire to
improve their working conditions, respondents and other employees of petitioners held their first formal meeting on
November 23, 2003 to discuss the formation of a union. The following day or on November 24, 2003, seventeen (17)
employees were barred from entering petitioners' factory premises and ordered to transfer to T&H Shopfitters'
warehouse purportedly because of its expansion. Afterwards, the said employees were repeatedly ordered to go on
forced leave due to the unavailability of work.

Respondents contended that the affected employees were not given regular work assignments, while subcontractors
were continuously hired to perform their functions. This development prompted respondents to seek the assistance of
the National Conciliation and Mediation Board. Subsequently, an agreement between petitioners and THS-GQ Union
was reached. Petitioners agreed to give priority to regular employees in the distribution of work assignments.
Respondents averred, however, that petitioners never complied with its commitment but instead hired contractual
workers.

Petitioners stress that T&H Shopfitters and Gin Queen are corporations separate and distinct from each other.
Consequently, T&H Shopfitters and Stinnes Huang, an officer of T&H Shopfitters, cannot be held liable for ULP for
the reason that there is no employer-employee relationship between the former and respondents. Further, Gin Queen
avers that its decision to implement an enforced rotation of work assignments for respondents was a management
prerogative permitted by law, justified by the decrease in the orders it received from its customers. It explains that its
failure to present concrete proof of its decreasing orders was due to the impossibility of proving a negative assertion.
It also asserts that the transfer from Castillejos to Cabangan was made in good faith and solely because of the
expiration of its lease contract in Castillejos.

In its Decision, dated December 21, 2005, the LA dismissed respondents' complaint and all their money claims for
lack of merit.Aggrieved, respondents appealed to the NLRC. In its July 24, 2007 Decision, the NLRC reversed the LA
decision and ruled in favor of respondents. Petitioners filed a motion for reconsideration but the NLRC denied the
same. Dissatisfied with the adverse ruling, petitioners instituted a petition for certiorari under Rule 65 of the Rules of
Court before the CA arguing grave abuse of discretion on the part of the NLRC in reversing the LA decision.
In its Decision, dated November 12, 2009, the CA sustained the NLRC ruling.

ISSUE:

Whether or not Unfair Labor Practice acts were committed by petitioners against respondents.

RULING:

As to the issue of ULP, petitioners' argument is utterly without merit.

In the case at bench, petitioners are being accused of violations of paragraphs (a), (c), and (e) of Article 257 (formerly
Article 248) of the Labor Code, to wit:

Article 257. Unfair labor practices of employers.— It shall be unlawful for an employer to commit any of
the following unfair labor practices:
(a)  To interfere with, restrain or coerce employees in the exercise of their right to self-organization;
xxxxxxxxx
(c)  To contract out services or functions being performed by union members when such will
interfere with, restrain, or coerce employees in the exercise of their right to self-organization;
xxxxxxxxx
(e)  To discriminate in regard to wages, hours of work, and other terms and conditions of
employment in order to encourage or discourage membership in any labor organization. . . .

In essence, ULP relates to the commission of acts that transgress the workers' right to organize. As
specified in Articles 248 [now Article 257] and 249 [now Article 258] of the Labor Code, the prohibited acts must
necessarily relate to the workers' right to self-organization.

The test of whether an employer has interfered with and coerced employees in the exercise of their right to
self-organization, that is, whether the employer has engaged in conduct which, it may reasonably be said, tends to
interfere with the free exercise of employees' rights; and that it is not necessary that there be direct evidence that any
employee was in fact intimidated or coerced by statements of threats of the employer if there is a reasonable
inference that anti-union conduct of the employer does have an adverse effect on self-organization and collective
bargaining.

The questioned acts of petitioners, namely: 1) sponsoring a field trip to Zambales for its employees, to the
exclusion of union members, before the scheduled certification election; 2) the active campaign by the sales officer of
petitioners against the union prevailing as a bargaining agent during the field trip; 3) escorting its employees after the
field trip to the polling center; 4) the continuous hiring of subcontractors performing respondents' functions; 5)
assigning union members to the Cabangan site to work as grass cutters; and 6) the enforcement of work on a
rotational basis for union members, all reek of interference on the part of petitioners.
Indubitably, the various acts of petitioners, taken together, reasonably support an inference that, indeed,
such were all orchestrated to restrict respondents' free exercise of their right to self-organization.

The Court is of the considered view that petitioners' undisputed actions prior and immediately before the
scheduled certification election, while seemingly innocuous, unduly meddled in the affairs of its employees in
selecting their exclusive bargaining representative. Petitioners had no business persuading and/or assisting its
employees in their legally protected independent process of selecting their exclusive bargaining representative. The
fact and peculiar timing of the field trip sponsored by petitioners for its employees not affiliated with THS-GQ Union,
although a positive enticement, was undoubtedly extraneous influence designed to impede respondents in their quest
to be certified. This cannot be countenanced.

Not content with achieving a "no union" vote in the certification election, petitioners launched a vindictive
campaign against union members by assigning work on a rotational basis while subcontractors performed the latter's
functions regularly. Worse, some of the respondents were made to work as grass cutters in an effort to dissuade
them from further collective action. Again, this cannot be countenanced.

More importantly, petitioners' bare denial of some of the complained acts and unacceptable explanations, a
mere afterthought at best, cannot prevail over respondents' detailed narration of the events that transpired. At this
juncture, it bears to emphasize that in labor cases, the quantum of proof necessary is substantial evidence, or that
amount of relevant evidence as a reasonable mind might accept as adequate to support a conclusion, even if other
minds, equally reasonable, might conceivably opine otherwise.

In fine, mindful of the nature of the charge of ULP, including its civil and/or criminal consequences, the Court
finds that the NLRC, as correctly sustained by the CA, had sufficient factual and legal bases to support its finding of
ULP.

You might also like